Текст
                    химия
Теория I Практика / Ключи и ответы
Курс самоподготовки
Технология решения заданий
2018
f

сдам ЕГЭ! ХИМИЯ Курс самоподготовки Технология решения заданий Учебное пособие для общеобразовательных организаций Москва «Просвещение» 2018
УДК 373.167.1:54 ББК 24я72.4 Я11 124- Авторы: А. А. Каверина, Ю. Н. Медведев, Г. Н. Молчанова, Н. В. Свириденкова, М. Г. Снастина, С. В. Стаханова Учебное пособие «Я сдам ЕГЭ! Химия. Курс самоподготовки. Технология решения заданий» создано авторским коллективом из числа членов Федеральной комиссии по разработке контрольных измерительных материалов ЕГЭ. Оно предназначено для эффективной подготовки учащихся 10—11 классов к государственной итоговой аттестации. Пособие содержит весь необходимый теоретический и фактологический материал, изучение которого является залогом успешной сдачи единого государственного экзамена. Для повышения эффективности работы с пособием в нём использованы различные подходы к предъявлению материала: тексты, схемы, иллюстрации, графики, таблицы. Для каждой из тем дан перечень ключевых понятий и приведены примеры заданий с комментариями и решениями. Данное пособие рекомендуется использовать в комплекте с пособием «Я сдам ЕГЭ. Химия. Типовые задания». Пособие адресовано школьникам, их родителям и учителям. Учебное издание Серия «Я сдам ЕГЭ!» Каверина Аделаида Александровна Медведев Юрий Николаевич Молчанова Галина Николаевна и др. ХИМИЯ Курс самоподготовки Технология решения заданий Учебное пособие для общеобразовательных организаций ЦЕНТР ЕСТЕСТВЕННО-МАТЕМАТИЧЕСКОГО ОБРАЗОВАНИЯ Редакция химии Зав. редакцией С. А. Сладков. Редактор Л. Н. Кузнецова. Ответственный за выпуск Л. Н. Кузнецова. Художественный редактор Т. В. Глушкова. Внешнее оформление и макет А. Г. Бушина. Техническое редактирование и компьютерная вёрстка Н. А. Разворотневой. Корректор Е. В. Барановская Налоговая льгота — Общероссийский классификатор продукции ОК 005-93—953000. Изд. лиц. Серия ИД №05824 от 12.09.01. Подписано в печать 15.11.17. Формат 84 х 108 716. Бумага типографская. Гарнитура Pragmatica. Доп. тираж 1000 экз. Заказ А-2948ТАТ. Акционерное общество «Издательство «Просвещение». 127521, Москва, 3-й проезд Марьиной рощи, 41. Отпечатано по заказу АО «ПолиграфТрейд» в типографии Полиграфическо-издательского комплекса «Идел-Пресс», филиала АО «ТАТМЕДИА». 420066, г. Казань, ул. Декабристов, 2. e-mail: id-press@yandex.ru http//www.idel-press.ru ISBN 978-5-09-057420-4 © Издательство «Просвещение», 2018 © Художественное оформление. Издательство «Просвещение», 2018 Все права защищены
ПРЕДИСЛОВИЕ Главная особенность учебного пособия «Я сдам ЕГЭ! Химия. Курс самоподготовки. Технология решения заданий» состоит в том, что отбор содержания проведён в соответствии со структурой «Кодификатора элементов содержания и требований к уровню подготовки выпускников общеобразовательных организаций для проведения в 2018 году ЕГЭ по химии». По аналогии с содержательными блоками кодификатора в пособии выделены четыре модуля: «Теоретические основы химии», «Неорганическая химия», «Органическая химия», «Количественные отношения в химии: расчёты по химическим формулам и уравнениям химических реакций». В каждом из этих модулей представлены те элементы содержания курса химии, усвоение которых обязательно проверяется на экзамене. Кроме того, в каждом модуле обозначен перечень формируемых умений, составленный в соответствии с требованиями к уровню подготовки выпускников. По сути это и есть указания на те действия и виды деятельности, которые необходимы для выполнения заданий экзаменационной работы. Важно также отметить особенности подходов к изложению учебного материала. В каждом модуле учебный материал излагается с учётом принципа целенаправленной систематизации и обобщения изученного. В этой связи много внимания уделяется тому, какие теоретические и фактологические знания имеют первостепенное значение в той или иной теме, каковы основные характеристики и границы применения ведущих понятий химии, в какой взаимосвязи находятся отдельные элементы содержания курса и др. Обращено также внимание на то, какой учебный материал может служить основой для построения заданий, аналогичных заданиям экзаменационной работы. В каждом модуле приведены примеры таких заданий и подробно прокомментировано их выполнение. Предлагаемое пособие максимально подчинено тому, чтобы оказать ученику, учителю, репетитору реальную помощь в организации работы по подготовке к экзамену. Для оптимального распределения времени на подготовку к ЕГЭ учащиеся могут использовать предложенное в пособии тематическое планирование. Учитель с помощью этого планирования может организовать подготовку к экзамену либо на уроках, либо на занятиях элективного курса. Рекомендуем использовать данное пособие в комплекте с пособием «Я сдам ЕГЭ. Химия. Типовые задания».
ТЕМАТИЧЕСКОЕ ПЛАНИРОВАНИЕ КУРСА «Я СДАМ ЕГЭ» (70 ч) Номер занятия Модули, темы занятий Количество часов МОДУЛЬ I. ТЕОРЕТИЧЕСКИЕ ОСНОВЫ ХИМИИ Формируемые умения: • раскрывать смысл основных химических понятий: атом, молекула, ион, валентность, степень окисления, электроотрицательность, окислитель, восстановитель; применять эти понятия при описании свойств эле- ментов, веществ и их превращений; • объяснять зависимость свойств химических элементов и их соединений от положения элемента в Перио- дической системе Д. И. Менделеева; • применять основные положения химических теорий (строения атома, химической связи, электролитиче- ской диссоциации, кислот и оснований, строения органических соединений, химической кинетики) для анализа строения и свойств веществ; • объяснять природу химической связи, сущность изученных видов химических реакций: электролитической диссоциации, ионного обмена, окислительно-восстановительных (и составлять их уравнения); влияние различных факторов на скорость химической реакции и на смещение химического равновесия 1 Строение электронных оболочек и электронная конфигурация атомов. Периодический закон и Периодиче- ская система химических элементов Д. И. Менделеева. Закономерности изменения химических свойств эле- ментов и их соединений по периодам и группам 2 2 Электроотрицательность, степень окисления и валентность химических элементов. Химическая связь. Веще- ства молекулярного и немолекулярного строения 2 3 Классификация и номенклатура неорганических веществ. Классификация химических реакций 2 4 Скорость реакции, её зависимость от различных факторов. Обратимые и необратимые химические реакции. Химическое равновесие 2 5 Электролитическая диссоциация. Сильные и слабые электролиты. Реакции ионного обмена 2 6 Гидролиз 2 7 Окислительно-восстановительные реакции 4 Итого 16
МОДУЛЬ II. НЕОРГАНИЧЕСКАЯ ХИМИЯ Формируемые умения: • объяснять зависимость свойств химических элементов и их соединений от положения элемента в Перио- дической системе Д. И. Менделеева; • классифицировать неорганические вещества на основе их состава и строения; • использовать основные положения изученных химических теорий для анализа свойств неорганических веществ; составлять уравнения изученных видов химических реакций (электролитической диссоциации, ионного обмена, окислительно-восстановительных) с участием неорганических веществ 8 Характерные химические свойства представителей основных классов неорганических веществ 4 9 Свойства галогенов и их соединений 2 10 Неметаллы VIA-группы: кислород, сера 2 11 Азот и его соединения 4 12 Фосфор и его соединения 2 13 Неметаллы IVA-группы: углерод, кремний 2 14 Общие свойства металлов. Металлы А-групп 2 15 Металлы В-групп 4 Итого 22 МОДУЛЬ III. ОРГАНИЧЕСКАЯ ХИМИЯ Формируемые умения: • классифицировать органические вещества на основе их состава и строения; • применять основные положения теории строения органических соединений для анализа свойств веществ и их генетической взаимосвязи; • составлять уравнения химических реакций с участием органических соединений изученных классов 16 Классификация и номенклатура органических веществ. Теория строения органических соединений 2 17 Алканы, циклоалканы 2 18 Алкены, алкадиены 2 19 Алкины 2 CJTI
03 Продолжение Номер занятия Модули, темы занятий Количество часов 20 Ароматические углеводороды 2 21 Спирты и фенолы 4 22 Карбонильные соединения: альдегиды и кетоны 2 23 Карбоновые кислоты, сложные эфиры 2 24 Амины, аминокислоты 2 25 Жиры. Углеводы 2 Итого 22 МОДУЛЬ IV. КОЛИЧЕСТВЕННЫЕ ОТНОШЕНИЯ В ХИМИИ: РАСЧЁТЫ ПО ХИМИЧЕСКИМ ФОРМУЛАМ И УРАВНЕНИЯМ ХИМИЧЕСКИХ РЕАКЦИЙ Формируемые умения: • проводить вычисления по химическим формулам и уравнениям, применяя знания о характерных свой- ствах соединений различных классов 26 Расчёты массы вещества или объёма газа по известному количеству вещества, массе или объёму одного из участвующих в реакции веществ 4 27 Расчёты с использованием понятия «массовая доля вещества в растворе» 4 28 Задачи на установление молекулярной и структурной формулы вещества 2 Итого 10
Модуль I. Теоретические основы химии 1 ЗАНЯТИЕ 1 Строение электронных оболочек и электронная конфигурация атомов. Периодический закон и Периодическая система химических эле- ментов Д. И. Менделеева. Закономерности изменения химиче- ских свойств элементов и их соединений по периодам и группам Основные элементы содержания: атом, атомное ядро, протон, нейтрон, электрон, атомная орбиталь, энергетический уровень, энергетический подуровень, электронная конфигурация атома, внешние и валентные электро- ны, основное и возбуждённое состояние атома, Периодический закон и Периодическая система химических элементов Д. И. Менделеева, атомный радиус, электроотрицательность. Атом — электронейтральная частица, состоящая из положительно заряженного ядра и вращающихся вокруг него отрицательно заряженных электронов. Ядро атома состоит из протонов и нейтронов (нуклонов). В целом атом элек- тронейтрален, поскольку число протонов в ядре равно числу электронов в атоме. Таблица 1 Основные характеристики протонов, нейтронов и электронов Частица Обозначение Заряд, усл. Массовое число Масса, а. е. м. Электрон ё — 1 0 5,5 • 10-4 Протон Р +1 1 1,0073 Нейтрон п 0 1 1,0087 Как установлено Г. Мозли, заряд ядра численно равен порядковому (атомному) номеру элемента в Периодической системе элементов Д. И. Менделеева: Порядковый номер Заряд _ Число протонов ядра “ в ядре Число электронов в атоме Поскольку химические свойства элементов определяются в первую очередь по- ведением электронов, при изучении химии подробно рассматривают особенности электронного строения атомов. В соответствии с квантовой механикой движение электрона вокруг ядра атома нельзя рассматривать как простое механическое пе- ремещение: электрон может находиться в любой точке вблизи ядра атома, но ве- роятность его пребывания в различных точках неодинакова. Строение электронных оболочек атомов Для характеристики состояния электрона в атоме используют понятие атомной орбитали. Атомная орбиталь (сокращённо АО) — это область пространства в ато- ме, в которой наиболее вероятно находится электрон1. 1 Более строгое определение атомной орбитали даётся в курсе химии высшей школы.
8 Каждая атомная орбиталь имеет определённую форму, орбитали разной сим- метрии обозначают буквами s, р, d и f. s-Орбитали имеют форму сферы (шара), р-орбитали — форму объёмной восьмёрки, вытянутой вдоль соответствующей оси координат, d- и f-орбитали — более сложную форму (рис. 1). Атомные орбитали группируются, образуя энергетические уровни и подуровни. Энергетические уровни обозначают числом (л = 1, 2, 3, 4 и т. д.). Каждый энергети- ческий уровень состоит из п подуровней (т. е. число подуровней в энергетическом уровне равно номеру этого уровня). Подуровень объединяет орбитали одного вида, поэтому говорят о s-подуровне, р-подуровне, d-подуровне, f-подуровне. Так, первый уровень состоит только из одного подуровня (Is-подуровень), второй уровень — из двух подуровней (2s и 2р), третий — из трёх подуровней (3s, Зр и 3d) и т. д.
9 Следует запомнить, что любой s-подуровень состоит из одной s-орбитали, р-подуровень — из трёх р-орбиталей, d-подуровень — из пяти d-орбиталей. Ска- занное поясняет рисунок 2. Рис. 2. Энергетические уровни и подуровни в атоме Число электронов на каждой атомной орбитали ограничено. В соответствии с принципом Паули, на атомной орбитали не может находиться более двух элек- тронов. Следовательно, первый энергетический уровень (одна АО) максимально вмещает только 2 электрона, второй уровень (четыре АО) — 8 электронов, третий уровень (девять АО) — 18 электронов, четвёртый уровень (16 АО) — 32 электрона. Электронная конфигурация показывает распределение электронов в атоме по энергетическим уровням, подуровням и орбиталям. Экспериментально установле- но, что атомные орбитали заполняются электронами в следующем порядке: 1s < 2s < 2р < 3s < Зр < 4s < 3d < 4р < 5s... Обратите внимание: сначала заполняется 45-атомная орбиталь, и лишь затем — 3d. Причины этого изучаются в высшей школе. Составляя электронную формулу атома, сначала цифрой указывают номер энер- гетического уровня, затем буквой (s, р, d, f) обозначают подуровень и, наконец, указывают число электронов на данном подуровне (в виде верхнего индекса). Так, запись 1s2 означает, что на Is-подуровне находятся два электрона. Обратите внимание на электронную конфигурацию хрома и меди, в атомах кото- рых происходит «провал» электрона с внешнего 4s- на предвнешний Зд-подуровень. Это связано с повышенной энергетической устойчивостью наполовину (d5) и пол- ностью заполненных (d10) орбиталей. В ряде случаев электронные конфигурации удобно изображать в виде графиче- ских схем, обозначая электроны стрелками. Электроны, находящиеся на последнем энергетическом уровне, называют- ся внешними электронами. Электроны, которые могут принимать участие в образовании химических связей, называются валентными электронами.
10 Таблица 2 Электронные конфигурации атомов элементов 1—4-го периодов Период Z Символ Электронная формула I 1 Н 1s1 2 Не 1s2 II 3 и 1s22s1 4 Be 1s22s2 5 В 1s22s22p’ 6 с 1s22s22p2 7 N 1s22s22p3 8 0 1s22s22p4 9 F 1s22s22p5 10 Ne 1s22s22p6 III 11 Na 1s22s22p63s1 12 Mg 1s22s22p63s2 13 Al 1s22s22p63s23p’ 14 Si 1s22s22p63s23p2 15 p 1s22s22p63s23p3 16 s 1 s22s22p63s23p4 17 Cl 1 s22s22p63s23p5 18 Ar 1 s22s22p63s23p6 IV 19 К 1s22s22p63s23p64s1 20 Ca 1s22s22pe3s23p64s2 21 Sc 1 s22s22p63s23p64s23d1 22 Ti 1 s22s22p63s23p64s23d2 23 V 1 s22s22p63s23p64s23cF 24 Cr 1s22s22p63s23p64s’3d5 «провал» электрона c s- на d-подуровень 25 Mn 1s22s22p63s23p64s23d5 26 Fe 1 s22s22p63s23p64s23d6 27 Co 1s22s22p63s23p64s23d7 28 Ni 1 s22s22p63s23p64s23d8 29 Cu 1 s22s22p63s23p64sl3d10 «провал» электрона c s- на d-подуровень 30 Zn 1 s22s22p63s23p64s23d10 31 Ga 1 s22s22p63s23pe4s23d,04p ’ 32 Ge 1 s22s22p63s23p64s23d'°4p2 33 As 1 s22s22p63s23pe4s23d’°4p3 34 Se 1 s22s22p63s23p64s23d104p4 35 Br 1 s22s22p63s23p64s23d104p5 36 Kr 1 s22s22p63s23p64s23d104p6
11 Таблица 3 Графические схемы электронных конфигураций атомов 1-го и 2-го периодов Атом Электронная конфигурация Графическое изображение электронной конфигурации внешнего уровня 1Н 1s1 1 le 2Не le2 11 1а 3Li Is^s1 ♦ 28 2р 4Ве 182282 1» 2s 2р 5в la22s22pl ♦ н 28 2р вс ls22s22p2 1 1 1» 28 2р 7n ls22s22p3 t ♦ ♦ 1» 2s 2р Для элементов главных подгрупп валентными являются внешние s- и р-электроны, число валентных электронов равно номеру группы: 17С1 ls22s22p6^3^5| — валентные электроны Для элементов побочных подгрупп валентными являются внешние s- и пред- внешние d-электроны: 24Сг ls22s22p63s23p6|3d54s1| — валентные электроны
12 Приобретая энергию, атом может из своего основного, самого устойчивого со- стояния перейти в возбуждённое состояние. Если основное состояние — одно- единственное, то возбуждённых состояний может быть множество. Сказанное поясняет рисунок 3 на примере основного и возбуждённого состояния атома алю- миния. 13А1 ls22s22p1 13А1* ls22s12p2 1 t tl 2s Н 2s Основное состояние Возбуждённое состояние Рис. 3. Основное и возбуждённое состояние атома алюминия Периодический закон и Периодическая система химических элементов Д. И. Менделеева Современная формулировка Периодического закона Д. И. Менделеева: «Свой- ства элементов, а также свойства образуемых ими соединений находятся в пе- риодической зависимости от величины заряда атомных ядер». Графическим выражением Периодического закона является Периодическая си- стема химических элементов, которую представляют обычно в виде таблицы. Каж- дый элемент в Периодической системе имеет своё место, определяемое номером группы и номером периода. Группами называют вертикальные столбцы Периоди- ческой системы. В классическом варианте таблицы группа состоит из двух под- групп — главной и побочной. Главную подгруппу обозначают латинской буквой А (группа А), а побочную — латинской буквой В (группа В). Так, пятую группу об- разуют элементы азот, фосфор, мышьяк, сурьма, висмут (группа VA) и ванадий, ниобий, тантал (группа VB). Атомы элементов, принадлежащих к одной группе, имеют, как правило, одинаковое число валентных электронов (в данном случае 5) и, следовательно, одинаковую высшую степень окисления (+5). Период — это горизонтальный ряд элементов, расположенных в порядке воз- растания зарядов ядер атомов. В Периодической системе Д. И. Менделеева семь периодов. Если порядковый номер элемента указывает на общее число электро- нов в атоме элемента, то номер периода указывает на число энергетических уров- ней, на которых расположены электроны в этом атоме в основном состоянии. Так, все 15 электронов в атоме фосфора (элемент № 15) расположены на трёх энер- гетических уровнях (так как фосфор — элемент 3-го периода). Все 26 электронов в атоме железа (элемент № 26) расположены на четырёх энергетических уровнях (так как железо — элемент 4-го периода). Изменение свойств элементов в главных подгруппах В главных подгруппах сверху вниз возрастает число заполненных электронных слоёв, следовательно, возрастают атомные радиусы элементов. В силу этого воз- растает лёгкость отдачи внешних электронов (уменьшается энергия ионизации), уменьшается электроотрицательность элементов, усиливаются металлические свойства и, напротив, ослабевают неметаллические свойства простых веществ, образованных этими элементами. Соответственно, кислотные свойства оксидов и
13 Рис. 4. Изменение радиусов атомов элементов IA- и ПА-групп гидроксидов сверху вниз по подгруппе уменьшаются, а оснбв- ные — усиливаются. Так для элементов VA-группы в ряду окси- дов N2O5 Р20б As2O5 — Sb,Ос — Bi,Ос имеем: св n2o5 КИСЛОТНЫЙ оксид « Н и 3S о д Р2О5 о кислотный оксид О ф As2O5 кислотный оксид м н 3 амфотерный оксид о ч Д ЬЬ2О5 д с преобладанием кислотных свойств о я * 8 амфотерный оксид Г Ь12°5 с преобладанием основных свойств Следовательно, происходит закономерное ослабление кис- лотных свойств оксидов (и гидроксидов) и усиление оснбвных. По группе сверху вниз возрастает также кислотность би- нарных водородных соединений (бескислородных кислот). Так, сила кислот возрастает в рядах: Н2О < H2S < H2Se < Н2Те HF < HCI < HBr < HI Изменение свойств элементов по периоду Рассмотрим в качестве примера характер изменения свойств периода: элементов 3-го nNa 12Mg 13А1 14Si 15Р 16S 17С1 18Аг С увеличением зарядов ядер атомов по периоду происходит последовательное увеличение числа электронов на внешнем уровне — от 1 (в атоме натрия) до 8 (в атоме аргона). Это приводит к усилению взаимодействия внешней оболочки с ядром и к уменьшению атомного радиуса: uNe 12Mg 13А1 14Si 15Р 16S 17С1 Рис. 5. Изменение радиусов атомов элементов 3-го периода Следствием уменьшения атомных радиусов является ослабление металлических свойств простых веществ, образованных элементами 3-го периода, и усиление неметаллических. С уменьшением атомных радиусов увеличивается электроотри- цательность элементов. Всё это обуславливает ослабление по периоду оснбвного характера оксидов и гидроксидов и усиление их кислотных свойств. Например, для неметаллов 2-го периода кислотные свойства водородных со- единений также усиливаются: NH3 < Н20 < HF
14 и Примеры заданий Пример 1. Какие из частиц имеют электронную конфигурацию 1 s22s22p63s23p6? 1) Li+ 2) К+ 3) Rb+ 4) Na+ 5) S2 Прежде всего, используя Периодическую систему химических элементов Д. И. Менделеева, составим электронные конфигурации атомов лития, калия, рубидия, натрия и серы: Li 1s22s1 К 1s22s22p63s23pe4s’ Rb 1 s22s22p63s23p64s23d,04p65s’ Na 1s22s22p63s1 s 1s22s22p63s23p4 В задании речь идёт о конфигурации ионов, как катионов (Li+, К+, Rb+, Na+), так и анионов (S2). Вспомним, что катион образуется при отдаче электрона атомом, а анион — при присоединении электрона. С учётом сказанного со- ставим электронные конфигурации ионов, указанных в условии задачи: Li* 1s2 K* 1 s22s22p63s23p6 Rb* 1 s22s22p63s23p64s23d104p6 Na* 1s22s22p6 s2- 1s22s22pe3s23p6 Теперь ответ очевиден: условию удовлетворяют электронные конфигурации ионов калия и серы. Ответ: 2 5 Пример 2. Установите соответствие между формулой частицы и её элек- тронным строением. ФОРМУЛА ЧАСТИЦЫ А) Б) В) Г) ЭЛЕКТРОННОЕ СТРОЕНИЕ 1s2 1s22s2 1s22s22p3 1s22s22p6 1s22s22p63s1 1 s22s22p63s23p6 конфигурации атомов 2) 3) 4) 5) 6) Так же, как в примере 1, составим электронные ионов, оговорённых в условии: Na1 Аг S2- +5 N
15 Na 1s22s22pe3s’ Na+ 1s22s22pe Ar 1s22s22p63s23p6 Ar 1s22s22p63s23p6 s 1s22s22p63s23p4 s2- 1s22s22p63s23p® N 1s22s22p3 5+ N 1s2 Сопоставим полученные нами конфигурации с приведёнными в задании и легко найдём ответ: Na+ 1s22s22p6 4 Ar 1 s22s22p63s23p6 6 s2- 1s22s22p63s23p6 6 +5 N 1s2 1 Ответ: Пример 3. Дан перечень химических элементов: 1) Na 2) Mg 3) Al 4) Si 5) P Определите, атомы каких из указанных в ряду элементов имеют на внешнем энергетическом уровне 1 неспаренный электрон. Запишите в поле ответа номера выбранных элементов. Используя Периодическую систему химических элементов Д. И. Менделеева, составим электронные конфигурации атомов указанных в условии элемен- тов. Ограничимся при этом только электронными конфигурациями внешнего уровня. Используя полученные знания, составим графические электронные формулы внешнего уровня: Atom Электронная конфигурация Графическое изображение электронной конфигурации внешнего уровня nNa 3s1 t 3s i2Mg 3s2 И 3s 13A1 Зз^Зр1 11 Н Зр 3s
16 НННВНННИННННННННМММ1 Атом Электронная конфигурация Графическое изображение электронной конфигурации внешнего уровня 3s23p2 ♦ ♦ Зр t ♦ 3s23p3 t t ♦ Ответ стал очевидным: один неспаренный электрон на внешнем энергетиче- ском уровне имеют атомы натрия и алюминия. 1 3 Пример 4. В главных подгруппах (A-группах) Периодической системы с увеличением заряда ядра атомов происходит 1) усиление неметаллических свойств элементов 2) уменьшение числа протонов в ядре 3) увеличение радиуса атомов 4) увеличение электроотрицательности 5) усиление восстановительных свойств элементов С увеличением заряда ядра атомов в главных подгруппах Периодической системы сверху вниз увеличивается число заполняемых электронных уров- ней (слоёв), что ведёт к увеличению радиуса атома. Следовательно, ответ 3 правильный. С увеличением атомного радиуса возрастает лёгкость отдачи электронов, т. е. уменьшается энергия ионизации. Ответ 5 также правиль- ный. А вот все остальные суждения следует признать неверными. Ответ: Пример 5. В ряду химических элементов Na — Mg — Al 1) уменьшается заряд ядер атомов 2) увеличивается число электронов на внешнем электронном уровне 3) уменьшается электроотрицательность 4) уменьшается радиус атомов 5) усиливаются металлические свойства Натрий, магний и алюминий — химические элементы, принадлежащие од- ному периоду, в данном случае — 3-му. Натрий — элемент 1А-группы, магний — элемент IIA-группы, алюминий — IIIA-группы. При переходе по периоду слева направо увеличивается порядковый номер (от 11 до 13), сле- довательно, возрастает заряд ядра (от +11 до +13). Таким образом, ответ 1 неверный. С увеличением заряда ядра увеличивается и число электронов в атоме (от 11 до 13), в том числе и на внешнем уровне (от 1 до 3). Следо- вательно, ответ 2 верный. Другой верный ответ указан под номером 4 — от
17 натрия к алюминию и дальше по периоду атомный радиус уменьшается. Все остальные варианты ответов неверные. Ответ: 2 4 Пример 6. Дан перечень химических элементов: 1) Na 2) Mg 3) К 4) Al 5) Са Выберите из них три элемента, находящихся в одном периоде Периодиче- ской системы Д. И. Менделеева, и расположите их в порядке увеличения атомного радиуса. Запишите в поле ответа номера выбранных элементов в нужной последовательности. Прежде всего вспомним, где в Периодической системе расположены указан- ные химические элементы. Обратимся к Периодической системе. Три эле- мента одного периода — это натрий, магний и алюминий. Как известно, по периоду с возрастанием заряда ядра происходит уменьшение атомного ра- диуса. Следовательно, атомный радиус натрия больше, чем атомный радиус магния, а магния больше, чем алюминия: Я(А1) < fl(Mg) < fl(Na) Ответ: 4 2 1 ЗАНЯТИЕ 2 Электроотрицательность, степень окисления и валентность химических элементов. Химическая связь. Вещества молекулярного и немолекулярного строения Основные элементы содержания: электроотрицательность, валентность, степень окисления, химическая связь, энергия связи, длина связи, валентные электроны, виды химической связи: ковалентная, ионная, металличе- ская, водородная, кратность связи, с- и л-связи, кристаллические решётки: ионные, атомные, молекулярные, металлические. Электроотрицательность — способность атомов в молекуле смещать к себе электронную плотность от соседних атомов. Значение ЭО увеличивается по груп- пе снизу вверх, а по периоду — слева направо. Самым электроотрицательным элементом считается фтор. Ряд электроотрицательности неметаллов приведён ниже: F>O>N>Cl>Br>I>S>C>P>H>B>Si Электроотрицательность уменьшается
Обратите внимание, что для некоторых элементов значения ЭО очень близки. Это имеет место для N и Cl, S и С, Р и Н. Электроотрицательность служит мерой неметалличности элементов — чем боль- ше значение ЭО, тем сильнее элемент проявляет неметаллические свойства и тем менее выражены у него металлические свойства. Валентность характеризует способность атомов данного химического эле- мента к образованию химических связей. В разные исторические периоды раз- вития химии это понятие имело различное содержание. В настоящее вре- мя валентность определяется числом ковалентных связей, образуемых данным атомом в соединении. Обратите внимание: валентность не имеет знака! Валентные возможности атома определяются числом: — неспаренных электронов; — неподелённых электронных пар; — вакантных валентных орбиталей. Сравним, например, валентные возможности азота и фосфора. Элемент азот находится во 2-м периоде, в VA-группе, следовательно, электрон- ная конфигурация внешнего электронного уровня 2s22p3, или 2s Имея 5 валентных электронов, атом азота, однако, не может проявлять высшую валентность, равную номеру группы, и вот почему. За счёт трёх неспаренных р-электронов атом азота может образовать макси- мально три ковалентные связи по механизму спаривания электронов. Ещё одну связь, четвёртую, атом азота может образовать по донорно-акцепторному меха- низму за счёт неподелённой 2в-электронной пары (являясь её донором). Посколь- ку во втором энергетическом (квантовом) слое нет вакантных орбиталей, то атом азота не может распарить 2в2-электронную пару и повысить свои валентные воз- можности. Таким образом, атом азота может проявлять валентности I, II, III и IV, но не V. Азот не может быть пятивалентным! Даже в молекуле азотной кислоты атом азота образует только четыре ковалентные связи, что видно из следующей формулы: Н —О—N л 'о® В отличие от атома азота атом фосфора может увеличить свои валентные воз- можности. Элемент фосфор находится в 3-м периоде. Электронная конфигурация его валентных электронов 3s23p3, или I I I I I 111111 3d ♦ <| зр Зз Являясь аналогом азота, фосфор также может проявлять валентности I, II, III и IV. Но, так как для элементов 3-го периода доступны вакантные Зс/-орбитали,
19 атом фосфора может перейти в возбуждённое состояние, переведя один из s-электронов на d-подуровень: t ♦ ♦ Таким образом, атом фосфора может образовать пять ковалентных связей по механизму спаривания электронов. Валентность V фосфор проявляет в молекулах PF5, Н3РО4, РОС13 и др.: о=р—он Степень окисления — это условный заряд атома в соединении в предположе- нии, что все связи в этом соединении ионные (т. е. все связывающие электронные пары полностью смещены к атому более электроотрицательного элемента). В от- личие от валентности, степень окисления имеет знак — она может быть отрица- тельной, нулевой или положительной. Для подсчёта степеней окисления имеется ряд простых правил. Суть их в сле- дующем: 1. Степень окисления элемента в составе простого вещества принимается рав- ной нулю. Если вещество находится в атомарном состоянии, то степень окисления его атомов также равна нулю. 2. Ряд элементов проявляют в соединениях постоянную степень окисления. Сре- ди них: фтор (-1), щелочные металлы (+1), щелочноземельные металлы, бериллий, магний и цинк (+2), алюминий (+3). 3. Кислород, как правило, проявляет степень окисления -2 за исключением пе- роксидов Н2О2(-1) и фторида кислорода OF2 (+2). 4. Водород в соединении с металлами (в гидридах) проявляет степень окисле- ния -1, а в соединениях с неметаллами, как правило, +1 (кроме SiH4, В2Н6). 5. Алгебраическая сумма степеней окисления всех атомов в молекуле должна быть равной нулю, а в сложном ионе — заряду этого иона. Высшая положительная степень окисления равна, как правило, номеру группы элемента в Периодической системе (исключения — О и F, благородные газы). На основании приведённых выше правил можно найти степень окисления эле- мента в любом веществе. В качестве примера найдём степень окисления серы в кислотах H2SO3, H2S2O5, H2S3O10. Очевидно, что степень окисления водорода равна +1, кислорода -2. Обозначим степень окисления серы как q. Тогда можно записать: 2 • (+1) + q + 3 • (-2) = О 2 • (+1) + 2q + 5 • (-2) = О q = +4 q = +4 2 • (+1) + 3q + 10 • (-2) = О q = +6 Таким образом, в первых двух кислотах степень окисления серы одинакова и равна +4, в последней кислоте +6. В органических соединениях можно находить степени окисления элементов исходя из структурной формулы. Так, в молекуле бромметана, в соответствии с
20 приведённым выше определением степени окисления, сместим все связывающие электронные пары в сторону более электроотрицательных атомов: н н :вг:с:н Атом углерода при этом имеет 6 электронов (на два больше, чем число валент- ных электронов), что отвечает условному заряду -2. Рассуждая аналогично, на- ходим, что в молекуле бромфторметана степень окисления углерода равна нулю: н н :вг:с:Ё: :bfjc(:f: н ” Под химической связью понимают взаимодействие, приводящее к образова- нию физически устойчивой двухатомной системы. Образование химической связи при взаимодействии двух атомов сопровождается выделением энергии, которую называют энергией связи: чем больше энергия связи, тем прочнее молекула. Так, сравнивая энергетические эффекты двух реакций: Cl + CI —> С12 + 243 кДж I + I -> 12 + 149 кДж приходим к выводу, что химическая связь в молекуле хлора более прочная, чем в молекуле иода. Химическая связь образуется за счёт валентных электронов. Валентными яв- ляются s- и р-электроны внешнего уровня (для непереходных элементов), а также d-электроны предвнешнего уровня (для переходных элементов): Cl ls22s22p6 3s23p5 Х \ валентные электроны Мп ls22s22p63s23p6 3d54s2 Ионная связь — химическая связь, осуществляемая за счёт электростатиче- ского притяжения разноимённо заряженных ионов. Ионом называют заряженную частицу, которая образуется в результате присоединения электрона к атому или молекуле или при отдаче электрона атомом или молекулой. Ионами, например, являются частицы: Н+, Н", Вг", Ca2+, SO2’ (рис. 6). Соединения с ионной связью образуются при взаимодействии атомов элемен- тов, резко различающихся по электроотрицательности, например при взаимодей- ствии щелочных или щелочноземельных металлов с кислородом или галогенами. Например: Na -> Na+ + ё 1 s22s22p63s1 -> 1 s22s22p6 + ё Cl + ё -> СГ 1 s22s22p63s23p5 + ё -> 1 s22s22p63s23p6 И в одном и в другом случае получаются ионы с полностью завершённым 8-элек- тронным внешним уровнем (подчёркнуто в формуле). Подобные частицы с полно- стью завершённым внешним уровнем обладают повышенной устойчивостью. Обра- зовавшиеся ионы Na+ и Cl- взаимодействуют и объединяются в ионную частицу: ё \ .. © Na- + 5,С1: ---------*- [Na] ССК]
21 Виды химической связи ! Hci । ; н2 I Н90 I I Вг, । г || z । ।_______। ।______। * Водородная | 8- 8+ 8- 8+ | IHF-HF Н2О-Н2О| Рис. 6. Виды химической связи В силу особенностей ионной связи (ненаправленность и ненасыщаемость) в ходе подобных реакций образуются не отдельные молекулы, а ионные кристаллы, состоящие из колоссального числа взаимодействующих друг с другом ионов. Ион- ная связь присутствует в таких соединениях, как NaCI, KI, CsBr, CaO, BaF2 и др. Другим примером соединений с ионной связью являются щёлочи, соли кислород- содержащих кислот, соли аммония. Так, в NaOH между катионом натрия и гидрок- сид-ионом действует ионная связь, в то время как между атомами водорода и кислорода в гидроксид-ионе — ковалентная связь: ® 0 Na О—Н Ковалентная связь осуществляется за счёт образования общих электронных пар. Ковалентная неполярная связь образуется между атомами одного и того же химического элемента, например в молекулах Н2, О2, Cl2> Na2. Ковалентная поляр- ная связь образуется между атомами (обычно неметаллов), различающимися по электроотрицательности, например в молекулах HCI, Н2О, NH3 (см. рис. 6). Существуют два механизма образования ковалентной связи — обменный ме- ханизм, т. е. механизм спаривания (или обобществления) электронов, и донорно- акцепторный механизм. В соответствии с первым механизмом ковалентная связь образуется при спари- вании электронов, принадлежавших разным атомам. Так, при взаимодействии двух атомов водорода (электронная конфигурация 1s1) происходит образование общей электронной пары: н- + -н ------------► н:н Аналогично образование общей электронной пары происходит и при взаимо- действии атомов водорода и хлора (конфигурация 3s23p5): н- + .ci: ----------► н:й:
22 В соответствии со вторым механизмом образования ковалентной связи, у одно- го атома (донора) должна быть неподелённая электронная пара, которая взаимо- действует с вакантной орбиталью другого атома (акцептора): н н— N — н I н И в этом случае химическая связь обусловлена появлением общей электрон- ной пары, которая до взаимодействия принадлежала одному из атомов. Донор- но-акцепторный механизм образования ковалентной связи реализуется во многих неорганических соединениях. Одна из связей образована по донорно-акцепторно- му механизму в ионе аммония NH4+, ионе гидроксония Н3О, молекулах озона О3 и азотной кислоты. Так, молекула аммиака взаимодействует с ионом водорода (протоном) с обра- зованием иона аммония: н I н — n: + н+ -----------► I н Одна из связей в ионе аммония образована по донорно-акцепторному механиз- му, но она ничем не отличается от остальных химических связей в этом ионе — ни длиной, ни прочностью (энергией). Если между двумя атомами образовалась одна ковалентная связь (одна общая электронная пара), то она называется одинарной. Между двумя атомами могут возникать и кратные связи — двойные (две общие электронные пары) и тройные (три общие электронные пары): н о: н—с—н :№n: :о=с = о: н—о—s—о—н н :о: Среди ковалентных связей принято выделять, в соответствии с симметрией электронного облака, ст- и л- связи. ст-Связи образуются при перекрывании элек- тронных облаков вдоль линии связи (рис. 7). Линия связи 8 S 8 р р р Рис. 7. Образование о-связей л-Связи образуются при боковом перекрывании р-электронных облаков над и под линией связи (рис. 8). Рис. 8. Образование л-связи ст-Связи более прочные, чем л-связи.
23 Основными характеристиками ковалентной связи являются длина и энергия. Под длиной связи понимают соответствующее межъядерное расстояние. Хими- ческая связь тем прочнее, чем меньше её длина. Так, в ряду однотипных соеди- нений HF — HCI — НВг — HI происходит увеличение длины связи, вызванное уве- личением размера атомов галогенов сверху вниз по подгруппе. Соответственно, в этом же ряду происходит и уменьшение прочности химической связи, уменьшение её энергии (табл. 4). Таблица 4 Длина и энергия связи в молекулах галоидводородов Молекула HF HCI НВг HI Длина связи, А 0,92 1,28 1,41 1,60 увеличивается Энергия связи, кДж/моль 565 431 364 217 увеличивается Ниже приведены и другие примеры, показывающие зависимость энергии связи от её кратности и длины (чем выше кратность, тем меньше длина и больше энер- гия связи) (табл. 5). Таблица 5 Длина и энергия некоторых простых и кратных связей Связь Длина, А Энергия, кДж/моль Связь Длина,А Энергия, кДж/моль с-с 1,54 343 с- О 1,43 351 с = с 1,33 615 с о 1,23 711 с = с 1,21 821 C-N 1,13 1096 Металлическая связь (см. рис. 6) существует в твёрдых металлах и их сплавах. Она осуществляется между атомами с небольшим числом валентных электронов, слабо удерживаемых ядром, и большим числом вакантных валентных орбиталей. Твёрдый металл представляет собой «каркас» из атомов и положительных ионов, находящихся в узлах кристаллической решётки и погружённых в «море» подвиж- ных электронов. Как и другие виды химической связи, металлическая связь имеет электростатическую природу. Водородная связь — особый тип межмолекулярного (иногда — внутримоле- кулярного) взаимодействия, обусловленного электростатическим притяжением положительно заряженного атома водорода и отрицательно заряженного атома одного из наиболее электроотрицательных элементов — фтора, кислорода или
24 азота. Например, жидкое состояние спиртов при обычных условиях объясняется наличием прочных межмолекулярных водородных связей: Вещества молекулярного и немолекулярного строения Твёрдые вещества чаще всего имеют кристаллическое строение, характеризу- ющееся правильным расположением частиц в определённых точках пространства. При соединении этих точек воображаемыми прямыми линиями образуется так на- зываемая кристаллическая решётка. Точки, в которых размещены частицы, на- зывают узлами кристаллической решётки. В узлах кристаллической решётки могут находиться ионы, атомы или молекулы. В зависимости от вида частиц, расположенных в узлах кристаллической решёт- ки, и характера связи между ними различают четыре типа кристаллических решё- ток: ионные, атомные, молекулярные и металлические (рис. 9; табл. 6). Ионную решётку образуют все вещества с ионным типом связи — соли, щё- лочи, бинарные соединения активных металлов с активными неметаллами (окси- ды, галогениды, сульфиды). Примеры: КОН, ВаСО3, СН3СООК, органические соли, алкоголяты, феноляты, соли аммония и аминов: NH4NO3, [CH3NH3]CI. Вещества с ионной кристаллической решёткой имеют, как правило, высокие температуры плав- ления, многие из них растворимы в воде, являются электролитами. Атомная кристаллическая решётка характерна для алмаза, графита, бора, кремния, германия, оксида кремния SiO2 (кремнезём, кварц, речной песок), кар- бида кремния SiC (карборунд). Вещества с атомной кристаллической решёткой имеют высокие температуры плавления, нерастворимы в воде, не относятся к электролитам. Металлическая решётка реализуется в кристаллах металлов и сплавов. В уз- лах решётки — атомы и катионы металла, при этом электроны металла обобщест- вляются и образуют так называемый «электронный газ», который движется между узлами решётки, обеспечивая её устойчивость. Металлы в силу своего строения Кристаллические решётки Рис. 9. Типы кристаллических решёток
25 электро- и теплопроводны, обладают высокой пластичностью, способны хорошо отражать свет, имеют характерный металлический блеск. В узлах молекулярной решётки находятся молекулы, которые слабо удержива- ются там с помощью непрочных межмолекулярных сил. Такую решётку имеют все органические вещества (кроме солей), газы и жидкости, легкоплавкие и летучие твёрдые вещества, в молекулах которых ковалентные связи (полярные и неполяр- ные), все вещества с запахом. Таблица 6 Характеристики кристаллических решёток различных типов Вид кристал- лической решётки Молекулярная Атомная Ионная Металлическая Частицы в узлах кри- сталлической решётки Молекулы Атомы Ионы Атомы и катионы металлов Характер сил взаимодей- ствия между частицами Слабые меж- молекулярные взаимодействия Ковалентная связь Ионная связь Обобществлённые валентные элек- троны — «элек- тронный газ» Характер- ные свойства веществ с данным типом решётки Малая твёр- дость, низкие температуры плавления Высокая твёр- дость, высокие температуры плавления, хруп- кость, отсутствие растворимости в воде Высокие темпе- ратуры плав- ления, высокая твёрдость, воз- можна раство- римость в воде, растворы и рас- плавы проводят электрический ток Ковкость, пла- стичность, те- плопроводность электропровод- ность Примеры веществ с данным типом решётки Большинство органических веществ, мно- гие неметаллы в твёрдом со- стоянии: сера, галогены, азот, кислород и др., твёрдый угле- кислый газ, га- логеноводороды и др. Алмаз, графит, кремний, крем- незём SiO2, кар- бид кремния SiC Большинство со- лей, щёлочи, ок- сиды металлов IA- и ПА-групп Металлы, сплавы
Примеры заданий Пример 1. Атом азота проявляет валентность, отличную от III, в соединениях 1) HNO3 2) HNO2 3) NBr3 4) NH3 5) NH4CI Приведём структурные формулы указанных веществ с учётом того, что хло- рид аммония имеет ионное строение: о // HO—N© Br— N—Br H—O—N=O Br H H—N—H H—N—H СГ H H в то время как в остальных молекулах — Совершенно очевидно, что в молекуле азотной кислоты и в хлориде аммо- ния атом азота четырёхвалентен, трёхвалентен. Ответ: +4, а валентность IV атом углерода про- 4) А14С3 5) СВг4 Пример 2. Степень окисления являет в соединениях 1) СН4 2) СО 3) Н2СО3 Прежде всего определим степень окисления углерода в указанных веще- ствах. Для этого используем правила, приведённые выше. Поскольку элек- троотрицательность углерода больше, чем у водорода и алюминия, то в СН4 и А14С3 углерод имеет отрицательную степень окисления, а именно -4. В трёх оставшихся веществах углерод проявляет положительную степень окисления, а именно +2 в СО и +4 в Н2СО3 и СВг4. В двух последних соединениях атом углерода образует по валентным: четыре ковалентные связи, т. е. является четырёх- Вг НОХ l/Br С=О НО Вг Вг Ответ: 3 5 4) Ва(СЮ2)2 5) С12О7 следовательно, высшая степень окисления хло- в задании веществах степень окисления хлора NaCIO, Пример 3. Максимально возможную степень окисления атом хлора про- являет в соединениях 1) СЮ3 2) НСЮ3 3) Хлор — элемент VIIA группы, ра равна +7. В приведённых равна:
27 Соединение Степень окисления хлора сю3 +6 нсю3 +5 NaCIO4 +7 Ва(СЮ2)2 +3 С12О7 +7 Ответ: 3 5 Пример 4. Степень окисления +3 атом углерода имеет в соединениях 1) Н2СО3 2) Н2С2О4 3) ВаСО3 4) КНСО3 5) C2F6 Используя правила, приведённые выше, найдём степень окисления углерода в соединениях: Соединение Степень окисления углерода Н2СО3 +4 Н2С2О4 +3 ВаСО3 +4 КНСО3 +4 сл +3 Степень окисления можно найти и через структурные формулы соединений. Структурные формулы угольной кислоты и карбонат-иона вам хорошо знако- мы, структурная формула щавелевой кислоты приведена ниже: нет хон Ответ: 2 5 Пример 5. Дан перечень химических элементов: 1) Si 2) Ti 3) Ge 4) Са 5) Se Выберите из них три элемента, находящихся в одном периоде Периодиче- ской системы Д. И. Менделеева и способных проявлять степень окисления +4. Запишите номера выбранных ответов в поле ответа.
28 Ответ: Ответ: 2 3 5 2 5 4) сероводород 5) нитрат калия Прежде всего вспомним, где в Периодической системе расположены указан- ные химические элементы. Кремний — элемент 3-го периода, титан, герма- ний, кальций и селен — элементы 4-го периода. Среди четырёх последних химических элементов кальций (IIA-группа) неспособен проявлять степень окисления +4. Остаются титан (IVB), германий (IVA) и селен (VIA). Эти эле- менты действительно способны проявлять указанную степень окисления, на- пример в оксидах ТЮ2, GeO2 и SeO2. Пример 6. Какие из указанных веществ образованы ионной химической связью? 1) оксид кремния 2) бромид калия 3) магний Вспомним, что ионная связь образуется между атомами элементов, наибо- лее сильно различающихся по электроотрицательности (например, между атомами щелочных или щелочноземельных элементов и кислородом или га- логенами). Такое вещество в списке есть — бромид калия (2). Кроме этого, ионная связь образуется между катионами металлов и анионами кислород- содержащих кислот. Такой пример в задании имеется, а именно нитрат ка- лия (5). Во всех остальных случаях (оксид кремния, магний и сероводород) ионной связи нет. Пример 7. Ковалентная полярная связь присутствует в молекулах 1) кислорода 4) водорода 2) ромбической серы 5) аммиака 3) воды Между атомами одного и того же химического элемента образуется кова- лентная неполярная связь (в нашем примере — кислород, сера и водород). Ковалентная полярная связь образуется между атомами с различной элек- троотрицательностью (если разница между значениями ЭО максимальна, то образуется ионная связь). В нашем примере ковалентная полярная связь реализуется в случае воды и аммиака. Ответ: 3 5 |<1 Пример 8. Ковалентная неполярная связь реализуется 1) в молекуле воды 4) в кристаллическом иоде 2) в кристалле SiO2 5) в молекуле сероводорода 3) в кристалле кремния Ковалентная неполярная связь образуется между атомами с одинаковой электроотрицательностью, т. е. между атомами одного и того же химическо-
29 инмнммммиммм неполярная кова- го элемента. В рассматриваемом примере простыми веществами являются кремний и иод — именно в этих веществах реализуется лентная связь. Ответ: 4 3 Na2 NH3 |f Пример 9. Ковалентная неполярная связь реализуется 1) в молекуле оксида углерода(1\/) 4) в молекуле 2) в молекуле S8 5) в молекуле 3) в кристаллическом льде Ковалентная неполярная связь образуется между атомами с одинаковой электроотрицательностью, т. е. между атомами одного и того же химическо- го элемента. В рассматриваемом примере неполярная ковалентная связь реализуется в молекулах S8 и Na2. Пусть вас не удивляет второй ответ. В кристаллическом натрии существует металлическая связь (как и в других твёрдых металлах и их сплавах). Однако в газовой фазе при высокой темпе- ратуре существуют двухатомные молекулы Na2, электронное строение кото- рых аналогично строению молекул Н2 или Cl2. Все эти молекулы объединяет наличие в них ковалентной неполярной связи. Ответ: 2 4 Пример 10. Химическая связь в молекуле водорода 1) ковалентная неполярная 4) двойная 2) ковалентная полярная 5) водородная 3) одинарная Между атомами водорода в молекуле Н2 действует ковалентная неполярная связь, как и в других аналогичных молекулах, образованных атомами с оди- наковой электроотрицательностью, т. е. атомами одного и того же химиче- ского элемента. Кратность химической связи в молекуле Н—Н равна 1 (оди- нарная связь). Ответ: 1 3 Пример 11. Химические связи в кристалле гидроксида бария 1) ковалентные неполярные 4) ионные 2) ковалентные полярные 5) водородные 3) металлические В гидроксиде бария Ва(ОН)2два типа химических связей. Во-первых, между катионом бария Ва2+ и гидроксид-ионом ОН действует ионная связь (веще- ство с ионной кристаллической решёткой). Кроме этого, в гидроксид-ионе между атомами кислорода и водорода действует ковалентная полярная связь: о—Н“ Ва2+ о—н Ответ:
30 ЗАНЯТИЕ 3 Классификация и номенклатура неорганических веществ. Классификация химических реакций Основные элементы содержания: классификация веществ и номенклатура неорганических веществ: вещества простые и слож- ные, оксиды, гидроксиды, кислоты, основания, соли, бинарные соединения; классификация химических реакций: реакции соединения, реакции разложения, реакции обмена, реакции за- мещения, каталитические реакции, реакции обратимые и практически необратимые, экзо- и эндотермические реакции. Основные классы неорганических веществ Для того чтобы легче ориентироваться в огромном многообразии неорганиче- ских веществ, их разделили на отдельные классы, включающие вещества, сходные по строению и свойствам. Все неорганические вещества можно разделить на простые и сложные. Простые вещества подразделяют на металлы и неметаллы. Сложные вещества, в свою очередь, подразделяют на несколько классов: окси- ды, гидроксиды, бескислородные кислоты, соли, бинарные соединения. На схе- ме 1 представлен один из возможных вариантов классификации неорганических веществ. Эта классификация, сложившаяся в XVIII—XIX вв., в настоящее время продолжает использоваться при первоначальном знакомстве с химией. Схема 1 Классификация неорганических веществ
31 Оксиды — сложные вещества, состоящие из двух элементов, один из которых кислород в степени окисления -2. Оксиды делят на две группы — солеобразую- щие и несолеобразующие (безразличные). Среди солеобразующих выделяют ос- нбвные, кислотные и амфотерные. Иногда среди оксидов выделяют группу двой- ных (солеобразных) оксидов. Сказанное поясняет схема 2. Схема 2 Классификация оксидов Названия оксидов состоят из слова «оксид» и названия элемента в родительном падеже с указанием в скобках степени окисления элемента римскими цифрами (если элемент проявляет постоянную степень окисления, то её можно не указывать): СаО — оксид кальция ЫО2— оксид азота(1\/) СиО — оксид меди(Н) N2O5— оксид азота(\/) Си2О — оксид меди(1) Иногда в литературе используют исторически сложившиеся, тривиальные назва- ния, например: Н2О — вода N2O — закись азота (веселящий газ) СО — угарный газ SiO2 — кремнезём СО2 — углекислый газ Fe3O4 — железная окалина SO2— сернистый газ Гидроксиды — дословно «гидратированные оксиды» (продукты взаимодействия оксидов с водой). Общая формула гидроксидов ЭОп(ОН)т, например АЮ(ОН) — метагидроксид алюминия, А1(ОН)3 — ортогидроксид алюминия. Гидроксиды металлов делят на оснбвные (основания) и амфотерные. (Кислород- содержащие кислоты иногда рассматривают как кислотные гидроксиды.) Основания с точки зрения теории электролитической диссоциации — слож- ные вещества, при диссоциации которых в качестве анионов образуются только гидроксид-ионы. По другой теории, основание — частица, являющаяся акцепто- ром протона. Сильные основания, образованные щелочными и щелочноземельны- ми металлами, называют щелочами. К основаниям относят гидроксиды щелочных и щелочноземельных металлов, а также гидроксиды магния, меди(Н), хрома(П), марганца(П) и железа(Н)2. 2 Строго говоря, оксиды и гидроксиды меди(П), марганца(П) и железа(П) проявляют незначительные амфотерные свойства. Однако в школьном курсе химии их принято относить к основным гидроксидам.
32 Амфотерными являются гидроксиды бериллия, цинка, алюминия, свинца(Н), олова(П) и олова(1\/), хрома(Ш), железа(Ш) и некоторые другие. Гидроксиды, об- разованные элементами-неметаллами и металлами в высоких степенях окисления (+5 и выше), являются кислотами. Так, к кислотам относятся гидроксиды хрома и марганца в высших степенях окисления: НМпО4, Н2СгО4, Н2Сг2О7. В соответствии с номенклатурными правилами при названии гидроксида после слова «гидроксид» следует указать элемент и его степень окисления римской циф- рой в скобках: КОН — гидроксид калия; Сг(ОН)2 — гидроксид хрома(П); Са(ОН)2— гидроксид кальция; Сг(ОН)3— гидроксид хрома(Ш). Кислоты — это сложные вещества, при электролитической диссоциации кото- рых образуются катионы водорода и анионы кислотных остатков. В соответствии с другим определением кислота — это частица, являющаяся донором протона. Кислоты делят на бескислородные и кислородсодержащие (последние можно рас- сматривать как гидроксиды, проявляющие кислотные свойства). Различают также кислоты сильные (серная, азотная, хлорная, хлорноватая, марганцовая) и слабые (азотистая, угольная, сероводородная и др.). Кислоты можно классифицировать и по числу атомов водорода в молекуле (одноосновные, двухосновные и т. д.). Названия кислот производятся от названия элементов, их образующих. Если эле- мент образует несколько кислородсодержащих кислот, это отражается в названии кислоты введением суффиксов -оватая, -истая, -оватистая (по мере уменьшения степени окисления элемента). В случае если одной степени окисления отвечает не- сколько кислот, то в название вводят префикс ди- (отражает наличие двух атомов кислотообразователя) или мета-, орто- (если кислоты различаются количеством свя- занной воды). В таблице 7 приведены названия изучаемых в средней школе кислот (в скобках приведены тривиальные названия). Таблица 7 Названия кислот и анионов Формула кислоты Название кислоты Название аниона HF Фтороводородная (плавиковая) Фторид HCI Хлороводородная (соляная) Хлорид нею Хлорноватистая Гипохлорит НСЮ2 Хлористая Хлорит НСЮ3 Хлорноватая Хлорат нсю4 Хлорная Перхлорат НВг Бромоводородная Бромид HI Иодоводородная Иодид НМпО4 Марганцовая Перманганат H2S Сероводородная Сульфид
33 Продолжение Формула кислоты Название кислоты Название аниона H2SO3 Сернистая Сульфит H2SO4 Серная Сульфат H2S2O7 Дисерная (пиросерная) Дисульфат (пиросульфат) H2CrO4 Хромовая Хромат H2Cr2O7 Дихромовая Дихромат hno2 Азотистая Нитрит HNO3 Азотная Нитрат H3PO3 Фосфористая Фосфит (HPO3)„ Метафосфорная Метафосфат H3PO4 Ортофосфорная, или фосфорная Ортофосфат, или фосфат h4p2o7 Дифосфорная (пирофосфорная) Дифосфат (пирофосфат) H2CO3 Угольная Карбонат CH3COOH Уксусная Ацетат H2SiO3 Метакремниевая (кремниевая) Метасиликат (силикат) H4SiO4 Ортокремниевая Ортосиликат H3BO3 Ортоборная Ортоборат Соли — сложные вещества, при диссоциации которых образуются катионы ме- талла (или аммония NH/) и анионы кислотных остатков. Соли можно рассматривать как продукты полного или частичного замещения атомов водорода в молекуле кислоты на металл или гидроксильных групп в моле- куле основания на кислотный остаток. Средние соли образуются при полном замещении атомов водорода в молекуле кислоты атомами металла или при полном замещении гидроксильных групп в фор- мульной единице основания кислотными остатками. Образование средних солей представлено следующими реакциями: 2NaOH + H2SO4= Na2SO4+ 2Н2О Ва(ОН)2+ H2SO4 = BaSOj + 2Н2О При неполной нейтрализации кислоты основанием образуются кислые соли, со- держащие в кислотном остатке атомы водорода: NaOH + H2SO4= NaHSO4+ Н2О Кислые соли образуют только многоосновные кислоты, содержащие более од- ного атома водорода в молекуле.
34 Оснбвные соли образуются в результате неполного замещения гидроксильных групп в молекуле основания на кислотные остатки: Mg(OH)2 + HCI = Мд(ОН)С1 + Н2О Существуют также двойные соли (образованы катионами двух металлов и одним кислотным остатком, например, KAI(SO4)2), смешанные соли (образованы одним металлом и двумя кислотными остатками, например, Ca(OCI)CI) и комплексные соли (содержат в своём составе комплексную частицу, например, K[AI(OH)J). Названия солей строятся из двух слов — названия аниона в именительном па- деже и названия аниона в родительном падеже, например CaF2 — фторид кальция. Для металлов с переменной степенью окисления её указывают в скобках после названия, например Cr(NO3)3 — нитрат хрома(1П). Названия кислых солей начинаются с префикса «гидро-» (указание на атомы во- дорода). Если атомов водорода два, то говорят «дигидро-». Например, NaHCO3 — гидрокарбонат натрия, Са(Н2РО4)2— дигидрофосфат кальция. Названия оснбвных солей начинаются с префикса «гидроксо-», например, Fe(OH)CI2 — гидроксохлорид железа(Ш). При необходимости назвать кристаллогидрат какой-либо соли к её названию следует добавить слово «гидрат», например, CuSO4 • 5Н2О имеет название «пен- тагидрат сульфата меди». При названии комплексных солей следует указывать лиганды (частицы, связан- ные с центральным атомом-комплексообразователем), их число, элемент-комплек- сообразователь, его степень окисления, например: Na[AI(OH)4] — тетрагидроксоалюминат натрия; K2[Zn(OH)4] — тетрагидроксоцинкат калия; K3[Fe(CN)6] — гексацианоферрат(Ш) калия. Двойные и смешанные соли называют следующим образом: KAI(SO4)2 • 12Н2О — кристаллогидрат сульфата калия-алюминия, или алюмокали- евые квасцы, Ca(CIO)CI — хлорид-гипохлорит кальция, или хлорная известь. Классификация химических реакций в неорганической химии В качестве классификационных признаков химических реакций могут быть вы- браны следующие: 1. Число и состав исходных веществ и продуктов реакции. 2. Изменение степеней окисления химических элементов в ходе реакции. 3. Знак теплового эффекта реакции. 4. Обратимость процесса и др. Рассмотрим более подробно каждый из типов реакций. 1. Классификация по числу и составу реагентов и конечных веществ Реакции соединения. В ходе реакций соединения из нескольких более про- стых по составу реагирующих веществ получается одно более сложное по составу вещество: A+B+C=D 2SO2+O2=2SO3 С2Н4 + Н2О кат- > С2Н5ОН
35 Как правило, эти реакции сопровождаются выделением тепла, т. е. приводят к образованию более устойчивых соединений. Реакции разложения. В ходе реакций разложения из одного сложного соеди- нения образуется два или более простых или сложных вещества: A=B+C+D СаСО3= СаО + СО2 4HNO3 = 2Н2О + 4NO2 + О2 ^18^38 * С9Н18 + С9Н20 Реакции замещения. Реакцией замещения называют взаимодействие простого вещества и сложного, в ходе которого атомы простого вещества замещают один из элементов, входящих в состав сложного вещества. Примеры таких реакций приведены ниже: Zn + 2HCI = ZnCI2+ Н2 8AI + 3Fe3O4 = 9Fe + 4AI2O3 2К + 2Н2О = 2КОН + Н2 Fe + CuSO4 = FeSO4 + Си В органической химии реакции замещения имеют свою специфику. Так, к реак- циям замещения относится хлорирование метана или нитрование бензола: СН4 + С12 свет > СН3С1 + HCI С6Н6 + HONO2 -> С6Н5—NO2 + Н2О Реакции обмена. Реакциями обмена принято называть реакции между двумя сложными веществами, в ходе которых молекулы сложных веществ обмениваются между собой своими составными частями: АВ + CD = AD + ВС В отличие от реакций замещения реакции обмена протекают без изменения сте- пеней окисления элементов. Это наиболее распространённая группа реакций между сложными веществами — оксидами, основаниями, кислотами и солями, например: AgNO3 + KBr = AgBr + KNO3 FeCI3 + ЗКОН = Fe(OH)3 + 3KCI Частным видом реакций ионного обмена является реакция нейтрализации, про- текающая между кислотой и основанием с образованием соли, например: Mg(OH)2 + H2SO4 = MgSO4 + 2Н2О NaOH + НВг = NaBr + Н2О 2. Классификация реакций по изменению степеней окисления химических элементов Химические реакции могут происходить без изменения степеней окисления эле- ментов, например: Na2SO4 + ВаС12 = BaSOj + 2NaCI СО2 + СаО = СаСО3 Степени окисления элементов в реакциях могут меняться: Мд + 2Н+ -> Мд2* + Н2Т
36 +2-1 +5 +3 +2 . +6 FeS2 + 8НМО3(конц.) = Fe(NO3)3 + 5NOT + 2H2SO4 + 2H2O Реакции, протекающие с изменением степеней окисления элементов, называют окислительно-восстановительными. 3. Классификация реакций по знаку теплового эффекта В зависимости от теплового эффекта химические реакции подразделяют на эк- зотермические и эндотермические. Экзотермическими называют реакции, проте- кающие с выделением энергии, например: 2Mg + О2 = 2МдО + Q Реакции, сопровождающиеся поглощением энергии, называют эндотермически- ми, например: СаСО3 = СаО + СО2 - О Термохимические уравнения включают в себя не только запись химической ре- акции, но и величину теплового эффекта, например: 2Наг,+ Оад = 2НгО,ж,+ 572 кДж А'гОз(тв)+ ЗЗО3(Ж) = AI2(SO4)3(tb) + 580 кДж Са(ОН)2(тв) = СаО(тв) + Н2О(Ж) - 108 кДж В термохимических уравнениях принято указывать агрегатное состояние ве- ществ, влияющее на величину теплового эффекта. 4. Обратимость процесса Химические реакции, которые в одних и тех же условиях могут протекать в пря- мом и обратном направлениях, называют обратимыми. Примерами обратимых хи- мических реакций могут служить синтез аммиака, диссоциация слабых электро- литов, взаимодействие углекислого газа с водой: N2 + 3H2*±2NH3 HNO2 Н+ + NO2 СО2+ Н2О^Н2СО3 К практически необратимым процессам относятся, например, реакции горения и некоторые другие: СН4+2О2-> СО2+2Н2О КСЮ4 = KCI + 2О2 Кроме рассмотренных признаков, химические реакции можно классифициро- вать: — по фазовым признакам (гомогенные и гетерогенные); — по необходимости использовать катализатор (каталитические и некаталитиче- ские) и по ряду других признаков.
37 Примеры заданий Пример 1. Выберите формулы трёх амфотерных оксидов и запишите со- ответствующие им номера в поле ответа: 1) ВеО 2) SeO3 3) Сг2О3 4) Rb2O 5) СО 6) А12О3 Охарактеризуем каждый из приведённых в задании оксидов. ВеО — в отличие от других элементов IIA-группы бериллий образует амфо- терный оксид; SeO3— оксид селена(\/1) является кислотным оксидом, ему соответствует ги- дроксид с кислотными свойствами — селеновая кислота; Сг2О3 — оксид хрома(Ш) является амфотерным оксидом, в то время как ок- сид хрома(П) — оснбвным, а оксид хрома(\/1) — кислотным; Rb2O — оксиды всех щелочных металлов являются основными, им отвечают сильные основания (щёлочи), в частности RbOH; СО — оксид углерода(П), или угарный газ, относится к несолеобразующим (безразличным) оксидам; А12О3 — оксид алюминия — один из наиболее распространённых амфотер- ных оксидов. 1 3 6 Пример 2. Выберите формулы трёх основных оксидов и запишите соот- ветствующие им номера в поле ответа. 1) ВаО 2) SO2 3) Сг2О3 4) Cs2O 5) МпО2 6) RaO К оснбвным оксидам относят оксиды металлов в невысоких степенях окис- ления (+1, +2). Среди приведённых в задании такими оксидами являются ВаО, Cs2O и RaO. Оставшиеся оксиды относятся или к кислотным (SO2), или к амфотерным (Сг2О3 и МпО2). Ответ: Пример 3. Установите соответствие между формулой вещества и клас- сом, к которому это вещество принадлежит: к каждой позиции, данной в первом столбце, подберите соответствующую позицию из второго столбца. ФОРМУЛА ВЕЩЕСТВА КЛАСС A) FeO Б) МпО2 В) РЬО 1) оксиды основные 2) оксиды кислотные 3) оксиды несолеобразующие 4) оксиды амфотерные
Оксид железа(Н) относится к основным оксидам, ему отвечает основание Fe(OH)2. Характер оксида железа(П) подтверждается лёгкостью его взаимо- действия с кислотами и отсутствием взаимодействия с щелочами. Оксид марганца(1У) и оксид свинца(И) являются амфотерными — они взаи- модействуют как с кислотами, так и с щелочами или основными оксидами, например: MnO2 + 4HF = MnF4 + 2Н2О МпО, + СаО = СаМпО, Ответ Пример 4. Установите соответствие между формулой вещества и клас- сом, к которому это вещество принадлежит. ФОРМУЛА ВЕЩЕСТВА КЛАСС A) NH4HSO4 1) соли средние Б) КСЮ4 2) оксиды кислотные В) N2O 3) оксиды несолеобразующие 4) соли кислые Запишите в таблицу выбранные цифры под соответствующими буквами. Гидросульфат аммония относится к кислым солям, в то время как хлорат калия (бертолетова соль) — к средним. Средние соли являются продуктом полной нейтрализации кислоты основанием. Кислые соли получаются в ре- зультате неполной нейтрализации кислоты основанием, поэтому в их составе присутствуют атомы водорода из исходной кислоты (в формуле такой атом водорода выделен: NH4HSO4). Закись азота (оксид азота(1)) относится к не- солеобразующим оксидам — он не взаимодействует с растворами кислот и щелочей с образованием соли. Ответ: А Б в 4 1 3 Пример 5. Выберите формулы трёх веществ, относящихся к классу кис- лот, и запишите соответствующие им номера в поле ответа: 1) Н3СОН 2) H2[SiF6] 3) AI(OH)3 4) H3N 5) HF 6) С2Н5СООН Существует несколько определений кислот. В соответствии с одним из них кислоты — это электролиты, в ходе диссоциации которых в качестве кати- онов образуются только ионы Н+. В соответствии с другим определением кислоты являются донорами протонов (Н+). Используя любое из опреде- лений, мы можем отнести к классу кислот гексафторокремниевую кислоту H2[SiF6], фтороводородную (плавиковую) кислоту HF и пропионовую кислоту С2Н5СООН. Первое вещество, приведённое в задании, относится к спиртам
39 (неэлектролитам), третье вещество является гидроксидом алюминия А1(ОН)3, четвёртое вещество — аммиак, проявляющий свойства основания. Ответ: 2 5 6 Пример 6. Установите соответствие между формулой вещества и клас- сом, к которому это вещество принадлежит: к каждой позиции, данной в первом столбце, подберите соответствующую позицию из второго столбца. ФОРМУЛА ВЕЩЕСТВА A) HCOONa Б) N2O5 В) N2O КЛАСС 1) соли средние 2) оксиды кислотные 3) оксиды несолеобразующие 4) соли кислые Запишите в таблицу выбранные цифры под соответствующими буквами. Первое соединение образовано катионом металла Na+ и анионом кислот- ного остатка НСОО и, следовательно, является солью (формиат натрия). Это средняя соль. Два других вещества, состоящие из двух элементов, один из которых кислород в степени окисления -2, являются оксидами. При этом N2O5 относится к кислотным оксидам (ему отвечает азотная кислота), a N2O — к несолеобразующим. Ответ: Пример 7. Реакция, уравнение которой 2AI + Fe2O3 = 2Fe + AI2O3, относится к реакциям: 1) разложения 2) соединения 3) замещения 4) обмена 5) окислительно-восстановительным Проанализируем предложенное уравнение реакции. Алюминий, взаимодей- ствуя с оксидом железа, замещает атомы железа в этом соединении. В ре- зультате образуется железо и оксид алюминия. Отсюда правильный ответ 3. Кроме этого, атомы алюминия и железа изменяют степени окисления: О 3+ AI - Зё = AI Fe + Зе = Fe Следовательно, это окислительно-восстановительная реакция (ответ 5). Ответ:
40 Пример 8. К реакциям ионного обмена относятся реакции, уравнения ко- торых 1) Na2O + CO2=Na2CO3 2) NaCI + AgNO3 = NaNO3 + AgCI 3) Zn + H2SO4 = ZnS04 + H2 4) 2Na + 2H2O = 2NaOH + H2 5) KOH + HNO3 = KNO3 + H2O Реакции ионного обмена протекают в водных растворах между электроли- тами. В первом, третьем и четвёртом уравнениях участвуют вещества-не- электролиты (углекислый газ, цинк и натрий). Среди приведённых в задании уравнений к реакциям обмена относится взаимодействие хлорида натрия и нитрата серебра, а также взаимодействие гидроксида калия и азотной кис- лоты. Эти реакции протекают практически до конца (напомним, что практи- чески полностью протекают реакции, в ходе которых выделяется газ, обра- зуется осадок или слабый электролит): Ад+ + СГ = AgCI ф Н+ + ОН' = Н2О Ответ: Пример 9. Взаимодействие раствора гидроксида кальция и соляной кис- лоты относится к реакциям 1) соединения 2) обмена 3) замещения 4) гомогенным 5) окислительно-восстановительным Запишем уравнение реакции: Са(ОН)2+ 2HCI = СаС12+ 2НгО Прежде всего это реакция, в которой участвуют электролиты (реакция об- мена, вещества обмениваются своими ионами). Реакции обмена происходят без изменения степени окисления атомов (т. е. это не окислительно-восста- новительная реакция). Кроме того, эта реакция осуществляется в растворе: и реагенты, и продукты реакции представляют собой однородную, т. е. го- могенную, систему (отсутствует поверхность раздела между отдельными веществами). Ответ: 2 4
и« Пример 10. Взаимодействие серы и кислорода относится к реакциям 4) гомогенным 5) окислительно-восстановительным 2) 3) соединения обмена замещения Запишем уравнение происходящей реакции: S(TB.) + ®2(г) — ^^2(г) В ходе этой реакции происходит соединение атомов серы и кислорода, т. е. из двух простых веществ образуется одно сложное вещество. Следователь- но, записанное уравнение относится к реакциям соединения. Так как атомы кислорода и серы меняют степени окисления, то эта реакция относится к окислительно-восстановительным. Ответ: ЗАНЯТИЕ 4 Скорость реакции, её зависимость от различных факторов. Обратимые и необратимые химические реакции. Химическое равновесие Основные элементы содержания: скорость химической реакции, гомогенные и гетерогенные реакции, химическое равновесие, смещение равновесия, катализатор, ингибитор. моль л • с Скорость химической реакции есть число элементарных актов химической ре- акции, происходящих в единицу времени в единице объёма (для гомогенных ре- акций) или на единице поверхности (для гетерогенных реакций). В соответствии с более простым определением скорость гомогенной химической реакции есть из- менение концентрации реагирующих веществ в единицу времени: |дс| v= ду1. размерность Рассмотрим влияние различных факторов на скорость реакции (схема 3). Природа реагирующих веществ. Природа вещества определяется строени- ем атомов, молекул и ионов, его составляющих. Она зависит от вида химических связей, типа кристаллической решётки, взаимного влияния атомов в веществе и определяет химическую активность реагентов. Так, скорость взаимодействия соля- ной кислоты с магнием выше, чем с железом (магний — более активный металл). Аналогично, цинк более активно реагирует с серной кислотой по сравнению с сер- нистой (серная кислота — более сильная). 41
42 Схема 3 Факторы, влияющие на скорость химической реакции Концентрация реагентов. Чем больше концентрация реагентов, тем больше молекул в единице объёма и тем больше вероятность их столкновения. Следо- вательно, увеличение концентрации реагентов приводит к увеличению скоро- сти реакции (закон действующих масс Гульдберга и Вааге). Так, аммиак горит в чистом кислороде, но не горит на воздухе, в котором объёмная доля кисло- рода всего 21 %. Цинк наиболее быстро реагирует с 20%-ной серной кислотой, а не с 5%-ной (чем больше концентрация кислоты, тем выше скорость реак- ции). В случае реакций, протекающих в газовой фазе, на скорость влияет давление в системе. Чем выше давление, тем больше скорость реакции. В случае гетерогенных процессов скорость зависит от площади соприкосно- вения реагентов: чем сильнее измельчены вещества, чем больше площадь их со- прикосновения, тем больше скорость реакции. Так, стружка металлического цин- ка вполне спокойно реагирует с соляной кислотой. Если взять «цинковую пыль» (мельчайший цинковый порошок), скорость реакции столь сильно возрастает, что содержимое буквально выбрасывает из пробирки. Температура. Чем выше температура, тем больше скорость химических реак- ций. При нагревании на каждые 10 °C скорость большинства химических реакций возрастает в среднем в 2—4 раза (правило Вант-Гоффа). Это связано с увеличе- нием при нагревании доли так называемых «активных молекул», т. е. таких моле- кул, столкновение которых приводит к образованию продуктов реакции. Наличие катализатора или ингибитора. Катализатор — вещество, ускоряющее химическую реакцию, но остающееся неизменным после окончания реакции. Инги- битор — вещество, замедляющее реакцию. Ошибочно считать, что катализатор не участвует в процессе. Из следующего примера видно, что действие катализатора основано на его непосредственном участии в реакции (на первой стадии NO всту- пает в реакцию, на последней стадии регенерируется). Суммарное уравнение каталитического процесса: 2SO, + О,N° 2SO3 Стадии процесса: 2NO + О2 = 2NO2 SO2 + NO2 = SO3 + NO Химические реакции, которые протекают только в одном направлении и завер- шаются полным превращением реагирующих веществ, называют необратимыми.
43 Примером такой реакции может служить разложение хлората калия (бертолетовой соли) при нагревании или горение метана: 2KCIO3 = 2KCI + ЗО2Т СН4+2О2-> СО2+2Н2О Большинство реакций являются обратимыми. Обратимыми называют реакции, которые могут протекать в двух взаимно противоположных направлениях. Приме- ром такой реакции может служить синтез аммиака из водорода и азота или раз- ложение карбоната кальция: N2+3H2t±2NH3 СаСО3 ** Са0 + С02 В системе, в которой осуществляется обратимый процесс, может установиться химическое равновесие. Под химическим равновесием подразумевают такое со- стояние системы, при котором скорость прямой реакции равна скорости обратной. Химическое равновесие динамическое, его можно смещать как в одну, так и в дру- гую сторону. Если на систему, находящуюся в состоянии химического равновесия, оказывать внешнее воздействие, то равновесие смещается в сторону, ослабляю- щую это воздействие (принцип Ле Шателье). Так, увеличение концентрации ка- кого-либо вещества смещает равновесие в сторону его расходования. Увеличение давления приводит к смещению равновесия в сторону уменьшения объёма. По- вышение температуры смещает равновесие в сторону эндотермической реакции. Катализатор, ускоряя как прямую, так и обратную реакции, не влияет на состояние равновесия (схема 4). Схема 4
44 3 Ответ: с и и H2SO4 (20%-ный р-р) H2SO4 (96%-ный р-р) наибольшей скоростью проте- Примеры заданий Пример 1. При комнатной температуре кает реакция между 1) Fe и Н28О4(1%-ный р-р) 3) Fe 2) Fe и H2SO4 (10%-ный р-р) 4) Fe В задании рассматривается взаимодействие железа с серной кислотой раз- ной концентрации. Как известно, скорость реакции зависит от концентрации реагента — чем больше концентрация, тем больше скорость реакции. Сле- довательно, скорость реакции железа с 10%-ной серной кислотой должна быть выше, чем с 1%-ным раствором её. Аналогично, скорость реакции с участием 20%-ного раствора кислоты будет больше, чем с участием 10%- ного раствора. А вот как быть дальше? Что будет происходить, если кон- центрация кислоты повысится до 96 %? Вы, конечно же, знаете, что концен- трированные серная и азотная кислоты на железо при обычных условиях не действуют. Это явление называется пассивацией металла. Итак, с наиболь- шей скоростью при комнатной температуре железо будет взаимодействовать не с 96%-ной, а с 20%-ной серной кислотой. Пример 2. Для увеличения скорости химической реакции ^еО(тв)+ СО(Г) ~ Fe(TB) + СО2(Г) +17 кДж 4) увеличить степень измельчения FeO 5) увеличить давление необходимо 1) увеличить концентрацию СО2 2) уменьшить концентрацию СО2 3) уменьшить температуру В задании фигурирует гетерогенная реакция, протекающая с участием твёрдо- го и газообразного веществ. Скорость подобных реакций зависит от темпера- туры, концентрации газа, давления в системе и степени измельчения твёрдого вещества (площади соприкосновения реагентов). С учётом этого легко найти правильный ответ: для увеличения скорости указанной реакции надо увели- чить степень измельчения реагента (FeO) и увеличить давление в системе. 5 4 Ответ: Пример 3. Для увеличения скорости химической реакции Zn(TB) + 2Н+(р.р) = Zn2+(p.p) + Н2(г) +154 кДж температуру концентрацию ионов цинка концентрацию кислоты необходимо уменьшить концентрацию ионов цинка добавить ингибитор увеличить увеличить увеличить 2) 3) 4) 5)
45 В задании фигурирует гетерогенная реакция, протекающая с участием твёр- дого вещества (цинка) и раствора кислоты (ионов Н+). Скорость подобных реакций зависит от температуры, концентрации реагента в растворе (в дан- ном случае — кислоты) и степени измельчения твёрдого вещества (в данном случае — цинка). С учётом этого легко найти правильный ответ: для уве- личения скорости указанной реакции надо увеличить температуру (скорость почти всех реакций возрастает с ростом температуры) и увеличить концен- трацию кислоты в растворе. Ответ: 3 5 Пример 4. Для смещения химического равновесия в системе 2SO2(r) + О2(г) 2SO3(r) + О в сторону исходных веществ необходимо 1) увеличить давление 4) понизить температуру 2) понизить давление 5) ввести катализатор 3) повысить температуру Рассмотрим влияние каждого из факторов на смещение химического равно- весия в системе. 1. Увеличение давления приведёт к смещению равновесия в сторону умень- шения объёма, т. е. в сторону образования продуктов реакции. 2. Понижение давления приведёт к смещению равновесия в сторону увели- чения объёма, т. е. в сторону образования исходных веществ. 3. Повышение температуры сместит равновесие в сторону протекания эндо- термической реакции, т. е. в сторону образования исходных веществ. 4. Понижение температуры сместит равновесие в сторону протекания экзо- термической реакции, т. е. в сторону образования продуктов реакции. 5. Катализатор не влияет на состояние равновесия. Подчёркнуты нужные факторы. 2 3 Пример 5. Установите соответствие между способом воздействия на рав- новесную систему ^Ог(г)+ ^'г(г) ** SO2CI2 + Q и направлением смещения химического равновесия в результате этого воз действия. ВОЗДЕЙСТВИЕ НА СИСТЕМУ А) уменьшение давления Б) добавление катализатора В) уменьшение температуры Г) уменьшение концентрации хлора НАПРАВЛЕНИЕ СМЕЩЕНИЯ ХИМИЧЕСКОГО РАВНОВЕСИЯ 1) в сторону продуктов реакции 2) в сторону исходных веществ 3) практически не смещается
46 Проанализируем процесс, обсуждаемый в условии задания. Взаимодействие сернистого газа с хлором — экзотермическая гомогенная реакция, протека- ющая в газовой фазе с уменьшением объёма (слева 2 моль газов, справа только один моль). Обсудим теперь действие каждого фактора. Уменьшение давления приведёт к смещению равновесия в сторону увеличения объёма, т. е. влево (2). Добавление катализатора, как известно, не влияет на состоя- ние равновесия (3). Уменьшение температуры сместит равновесие в сторо- ну экзотермического процесса (1). Уменьшение концентрации хлора вызовет смещение равновесия в сторону его образования, т. е. влево (2). Ответ: ЗАНЯТИЕ 5 Электролитическая диссоциация. Сильные и слабые электролиты. Реакции ионного обмена Основные элементы содержания: электролиты, электролитическая диссоциация, ионы, катион, анион, степень диссоциации, электролиты сильные и слабые, ионные уравнения реакций, практически осуществимые реак- ции ионного обмена. Электролиты — вещества, водные растворы или расплавы которых проводят электрический ток. Неэлектролиты — вещества, водные растворы которых не проводят электрический ток. Типичными электролитами являются кислоты, соли и основания. Большинство органических веществ электролитами не являются. Так, не отно- сятся к электролитам углеводороды, спирты, альдегиды, углеводы и др. Электропроводность электролитов обусловлена присутствием в их растворах (или расплавах) ионов, которые обеспечивают прохождение электрического тока. Ионы образуются в результате электролитической диссоциации. Электролитиче- ская диссоциация — процесс распада электролита на ионы при его растворении или плавлении. Движущими силами процесса электролитической диссоциации в водном раство- ре являются высокие значения энергии гидратации ионов и высокое значение ди- электрической константы воды (ен0= 81, т. е. вода в 81 раз ослабляет притяжение ионов друг к другу). Сильные электролиты в разбавленных водных растворах практически полно- стью диссоциируют на ионы (рис. 10). К сильным электролитам относятся все щё- лочи, многие кислоты (такие, как HCIO4, НСЮ3, HCI, HNO3, H2SO4, НМпО4 и некото- рые другие), а также практически все соли, растворимые в воде. Сильные электролиты диссоциируют практически полностью, например: HCI = Н+ + СГ hno3= н+ + no;
47 Рис. 10. Образование гидратированных ионов в процессе электролитической диссоциации K2SO4 = 2Ю + SOf Ва(ОН)2 = Ва2++2ОН Слабые электролиты лишь в малой степени диссоциируют на ионы. К слабым электролитам относятся вода, органические кислоты (уксусная, щавелевая, лимон- ная и т. д.), многие неорганические кислоты (сернистая, сероводородная, плавико- вая, азотистая, хлорноватистая и др.). В отличие от сильных электролитов, диссо- циация слабых электролитов обратима, что отражают в уравнениях диссоциации: HNO2 s=t H++NO2 HF H* + F" NH3H2O^NH4+ + OH’ Количественной характеристикой силы электролита могут служить значения сте- пени диссоциации. Степень диссоциации а равна отношению числа молекул, про- диссоциировавших на ионы, к исходному числу растворённых молекул: а = ^!“.100% Степень диссоциации сильных электролитов в разбавленных водных растворах практически равна 100%. Степень диссоциации слабых электролитов в разбавлен- ных водных растворах не превышает, как правило, 10—20 %, например: Слабый электролит а, % (С = 0,1М) H2SO3 20 HF 8 NH3 • Н2О 1,4 СН3СООН 1,4 Н2СО3 0,2 H2S 0,07
48 Степень диссоциации зависит от природы и концентрации электролита, а также от температуры раствора. Так, а растёт при нагревании (поскольку диссоциация — процесс эндотермический и нагревание смещает равновесие вправо, в сторо- ну продуктов диссоциации). При уменьшении концентрации вещества в растворе (т. е. при разбавлении) степень диссоциации также увеличивается. Многоосновные кислоты и основания диссоциируют ступенчато, по стадиям: H2St±H++HS’ HS- Н+ + S2- При этом степень диссоциации по каждой последующей ступени намного мень- ше, чем по предыдущей, например: Н3РО4 Н+ + Н2РО4 (а = 23,5 %) Н2РО4 Н+ + HPOf (а = 3 • 10-4 %) HPOf Н+ + POf (а = 2 • 10-9 %) Причина этого состоит в том, что в соответствии с принципом Ле Шателье про- исходит смещение химического равновесия в каждой последующей стадии диссо- циации под действием катионов водорода, образовавшихся в результате диссо- циации на предыдущей стадии. Подчеркнём ещё раз, что все растворимые соли относятся к сильным электро- литам, полностью распадаясь в водных растворах на ионы: К2СО3 = 2К+ + COf AI(NO3)3= AI3++3NO3 Однако кислые и основные соли диссоциируют ступенчато, например: NaHSO3= Na++ HSO3 (первая ступень) HSO3 <=* Н+ + SOf (вторая ступень) (ZnOH)2SO4= 2ZnOH+ + SOf (первая ступень) ZnOH+ Zn2t + ОН- (вторая ступень) Диссоциация по первой стадии протекает полностью (сильный электролит, как и все соли), по второй стадии — обратимо (слабый электролит, как и все много- основные кислоты или основания). Так как электролиты в воде диссоциируют на ионы, то реакции между электро- литами в растворе следует рассматривать как процесс взаимодействия соответ- ствующих ионов. Ионные уравнения реакций отражают суть тех изменений, ко- торые происходят при взаимодействии веществ-электролитов. При написании уравнений реакций в ионном виде следует помнить, что хорошо растворимые сильные электролиты записывают как набор ионов, образовавшихся при их полной диссоциации, в то время как неэлектролиты и слабые электролиты (а также газы и осадки) записывают в молекулярном виде. Алгоритм составления ионного уравнения реакции 1. Записать молекулярное уравнение реакции и расставить коэффициенты. 2. Переписать это уравнение, записывая хорошо растворимые сильные электро- литы в виде соответствующих ионов, а слабые электролиты, нерастворимые веще- ства и газы — в молекулярном виде. 3. Сократить одинаковые ионы в левой и правой частях уравнения и записать сокращённое ионное уравнение.
Пример составления уравнения реакции в ионном виде: 1) Cu(OH)2+ 2HNO3 = Cu(NO3)2+ 2Н2О 2) Cu(OH)2 + 2Н+ + 2NO3 = Cu2+ + 2NO3‘ + 2H2O 3) Cu(OH)2+2H+ = Cu2++2H2O Важно помнить, что реакции в растворах электролитов идут практически до кон- ца в том случае, если происходит связывание исходных ионов с образованием: — слабого электролита; — осадка практически нерастворимого вещества; — газообразного продукта. Примеры заданий Пример 1. К электролитам относятся 1) С2Н5ОН 2) С^Н-О.. 3) Са(ОН)2 4) СО 5) НСООН Напомним, что электролитами являются вещества, принадлежащие к кисло- там, основаниям и солям. С этих позиций проанализируем вещества, ука- занные в задании. Этиловый спирт (1), сахароза (2) и угарный газ (4) не относятся к электролитам. И только гидроксид кальция (3) и муравьиная кислота (5) — типичные электролиты. Их водные растворы проводят элек- трический ток. Ответ: 5 3 Пример 2. Сильными электролитами являются 1) угольная кислота 4) азотная кислота 2) сероводородная кислота 5) плавиковая кислота 3) гидроксид бария Сильные электролиты — это электролиты, которые в разбавленных водных растворах полностью диссоциируют на ионы. Среди приведённых в зада- нии веществ сильными электролитами являются гидроксид бария (щёлочь) и азотная кислота. Все остальные вещества лишь в малой степени диссо- циируют в растворе на ионы. Ответ: 3 4 Пример 3. В водном растворе практически полностью взаимодействуют вещества 1) ВаС12 и КОН 4) СаС12 и К3РО4 2) HCI и CsOH 5) NaCI и H2SO4 3) К3РО4 и NaNO3 Реакции в растворах электролитов — это реакции между ионами. Возможны только те процессы, в ходе которых происходит прочное связывание ионов друг с другом. Это имеет место, если реакция сопровождается выпадением осадка, выделением газа, образованием слабого электролита. С этих пози- ций становится ясно, что практически осуществимы только вторая и четвёр- тая реакции:
50 2) HCI + CsOH = CsCI + H2O h+ + oh-=h2o 4) 3CaCI2 + 2K,PO4 = Ca,(PO4U + 6KCI 3Ca2+ + 2PQ3- = Ca3(PO4)24- В ходе реакции 2 образуется слабый электролит (вода), в ходе реакции 4 выпадает осадок. 2 4 Пример 4. Верны ли следующие суждения о диссоциации фосфорной кислоты? А. Фосфорная кислота диссоциирует ступенчато. Б. В растворе фосфорной кислоты присутствуют только ионы Н+ и РО£". 1) верно только А 3) верны оба суждения 2) верно только Б 4) оба суждения неверны Фосфорная кислота Н3РО4 относится к слабым многоосновным кислотам. Как и все слабые кислоты, фосфорная кислота диссоциирует обратимо. Как и все многоосновные кислоты, фосфорная кислота диссоциирует ступенчато: Н3РО4 Н* + н2ро; н2ро4-«*н++нро42- НРО2- Н+ + POf Таким образом, первое суждение верное. А вот второе суждение ошибоч- но — в растворе фосфорной кислоты, кроме указанных в условии ионов, присутствуют ионы Н2РО4 и НРО2-, а также молекулы Н3РО4. Ответ: 1 Пример 5. Краткое ионное уравнение н+ + он-=н2о отвечает взаимодействию веществ 1) СН3СООН и КОН 4) HCI и Fe(OH)2 2) СН3СООН и Fe(OH)2 5) Ва(ОН)2и HNO3 3) HCI и КОН Будем исходить из того, что символ «Н+» в ионном уравнении реакции обо- значает не любую кислоту, а только сильную. Точно так же, как символ «ОН-» обозначает только сильное основание (щёлочь). С учётом сказанного легко найти верные ответы: 3 и 5. Именно в этих реакциях сильная кислота взаи- модействует со щёлочью. Ответ: 3 5
51 Пример 6. Даны вещества: дихромат калия, гидрокарбонат кальция, азотная кислота, оксид углерода(П), хлорид серебра. Из предложенного перечня веществ выберите вещества, между которыми возможна реакция ионного обмена. Запишите молекулярное, полное ионное и сокращённое ионное уравнения этой реакции. Реакции ионного обмена протекают в растворах с участием кислот, солей и оснований. Они не являются окислительно-восстановительными. Обязательным условием протекания таких реакций является взаимодействие ионов, которое сопровождается образованием осадка нерастворимого вещества, выделением газа (СО2, SO2, H2S, NH3) или образованием слабого электролита (слабой кислоты, гидрата аммиака, воды). Среди представленных веществ в реакцию ионного обмена могут вступать гидрокарбонат кальция и азотная кислота. Эта реакция сопровождается выделением газа: Са(НСО3)2 + 2HNO3 = Ca(NO3)2 + 2Н2О + 2СО2Т Са2+ + 2НСО3 + 2Н+ + 2NO3 = Са2* + 2NO3 + 2Н2О + 2СО2Т НСО3+Н+=Н2О + СО2Т' ЗАНЯТИЕ 6 Гидролиз Основные элементы содержания: гидролиз, гидролиз солей, типы гидролиза, полный гидролиз, совместный гидролиз. В общем случае гидролизом называют реакцию обмена между веществом и водой. Многие вещества, например бинарные соединения (карбиды, гидриды, ни- триды, ковалентные галогениды и др.) полностью разрушаются при взаимодей- ствии с водой: СаС2 + 2Н2О = Са(ОН)2 + С2Н2Т РВг3 + ЗН2О = Н3РО3 + ЗНВг РС15 + 4Н2О = Н3РО4 + 5HCI NaH + Н2О = NaOH + Н2Т Подобные реакции протекают, как правило, необратимо и в одну стадию. Соли также могут гидролизоваться. Однако большинство солей реагируют с во- дой обратимо, т. е. полного разложения соли не происходит. Гидролизом соли называют взаимодействие ионов соли с водой, приводящее к образованию слабого электролита и изменению среды раствора. Соли, образованные сильной кислотой и сильным основанием, например хло- рид натрия, не подвергаются гидролизу, поскольку ионы соли с молекулами воды не реагируют (не образуют слабых электролитов). Среда растворов таких солей нейтральная.
52 Соли, образованные слабой кислотой и сильным основанием, гидролизуются по аниону. Так, в растворе нитрита натрия NaNO2 нитрит-ионы NO2 обратимо взаимо- действуют с молекулами воды, образуя слабый электролит — азотистую кислоту: h2o + no2-t±hno2+oh- В результате накопления в растворе гидроксид-ионов среда раствора становит- ся щелочной. Взаимодействие с водой многозарядных анионов протекает ступен- чато, например: S2- + Н2О HS’ + ОН- HS- + Н2О H2S + он Гидролиз подобных солей протекает главным образом по первой стадии. Соли, образованные сильной кислотой и слабым основанием, гидролизуются по катиону. Например, гидролиз хлорида аммония NH4CI описывается следующим уравнением: NH;+ Н2О*± NH,- Н2О + Н* 4 2 3 2 В результате гидролиза в растворе накапливаются ионы Н+, поэтому среда ста- новится кислотной. Взаимодействие с водой многозарядных катионов протекает ступенчато, напри- мер: Си2* + Н2О СиОН* + Н* СиОН* + Н2О Си(ОН)2 + Н* Гидролиз подобных солей протекает главным образом по первой стадии. Гидролиз солей, образованных слабой кислотой и слабым основанием (напри- мер, ацетата аммония CH3COONH4), идёт и по катиону, и по аниону: NH; + СН,СОСГ + Н2О NH3 • Н2О + СН3СООН ДО d ос о Гидролиз подобных солей протекает довольно глубоко (так как образуются два слабых электролита). Реакция среды в растворах таких солей определяется взаим- ной силой образующихся кислоты и основания. Следует отметить, что иногда гидро- лиз подобных солей протекает практически необратимо, т. е. до конца, например: AI2S3 + 6Н2О = 2А1(ОН)34, + 3H2S? В этом случае оба продукта гидролиза уходят из сферы реакции, делая процесс гидролиза практически необратимым. По этой причине не существуют такие соли, как карбонаты трёхвалентных алюминия, железа, хрома. При сливании водных рас- творов, содержащих, например, ионы алюминия и сульфид-ионы, не происходит осаждения сульфида алюминия. В результате совместного (или взаимоусиливаю- щегося) гидролиза сразу же выпадает осадок гидроксида алюминия и выделяется сероводород: 2AI3* + 3S2- + 6Н2О = 2AI(OH)3i + 3H2ST Степень гидролиза, так же как и степень электролитической диссоциации элек- тролита, увеличивается с ростом температуры и с увеличением степени разбавле- ния раствора соли.
53 Примеры заданий Пример 1. Установите соответствие между формулой соли и отношением её к гидролизу. ФОРМУЛА СОЛИ ОТНОШЕНИЕ К ГИДРОЛИЗУ А) KNO3 1) гидролизуется по катиону Б) Na2S 2) гидролизуется по аниону В) AICI3 3) гидролизуется по катиону и аниону Г) LiNO2 4) не гидролизуется Рассмотрим отношение к гидролизу каждой из четырёх предложенных со- лей. Соль KNO3 образована сильной кислотой HNO3 и сильным основанием КОН. Соли такого типа гидролизу не подвергаются. Соль Na2S образована слабой кислотой H2S и сильным основанием NaOH. Гидролиз таких солей протекает по аниону. Соль AICI3 образована сильной кислотой HCI и слабым основанием А1(ОН)3. Такие соли гидролизуются по катиону. Соль LiNO2 образована слабой кислотой HNO2 и сильным основанием LiOH. Гидролиз протекает по аниону. Ответ А Б В Г 4 2 1 2 Пример 2. Установите соответствие между названием соли и отношени- ем её к гидролизу. НАЗВАНИЕ СОЛИ А) пропионат аммония Б) сульфид цезия В) сульфид алюминия Г) карбонат натрия ОТНОШЕНИЕ К ГИДРОЛИЗУ 1) не гидролизуется 2) гидролизуется по катиону 3) гидролизуется по аниону 4) гидролизуется по катиону и аниону Алгоритм выполнения этого задания такой же, как и в предыдущем случае. Пропионат аммония CH3CH2COONH4 образован слабой органической пропи- оновой кислотой и слабым основанием NH3 H2O. Следовательно, гидролиз этой соли протекает и по катиону, и по аниону. Сульфид цезия образован слабой сероводородной кислотой и сильным ос- нованием CsOH. Гидролиз этой соли протекает по аниону.
54 Сульфид алюминия образован и слабой кислотой, и слабым основанием. Ги- дролиз этой соли протекает и по катиону, и по аниону. Карбонат натрия образован слабой угольной кислотой и сильным основани- ем NaOH. Гидролиз этой соли протекает по аниону. Ответ: А Б в Г 4 3 4 3 Пример 3. Установите соответствие между формулой соли и реакцией среды её водного раствора. РЕАКЦИЯ СРЕДЫ ФОРМУЛА СОЛИ A) NH4NO3 Б) ZnSO4 1) щелочная 2) кислотная В) CH3COONa 3) нейтральная Г) NaBr Рассмотрим отношение к гидролизу каждой из солей. Нитрат аммония NH4NO3 образован сильной кислотой HNO3 и слабым осно- ванием NH3 • Н2О. Гидролиз этой соли протекает по катиону, а среда раство- ра становится кислотной. Сульфат цинка ZnSO4 образован сильной серной кислотой и гидроксидом цинка — слабым основанием. Гидролиз этой соли, как и в предыдущем слу- чае, протекает по катиону, среда раствора становится кислотной. Ацетат натрия CH3COONa образован слабой органической уксусной кислотой и сильным основанием NaOH. Следовательно, гидролиз этой соли протекает по аниону, реакция среды становится щелочной. Бромид натрия NaBr образован и сильной кислотой, и сильным основанием. Значит, эта соль гидролизу не подвергается, среда раствора остаётся ней- тральной.
55 ЗАНЯТИЕ 7 Окислительно-восстановительные реакции Основные элементы содержания: процессы окисления и восстановления; окислитель, восстановитель, окислительно-восстанови- тельные реакции. Основные положения теории окислительно-восстановительных процессов Окислительно-восстановительные реакции — такие реакции, в которых про- исходит изменение степеней окисления атомов, входящих в состав молекул реа- гирующих веществ. Теория окислительно-восстановительных процессов основана на следующих ос- новных положениях. Окислением называют процесс отдачи электронов атомом, молекулой или ио- ном, а восстановлением — процесс присоединения электронов атомом, молеку- лой или ионом. Окислитель — это вещество, в состав которого входят атомы, понижающие свою степень окисления. Принимая электроны от восстановителя, окислитель восстанавливается. Восстановитель — это вещество, в состав которого входят атомы, повышаю- щие свою степень окисления. Отдавая электроны окислителю, восстановитель окисляется. Понятия «окислитель» и «восстановитель» можно отнести не только к веществам, но и к входящим в их состав ионам и атомам. Так, в реакции +3 -1 +5 -2 KNO2 + Н2О2 = KNO3 + Н2О восста- окисли- новитель тель участвуют вещество-восстановитель KNO2, ион-восстановитель NO2, а также атом- восстановитель N(+3). В окислительно-восстановительных реакциях, кроме окислителя и восстановите- ля, могут принимать участие вещества, создающие ту или иную реакцию среды (кислотную, щелочную), благоприятствующую протеканию реакции. Для создания кислотной среды часто используют серную кислоту, например: +3 +4 +5 +2 NaNO2 + РЬО2 + H2SO4 = NaNO3 + PbSO4 + Н2О восста- окисли- среда новитель тель Щелочную среду обычно создают добавлением гидроксидов натрия или калия: +2 +1 +5 -1 2NO + 3KCIO + 2КОН = 2KNO3 + 3KCI + Н2О восста- окисли- среда новитель тель Рассмотрим пример выполнения задания, направленного на проверку усвоения основных положений теории окислительно-восстановительных процессов.
56 Пример 1. Установите соответствие между схемой реакции и формулой вещества-восстановителя в ней. СХЕМА РЕАКЦИИ А) Cr2O3 + Cl2 + КОН -> KCI + К2СгО4 + Н2О Б) В) Г) ФОРМУЛА ВОССТАНОВИТЕЛЯ Н2О2 + Ад2О -» Н2О + Ад + О2 H2O2+H2S->H2O + S 2' 2^ ' '2 1) Cl2 2) H2S 3) Н2О2 4) Ад2О и Сг2О3 5) 6) определить степени окисле- При выполнении этого задания нужно сначала ния элементов, а затем найти вещество-восстановитель, в составе которого находится элемент, отдающий электроны и повышающий степень окисления. +3 -1 +6 В реакции Сг2О3 + КОН —> KCI + ^СгОд + Н2О степень окисления атома хлора меняется от 0 до -1, а атома хрома — от +3 до +6. Напомним, что восстано- вителем является тот атом, степень окисления которого повышается. В дан- ном примере это атом хрома, а вещество-восстановитель — Сг2О3. Отметим, что в этой реакции участвует гидроксид калия, создающий щелочную среду. При анализе схем реакций Б и В важно не забыть, что степень окисления кис- -1 о лорода в составе Н2О2 равна -1. В ходе реакции Н2О2 + Ад2О -> Н2О + Ад + О2 она повышается до 0, следовательно, восстановителем является пероксид водорода. -1 -2 О В реакции Н2О2 + H2S -> Н2О + S, наоборот, степень окисления атома кислоро- да понижается от -1 до -2, а степень окисления атома серы повышается от -2 до 0. Следовательно, восстановителем в данном примере является серо- водород. А вот в реакции H2S + Li -► Li2S + Н2 степень окисления атома серы не изменя- ется. Сероводород будет окислителем за счёт понижения степени окисления атома водорода от +1 до 0. Восстановителем же в этой реакции является литий. Ответ: А Б в Г 6 3 2 5 Составление уравнений окислительно-восстановительных реакций. Метод электронного баланса Для расстановки коэффициентов в уравнениях окислительно-восстановитель- ных реакций можно использовать различные методы; наиболее распространённым среди них является метод электронного баланса. Рассмотрим его суть на несколь- ких примерах.
57 Пример 2. Методом электронного баланса подберите коэффициенты в схеме реакции KMnO4 + KBr + H2SO4 -> MnS04 + K2SO4 + Br2 + Н20 1. Определим степени окисления тех атомов, которые изменяют её в про- цессе реакции, и запишем их значения над символами элементов: +7 -1 +2 о КМпО4 + KBr + H2SO4 -4- MnSO4 + K2SO4 + Br2 + Н2О 2. По изменению степени окисления определим, какова роль каждого из ве- ществ в данной реакции: степень окисления атомов марганца понизилась от +7 до +2, следовательно, КМпО4 является окислителем. Степень окисления атомов брома повысилась с -1 до 0, следовательно, бро- мид калия является восстановителем. Кроме окислителя и восстановителя, в реакции участвует серная кислота. Входящие в её состав атомы не меняют степени окисления, но она создаёт кислотную среду, благоприятную для протекания реакции. Часто такое ве- щество, создающее ту или иную реакцию среды, кратко называют «среда». 3. Запишем уравнения электронного баланса, подбирая множители так, что- бы суммарное число электронов, отданных всеми атомами восстановителя, было равно числу электронов, принятых всеми атомами окислителя: -1 о 2Вг - 2ё -> Вг2 5 Мп + 5ё -> Мп 2 4. Найденные таким образом коэффициенты подставим в уравнение реакции: 2КМпО4 + 1ОКВг + H2SO4 -> 2MnSO4 + K2SO4 + 5Br2 + H2O 5. Подберём коэффициенты перед формулами остальных реагентов в сле- дующей последовательности: K2SO4-> H2SO4-> Н2О. Окончательно получаем: 2КМпО4 + 1ОКВг + 8H2SO4 = 2MnSO4 + 6K2SO4 + 5Br2 + 8H2O 6. Чтобы убедиться в правильности подбора коэффициентов, подсчитаем число атомов кислорода в левой и правой частях уравнения. И в левой, и в правой части уравнения находится по 40 атомов кислорода, следовательно, коэффициенты подобраны верно. Пример 3. Методом электронного баланса подберите коэффициенты в схеме реакции KMnO4 + HCI -> MnCI2 + KCI + Cl2 + Н2О Как и в предыдущем примере, сначала определим степень окисления тех атомов, которые изменяют её в процессе реакции: +7 -1 -1 -1 0 КМпО4 + HCI -» MnCI2 + KCI + Cl2 + Н2О Окислителем в реакции является КМпО4, а восстановителем — HCI. Однако эта реакция имеет характерную особенность: можно заметить, что не все атомы хлора переходят из степени окисления -1 в степень окисления 0. В правой части уравнения СГ входит в состав хлоридов марганца и калия. Нминммннмм1
при подборе коэффициентов. Дело в том, что в этой реакции HCI выполняет роль и восстановителя, и среды одновременно. Будем учитывать это Составим электронный баланс: -1 о 2CI - 2ё -> Cl2 + 7 +2 Мп + 5ё -> Мп 5 2 реакции. Коэффициент 5 ста- в котором содержатся атомы Подставим найденные коэффициенты в схему вим только перед формулой того вещества, хлора, поменявшие степень окисления (т. е. перед формулой С12, но не пе- ред формулой HCI): 2КМпО.+ HCI -> 2MnCL+ 5CL+ KCI + Н2О 4 ice. Далее подбираем коэффициенты перед формулами остальных реагентов в следующей последовательности: KCI -> HCI Н2О. Окончательно получаем: 2КМпО4+ 16HCI = 2МпС12+ 5CI2+ 2KCI + 8Н2О Убеждаемся в правильности подбора коэффициентов, сравнив число атомов кислорода в левой и правой частях уравнения (по 8 атомов). «Пример 4. Методом электронного баланса подберите коэффициенты в схеме реакции FeS + О2 —> Fe2O3 + SO2 После определения степеней окисления элементов становится очевидна особенность этой реакции: в ней участвует один атом-окислитель (кисло- род), но два атома-восстановителя — железо и сера: + 2 -2 0 +3 -2 +4 -2 FeS + О2 —> Fe2O3 + SO2 Атом железа отдаёт один электрон: + 2 +3 Fe - ё -> Fe Атом серы отдаёт шесть электронов: -2 +4 S - 6ё -> S Для составления электронного баланса оба процесса окисления можно объ- единить: +3 +4 FeS — 7е —> Fe+ S 4 О -2 О2 + 4ё -> 20 7 С учётом найденных коэффициентов получаем:
59 Окислительно-восстановительные свойства веществ. Прогнозирование продуктов окислительно-восстановительных реакций При выполнении наиболее сложных заданий данной темы, кроме подбора ко- эффициентов, бывает необходимо определить продукты окислительно-восстанови- тельной реакции: вписать в уравнение формулы недостающих веществ или полно- стью составить уравнение окислительно-восстановительной реакции. Выполнять такие задания нужно, опираясь на знания о закономерностях протекания окис- лительно-восстановительных реакций. Кроме того, необходимо познакомиться со свойствами важнейших окислителей и восстановителей, применяемых в промыш- ленных процессах и лабораторной практике, знать, какие продукты восстановления и окисления соответственно они образуют. Итак, напомним несколько правил, позволяющих характеризовать окислительно- восстановительные свойства того или иного вещества. Атомы, находящиеся в высшей степени окисления, могут быть только окислителями, поскольку способны только принимать электроны. Например, сера в составе H2SO4 находится в своей высшей степени окисления +6 и, следователь- но, может проявлять только окислительные свойства. Аналогично, только окисли- тельные свойства проявляет азот в составе азотной кислоты и нитратов, хлор в составе кислоты НСЮ4 и её солей и т. д. Атомы, находящиеся в низшей степени окисления, могут быть только вос- становителями, поскольку способны только отдавать электроны. Поэтому только восстановительные свойства проявляют, например, сера в составе H2S и сульфи- дов, азот в составе NH3, NH4 и нитридов, хлор в составе HCI и хлоридов и т. д. Атомы, находящиеся в промежуточной степени окисления, могут и при- нимать, и отдавать электроны. Вещества, содержащие такие атомы, облада- ют окислительно-восстановительной двойственностью: они выступают в роли окислителя или восстановителя в зависимости от свойств реагента, с которым взаимодействуют. Например, в молекуле пероксида водорода Н2О2 кислород находится в проме- жуточной степени окисления -1; следовательно, это вещество может проявлять как окислительные, так и восстановительные свойства. Окислительные свойства Н2О2 преобладают и проявляются в реакциях со многи- ми типичными восстановителями, например с сульфидами: -1 -2 PbS + 4Н2О2 = PbSO4 + 4Н2О Однако при действии на Н2О2 ещё более энергичного окислителя, чем он сам, например перманганата калия, пероксид водорода выступает в роли восстанови- теля: 2КМпО4 + 5Н2О2 + 3H2SO4 = 2MnSO4 + K2SO4 + 5О2 + 8Н2О Вещества, содержащие элементы в промежуточной степени окисления, облада- ют ещё одной особенностью: они способны к реакциям самоокисления-самовос- становления, или диспропорционирования. В таких реакциях в роли окислителя и восстановителя выступают атомы одного и того же элемента, находившиеся до начала реакции в одной степени окисления. Например, при взаимодействии Cl2 с холодным раствором гидроксида калия один из атомов хлора выступает в роли окислителя, понижая свою степень окисления, а другой атом хлора является вос- становителем, повышая свою степень окисления:
60 C°l2 + 2КОН = KCI + КСЮ + H2O О -1 Cl + ё -> Cl О +1 Cl - ё Cl К этому же типу реакций относится и разложение пероксида водорода на воду и кислород: 1 2 0 2Н2О2= 2Н2О + О2 При анализе окислительно-восстановительных свойств веществ нужно учитывать возможности всех атомов, входящих в его состав, отдавать или принимать элек- троны. Так, в составе сероводорода атом водорода находится в высшей степени окисления +1 и проявляет свойства окислителя, например: +1 о H2S + Mg = MgS + Н2 В этом же веществе атом серы находится в низшей степени окисления и явля- ется восстановителем: 2 0 H2S + Br2= S+2НВг Таким образом, сероводород способен быть окислителем за счёт атома водо- рода в степени окисления +1 и восстановителем за счёт атома серы в степени окисления -2. В некоторых случаях атом-окислитель и атом-восстановитель могут входить в состав одного и того же вещества. К этому типу относится, например, реакция разложения дихромата аммония -3+6 0+3 (NH4)2Cr2O7 = N2 + Cr2O3 + 4Н2О, в которой восстановителем является атом азота, а окислителем — атом хрома, до начала реакции находившиеся в составе (NH4)2Cr2O7: 2Сг + 6ё -> 2Сг -3 о 2N - 6ё -> N2 Пример 5. Определите, в составе каких из веществ атом азота проявляет толь- ко восстановительные свойства. 1) Ca3N2 2) HNO3 3) Ba(NO2)2 4) N2H4 5) NH4HS Атом азота проявляет только восстановительные свойства в том случае, если он находится в низшей степени окисления. Низшей степенью окисления для азота яв- ляется степень окисления -3. Вещества, в которых она реализуется, — это Ca3N2 и NH4HS. 1 5 Пример 6. Установите соответствие между схемой реакции и изменением сте- пени окисления восстановителя в ней.
< : - ;< •.... • . c : •• - СХЕМА РЕАКЦИИ ИЗМЕНЕНИЕ СТЕПЕНИ ОКИСЛЕНИЯ ВОССТАНОВИТЕЛЯ A) KCIO4 —> KCI + O2 Б) KCIO3 -» KCI + KCIO. B) KCIO3->KCI + O2 Г) KCIO-> KCI + KCIO3 +7 -1 1) CI—► CI -2 О 2) О —> О +5 -1 3) CI->CI +1 -1 4) Cl -> Cl +1 +5 5) Cl -> Cl +5 +7 6) Cl —► Cl Схемы предложенных к рассмотрению реакций выглядят несколько необыч- но: в их левых частях содержатся формулы только одного вещества. Следо- вательно, атом-окислитель и атом-восстановитель находятся в составе одно- го соединения. Запишем схемы электронного баланса для всех предложенных реакций. А) КСЮ4->КС1 + О2 С1 + 8ё->С1 -2 О 2О-4ё—> О2 Степень окисления меняют атомы хлора и кислорода. Восстановителем яв ляется кислород в степени окисления -2 (ответ 2). Б) КСЮ3 -> KCI + КСЮ4 С1 + 6ё->С1 +5 +7 С1-2ё —> CI Поскольку степень окисления меняют атомы одного и того же элемента, на- ходившиеся изначально в одинаковой степени окисления, данный процесс является реакцией диспропорционирования. И окислителем, и восстанови- телем являются атомы хлора. Восстановитель — тот из них, который повы- шает степень окисления: С1->С1 (ответ 6). В) КСЮ3 -> KCI + О. С1 + 6ё-»С1 2О-4ё—> О, Как и в случае А, степень окисления меняют атомы хлора и кислорода. Вос- становителем является кислород в степени окисления -2 (ответ 2). Г) КСЮ -> KCI + КСЮ. С1 + 2ё->С1 +1 +5 С1-4ё->С1
62 -'««>!? vmsm'W; ..... • j-.v : р. »•; . * Г' ; * «., * Это реакция диспропорционирования. Восстановителем является тот атом +1 ♦« хлора, который повышает степень окисления: Cl -» CI (ответ — 5). Ответ: А Б в Г 2 6 2 5 .... . : . . .И- Приведённые выше закономерности позволяют проанализировать и охарактери- зовать окислительно-восстановительные свойства веществ. Однако, опираясь толь- ко на эти правила, не всегда можно сделать вывод о том, насколько выраженными будут эти свойства и какие именно продукты образуются в результате реакции. Для ответа на эти вопросы часто требуется знать особенности химических свойств веществ, которые изучаются в курсе химии. Поэтому тренировать умение состав- лять уравнения окислительно-восстановительных реакций необходимо на протяже- нии всего курса повторения неорганической, а затем и органической химии. На данном этапе обучения целесообразно познакомиться с некоторыми важнейшими окислителями и восстановителями. Важнейшие окислители 1. Простые вещества, образованные атомами с высокой электроотрица- тельностью: F2, Cl2, Вг2, О2 и т. п. Принимая электроны, они восстанавливаются до низших степеней окисления: О -1 F2+2e->2F о -1 С12 + 2ё^2С1 О -1 Вг2 + 2ё-> 2Вг О -2 02+4ё->20 В ряду галогенов от фтора к брому окислительные свойства ослабевают, окис- лительные свойства иода значительно слабее, чем у остальных галогенов. 2. Среди кислородсодержащих кислот к важнейшим окислителям относятся азотная и концентрированная серная кислоты. Концентрированная серная кислота обладает выраженными окислительными свойствами за счёт атома серы в степени окисления +6. В отличие от разбав- ленной, концентрированная H2SO4 окисляет некоторые металлы, расположенные в ряду напряжений после водорода, а также многие неметаллы: 2Н25О4(конц.) + 2Ag = Ag2SO4 + SO2 + 2Н2О 2Н25О4(конц.) + С = СО2+ 2SO2+ 2Н2О Чаще всего продуктом восстановления серной кислоты является SO2. Однако в зависимости от условий проведения реакции и силы восстановителя можно полу- чить и другие продукты — серу и сероводород: 4Н28О4(конц.) + 3Zn = 3ZnSO4+ S + 4Н2О 5Н25О4(конц.) + 4Mg = 4MgSO4+ H2S + 4H2O
63 Азотная кислота, особенно концентрированная и дымящая, — сильный окис- литель. Она взаимодействует со многими неметаллами и сложными веществами, окисляя большинство элементов до их высших степеней окисления: S + 6НМО3(конц.) = H2SO4+ 6NO2+ 2Н2О Cu2S+ 14НМО3(конц.) = 2Cu(NO3)2+ H2SO4+ 10NO2 + 6H2O При взаимодействии HNO3 с металлами окислителем является атом азота, нахо- дящийся в степени окисления +5. Поэтому водород в таких реакциях практически не выделяется, а образуются различные продукты восстановления нитрат-иона, в зави- симости от концентрации кислоты, активности металла и некоторых других факторов. При взаимодействии горячей концентрированной азотной кислоты с металлами и неметаллами в большинстве случаев выделяется бурый газ NO2. При действии разбавленной, 30—35%-ной HNO3 на малоактивные металлы в основном образует- ся оксид азота(П). Сильно разбавленная азотная кислота при действии на актив- ные металлы (Mg, Zn, Са) может восстанавливаться до иона аммония, образующе- го с нитрат-анионом нитрат аммония. Примеры реакций: Си + 4НМО3(конц.) = Cu(NO3)2+ 2NO2+ 2Н2О ЗСи + 8НМО3(разб.) = 3Cu(NO3)2+ 2NO + 4Н2О 4Мд + ЮНЫО3(очень разб.) = 4Mg(NO3)2+ NH4NO3+ ЗН2О 3. Перманганат калия КМпО4 проявляет сильные окислительные свойства за счёт атома марганца в степени окисления +7. Продукты его восстановления, об- разующиеся при взаимодействии с одними и теми же реагентами, зависят от ха- рактера среды (кислотной, нейтральной, щелочной), в которой протекает реакция. В кислотной среде КМпО4 восстанавливается до катиона Мп2+, в нейтральной — до оксида марганца(1\/), а в щелочной — до манганата калия KjMnO., (схема 5). Схема 5 Восстановление перманганат-аниона в различных средах кислотная „ „ I среда + Мп КМпО4 Чей^ьная + Зс \ щелочная , . +6 среда + ® К2МПО4 Примеры реакций: кислотная среда 2КМпО4 + 5KNO2 + 3H2SO4 = 2MnSO4 + 5KNO3 + K2S04 + 3H2O нейтральная среда 2KMnO4 + 3KNO2 + H2O = 2MnO2 + 3KNO3 + 2КОН щелочная среда 2КМпО4 + KNO2 + 2КОН = 2K2MnO4 + KNO3 + Н2О 4. Хромат и дихромат калия (К2СгО4 и К2Сг2О7) проявляют окислительные свой- ства за счёт атома хрома, находящегося в степени окисления +6. Эти окислите- ли используют обычно в кислотной среде, продуктом их восстановления является обычно ион Сг3+, например: K2Cr2O7 + 3KNO2 + 4H2SO4 = Cr2(SO4)3 + 3KNO3 + I^SC^ + 4H2O
64 5. Кислородсодержащие соединения галогенов (NaCIO, КСЮ3, НСЮ4, КВгО3 и т. д.) содержат атомы галогенов в неустойчивых положительных степенях окисле- ния и проявляют за счёт этого сильные окислительные свойства. Атомы галогенов, как правило, восстанавливаются до наиболее устойчивой для них степени окисле- ния -1, например: 5КСЮ3 + 6Р = 5KCI + ЗР2О5 Таблица 8 Некоторые важнейшие окислители и продукты их восстановления Окислители Преимущественно образующиеся продукты восстановления Н2ЗО4(конц.) H2S, S, SO2 (в зависимости от силы восстановителя) HNO3 NH4, N2, N2O, NO, NO2 (в зависимости от силы восста- новителя и концентрации кислоты) КМпО4 перманганат калия Мп2*, МпО2, МпО4~(в зависимости от характера среды) KjCr2O7 дихромат калия Сг3* МпО2 Мп2* Кислородсодержащие соединения галогенов КСЮ3 КСЮ НСЮ4 КВгО3 и т. д. Галогенид-ионы: CI’, Вг" Важнейшие восстановители 1. Простые вещества, образованные атомами с низкой электроотрица- тельностью: металлы (Na, Са, Mg, AI и т. п.), углерод, водород. Эти восстановите- ли часто используют в процессах получения металлов, протекающих при высоких температурах, например: Cr2O3 + 2AI = 2Сг + А12О3 TiCI4+ 2Mg = Ti + 2MgCI2 ZnO + C = Zn + CO CuO + H2= Cu + H2O 2. Сложные вещества, содержащие атом в низшей степени окисления: HI, KI, H2S, Na2S, NH3, PH3 и т. п. 3. Сложные вещества, содержащие катионы металлов, заряд которых может возрасти, например: Fe2+, Сг2+. 4. Сложные вещества, содержащие атомы в промежуточной степени окисления, могут проявлять как окислительные, так и восстановительные свойства. Однако для некоторых из них, таких, как угарный газ СО, сульфиты, нитриты, восстанови- тельные свойства преобладают над окислительными, поэтому их часто используют в промышленности и лабораторной практике в качестве восстановителей. Нитриты обычно окисляются до нитратов, а сульфиты — до сульфатов. Примеры реакций:
Fe3O4 + 4C0 = 3Fe + 4CO2 KNO2 + Br2 + 2K0H = KNO3 + 2KBr + H2O Na2SO3 + CL + H20 = Na2S04 + 2HCI 2 О 2 2 2 4 Таблица 9 Некоторые важнейшие восстановители и продукты их окисления Восстановители Преимущественно образующиеся продукты восстановления HI, KI H2S, Na2S, ZnS S, SO2, SO*" (в зависимости от силы окислителя и условий реакции) NH3 N2, NO (в зависимости от условий реакции) PH3, P2O,, H,PO3 РОЗ Fe2+, FeO Fe3+, Fe2O3 (в зависимости от условий реакции) Cu2O, Cu2S Си2*, СиО (в зависимости от условий реакции) KNO2 KNO3 K2SO3 K2SO4 Рассмотрим на нескольких примерах, как применить на практике полученные знания для определения продуктов окислительно-восстановительных реакций. Пример 7. Установите соответствие между схемой реакции и формулой недостающего в ней вещества. СХЕМА РЕАКЦИИ ФОРМУЛ? ( ВЕЩЕСТВА А) Н2О2+К1 -> КЮ3+... 1) KNO3 Б) H2O2+CI2-> HCI + ... 2) о2 В) KNO2+Br2+KOH -> КВг+Н2О + ... 3) NO Г) KNO2+ KI + Н2О -► l2+ ... + КОН 4) N2O3 5) Н2О 6) н2 Напомним, что в молекуле пероксида водорода атом кислорода находится в промежуточной степени окисления -1, следовательно, это вещество может играть роль и окислителя, и восстановителя. Будем рассуждать следующим образом. В схеме первой реакции Н2О2+ KI -> КЮ3+... указано, что из иоди- да калия образуется иодат калия КЮ3. Степень окисления атомов иода повы- шается, следовательно, иод в этой реакции является восстановителем. Тогда пероксид водорода должен быть окислителем, степень окисления атома кис- лорода будет понижаться и в продукте реакции станет равной -2. Варианты ответов предусматривают только одно такое вещество — Н2О (ответ 5).
66 Из схемы второй реакции Н2О2 + С12—> HCI + ... видно, что хлор выступает в роли окислителя. Тогда пероксид водорода будет восстановителем, и сте- пень окисления атома кислорода повысится с -1 до 0. Значит, недостающим продуктом реакции является кислород О2 (ответ 2). Теперь рассмотрим третью и четвёртую реакции. Нитрит калия также спо- собен проявлять как свойства окислителя, так и свойства восстановителя за счёт изменения степени окисления атома азота, находящегося в проме- жуточной степени окисления +3. В третьей реакции окислителем является бром. Тогда нитрит калия KNO2 должен быть восстановителем, и степень окисления атома азота будет повышаться. Варианты ответа предусматрива- ют только одно вещество с более высокой степенью окисления атома азо- та — нитрат калия (ответ 1). И наконец, в последней реакции иодид калия является восстановителем, тогда KNO2 будет окислителем, и степень окисления атома азота понизится. Недостающим продуктом реакции будет в этом случае NO (ответ 3). А Б 5 2 В г 1 3 Пример 8. Используя метод электронного баланса, составьте уравнение реакции Na2SO3+ ... + КОН -> К2МпО4+ ... + Н2О Прежде всего определим недостающие в уравнении реакции вещества. В правой части уравнения находится манганат калия К2МпО4, а в левой части в качестве вещества-среды указан гидроксид калия. Известно, что манганаты получаются при восстановлении перманганатов в щелочной среде (табл. 8). Следовательно, недостающее в левой части уравнения вещество — перман- ганат калия. Таким образом, КМпО4 в этой реакции окислитель, а это зна- чит, что сульфит натрия выступает в данном случае в роли восстановителя. Окисляясь, он образует в качестве продукта сульфат натрия Na2SO4 (табл. 9). Итак, мы установили формулы всех веществ, участвующих в реакции: Na2SO3+КМпО4+КОН -> I^MnO, + Na2SO4 + Н2О Теперь составим электронный баланс: +7 +6 2 Мп + ё —> Мп 1 S - 2ёS Укажем, что сера в степени окисления +4 (или сульфит натрия) является восстановителем, а марганец в степени окисления +7 (или перманганат ка- лия) — окислителем. Расставляем коэффициенты в уравнении реакции: Na2SO3 + 2КМпО4 + 2КОН = Na2SO4 + 2К2МпО4 + Н2О
е/ Правильность расстановки коэффициентов проверяем, сравнивая суммарное количество атомов кислорода в левой и правой части уравнения. Пример 9. Используя метод электронного баланса, составьте уравнение реакции Zn + KNO3+... -> K2ZnO2+NH3 + ... Определите окислитель и восстановитель. В данном уравнении уже указаны окислитель (KNO3) и восстановитель (Zn) и продукты их восстановления (K2ZnO2) и окисления (NH3) соответственно. Следовательно, в левой части уравнения необходимо дописать формулу ве- щества, выполняющего роль среды. Поскольку одним из продуктов является цинкат калия, то очевидно, что реакция протекает в щелочной среде, а не- достающее в левой части уравнения вещество — гидроксид калия КОН. В правую часть уравнения добавляем Н2О: Zn + KNO3 + КОН -> К^пОг + NH3 + Н2О Составляем электронный баланс: 4 1 О +2 Zn - 2е —> Zn +5 -3 N+8e -> N Указываем, что цинк в степени окисления 0 является восстановителем, а азот в степени окисления +5 (или нитрат калия) — окислителем. Расставля- ем коэффициенты: 4Zn + KNO3 + 7КОН = 4K2ZnO2 + NH3 + 2H2O Правильность расстановки коэффициентов проверяем, сравнивая суммарное количество атомов кислорода в левой и правой части уравнения. Пример 10. Даны вещества: перманганат калия, гидрокарбонат калия, сульфит натрия, хлорид серебра, гидроксид калия. Из предложенного пе- речня веществ выберите вещества, между которыми возможна окисли- тельно-восстановительная реакция, и запишите уравнение этой реакции. Составьте электронный баланс, укажите окислитель и восстановитель. Допустимо использование водных растворов этих веществ. Проанализируем окислительно-восстановительные свойства представленных веществ. Типичным окислителем является перманганат калия (см. табл. 8), а выраженными восстановительными свойствами обладает сульфит натрия (см. табл. 9). Продуктом окисления сульфита натрия будет его сульфат, а вот продукт восстановления перманганата калия зависит от того, в какой среде — кислотной, щелочной или нейтральной — проводили реакцию (см. схему 5). Условие задания даёт возможность составить два уравнения. В первом случае, кроме окислителя и восстановителя, будем использовать гидроксид калия для создания щелочной среды:
68 «г^ЖШми^ей^у^ййч'*-- йШйи^кШ. ; .гг»л • Ц; Na2SO3 + 2KMnO4 + 2K0H = Na2SO4 + г^МпО, + H2O +7 +6 Мп + е -» Мп 2 +4 +6 S - 2ё -> S Сера в степени окисления +4 (или сульфит натрия) является восстановителем, марганец в степени окисления +7 (или перманганат калия) — окислителем. Поскольку допускается использование водных растворов веществ, то возможен и другой вариант ответа: 3Na2SO3 + 2КМпО4 + Н2О = 3Na2SO4 + 2MnO2 + 2КОН Мп + 3ё -> Мп 2 Мп + 3ё -> Мп +4 +6 S - 2ё -> S Сера в степени окисления +4 (или сульфит натрия) является восстановителем, марганец в степени окисления +7 (или перманганат калия) — окислителем. На экзамене можно будет написать любую из возможных окислительно-вос- становительных реакций, отвечающих условию задания. Правила проведе- (£ ния экзамена предусматривают такую возможность, и в случае полного и правильного выполнения всех элементов задания оно будет оценено макси- мальным баллом. Электролиз Электролиз — это совокупность окислительно-восстановительных процессов, протекающих на поверхности электродов, помещённых в расплав или раствор электролита, при пропускании электрического тока. Катодом является электрод, на котором протекают реакции восстановления ионов и молекул. Анодом является электрод, на котором протекают реакции окис- ления ионов и молекул. Школьная программа предполагает знакомство с процес- сами электролиза, протекающими на так называемых инертных электродах, т. е. электродах, которые сами не изменяются в процессе электролиза. Такие электро- ды могут быть изготовлены, например, из платины или графита. Наиболее простым в химическом отношении случаем является электролиз расплавов солей. В качестве примера рассмотрим процессы, протекающие при электролизе расплава хлорида натрия. Хлорид натрия в расплаве диссоциирует с образованием катионов натрия и анионов хлора: NaCI Na+ + Cl- При пропускании электрического тока на катоде протекает восстановление ионов натрия до металлического натрия, а на аноде — окисление хлорид-ионов: Катод(-): №+ + ё-> Na О ж Анод(+): 2CI-- 2ё-> С12Т Складывая полуреакции окисления и восстановления, получаем суммарное уравнение электролиза: 2NaCI электРолиэ > 2Na + С12
69 Другой пример — электролиз расплава щёлочи, суммарное уравнение которого будет выглядеть так: 4NaOH - Электре™3 » 4Na + О2 + 2Н2О В случае водных растворов в процессах, протекающих на электродах, кроме ионов электролита, могут участвовать и молекулы воды. В случае электролиза вод- ного раствора соли на катоде может выделяться не только металл, но и водород (из воды). Аналогично на аноде возможно выделение не только продукта окисле- ния анионов соли, но и кислорода из воды. Для выбора катодного процесса следует руководствоваться положением метал- ла в ряду напряжений. Если металл расположен в ряду напряжений левее марганца, то на катоде происходит восстановление молекул воды и выделяется водород: О ж 2Н2О + 2ё = Н2Т + 2ОН- Если металл расположен в ряду напряжений правее водорода, то на катоде вы- деляется только металл, например: О Си2++ 2е= Си Металлы от марганца включительно и до свинца выделяются на катоде одно- временно с водородом. Li К Ba Са Na Mg Al Мп Zn Cr Fe Cd Ni Sn Pb H2 Cu Hg Ag Pt Au ' V ' ' V ' ' V ' H2 из воды Металл + H2 Металл Анодный процесс. Если анион бескислородный (CI-, Br-, I-, S2-), то на аноде окисляется сам анион, например: о 2Вг - 2ё -> Вг2 Исключением является ион F-: при электролизе растворов фтороводорода и фторидов на аноде выделяется кислород. Если электролизу подвергаются водные растворы кислородсодержащих кислот или их солей (а также фтороводорода и фторидов), то на аноде происходит окис- ление воды с выделением кислорода: 2Н2О-4ё -4- 4Н+ + О2Т При электролизе солей карбоновых кислот на аноде протекает окисление ани- она по схеме: 2RCOO- - 2ё -+ R—RT + 2СО2Т Пример 11. Установите соответствие между формулой соли и продуктом, образующимся на катоде при электролизе её водного раствора. я* ФОРМУЛА СОЛИ ПРОДУКТ НА КАТОДЕ Л- A) CuBr2 1) н2 Б) CuSO4 2) Си В) NaNO3 3) Na Г) Ba(NO3)2 4) Ва 5) NO2 6) Вг2
70 Решение. Напомним, что на катоде протекает процесс восстановления (при- соединения электронов). Следовательно, при электролизе растворов солей на катоде могут выделяться только металлы или водород. Поэтому два по- следних варианта ответа можно сразу исключить из рассмотрения как за- ведомо неверные. Далее руководствуемся правилами определения катодных продуктов электролиза. Медь расположена в ряду напряжений металлов пра- вее водорода, значит, при электролизе и раствора CuBr2, и раствора CuSO4 на катоде будет выделяться медь (ответ 2). Барий и натрий расположены левее марганца, следовательно, при электролизе водных растворов их солей на катоде выделяется водород из воды (ответ 1). Ответ: А Б В г 2 2 1 1 Пример 12. Установите соответствие между формулой соли и продуктом, который образуется на инертном аноде при электролизе её водного рас- твора. ФОРМУЛА СОЛИ ПРОДУКТ НА АНОДЕ A) CuSO4 1) водород Б) K2S 2) кислород В) ВаС12 3) металл Г) Pb(NO3)2 4) хлор 1 5) сера 6) азот Решение. Воспользуемся приведёнными выше правилами для определения анодных продуктов электролиза. Поскольку CuSO4 и Pb(NO3)2 содержат кис- лородсодержащие анионы, при электролизе водных растворов этих солей на аноде окисляются молекулы воды с образованием кислорода (ответ 2). Так как сульфид калия является солью бескислородной кислоты, анодным продуктом электролиза его раствора будет сера (ответ 5). Аналогично, при электролизе раствора ВаС12 на аноде выделится хлор (ответ 4). Ответ: А Б В Г 2 5 4 2
Модуль II. Неорганическая химия ЗАНЯТИЕ 8 Характерные химические свойства представителей основных классов неорганических веществ Основные элементы содержания: оксиды, оксиды кислотные, оснбвные, амфотерные, двойные оксиды, несолеобразующие окси- ды, основания, щёлочи, амфотерные гидроксиды, кислоты, общие и специфические свойства кислот, соли. Характерные химические свойства оксидов Химические свойства оснбвных оксидов Все оснбвные оксиды взаимодействуют с кислотами, с кислотными и амфотер- ными оксидами с образованием солей, например: Na2O + 2HCI = 2NaCI + Н2О ВаО + H2SO4 = BaSO4 + Н2О ВаО + SO3 = BaS04 3Li2O + Р2О5 = 2Li3PO4 Na2O + AI2O3 = 2NaAIO2 Na2O + PbO = Na2PbO2 Оксиды щелочных и щелочноземельных металлов взаимодействуют с водой с образованием растворимых в воде оснований — щелочей: Na2O + Н2О = 2NaOH ВаО + Н2О = Ва(ОН)2 Некоторые из оснбвных оксидов могут восстанавливаться под действием вос- становителей или разлагаться при нагревании: СиО + Н2 = Си + Н2О FeO + СО = Fe + СО2 4СиО = 2Си2О + О2 Химические свойства кислотных оксидов Кислотные оксиды взаимодействуют с водой с образованием соответствующих кислот (исключение — оксид кремния(1\/): Р2О5+ЗН2О = 2Н3РО4 SO3+ Н2О = H2SO4 Кислотные оксиды взаимодействуют с основаниями, с оснбвными и амфотерны- ми оксидами с образованием солей: СО2 + 2NaOH = Na2CO3 + Н2О Р2О5 + 6КОН = 2К3РО4 + ЗН2О Р2О5 + 4NaOH = 2Na2HPO4 + Н2О СО2 + СаО = СаСО3
12 S03+ Na2O = Na2SO4 3SiO2 + ALO, = AI2(SiO3)3 Некоторые из кислотных оксидов могут разлагаться при нагревании, а также вступать в окислительно-восстановительные реакции, проявляя как окислительные, так и восстановительные свойства: 2SO3 = 2SO2 + O2 2N2O5 = 4NO2+О2 2NO2 +4Н2= N2 +4Н2О SO2 + 2H2S = 3S + 2H2O 2SO2+O2=2SO3 SO2 + CI2=SO2CI2 При нагревании некоторые кислотные оксиды вытесняют более летучие из их солей. Так, при производстве стекла протекают реакции: СаСО3 + SiO2 = CaSiO3 + СО2Т Na2CO3 + SiO2 = Na2SiO3 + CO2t Химические свойства амфотерных оксидов Амфотерные оксиды взаимодействуют как с кислотами, так и с щелочами с об- разованием солей: А12О3 + 3H2SO4 = AI2(SO4)3 + ЗН2О AI2O3+ 2NaOH = 2NaAIO2+ Н2ОТ А12О3 + 2NaOH + ЗН2О = 2Na[AI(OH)4] Амфотерные оксиды могут взаимодействовать с кислотными и оснбвными окси- дами с образованием солей: ZnO + SO3 = ZnSO4 ZnO + BaO = BaZnO2 Некоторые из амфотерных оксидов при нагревании реагируют с солями: А12О3 + Na2CO3 = 2NaAIO2 + СО2? Амфотерные оксиды могут вступать в окислительно-восстановительные реакции: Cr2O3 + 2AI = А12О3 + 2Сг MnO2 + 4HCI = МпС12 + С12 + 2Н2О При нагревании некоторые амфотерные оксиды могут разлагаться: 4МпО2 = 2Мп2О3+О2 6Fe2O3 = 4Fe3O4 + О2 Характерные химические свойства оснований Общие свойства оснований связаны с наличием в их растворе гидроксид-ионов, образующихся в ходе электролитической диссоциации: NaOH = Na+ + OH- Ва(ОН)2= Ва2++2ОН-
73 Все основания реагируют с кислотами с образованием солей (как средних, так и кислых или оснбвных): 2NaOH + H2SO4 = Na2SO4 + 2Н2О NaOH + H2SO4= NaHSO4+ H2O Mg(OH)2 + HCI = (MgOH)CI + H2O Основания реагируют с кислотными и амфотерными оксидами с образованием солей: 2NaOH + СО2 = Na2CO3 + Н2О 2КОН + ZnO + Н2О -► K2[Zn(OH)4] 2NaOH + Сг2О3 -?плавл- > 2NaCrO2 + Н2О Основания реагируют с амфотерными гидроксидами с образованием солей: 2NaOH + Zn(OH)2 сплавл~ > Na2ZnO2 + Н2ОТ 2NaOH + Zn(OH)2 -» Na2[Zn(OH)4] Нерастворимые в воде основания (а также гидроксид лития) разлагаются при нагревании с образованием соответствующего оснбвного оксида и воды: Мд(ОН)2 = МдО + Н2О 2LIOH = Li2O + Н2О Щёлочи могут взаимодействовать с солями, а также с некоторыми металлами и неметаллами: 2КОН + CuCI2= Cu(OH)2i + 2KCI КОН + NH4CI = KOI + Н2О + NH3t NaOH + NaHCO3 = Na2CO3 + H2O 2AI + 6KOH + 6H2O = 2Кз[А1(ОН)6] + 3H2t Zn + 2NaOH = Na2ZnO2+ H2T Si + 2NaOH + H2O = Na2SiO3 + 2H2T 3CI2 + 6KOH = KCIO3 + 5KCI + 3H2O 3S + 6NaOH = 2Na2S + Na2SO3 + 3H2O Химические свойства амфотерных гидроксидов Амфотерные гидроксиды реагируют как с кислотами, так и с щелочами: Zn(OH)2 + H2SO4 = ZnSO4 + 2Н2О Zn(OH)2 + 2КОН(р.р) =K2[Zn(OH)4] Zn(OH)2+2NaOH(TB.) = Na2ZnO2 + 2H2O Возможны также реакции амфотерных гидроксидов с кислотными и основными оксидами: Zn(OH)2 + SO3 = ZnSO4 + Н2О Zn(OH)2+ CaO = CaZnO2+ H2O
74 В то же время амфотерные гидроксиды не реагируют с кислотными оксидами, отвечающими очень слабым кислотам: А1(ОН)3 + СО2* При нагревании все амфотерные гидроксиды разлагаются: 2А1(ОН)3 =А12О3 + ЗН2О Характерные химические свойства кислот Общие свойства кислот связаны с наличием в их водных растворах гидрати- рованных ионов водорода, образующихся в процессе электролитической диссо- циации: HCI = Н++С1- Все кислоты взаимодействуют с основаниями (растворимыми, нерастворимы- ми): HNO3 + NaOH = NaNO3 + Н2О H2SO4 + Mn(OH)2 = MnSO4 + 2H2O В ходе этих реакций возможно образование не только средних, но и кислых или основных солей: Си(ОН)2+ НС1( недостаток) = (CuOH)CI + Н2О КОН + H2SO4( избыток) = KHSO4+ Н2О Кислоты взаимодействуют с оксидами — основными и амфотерными. При этом образуются соль и вода: 2HNO3 + Na2O = 2NaNO3 + Н2О H2SO4 + ZnO = ZnSO4 + 2H2O Кислоты взаимодействуют с металлами, стоящими в ряду напряжений металлов до водорода, с образованием соли и водорода: Fe + 2HCI = FeCI2 + H2t Zn + H2SO4 = ZnS04 + H2T 2AI + 6HCI = 2AICI3 + 3H2T В некоторых случаях металлы не взаимодействуют с растворами кислот, хотя стоят в ряду активности металлов до водорода. Так, свинец не реагирует с раз- бавленной соляной кислотой (образующийся в ходе реакции нерастворимый хло- рид свинца покрывает металл плёнкой): Pb + 2HCI * Кислоты могут взаимодействовать с солями. Этот тип реакций возможен в слу- чае выделения газа, образования осадка или слабого электролита: H2S + CuSO4 = CuS-l + H2SO4 H2SO4 + CaF2 = CaSO4 + 2HF HCI + NaHCO3 = NaCI + H2O + CO2t Некоторые кислоты могут разлагаться при нагревании, например: H2SiO3 = SiO2 + Н2О 4HNO3 = 4NO2 + О2 + 2Н2О
75 Кислоты могут участвовать не только в кислотно-оснбвных, но и в окислитель- но-восстановительных реакциях. Кислоты, содержащие элемент в низшей степени окисления, могут проявлять восстановительные свойства. Напротив, кислоты, со- держащие элемент в высшей степени окисления, могут проявлять окислительные свойства: H2S + Br2= S + 2НВг 4НВг + МпО2= МпВг2+ Вг2+ 2Н2О 4HNO, + С = 4NO2 + СО2 + 2Н2О Концентрированная серная кислота обладает некоторыми специфическими свойствами. Например, она относится к эффективным водоотнимающим СреД- СТВаМ: НСООН н2о + соф Концентрированная серная кислота является довольно сильным окислителем, особенно при нагревании. Так, она окисляет бромид-, иодид-ионы, некоторые простые вещества (серу, уголь) и др.: 8KI + 9Н28О4(конц.) = 8KHSO4+H2S + 4I2 + 4H2O С + 2Н28О4(конц.) = СО2 + 2SO2 + 2Н2О Концентрированная серная кислота может окислять некоторые малоактивные металлы, стоящие в ряду активности после водорода (Си, Нд, Ад). При взаимо- действии с металлами водород обычно не выделяется: Си + 2Н28О4(конц.) = CuSO4 + SO2 + 2Н2О 4Mg + 5Н28О4(конц.) = 4MgSO4+ H2S + 4Н2О Концентрированная серная кислота не растворяет (пассивирует) такие металлы, как алюминий, железо, хром. Азотная кислота, особенно концентрированная, проявляет сильные окисли- тельные свойства. Так, она окисляет сульфиды, сульфиты, многие простые веще- ства (иод, углерод, фосфор, серу) и др.: 2HNO3 + H2S = S + 2Н2О + 2NO2 5HNO3 + P = H3PO4 + H2O + 5NO2 Азотная кислота (даже разбавленная) растворяет многие малоактивные металлы (медь, серебро, ртуть). Поскольку азотная кислота, даже разбавленная, является окислителем за счёт азота(\/), то продуктом её восстановления являются различ- ные соединения азота: NO2, NO, N2O, N2, NH4+ (схема 6). Вероятность выделения NO2 наибольшая в случае использования неактивного металла и концентрирован- ной азотной кислоты. Напротив, образование ионов аммония возможно при рас- творении в очень разбавленной азотной кислоте активного металла. Схема 6 Продукты восстановления азотной кислоты +4 +2 +1 О -3 Металл + HNO3-* NO2—NO—N2O—N2—NH4 возрастает концентрация кислоты увеличивается активность металла
76 Примеры реакций: Си + 4Н1ЧО3(конц.) = Cu(NO3)2 + 2NO2 + 2Н2О ЗСи + 8НМО3(разб.) = 3Cu(NO3)2+ 2N0 + 4Н2О 4Мд + 10НМО3(очень разб.) = 4Mg(NO3)2 + NH4NO3 + ЗН2О Концентрированная азотная кислота не растворяет (пассивирует) такие метал- лы, как алюминий, железо, хром. Характерные химические свойства солей Соли — сложные вещества, при диссоциации которых в водном растворе об- разуются катионы металлов и анионы кислотных остатков. Химические свойства солей определяются природой катиона и аниона, из которых соль образована. В водных растворах или расплавах соли диссоциируют на составляющие ионы: KCI -> К+ + СГ Na2SO4 -» 2Na+ + SO42- Соли могут вступать в реакции ионного обмена с кислотами, основаниями и другими солями. Условием возможности таких реакций является выделение газа, образование осадка или малодиссоциирующего соединения (слабого электролита): CaF2 + H2SO4 = CaSO4 + 2HF 2CH3COONa + H2SO4 = Na2SO4 + 2CH3COOH CuCI2 + 2KOH = Си(ОН)2Ф + 2KCI FeS + 2HCI = FeS + H2ST Na2SO4+ BaCI2= BaSOj + 2NaCI Соли могут вступать в реакции замещения с металлами. В ходе этих реакций более активный металл вытесняет менее активный из раствора его соли: Zn + CuSO4 = Си + ZnSO4 Си + 2AgNO3= 2Ag + Cu(NO3)2 Реакции вытеснения одних металлов другими могут идти не только в растворе. Так, первый металлический алюминий был получен вытеснением алюминия калием из расплавленного хлорида алюминия: AICI3 + ЗК = Al + 3KCI Соли могут участвовать в окислительно-восстановительных реакциях. Одни соли проявляют сильные восстановительные свойства, другие — окислительные. Сильными восстановителями являются SnCI2, CrCI2, FeCI2, CuCI, K2S, K2SO3 и др. Окислительные свойства проявляют KMnO4, K2Cr2O7, KNO3, КСЮ3 и др. Примеры окислительно-восстановительных реакций с участием солей: 5КСЮ3 + 6Р = ЗР2О5 + 5KCI 2FeCI3+ Fe = 3FeCI2 2KMnO4 + 16HCI = 2KCI + 8Н2О + 5CI2 + 2MnCI2 2K2S + K2SO3 + 6HCI = 6KCI + 3S + ЗН2О Некоторые соли разлагаются при нагревании. К таким солям относятся кар- бонаты, нитраты, перманганаты, хлораты, перхлораты, дихроматы, а также соли аммония:
11 2KMnO4 = K2MnO4+ MnO2+ 02 CaCO3 = CaO + CO2 2NaHCO3 = Na2CO3 + CO2T + H2O NH4CI = NH3+HCI Кислые соли под действием основэний переходят в средние соли. Ниже при- ведены урзвнения реакций с учзстием кислых солей: KHSO4 + КОН = K2SO4 + Н2О 2NaHSO4 + Zn = Na2SO4 + ZnSO4 + H2 2NaHCO3 = Na2CO3+ CO2+ H2O KHCO3 + HCI = KCI + H2O + CO2 Примерами комплексных солей могут служить гидроксокомплексы алюминия и цинка, образование и некоторые свойства которых представлены ниже: AI(OH)3+ NaOH = Na[AI(0H)4] Na[AI(OH )4] + СО2 = NaHCO3 + AI(OH)3 Na2[Zn(OH)4] + 2H2SO4 = Na2SO4 + ZnSO4 + 4H2O Примеры заданий Пример 1. Оксид цинка взаимодействует с каждым из веществ: 1) Н2О, Fe 2) HNO3, С 3) HCI, Си 4) NaOH, О2 5) КОН, ВаО Оксид цинка относится к амфотерным оксидам, поэтому должен взаимодей- ствовать с кислотами, щелочами, некоторыми кислотными и основными ок- _______ ______ сидами. Кроме того, он должен вступать в окислительно-восстановительные реакции, восстанавливаясь сильными восстановителями до металлического цинка. Этим условиям удовлетворяют ответы 2 и 5. Ниже приведены урав- нения реакций: ZnO + 2HNO3 = Zn(NO3)2 + Н2О ZnO + С = Zn + СО ZnO + 2КОН = K2ZnO2 + H2O ZnO + ВаО = BaZnO2 Ответ: 2 5 Пример 2. Оксид железа(П) реагирует с каждым из веществ: 1) водородом и серной кислотой 2) гидроксидом калия и углекислым газом 4) азотом и водородом 5) углеродом и кислородом 3) оксидом бария и азотом Оксид железа(Н) относится к оснбвным оксидам, поэтому должен взаимо- действовать с кислотами, некоторыми кислотными и амфотерными оксида-
78 ми. Кроме того, он должен вступать в окислительно-восстановительные ре- акции: железа воряют восстанавливаться сильными восстановителями до металлического или окисляться до соединений железа(Ш). Этим условиям удовлет- ответы 1 и 5. Ниже приведены уравнения реакций: FeO + H2SO4 = FeSO4 + Н2О FeO + Н2 = Fe + Н2О ‘ FeO + С = Fe + СО 4FeO + О2 = 2Fe2O3 Ответ: водой соляной кислотой водородом гидроксидом калия оксидом фосфора(\/) Пример 3. И оксид алюминия, и оксид натрия реагируют с с с с с 2) 3) 4) 5) Оксид алюминия относится к амфотерным оксидам, оксид натрия — к ос- нбвным. Число реактивов, с которыми взаимодействуют и амфотерные, и основные оксиды, невелико. Это главным образом вещества кислотной при- роды — кислоты и кислотные оксиды. Этим условиям удовлетворяют от- веты 1 и 5. Ниже приведены уравнения реакций оксидов алюминия и калия с соляной кислотой и оксидом фосфора(У): А12О3 + 6HCI = 2AICI3 + ЗН2О Na2O + 2HCI = 2NaCI + Н2О AI2O3+ Р2О5 = 2А1РО4 3Na2O + P2O5 = 2Na3PO4 Ответ: Пример 4. Установите соответствие между реагирующими веществами и продуктами их взаимодействия. РЕАГИРУЮЩИЕ ВЕЩЕСТВА А) СО2+Н2О -> Б) СО2 + СаО —> В) СО2+Са(ОН)2(избыток) -> Г) СО2(избыток) + Са(ОН)2 -> ПРОДУКТЫ ВЗАИМОДЕЙСТВИЯ СаСО3 СаСО3 + Н2О Са(НСО3)2 Са(НСО3)2+ Н2О СО + Н2 Н2СО3 2) 3) 4) 5) 6) В задании проверяется знание свойств углекислого газа как кислотного ок- сида. В первой реакции углекислый газ реагирует с водой, в ходе подоб-
ных реакций образуется кислота, в данном случае угольная кислота (ответ 6). Во второй реакции кислотный оксид взаимодействует с основным окси- дом (оксидом кальция). Подобная реакция приводит к образованию соли — карбоната кальция (ответ 1). Третья и четвёртая реакции осуществляются между кислотным оксидом и щёлочью. В зависимости от условий здесь возможно образование различных продуктов. При избытке щёлочи образу- ется средняя соль (ответ 2), при избытке кислотного оксида — кислая соль (ответ 3). Ответ: Пример 5. Установите соответствие между названиями оксидов и переч- нем веществ, с которыми они могут взаимодействовать. НАЗВАНИЕ ОКСИДА А) оксид кремния(1\/) Б) оксид азота(1\/) В) оксид лития Г) оксид алюминия ВЕЩЕСТВА 1) СаСО3, NaOH 2) СО, NaOH 3) СО2, КОН 4) Н2О, CaSO4 5) Н2О, HNO3 6) HCI, О2 Оксид кремния(1\/) является кислотным оксидом. Он способен реагировать с карбонатом кальция и щёлочью: SiO2 + СаСО3 = CaSiO3 + СО2 SiO2 + 2NaOH = Na2SiO3 + Н2О Оксид азота(1\/), будучи кислотным оксидом, реагирует с гидроксидом на- трия. Как окислитель реагирует с угарным газом: 2NO2 + 2NaOH = NaNO2 + NaNO3 + H2O 2NO2 + 4CO = N2 + 4CO2 Оксид лития реагирует с водой и азотной кислотой: l_i2O + Н2О = 2UOH Ш2О + 2HNO3 = 2LiNO3 + Н2О Оксид алюминия, будучи амфотерным, реагирует с карбонатом кальция (при нагревании) и щёлочью: А12О3 + СаСО3 = Са(АЮ2)2 + СО2 А12О3+ 2NaOH = 2NaAIO2+ Н2О Ответ:
80 Пример 6. Гидроксид меди(П) при обычных условиях реагирует 1) с МаОН(разб.) 2) с H2SO4(p-p) 3) с СаО 4) с СН3СООН 5) с SiO2 Гидроксид меди(П) относится к основаниям, следовательно, он должен взаи- модействовать с веществами кислотной природы. Кроме этого, соединения меди(П) могут проявлять слабые окислительные свойства и восстанавливать- ся до меди(1) или металлической меди. В списке реагентов есть две кислоты (серная и уксусная) — именно с ними и возможны химические реакции: Cu(OH)2 + H2SO4 = CuSO4 + 2Н2О Cu(OH)2 + 2СН3СООН = Cu(CH3COO)2 + 2Н2О С оксидом кремния (кислотным оксидом) реакция возможна, но только при нагревании — при обычных условиях она не протекает (мала химическая ак- тивность оксида кремния). Ответ: 2 4 It Пример 7. Гидроксид азота(\/) 1) относится к сильным электролитам 2) не разлагается при нагревании 3) растворяет серебро 4) проявляет восстановительные свойства 5) имеет формулу N(OH)5 Напомним, что гидроксиды — это вещества с общей формулой ЭОт(ОН)п, которые можно рассматривать как продукт взаимодействия оксида с водой. Гидроксид азота(У) — не что иное, как азотная кислота (продукт взаимодей- ствия оксида азота(\/) с водой): о н—о—N NO2(OH) HNO3 V Азотная кислота относится к сильным электролитам (ответ 1 верный). Буду- чи сильным окислителем, азотная кислота растворяет некоторые неактивные металлы, стоящие в ряду напряжений после водорода, в частности медь, ртуть и серебро (ответ 3 верный). Ответ: 3 Пример 8. Гидроксид калия взаимодействует с каждым из двух веществ цинком и едким натром алюминием и сероводородом серной кислотой и гашёной известью гидроксидом натрия и содой гидроксидом алюминия и алюминием Гидроксид калия относится к сильным основаниям (щелочам). Щёлочи реа- гируют с веществами кислотной природы, а также с амфотерными оксидами 2) 3) 4) 5)
№ чГгЛ-V Г ,;Ч ; :• : .. л:.,;;. .-Л ... .< : . • и гидроксидами. Возможны также реакции с солями, некоторыми металлами и неметаллами. Проанализируем предложенные варианты ответов. 1) Гидроксид калия взаимодействует с цинком, но не реагирует с едким на- тром. 2) Гидроксид калия реагирует и с алюминием, и с сероводородом (кисло- той): 2КОН + 2AI + 6Н2О = 2К[А1(ОН)4] + ЗН2 2КОН + H2S = K2S + 2Н2О 3) Гидроксид калия реагирует с серной кислотой, но не реагирует с гашёной известью (гидроксидом кальция, щёлочью). 4) Гидроксид калия не реагирует с NaOH и Na2CO3. 5) Гидроксид калия реагирует и с амфотерным гидроксидом алюминия, и с металлическим алюминием: КОН + AI(OH)3= K[AI(OH)J 2КОН + 2AI + 6Н2О = 2К[А1(ОН)4] + ЗН2 2 5 Пример 9. Верны ли следующие суждения о гидроксидах? А. Гидроксид бериллия не относится к щелочам. Б. Гидроксид калия может реагировать и с углекислым, и с угарным газом. 1) верно только А 2) верно только Б 3) верны оба суждения 4) оба суждения неверны Бериллий — элемент IIA-группы, однако к щелочноземельным металлам он не относится (слишком велика разница в свойствах бериллия и других эле- ментов группы). Из-за малого ионного радиуса бериллия его гидроксид не только проявляет наиболее слабые основные свойства в ряду Ве(ОН)2 — Ra(OH)2, но и является амфотерным. Таким образом, Ве(ОН)2 к щелочам не относится. Первое утверждение верно. Что касается гидроксида калия, то он, являясь сильным основанием (щёлочью), легко взаимодействует с угле- кислым газом, проявляющим кислотные свойства: 2КОН + СО2 = К2СО3 + Н2О Угарный газ относится к несолеобразующим оксидам, что, казалось бы, ис- ключает возможность его взаимодействия с щёлочью. Однако это утвержде- ние справедливо только для водных растворов щелочей, взятых при обыч- ных условиях. В особых условиях СО реагирует со щёлочью с образованием формиатов (солей муравьиной кислоты): КОН + СО НСООК Ответ: 3
82 Пример 10. Установите соответствие между исходными веществами и алюминийсодержащим продуктом их взаимодействия. ПРОДУКТ ВЗАИМОДЕЙСТВИЯ 1) NaAIO2 2) Na[AI(OH)J 3) KAIO2 4) K[AI(OH)J ИСХОДНЫЕ ВЕЩЕСТВА A) AI(OH)3+ NaOH(p-p) -> Б) AI(OH)3+Na2O — В) AI(OH)3+KOH— Г) AI(OH)3 + Na2CO3 —► Гидроксид алюминия — типичный амфотерный гидроксид. В реакциях с веществами оснбвной природы в зависимости от условий могут получать- ся различные продукты. В первой реакции гидроксид алюминия, проявляя свои амфотерные свойства, растворяется в водном растворе щёлочи с образованием тетрагидроксо- или гексагидроксоалюмината натрия (при из- бытке щёлочи). Подобным образом с щелочами реагируют все амфотерные гидроксиды: Я 1 AI(OH)3+NaOH(p.p) —> Na[AI(0H)4] Во второй реакции, происходящей при нагревании сухих реагентов, об- разуется оксоалюминат (метаалюминат) натрия: 2AI(OH)3+ Na2O = 2NaAIO2 + ЗН2О Аналогичный продукт образуется в третьей и четвёртой реакциях: А1(ОН)3 + КОН = КАЮ2 + 2Н2О 2AI(OH), + Na2CO, = 2NaAIO2 + ЗН2О + СО2 * Г 0 fc О С* & С- Ответ: А Б в г 2 1 3 1 Пример 11. Гидросульфат натрия в водном растворе реагирует 1) с КОН 2) с H2SO4 3) с Fe 4) с СН3СООН 5) с K2SO4 Гидросульфат натрия NaHSO4 относится к кислым солям. Эта соль образу- ется при неполной нейтрализации сильной серной кислоты щёлочью, сле- довательно, атом водорода, входящий в состав гидросульфат-иона, доволь- но кислотен. Это определяет свойства гидросульфата натрия — его раствор должен реагировать со щёлочью и растворять такие металлы, как железо: 2NaHSO4 + 2КОН = Na2SO4 + K2SO4 + 2Н2О 2NaHSO4 + Fe = Na2SO4 + FeSO4 + H2 Ответ: 3
83 ЗАНЯТИЕ 9 Свойства галогенов и их соединений Основные элементы содержания: свойства простых веществ — галогенов и их соединений: галогеноводородов, оксидов, кисло- родсодержащих кислот, солей. Общая характеристика галогенов Элементы VIIA-группы — фтор, хлор, бром, иод и астат — называют галогенами (от греческого «рождающие соли»). Действительно, фтор, хлор, бром и иод встре- чаются в природе практически только в составе солей, которые содержатся как в земной коре, так и в морской воде. Астат чрезвычайно редкий элемент, поскольку имеет только очень нестабильные радиоактивные изотопы. В земной коре астат присутствует лишь в следовых коли- чествах. Поэтому свойства астата изучены слабо, и далее мы его рассматривать не будем. Атомы галогенов имеют одинаковое строение внешнего электронного слоя ns2nps, т. е. являются электронными аналогами, чем и объясняется общность их свойств. Все галогены обладают высокой электроотрицательностью (табл. 10), а фтор является наиболее электроотрицательным среди всех элементов. Атомы га- логенов легко присоединяют электрон, образуя отрицательно заряженные ионы, обладающие устойчивой электронной конфигурацией соответствующего благород- ного газа ns2np6. Поэтому для всех галогенов наиболее устойчива степень окис- ления -1. В ряду галогенов от фтора к астату увеличиваются радиусы атомов, уменьшается электроотрицательность, ослабевают неметаллические свойства и окислительная способность простых веществ. Фтор — самый электроотрицательный элемент, поэтому он проявляет в соеди- нениях только степень окисления -1. Важнейшие соединения фтора — это фторо- водородная (плавиковая) кислота HF и её соли — фториды. Соединение фтора с кислородом OF2 называют фторидом кислорода, поскольку именно фтор находится в отрицательной степени окисления, а кислород — в степени окисления +2. Таблица Ю Электронное строение и характеристики атомов галогенов3 Атом Строение внешнего электронного слоя Характерные степе- ни окисления Радиус атома, нм Электроотрица- тельность F 2s22p5 -1. о 0,058 4,0 CI 3s23p5 -1, 0, +1, +3, +5, +7 0,099 3,2 Вг 4s24p5 -1, 0, +1, +3, +5, +7 0,114 3,0 I 5s25p5 -1, 0, +1, +3, +5, +7 0,133 2,7 3 Данные взяты в справочнике Дж. Эмсли «Элементы» (M.: Мир, 1993).
84 Хлор, бром и иод, кроме наиболее устойчивой степени окисления -1, образуют соединения и в положительных степенях окисления, преимущественно в нечётных: +1, +3, +5, +7. В основном это кислородсодержащие соединения — оксиды, кис- лоты и соответствующие им соли (табл. 11). Известны также оксиды галогенов в степенях окисления +4 и +6. Таблица 11 Важнейшие оксиды, кислоты и соли, образованные хлором Степень окисления —1 4-1 +3 +5 +7 Оксиды CI2O — — CI2O7 Кислоты HCI хлороводородная нею хлорноватистая нсю2 хлористая нсю3 хлорноватая нсю4 хлорная Соли NaCI хлориды NaCIO гипохлориты NaCIO2 хлориты NaCIO3 хлораты NaCIO„ перхлораты Простые вещества — галогены Молекулы простых веществ — галогенов двухатомны. Атомы связаны между со- бой ковалентной неполярной связью, и молекулы галогенов в целом неполярны. Поэтому галогены хорошо растворяются в неполярных органических растворите- лях. В полярном растворителе — воде — галогены растворяются плохо. При 20 °C концентрация насыщенного водного раствора хлора составляет всего 0,63 %, а брома — 3,6 %. Соответствующие растворы называют хлорной и бромной водой. Растворимость в воде иода ещё более низкая, однако она существенно повышает- ся при добавлении в раствор иодида калия, полученный раствор называют иодной водой. Водный раствор фтора получить нельзя, так как фтор энергично реагирует с водой с выделением кислорода. В твёрдом состоянии галогены имеют молекулярную кристаллическую решётку. Молекулы удерживаются между собой слабыми межмолекулярными взаимодей- ствиями, поэтому все галогены — летучие вещества (табл. 12). Галогены имеют резкий неприятный запах и чрезвычайно ядовиты. Таблица 12 Характеристики простых веществ — галогенов Галоген Состояние при обычных условиях L, “С ПЛ’ t , °C кил’ Примечание F, желтоватый газ -219 -188 Активно реагирует почти со всеми веще- ствами, разрушает стекло, воду Cl2 жёлто-зелёный газ, легко сжижается -101 -34 Влажный хлор отбеливает, обладает обез- зараживающими свойствами Вг2 бурая жидкость -8 58 Бромная вода является реактивом на на- личие кратных связей в молекулах орга- нических веществ к тёмно-серые кристал- лы, пары фиолетовые 114 184 Возгоняется (переходит из твёрдого со- стояния в газообразное, минуя жидкое); даёт синюю окраску с крахмалом
85 Химические свойства галогенов Химическая активность галогенов чрезвычайно высока. Все они являются силь- ными окислителями. Галогены энергично реагируют с металлами и большинством неметаллов, способны окислять многие сложные вещества. Химическая активность и окислительная способность галогенов снижается от фтора к иоду (схема 7). Фтор является самым активным неметаллом, наиболее сильным окислителем среди всех веществ. Он обладает чрезвычайной химической активностью, энергич- но реагирует с большинством простых и сложных веществ. Непосредственно фтор не реагирует лишь с лёгкими благородными газами — гелием, неоном, аргоном. Реакции протекают с выделением большого количества теплоты и могут сопрово- ждаться взрывом: 2Fe + 3F2= 2FeF3 H2+F2=2HF (co взрывом) Si + 2F2=SiF4 Характерная особенность фтора — способность окислять воду (пары воды могут гореть в атмосфере фтора): 2Н2О + 2F2 = 4HF +О2 Реагируя с диоксидом кремния, фтор разру- шает стекло: SiO2 + 2F2 = Si F4 + O2 Фтор — единственное вещество, которое взаимодействует с тяжёлыми благородными газами (Хе, Кг, Rn): Хе + F2 = XeF2 Работа со фтором чрезвычайно опасна, по- этому на практике его почти не используют, однако соединения фтора применяют в самых Схема 7 Галогены - активные неметаллы и сильные окислители cd 3 6 д о ф 3 к 3 ф н б Ч о S К о F2 + 2ё —► 2F" С12 + 2ё —► 2СГ Вг2 + 2ё —► 2Вг“ 12 + 2ё —2Г разнообразных целях. Хлор — второй по активности галоген. В обычных условиях или при слабом на- гревании он реагирует с большинством простых веществ, как металлов, так и не- я 8 металлов, выступая в роли окислителя: Н2 + С12=2НС1 2Fe + 3CI2= 2FeCI3 О2 + С12 * Реакция хлора с водородом начинается на свету и может сопровождаться взры- вом. С кислородом хлор непосредственно не взаимодействует, оксиды хлора по- лучают косвенным путём. Среди реакций со сложными веществами наиболее важны взаимодействие хло- ра с водой и растворами щелочей. Хлор немного растворяется в воде, его водный раствор называют хлорной во- дой. Часть растворённого хлора вступает с водой в обратимое взаимодействие с образованием хлороводородной и хлорноватистой кислот: о -1 +1 CI2+H2O^HCI +нею
86 В этой реакции хлор является и окислителем, и восстановителем. Таким обра- зом, этот процесс относится к реакциям самоокисления-самовосстановления, или диспропорционирования. Образующаяся хлорноватистая кислота очень неустойчи- ва и постепенно разлагается, выделяя атомарный кислород: НСЮ = HCI + [О] Именно выделением атомарного кислорода в этом процессе объясняются от- беливающие свойства влажного хлора (сухой хлор не отбеливает). Хлор вступает в реакции диспропорционирования и при взаимодействии с рас- творами щелочей, причём состав образующихся продуктов зависит от температу- ры процесса. При охлаждении или комнатной температуре образуются хлориды и гипохлориты: Cl2 + 2КОН = KCI + КСЮ + Н2О Если же хлор пропускать в раствор щёлочи, нагретый до 60—90 °C, продуктами реакции будут хлориды и хлораты: 3CI2+ 6NaOH = 5NaCI+ NaCIO3+ ЗН2О Хлор как более сильный окислитель вытесняет менее активные галогены из их соединений: С12 + СаВг2 = Вг2 + СаС12 Cl2 + 2HI = 12 + 2HCI Работать с хлором необходимо с чрезвычайной осторожностью, так как вдыха- ние даже небольших его количеств вызывает удушье и тяжёлое отравление. Хлор традиционно применяют для обеззараживания питьевой воды, отбеливания тканей и бумаги. В настоящее время от использования хлора в этих целях постепенно от- казываются, заменяя его другими, менее опасными реагентами. Химические свойства брома и иода аналогичны свойствам хлора. Они также вступают в реакции с металлами и неметаллами, растворами щелочей, выступают как окислители в реакциях со сложными веществами, например: H2S + Br2=2HBr + S4< 3I2 + 6NaOH = 5Nal + NalO3 + 3H2O Тем не менее реакционная способность и окислительные свойства брома и осо- бенно иода ниже, чем у хлора. Снижение химической активности в ряду F2->CI2-> -> Вг2-> 12 можно проиллюстрировать на примере реакции галогенов с водородом. Обратите внимание на то, как снижаются тепловые эффекты и меняются условия и признаки реакции: ^2(г)+ ^2(D = 2HF(r) + 547 кДж (со взрывом даже при охлаждении) ^2(г)+ С12(г)= 2НС1(Г) + 185 кДж (со взрывом на свету) ВГ2(г) + Н2(г)= 2НВГ(г) + 73 КД* (протекает при нагревании) 12(кР)+ Н2 i 2HI - 53 кДж (обратимая эндотермическая реакция)
87 Иод является самым слабым окислителем среди галогенов. В отличие от осталь- ных галогенов иод не способен, например, при взаимодействии с железом пере- вести его в степень окисления +3, а образует только иодид железа(И) Fel2. Анало- гично не существует иодида двухвалентной меди — образуется только Cui. Рассмотрим способы получения галогенов. Единственно возможный способ получения фтора — электролиз расплавов фторидов металлов. Обычно в промыш- ленности проводят электролиз расплава KF в присутствии HF, фтор при этом вы- деляется на аноде. Хлор можно получить электролизом как растворов, так и расплавов хлоридов. В промышленности его получают обычно электролизом концентрированного водного раствора NaCI. Одновременно с хлором (на аноде) получают водород (на катоде); в растворе остаётся гидроксид натрия: 2NaCI + 2Н2О -^.^.ролиз^ + 2NaOH + С12Т катод анод Лабораторные способы получения хлора основаны на окислении хлорид-иона такими сильными окислителями, как перманганат калия, дихромат калия, оксид марганца(1\/), в кислой среде. В качестве исходного вещества чаще всего исполь- зуют концентрированную соляную кислоту: MnO2 + 4HCI = МпС12 + С12 + 2Н2О 2КМпО4 + 16HCI = 5С12Т + 2МпС1г + 2KCI + 8Н2О K2Cr2O7 + 14HCI = 3CI2T + 2CrCI3 + 2KCI + 7Н2О Бром и иод извлекают из морских или буровых вод, вытесняя их газообразным хлором из соответствующих галогенидов: Cl2+2NaBr = Br2+2NaCI Cl2+2Nal = l2+2NaCI Соединения галогенов с водородом Из всех соединений галогенов наибольшее значение имеют галогеноводороды и соли галогеноводородных кислот. Химическая связь в молекулах галогеноводородов ковалентная полярная. По- лярность связи убывает в ряду HF -» HCI -> НВг -> HI. При комнатной температуре все галогеноводороды — бесцветные газы с рез- ким запахом. Благодаря полярности молекул газообразные галогеноводороды, в отличие от самих галогенов, очень хорошо растворяются в воде. Образующиеся при этом растворы собственно и представляют собой соответствующие галоге- новодородные кислоты: фтороводородную (плавиковую), хлороводородную (соля- ную), бромоводородную и иодоводородную. Например, в одном литре воды при 20 °C растворяется около 400 л газообразного HCI. Массовая доля хлороводорода в полученном растворе будет около 37 %. Такой раствор называют концентриро- ванной соляной кислотой. Химические свойства галогеноводородов обусловлены тем, что: • водные растворы HF, HCI, НВг и HI проявляют свойства кислот, • галогеноводороды и соли соответствующих им кислот проявляют восстанови- тельные свойства за счёт атома галогена, находящегося в низшей степени окисления.
88 Кислотные свойства галогеноводородных кислот усиливаются в ряду HF -> -> HCI -> НВг -> HI. Фтороводородная кислота является слабой, хлороводородная, бромоводородная и иодоводородная кислоты относятся к сильным кислотам. Эти вещества проявляют все общие свойства кислот: диссоциируют в водном раство- ре с образованием катиона водорода, реагируют с металлами, оснбвными и ам- фотерными оксидами, основаниями, вступают в реакции ионного обмена с раство- рами солей. Примеры реакций: Zn + 2HBr = ZnBr2 + Н2Т Fe2O3 + 6HCI = 2FeCI3 + ЗН2О Са(ОН)2 + 2HF = CaF-Д + 2Н2О AgNO3 + HCI = AgCli + HNO3 NaHCO3 + HI = Nal + H2O + CO2T 0 fc Восстановительные свойства галогеноводородов также усиливаются в ряду HCI -> НВг -> HI (схема 8). Фторид-ионы образованы наиболее электроотрицательным элементом — фто- ром. Поэтому среди химических веществ нет окислителей настолько сильных, что- бы быть способными оторвать электрон у иона F". Фторид-ионы проявляют вос- становительные свойства только в процессе электролиза (см. получение фтора). Соляная кислота и её соли отдают электроны только под действием наиболее сильных окислителей — перманганата калия, дихромата калия и некоторых других (см. лабораторные способы получения хлора). Усиление восстановительных свойств галогеноводородов можно проиллюстри- ровать следующими реакциями: CaF2 + Н28О4(конц.) = 2HF? + CaSOj (1) NaCI + Н28О4(конц.) = HClT + NaHSO4 (2) 2HBr + Н28О4(конц.) = Br2+ SO2T+ 2H2O (3) 8HI + Н2ЗО4(конц.) = 4I2 + H2ST + 4H2O (4) Схема 8 Реакции концентрированной серной кисло- ты, обладающей выраженными окислительны- ми свойствами, с фторидами и хлоридами не сопровождаются изменением степеней окис- ления. Более того, в лаборатории HF и HCI получают именно взаимодействием твёрдых галогенидов с концентрированной H2SO4 (реак- ции 1 и 2). Бромоводород и иодоводород не удаётся получить подобным образом, посколь- ку Вг' и Г уже способны окисляться серной кислотой до свободных галогенов (реакции 3 и 4). Иодид-ион как наиболее сильный восста- новитель обеспечивает максимальную глубину восстановления серной кислоты — до H2S. Бромид-ионы окисляются не только концентрированной серной кислотой, но и многими другими типичными лабораторными окислителями, например дихроматом калия: К2Сг2О7 + 6КВг + 7H2SO4 = Cr2(SO4)3 + 3Br2 + 41^804 + 7Н2О
89 Иодоводородная кислота и её соли проявляют наиболее выраженные восста- новительные свойства. Они медленно окисляются даже под действием кислорода воздуха: 4HI + О2= 21г+2Н2О Протеканием этого процесса объясняется появление желтоватой окраски дли- тельно хранившихся растворов иодидов. Иодид-ион достаточно сильный восстановитель для того, чтобы восстановить Fe(lll) до Fe(ll) и Cu(ll) до Си(1): 2HI + Fe2(SO4)3 = 2FeSO4 + 12Ф + H2SO4 2CuSO4 + 4KI = 2Си1Ф + U + 2K?SO4 Фтороводород и плавиковая кислота обладают рядом особенностей. Во-первых, в отличие от молекул остальных галогеноводородов молекулы HF связаны между собой водородными связями: Н—р-н—F Поэтому температура кипения HF достаточно высока — почти 20 °C. Ниже этой температуры фтороводород находится в жидком состоянии. Во-вторых, HF взаимодействует с оксидом кремния(1\/), входящим в состав стекла: 4HF + SiO2= SiF4T + 2Н2О Поэтому для работы с плавиковой кислотой нельзя использовать стеклянную по- суду. При работе с HF используют посуду из полиэтилена или тефлона. Работать с плавиковой кислотой нужно с большой осторожностью, так как она способна вы- звать тяжёлые ожоги. В промышленном отношении наиболее важными являются хлороводород и со- ляная кислота. В промышленности хлороводород получают в основном сжиганием водорода в хлоре: Н2+С12=2НС1 Лабораторный способ получения хлороводорода заключается в действии кон- центрированной серной кислоты на твёрдые хлориды при 150—200 °C: NaCI(TB) + Н2ЗО4(конц.) = NaHSO4 + HClT Следует обратить внимание на условия проведения этой реакции. Данная ре- акция является реакцией ионного обмена и подчиняется общим закономерностям, необходимым для их протекания. Поскольку осадка или слабого электролита здесь образоваться не может, необходимо получить хлороводород обязательно в газоо- бразном состоянии. Поэтому используют твёрдый хлорид натрия и концентриро- ванную серную кислоту, а не их водные растворы. В противном случае образую- щийся хлороводород будет растворяться в воде и не выделится в виде газа. При взаимодействии нитрата серебра с растворами, содержащими ионы CI-, Вг и I- образуются осадки галогенидов серебра белого, желтоватого и жёлтого цвета соответственно: Ag+ + Cl- = AgCl4< (белый) Ад" + Вг = АдВг4< (светло-жёлтый) Ад* + Г = Ад|ф (жёлтый)
90 Эти взаимодействия являются качественными реакциями на галогенид-ионы. В отличие от остальных галогенидов фторид серебра растворим в воде. Кислородсодержащие соединения хлора Хлор образует ряд кислородсодержащих соединений — оксидов, кислот и со- лей (табл. 12). Во всех этих соединениях хлор находится в положительных степе- нях окисления. Положительные степени окисления для хлора неустойчивы. Поэтому кисло- родсодержащие соединения хлора : • проявляют свойства окислителей', • термически нестабильны, т. е. способны разлагаться при нагревании. Наиболее устойчивыми из оксидов хлора являются С12О, СЮ2 и С12О7. Все ок- сиды хлора являются кислотными. При реакциях с водой С12О и С12О7 образуют соответствующие кислоты: CI2O + Н2О = 2HCIO С12О7+Н2О = 2НСЮ4 В реакциях с растворами щелочей образуются соли этих кислот: CI2O + 2КОН = 2KCIO + Н2О С12О7 + Са(ОН)2 -> Са(СЮ4)2 + Н2О При взаимодействии СЮ2 с водой протекает реакция диспропорционирования: образуются хлористая и хлорноватая кислоты: 2СЮ2 + Н2О = НСЮ2 + НСЮ3 В ряду кислородсодержащих кислот хлора (схема 9) сила кислот увеличивается от хлорноватистой кислоты к хлорной. Хлорноватистая и хлористая кислоты сла- бые, а хлорноватая и хлорная сильные. Кислородсодержащие кислоты и соли хлора являются сильными окислителями. Обычно в ходе окислительно-восстановительной реакции атом хлора переходит в наиболее устойчивую степень окисления -1: НСЮ + 2Н1 = НС1 + 12+Н2О NaCIO3 + 6KI + 3H2SO4 = NaCI + 3I2 + 3K2SO4 + ЗН2О
91 Хлорат калия КСЮ3 называют также бертолетовой солью. Это сильный окис- литель, смеси КСЮ3 с восстановителями (сахарной пудрой, красным фосфором) вспыхивают при механическом воздействии: трении, ударе и т. д. Например, при зажигании спички протекает реакция 5КСЮ3 + 6Р = 5KCI + ЗР2О5 Бертолетова соль разлагается при нагревании (катализатор — МпО2) с выделе- нием кислорода: 2КСЮ3 = 2KCI + ЗО2 Напомним, что хлор не соединяется непосредственно с кислородом, поэтому его кислородсодержащие соединения получают косвенным путём. В частности, гипохлориты и хлораты можно получить при взаимодействии хлора с растворами щелочей. Примеры заданий, при выполнении которых нужно продемонстрировать знание свойств галогенов и их соединений Пример 1. Установите соответствие между формулой соли и отношением этой соли к гидролизу. СРЕДА РАСТВОРА ФОРМУЛА СОЛИ A) NaF Б) Cal2 В) NH4CIO4 Г) КСЮ 1) гидролизуется по катиону 2) гидролизуется по аниону 3) гидролизу не подвергается 4) гидролизуется по катиону и аниону При выполнении этого задания нужно наряду с пониманием сущности гидро- лиза солей продемонстрировать знание особенностей свойств галогеново- дородных кислот и кислородсодержащих кислот хлора: необходимо вспом- нить, какие из них являются сильными, а какие — слабыми. Рассмотрим отношение к гидролизу каждой из солей. Фторид натрия об- разован сильным основанием NaOH и слабой фтороводородной кислотой. Гидролиз таких солей протекает по аниону. Иодид кальция образован и сильным основанием, и сильной иодоводород- ной кислотой. Эта соль гидролизу не подвергается. |h Перхлорат аммония NH4CIO4 образован слабым основанием NH3 • Н2О, но сильной хлорной кислотой. Гидролиз этой соли будет протекать по катиону. Гипохлорит калия образован сильным основанием и слабой хлорноватистой кислотой. Значит, гидролиз будет протекать по аниону. Ответ:
Пример 2. Установите соответствие между веществами и продуктами их взаимодействия с водным раствором гидроксида калия. ВЕЩЕСТВО ПРОДУКТЫ ВЗАИМОДЕЙСТВИЯ С РАСТВОРОМ КОН А) Cl2 Б) С12О В) С12О7 Г) НСЮ4 1) КСЮ + Н2О 2) КСЮ3 + Н2О 3) КС1 + КСЮ + Н2О 4) КСЮ4+Н2О 5) КС1 + КСЮ4 + Н2О Чтобы выполнить это и подобные задания без ошибок, настоятельно реко- мендуем составлять уравнения рассматриваемых реакций и расставлять ко- эффициенты. Хлор в растворах щелочей диспропорционирует с образовани- ем хлоридов и гипохлоритов (при охлаждении или при комнатной темпера- туре): Cl2 + 2КОН = KCI + КСЮ + Н2О Оксиды С12О и С12О7 являются кислотными, им соответствуют хлорноватистая кислота НСЮ и хлорная кислота НСЮ4. В реакциях с растворами щелочей образуются соли этих кислот: С12О + 2КОН = 2КСЮ + Н2О С12О7 + 2КОН = 2КСЮ4 + Н2О При взаимодействии хлорной кислоты с гидроксидом калия образуются перхлорат калия и вода: НСЮ4 + КОН = КСЮ4 + Н2О Ответ: Пример 3. Хлорат калия нагрели в присутствии катализатора. Получен- ную в результате соль растворили в воде и подвергли электролизу. На ано- де выделился жёлто-зелёный газ, который пропустили через раствор иоди- да натрия. Образовавшееся в результате этой реакции простое вещество прореагировало при нагревании с раствором гидроксида калия. Напишите уравнения четырёх описанных реакций. При выполнении подобных заданий наряду с пониманием основных законо- мерностей протекания различных типов реакций потребуется продемонстри- ровать и знание некоторых особенностей свойств соединений галогенов. Так, хлорат калия, как и другие кислородсодержащие соли галогенов, тер- мически нестабилен и разлагается при нагревании. В ходе соответствующей окислительно-восстановительной реакции атом хлора восстанавливается до наиболее устойчивой для него степени окисления -1, образуется хлорид ка- лия и выделяется кислород:
93 аж ли. 2KCI0, = 2KCI + ЗО2 о С Электролиз водных растворов хлоридов применяют для получения галогенов 2KCI + 2Н2О = 2КОН + Н2Т + С12Т (С12 — жёлто-зелёный газ) Хлор является более активным неметаллом и более сильным окислителем, чем иод, поэтому способен вытеснить его из иодида натрия: 2Nal + Cl2 = 2NaCI + 12 (12 — простое вещество) Наконец, иод, так же как хлор и бром, диспропорционирует в растворах ще лочей: 6КОН + 312 —*-► 5KI + КЮ3 + ЗН2О Пример 4. Твёрдый хлорид калия нагрели с концентрированной серной кислотой, при этом выделился газ, который растворили в воде. При взаимо- действии полученного раствора с перманганатом калия образовалось про- стое газообразное вещество жёлто-зелёного цвета. При горении железной проволоки в этом веществе получили соль. Соль добавили к раствору ни- трата серебра, образовался осадок. Напишите уравнения четырёх описан- ных реакций. Нагревание твёрдых хлоридов с концентрированной серной кислотой — это лабораторный способ получения хлороводорода: КС1(тв.) + Н2ЭО4(конц.) = НОТ + KHSO4 можно написать и другое уравнение: 2КС1(тв.) + Н2ЭО4(конц.) = 2HCIT + K2SO4 так реакция протекает при более сильном нагревании. При растворении хлороводорода в воде образуется соляная кислота. Хими- ческих реакций при этом не происходит, следовательно, уравнение реакции писать не нужно. При взаимодействии перманганата калия с концентриро- ванной соляной кислотой выделяется хлор (см. лабораторные способы полу- чения хлора): 2КМпО4 + 16HCI = 2МпС12+ 5CI2+ 2KCI + 8Н2О Если внести нагретую железную проволоку в сосуд с хлором, начинается реакция горения железа в хлоре, сопровождающаяся выделением большого количества бурого дыма, состоящего из частичек хлорида железа(Ш). Урав- нение реакции: 2Fe + 3CI2 = 2FeCI3 Наконец, хлорид железа(Ш) вступает в реакцию ионного обмена с нитратом серебра с образованием белого осадка хлорида серебра: FeCI3 + 3AgNO3 = 3AgCI^ + Fe(NO3)3
94 ЗАНЯТИЕ 10 Неметаллы VIA-группы: кислород, сера Основные элементы содержания: общая характеристика кислорода и серы в связи с их положением в Периодической системе Д. И. Менделеева; кислород и озон; особенности окислительно-восстановительных свойств пе- роксида водорода; сера и её соединения: сероводород, оксид cepbi(IV) и оксид серы(У1), сер- нистая и серная кислоты, сульфиды, сульфиты и сульфаты; общие научные принципы химиче- ского производства на примере производства серной кислоты. Общая характеристика элементов VIA-группы Кислород, сера, а также селен и теллур — первые четыре элемента VIA-группы, имеющие неметаллический характер, — носят групповое название «халькогены», что в переводе с греческого означает «рождающие медь». Последний элемент группы — полоний — является металлом. Он радиоактивен и в природе практи- чески не встречается. Атомы элементов VIA-группы имеют сходное электронное строение (табл. 13). На внешнем энергетическом уровне этих атомов содержится по шесть электронов. И атомы кислорода, и атомы серы способны принимать электроны и достраивать внешнюю электронную оболочку до восьмиэлектронной. При этом достигается низшая степень окисления -2. Сера проявляет эту степень окисления в соедине- ниях с водородом и металлами. Для кислорода степень окисления -2 наиболее устойчива, он проявляет её в большинстве соединений. Однако из этого правила есть исключения. Так, в со- единениях O2F2 и OF2 с более электроотрицательным элементом фтором степень окисления кислорода равна соответственно +1 и +2. А в пероксидах, например в Н2О2 и Na2O2, его степень окисления равна -1. Для кислорода, как и для фтора, высшая степень окисления, равная номеру группы, не реализуется. А вот для серы характерны и положительные степени окисления: высшая степень окисления +6, равная номеру группы, а также степень окисления +4. Таблица 13 Электронное строение и характеристики атомов кислорода и серы Электронное строение Характерные степени окисления Радиус атома, нм Электроотрица- тельность О 1s22s22p* -2, (-1)(+1), (+2) 0,066 3,44 S 1s22s22p63s^3pj -2, (+2), +4, +6 0,104 2,58 КИСЛОРОД И ЕГО СОЕДИНЕНИЯ Как элемент кислород входит в состав очень многих соединений (воды, ок- сидов, гидроксидов, солей) и является самым распространённым элементом на Земле. Он образует два простых вещества — кислород О2 и озон О3. Объём-
95 ное содержание О2 в составе атмосферного воздуха составляет примерно 21 %. Озон в природе находится в основном в одном из верхних слоёв атмосферы — стратосфере — на высоте 15—30 км от земной поверхности, об- разуя так называемый озоновый слой. Озон поглощает жёсткое ультрафиолето- вое излучение Солнца, предохраняя живые организмы от его губительного дей- ствия. Свойства и способы получения кислорода В обычных условиях кислород представляет собой газ без цвета, вкуса, запа- ха, плохо растворимый в воде. Только при очень низких температурах, близких к -200 °C, он конденсируется в голубую жидкость, а затем затвердевает. В твёрдом состоянии кислород имеет молекулярную кристаллическую решётку. Кислород химически активен. Он реагирует с большинством простых веществ и многими сложными веществами, выступая в роли окислителя. Характерной осо- бенностью многих реакций с его участием является выделение большого количе- ства теплоты и света. Продуктами реакций, как правило, являются оксиды: 2Са + О2 = 2СаО Si + О2 = SiO2 4Li + О2 = 2Li2O Щелочные металлы (кроме лития) взаимодействуют с кислородом с образова- нием пероксидов4; 2Na + O2=Na2O2 Напрямую не соединяются с кислородом лишь галогены, серебро, золото и не- которые другие вещества. Тем не менее оксиды галогенов, серебра и золота су- ществуют, их можно получить косвенным путём. При горении в кислороде или на воздухе сложных веществ, как правило, обра- зуются оксиды каждого из элементов, его образующих: СН4 + 2О2 = СО2 + 2Н2О 2РН3 +4О2= Р2О5+ЗН2О 2ZnS + ЗО2 = 2ZnO + 2SO2 Одним из исключений из этого правила является аммиак, горение которого приводит к образованию азота и воды: 4NH3 +ЗО2= 2N2 +6Н2О Образование оксида азота(П) происходит только в присутствии катализатора, т. е. не при горении, а при каталитическом окислении аммиака: 4NH3+5O2 = 4NO + 6H2O Получить кислород в лаборатории можно термическим разложением перманга- ната калия, хлората калия (бертолетовой соли), пероксида водорода: 2КМпО4 = К2МпО4 + МпО2 + О2Т 4 При взаимодействии калия, рубидия и цезия с кислородом, кроме пероксидов, образуются надпероксиды, например КО2.
96 2KCIO3 = 2KCI + ЗО2Т 2Н2О2 = 2Н2О + 02Т В промышленности 02 получают сжижением и последующим испарением воз- духа. Первым из жидкого воздуха испаряется азот, а кислород остаётся жидким. Кислород хранят в баллонах голубого цвета. Его применяют как окислитель в ме- таллургии, химической промышленности, при газовой сварке и резке металлов, в составе дыхательных смесей в медицине и т. п. Свойства и способы получения озона Озон при обычных условиях представляет собой газ с резким неприятным за- пахом. При сильном охлаждении он переходит в тёмно-синюю жидкость, а затем в фиолетово-чёрные кристаллы с молекулярной кристаллической решёткой. Озон — сильнейший окислитель. Реакционная способность озона значительно выше, чем у кислорода. В отличие от кислорода он активно взаимодействует с серебром при комнатной температуре, окисляет сульфиды до сульфатов. Обра- тите внимание на то, что в реакциях с участием озона обычно выделяется кис- лород: 2Ад + О3 = Ад2О + О2 PbS + 4О3 = PbSO4 + 4О2 При длительном хранении или нагревании озон разрушается с образованием кислорода: 2О3=ЗО2 Получают озон в специальных приборах — озонаторах путём перевода О2 в О3 под действием электрических разрядов: ЗО2 2О3 Поскольку данная реакция обратима, достигнуть 100%-ного перевода кислорода в озон никогда не удаётся. Озон может образоваться во время грозы при разряде молнии. Появление та- ких небольших количеств озона в воздухе способствует его очищению, поэтому после грозы обычно появляется ощущение свежести. Однако следует помнить, что в более высоких концентрациях озон чрезвычайно токсичен из-за своей высокой окислительной способности. Озон используют для дезинфекции помещений, в том числе в медицинских уч- реждениях, где небольшие количества этого газа получают при работе кварцевых ламп. Поскольку при разложении озона не образуется вредных продуктов, при обеззараживании воды на станциях водоподготовки или в бассейнах всё чаще хлорирование заменяют озонированием. Свойства пероксида водорода В молекуле Н2О2 кислород находится в степени окисления -1. При этом валент- ность кислорода в этом соединении равна II, так как атомы кислорода связаны друг с другом, а также каждый атом кислорода образует связь с атомом водорода: Н—О—О—Н
07 Благодаря тому что кислород находится в промежуточной степени окисления, пероксид водорода в окислительно-восстановительных реакциях может выступать в роли как окислителя, так и восстановителя. Однако его окислительные свойства выражены гораздо сильнее. Именно в качестве окислителей чаще всего использу- ют растворы Н2О2 в лабораторной практике: 2KI + Н2О2 + H2SO4 = 12 + K2SO4 + 2Н2О 2Fe(OH)2 + Н2О2 = 2Fe(OH)3 Однако при действии ещё более сильных, чем он сам, окислителей Н2О2 мо- жет проявлять восстановительные свойства. В таких реакциях выделяется кисло- род: 2КМпО4 + 5Н2О2 + 3H2SO4 = 2MnSO4 + 5О2 + K2SO4 + 8Н2О Н2О2 + Cl2 = 2HCI + О2 Степень окисления -1 для кислорода не является устойчивой. Поэтому при дли- тельном хранении, нагревании или действии катализатора пероксид водорода раз- лагается на воду и кислород. Использование пероксида водорода связано с его окислительными свойствами: он применяется для отбеливания тканей и бумаги, дезинфекции различных по- верхностей, тары в пищевой промышленности и т. п. Благодаря антисептическим свойствам используется в медицине для промывания небольших поверхностных ран. Обратите внимание на то, что для этих целей можно использовать растворы с концентрацией не выше 3 %, так как более концентрированные растворы из-за окислительных свойств Н2О2 могут вызвать ожоги. СЕРА И ЕЁ СОЕДИНЕНИЯ Для серы наиболее характерно образование соединений в степенях окисления -2, +4 и +6 (табл. 14). Существуют соединения, в которых степень окисления серы равна -1 (пирит, или колчедан, FeS2), +1 (S2CI2) и +2 (SCI2). Таблица 14 Важнейшие соединения, образованные серой в различных степенях окисления -2 +4 +6 Сероводород H2S; гидросульфиды и суль- фиды: NaHS, K2S, FeS, AI2S3, (NH4)2S и др. Оксид cepbi(IV) SO2 (сернистый газ); сернистая кислота H2SO3; гидросульфиты и сульфиты: NaHSO3, K2SO3, (NH4)2SO3, CaSO3 и др. Оксид серы(\/1) SO3 (серный ан- гидрид); серная кислота H2SO4; гидросульфаты и сульфаты: KHSO4, Na2SO4, (NH4)2SO4, CaSO4, Fe2(SO4)3 и др. Сера как простое вещество При обычных условиях сера — кристаллическое легкоплавкое вещество жёлтого цвета. Сера существует в виде трёх аллотропных модификаций: ромбическая, мо- ноклинная и пластическая. Наиболее устойчивой при обычных условиях является
98 ромбическая сера. Кристаллы как ромбической, так и моноклинной серы построе- ны из коронообразных циклических молекул S8: zs\ s\s s^s~7s Таким образом, в твёрдом состоянии сера имеет молекулярную кристалличе- скую решётку. Если ромбическую серу расплавить и расплав быстро охладить, вылив его в холодную воду, образуется коричневая масса, напоминающая рези- ну, — пластическая сера. Дело в том, что при высокой температуре кольцеобраз- ные молекулы S8 разрываются и объединяются в более крупные молекулы, пред- ставляющие собой цепочки, которые могут содержать до нескольких тысяч атомов серы. По сути дела, пластическая сера представляет собой неорганический поли- мер, состав которого можно отразить формулой Sn. А вот в газообразном состоя- нии молекулы серы состоят из меньшего числа атомов: двух, четырёх или шести в зависимости от температуры. Сера достаточно химически активна и взаимодействует с большинством простых веществ: как с металлами, так и с неметаллами. С наиболее активными металла- ми (щелочными и щелочноземельными), а также с ртутью сера реагирует даже без нагревания, со многими другими металлами — при нагревании. Продуктами реакции являются сульфиды металлов: 2Na + S = Na2S t Fe + S = FeS 2AI + 3S = AI2S3 Сера горит на воздухе, образуя оксид cepbi(IV) — сернистый газ5: S + О2 = SO2 При нагревании сера реагирует с водородом, причём реакция обратима: S + Н2 H2S Сера взаимодействует и с остальными неметаллами, за исключением азота, иода и благородных газов. Так, при взаимодействии с галогенами образуются фто- рид cepbi(VI) SF6, хлориды и бромиды серы различного состава: S2CI2, SCI2, SBr2. При взаимодействии серы с углём образуется сероуглерод CS2, а с фосфором — сульфиды фосфора различного состава, например P2S5. Сложных веществ, с которыми взаимодействует сера, не так много. С во- дой, разбавленными кислотами, растворами большинства солей сера не реаги- рует. Концентрированной азотной кислотой сера окисляется до серной кислоты: S + 6НМО3(конц.) = H2SO4 + 6NO2+ 2Н2О При взаимодействии с концентрированной серной кислотой образуется диоксид серы: S + 2Н25О4(конц.) = 3SO2 + 2Н2О 5Обратите внимание на то, что оксид серы(\/1) SO3 в этой реакции практически не образуется, его получение возможно только в присутствии катализатора.
99 При кипячении в концентрированных растворах щелочей сера, как и галогены, диспропорционирует: 3S + 6КОН = 2K2S + K2SO3 + ЗН2О Сера встречается в природе в виде простого вещества, поэтому для промыш- ленных нужд организована добыча самородной серы. Кроме того, в промышленности серу получают из сероводорода, окисляя его сернистым газом (процесс Клауса): 2H2S + SO2 = 3S + 2Н2О или сжигая H2S при недостатке кислорода: 2H2S + О2 = 2S + 2Н2О Соединения серы в степени окисления -2: сероводород и сульфиды Сероводород H2S в обычных условиях — бесцветный газ с отвратительным за- пахом (тухлых яиц), очень ядовит. Сероводород плохо растворим: при обычных ус- ловиях в 1 л воды растворяется около 2,5 л H2S, что приводит к получению очень разбавленного раствора с концентрацией около 0,3 %. Химические свойства сероводорода определяются тем, что • его водный раствор представляет собой слабую кислоту, • за счёт атома серы, находящейся в низшей степени окисления, сероводород и проявляет восстановительные свойства. Раствор сероводорода в воде — сероводородная кислота — является очень слабой двухоснбвной кислотой. Из-за низкой растворимости сероводорода в воде получить её в концентрированном виде невозможно. Сероводородная кислота взаимодействует с оснбвными оксидами, основания- ми, вступает в реакции обмена с солями, например: Na2O + H2S = Na2S + Н2О КОН + Н25(изб.) = KHS + Н2О 2КОН(изб.) + H2S = K2S + 2Н2О CuSO4 + H2S = CuSJ- + H2SO4 H2S проявляет восстановительные свойства за счёт атома серы в степени окис- ления -2. Важнейшим примером окислительно-восстановительной реакции с участием се- роводорода является его взаимодействие с кислородом — горение: 2H2S + ЗО2 = 2SO2 + 2Н2О При недостатке кислорода при горении H2S вместо SO2 может образоваться сера: 2H2S + О2( недостаток) = 2S + 2Н2О При взаимодействии с растворами типичных окислителей — брома, иода, ди- хромата калия, перманганата калия — сероводород и растворимые сульфиды обычно окисляется до серы, например: H2S + Br2 = S + 2НВг 3H2S + K2Cr2O7 + 4H2SO4 = Cr2(SO4)3 + 3S + K2SO4 + 7H,0
100 А более сильные окислители — хлор, концентрированная азотная кислота — могут окислить сероводород до серной кислоты: H2S + 4С12 + 4Н2О = H2SO4 + 8HCI H2S + 8HNO3(KOHU.) = H2SO4+ 8NO2T + 4H2O Сероводород получают нагреванием серы с водородом: S + Н2 H2S или действием растворов кислот на некоторые сульфиды: FeS + 2HCI = H2St + FeCI2 ZnS + Н2ЗО4(разб.) = H2S? + ZnSO4 Сероводород образует два ряда солей — средние (сульфиды) и кислые (гидро- сульфиды). Сульфиды образуются при взаимодействии H2S с избытком щёлочи: 2КОН + H2S = K2S + 2Н2О Гидросульфиды образуются при избытке сероводорода: КОН + H2S = KHS + Н2О Растворимые сульфиды вступают в реакции ионного обмена, например: K2S + ZnSO4 = ZnS^ + K2SO4 K2S + 2HCI = H2ST + 2KCI Сульфиды щелочных металлов подвергаются гидролизу по аниону, поскольку они образованы катионами сильных оснований и анионом слабой кислоты. Их рас- творы имеют щелочную среду. Если же в состав сульфида входит катион слабого основания, то гидролиз протекает особенно глубоко. Например, сульфид алюми- ния подвергается полному гидролизу: ALS, + 6Н2О = 2А1(ОН),4- + 3H2ST При взаимодействии растворов сульфидов с растворами солей алюминия и хрома(Ш) протекает совместный гидролиз, например: 2AICL + 3Na2S + 6Н2О = 2АЦОН) J + 3H2ST+ 6NaCI Сульфиды проявляют восстановительные свойства за счёт атома серы в степени окисления -2. Растворимые сульфиды взаимодействуют с типичными лаборатор- ными окислителями с образованием серы, например: (NH4)2S + Br2=S + 2NH4Br 2КМпО4 + 3K2S + 4Н2О = 2МпО2 + 3S + 8КОН Нерастворимые сульфиды также могут выступать в роли восстановителей в окислительно-восстановительных реакциях. Практически важными среди таких реакций является обжиг сульфидов: 2ZnS + ЗО2 = 2ZnO + 2SO2T, а также их взаимодействие с горячей концентрированной азотной кислотой: Ag2S + 10НМО3(конц.) = 2AgNO3+ H2SO4+ 8NO2T + 4Н2О
101 Эти реакции применяются при переработке и анализе сульфидных руд. Обрати- те внимание на то, что металлы, находящиеся в составе сульфидов в низкой не- устойчивой степени окисления — Cu(+1), Fe(+2), — в этих процессах также окис- ляются до более устойчивых степеней окисления — Cu(+2), Fe(+3): 4FeS + 7О2 = 2Fe2O3 + 4SO2? Cu2S + 12Н1\Ю3(конц.) = Cu(NO3)2 + CuSO4 + 10NO2T + 6H2O Таким образом, в приведённых выше реакциях принимают участие один атом- окислитель — О или N(+5) и два атома-восстановителя — Си(+1) или Fe(+2) и S(-2). Соединения серы в степени окисления +4: оксид серы(1У), сернистая кислота и сульфиты Оксид cepbi(IV) (сернистый газ) — бесцветный газ с резким запахом. Запах SO2 можно почувствовать, например, при зажигании спички. Сернистый газ хоро- шо растворим в воде: при комнатной температуре в 1 объёме воды растворяется около 40 объёмов SO2. Оксид серы(1\/) проявляет все общие свойства, присущие кислотным оксидам: взаимодействует с водой, с основаниями и с оснбвными оксидами. При растворении SO2 в воде образуется сернистая кислота: SO2 + Н2О з=ь H2SO3 Будучи двухоснбвной, сернистая кислота образует два ряда солей: средние соли — сульфиты (например, Na2SO3, CaSO3) и кислые соли — гидросульфиты (например, NaHSO3, Ca(HSO3)2). При взаимодействии SO2 с растворами щелочей в зависимости от соотношения по количеству, в котором взяты компоненты, об- разуются либо средние, либо кислые соли: SO2 + 2МаОН(изб.) = Na2SO3 + Н2О ЭО2(изб.) + NaOH = NaHSO3 Сульфиты можно перевести в гидросульфиты, пропуская SO2 через их водный раствор: Na2SO3 + Н2О + SO2 = 2NaHSO3 Отметим, что сульфиты щелочноземельных металлов, например кальция, нерас- творимы в воде, а их гидросульфиты растворимы. Если взболтать осадок CaSO3 с водой и пропустить через полученную взвесь SO2, то образуется прозрачный рас- твор, содержащий Ca(HSO3)2: CaSO3 + Н2О + SO2 = Ca(HSO3)2 осадок растворимая соль Сернистый газ проявляет и окислительные, и восстановительные свойства за счёт атома серы в промежуточной степени окисления +4. В большинстве окисли- тельно-восстановительных реакций SO2 выступает в роли восстановителя. Напри- мер, оксид серы(1\/) в присутствии катализатора окисляется кислородом до оксида cepbi(VI): t 2SO2+O2^2SO3
102 При использовании типичных лабораторных окислителей в водном растворе продуктом окисления SO2 обычно является серная кислота, например: SO2 + Вг2 + 2Н2О = H2SO4 + 2НВг SO2+H2O2=H2SO4 Примером реакции, в которой SO2 выступает в роли окислителя, является его взаимодействие с сероводородом: SO2 + 2H2S = 3S + 2Н2О В промышленности SO2 получают обжигом сульфидных руд: 4FeS2 + 1102 = 8SO2 + 2Fe2O3 £. £. с. О или сжиганием серы: t S + О2 = SO2 Основной лабораторный способ получения SO2 — действие концентрированной серной кислоты на твёрдые сульфиты: Na2SO3 + Н2ЗО4(конц.) = Na2SO4+ SO2T + Н2О Сернистая кислота в свободном состоянии не выделена. Она образуется при растворении SO2 в воде, является слабой кислотой и разлагается уже при непро- должительном хранении. Сульфиты вступают в реакции ионного обмена, например: K2SO3 + BaCI2 = BaSOj + 2KCI K2SO3 + 2HCI = 2KCI + SO2t + Н2О Сульфиты подвергаются гидролизу, поскольку образованы анионом слабой кис- лоты. При смешивании растворов солей хрома(Ш) или алюминия с растворами сульфитов протекает совместный гидролиз. При этом в осадок выпадает гидрок- сид соответствующего металла, a SO2 выделяется в виде газа, например: 2AICL + 3K2SO3 + ЗН2О = 2AI(OH)a + 6KCI + 3SO2T О £ О £ * ' О С, Как и диоксид серы, сульфиты проявляют окислительно-восстановительную двойственность. Восстановительные свойства сульфитов выражены значительно сильнее, чем окислительные. Сульфиты используют в лабораторной практике как типичные восстановители: K2Cr2O7 + 3K2SO3 + 4H2SO4 = Cr2(SO4)3 + 4K2SO4 + 4H2O 2KMnO4 + K2SO3 + 2КОН = З^МпО^ K2SO4 + Н2О Тем не менее сульфиты способны проявлять и слабые окислительные свойства: K2SO3 + 2K2S + 6HCI = 3S + 6KCI + ЗН2О Соединения серы в степени окисления +6: оксид cepbi(VI) и серная кислота Оксид серы(\/1) SO3 (серный ангидрид) при обычной температуре представляет собой бесцветную жидкость. Так же как и SO2, оксид cepbi(VI) проявляет все ха- рактерные для кислотных оксидов свойства: реагирует с водой, с основаниями и с оснбвными оксидами.
103 Концентрированная серная кислота — тяжёлая, очень гигроскопичная масляни- стая бесцветная жидкость. Поглощает влагу, выделяя огромное количество тепло- ты. Поэтому нельзя приливать воду к кислоте, так как это может привести к вски- панию и разбрызгиванию раствора. Надо, наоборот, концентрированную серную кислоту осторожно тонкой струйкой вливать в воду. Серная кислота проявляет свойства, характерные для сильных кислот: взаимо- действует с металлами, с оснбвными и амфотерными оксидами и гидроксидами, аммиаком, образуя два ряда солей — сульфаты и гидросульфаты: Н2ЗО4(разб.) + Mg = MgSO4+ Н2Т 2КОН(изб.) + H2SO4= K2SO4+ 2Н2О КОН + Н28О4(изб.)= KHSO4+H2O, вступает в реакции ионного обмена с солями: H2SO4 + ВаС12 = BaSOj + 2HCI H2SO4 + Na2SO, = Na2SO4 + SO2T + H2O Обратите внимание на то, что с металлами разбавленная серная кислота ре- агирует по общим для всех кислот правилам: она взаимодействует только с ме- таллами, стоящими в ряду напряжений левее водорода, в результате реакции вы- деляется водород: Н2ЗО4(разб.) + Zn = ZnSO4+ Н2? Н2ЗО4(разб.) + Fe = FeSO4 + H2T Н2ЗО4(разб.) + Си -А Концентрированная серная кислота по свойствам существенно отличается от разбавленной. Концентрированная H2SO4— сильный окислитель. Она реагирует в том числе и с металлами, расположенными в ряду напряжений после водорода (не взаимодействует только с Pt и Au). В этих реакциях выделяется не водород, а чаще всего SO2: 2Н2ЗО4(конц.) + 2Ag = Ag2SO4 + SO2T + 2Н2О При взаимодействии концентрированной серной кислоты с активными металла- ми продуктами её восстановления могут быть сера или даже сероводород, напри- мер: 5Н2ЗО4(конц.) + 4Zn = 4ZnSO4+ H2ST + 4Н2О В то же время концентрированная серная кислота, если нет нагревания, пас- сивирует некоторые металлы, к числу которых относятся, например, алюминий, хром и железо. При контакте с H2SO4 эти металлы покрываются плёнкой оксида, который предохраняет их от дальнейшего действия кислоты. Поэтому алюминий, хром и железо не растворяются в холодной концентрированной серной кислоте. При нагревании пассивация снимается и реакция протекает, причём, поскольку серная кислота проявляет сильные окислительные свойства, железо переходит в степень окисления +3: 2Fe + 6Н2ЗО4(конц.) = Fe2(SO4)3 + 3SO2t + 6Н2О Как окислитель серная кислота взаимодействует с некоторыми неметаллами и сложными веществами. В зависимости от силы взятого для реакции восстановите- ля продуктами восстановления H2SO4 могут быть SO2 или H2S, например:
104 S + 2Н28О4(конц.) = 3SO2+ 2H2O С + 2Н28О4(конц.) = СО2Т + 2SO2? + 2Н2О 8KI + 5Н28О4(конц.) = 4I2+ H2S? + 4K2SO4 + 4Н2О Производство и применение серной кислоты Существует несколько способов производства серной кислоты, наиболее из- вестным среди которых является контактный способ. В качестве сырья для получе- ния H2SO4 можно использовать пирит, или колчедан, FeS2, сероводород или серу. В современном производстве чаще используют серу. На первой стадии проводят окисление сырья, например, пирит подвергают об- жигу: 4FeS2 +110, = 8SO, + 2Fe2O3 Поскольку пирит — твёрдое вещество, реакция обжига является гетерогенной. Чтобы повысить скорость реакции, пирит измельчают (это увеличивает площадь соприкосновения реагентов). Обжиг FeS2 осуществляют в обжиговой печи, исполь- зуя метод кипящего слоя. В такую печь снизу подаётся мощная струя воздуха, обогащённого кислородом. Частички FeS2 перемешиваются этими потоками возду- ха, благодаря чему создаётся иллюзия кипения. В печи постоянно поддерживается повышенная температура (около 800 °C). Затем выделившийся при обжиге сернистый газ очищают от пыли, подают в контактный аппарат, в котором протекает обратимая экзотермическая каталитиче- ская реакция окисления 8О2до SO3: t, Р, кат. 2SO2 + О2 < > 2SO, + Q с С о Для осуществления этой реакции используют катализатор на основе V2O5. Наконец, необходимо превратить SO3 в H2SO4. Казалось бы, это просто, нужно только провести реакцию SO3 + Н2О = H2SO4 Однако она протекает с выделением такого большого количества теплоты, что при поглощении SO3 непосредственно водой серная кислота частично испаряется и образуется взвесь мельчайших её капелек в воздухе — сернокислотный туман. Чтобы избежать этого, оксид серы(\/1) поглощают не водой, а концентрированной серной кислотой. Процесс ведут в поглотительной башне. Образуется олеум — раствор SO3 в 100%-ной серной кислоте. Олеум и является продуктом сернокис- лотного производства. Поскольку олеум, как и концентрированная серная кислота, пассивирует железо, его можно перевозить в стальных цистернах. И затем уже по- требитель, разбавляя олеум водой, может получить серную кислоту необходимой концентрации. Мировое производство серной кислоты — более 200 млн т в год. Серная кис- лота используется в производстве минеральных удобрений, взрывчатых веществ, в металлургии цветных металлов, в производстве соляной и уксусной кислот, ор- ганических красителей и др.
105 ВШЙ! Примеры заданий, при выполнении которых нужно продемонстрировать знание свойств кислорода, серы и их соединений Пример 1. Задана следующая схема превращений веществ: х Y CuS —> SO? —> NsuSO4 Определите, какие из указанных веществ являются веществами X и Y: 1) О2 2) H2SO3 3) Na2SO4 4) NaOH 5) Н3РО4 Проанализируем каждое из превращений. Первое из них соответствует окис- лительно-восстановительному процессу: степень окисления серы повышает- ся от -2 до +4. Для осуществления этого превращения на сульфид меди(П) действуют окислителем, обычно кислородом. Этот процесс в промышленно- сти называют обжигом сульфидов: 2CuS + ЗО2 = 2CuO + 2SO2T Второе превращение не сопровождается изменением степени окисления. Чтобы кислотный оксид SO2 перевести в соль соответствующей ему кислоты, нужно провести реакцию со щёлочью: SO2+ 2NaOH = Na2SO3+ Н2О Ответ: X Y 1 4 РЕАГИРУЮЩИЕ ВЕЩЕСТВА А) Б) В) Г) SO2 и Н2О SO2 и Н2О2 ЭО2и КОН (изб.) КОН и ЗО2(изб.) Ж® 2) 3) 4) 5) 6) Пример 2. Установите соответствие между реагирующими веществами и содержащим серу продуктом реакции, образующимся при их взаимодействии. ПРОДУКТ РЕАКЦИИ H2SO3 H2SO4 KHSO3 K2SO3 KHSO4 K2SO4 Одним из реагирующих веществ во всех случаях является оксид серы(1У), поэтому выполнение задания начнём с характеристики его свойств. Это кис- лотный оксид, а значит, взаимодействует с водой с образованием соответ- ствующей кислоты. Важно помнить, что степень окисления серы при этом не меняется и образуется сернистая кислота: SO2+H2O = H2SO3 При взаимодействии SO2 со щелочами образуются соли сернистой кислоты, причём, если щёлочь взята в избытке, получится средняя соль, а если в из- бытке SO2 (кислотный компонент), то образуется кислая соль: SO2 + 2КОН = K2SO3 + Н2О SO2+KOH = KHSO3
106 di 14 Д-: Обратите внимание на то, что и в этих реакциях не происходит изменения степени окисления серы и образуются сульфиты, а не сульфаты. Наконец, рассмотрим взаимодействие оксида серы(1У) с сильным окислите- лем — пероксидом водорода. В этом случае SO2 возьмёт на себя функцию восстановителя, степень окисления серы повысится до +6, продуктом реак- ции будет серная кислота: SO2+H2O2=H2SO. Ответ: А Б В Г 1 2 4 3 Пример 3. Установите соответствие между формулой вещества и реаген- тами, с каждым из которых это вещество может взаимодействовать. ФОРМУЛА ВЕЩЕСТВА A) S Б) SO2 В) Н2ЭО4(разб.) Г) AI2(SO4)3 (р-р) 2) 3) 4) 5) РЕАГЕНТЫ Fe, CuO, Mg(NO3)2 H2S, CaO, Вг2(водн.) Hg, Cl2, HNO3 Na2S, KOH, Ba(NO3)2 HCI, Fe2O3, Zn(OH)2 При выполнении предложенного задания фактически нужно рассмотреть ре- акционную способность четырёх веществ, относящихся к разным классам неорганических соединений: простого вещества серы, кислотного оксида SO2, серной кислоты и соли этой кислоты — сульфата алюминия. Начинать выполнение задания нужно с характеристики свойств каждого вещества. Так, известно, что сера взаимодействует со многими простыми веществами, ме- таллами и неметаллами, а среди сложных веществ — с кислотами-окисли- телями (HNO3, H2SO4), а также диспропорционирует в растворах щелочей. Взаимодействие с солями и оксидами для серы не характерно. После такой характеристики становится понятным и набор реагентов, с которыми сера будет взаимодействовать: это Hg, Cl2, HNO3. Для SO2, кроме свойств, присущих кислотным оксидам, характерна ещё и окислительно-восстановительная двойственность, поскольку атом серы нахо- дится в промежуточной степени окисления +4. Поэтому SO2 способен реаги- ровать и с СаО (как кислотный оксид), и с H2S (как окислитель), и с бромной водой (как восстановитель): СаО + SO2 = CaSO3 2H2S + SO2 = 3S + 2H2O SO2 + Br2 + H2O = H2SO4 + 2HBr Разбавленная серная кислота проявляет все общие свойства, характерные для кислот. Это довольно обширный круг реакций: взаимодействие с метал- лами, расположенными в ряду напряжений левее водорода, с оснбвными и амфотерными оксидами, основаниями и амфотерными гидроксидами, реак- Й /у ч • й if!
107 ции ионного обмена с солями. Поэтому для нахождения ответа можно дей- ствовать методом от противного: найдём вещества, с которыми H2SO4 не ре- агирует, и отбросим заведомо неверные варианты ответов. Серная кислота не будет взаимодействовать с Вг2(водн.), Hg, HCI — тогда варианты ответов 2, 3 и 5 исключаем. Остаётся проанализировать варианты ответов 1 и 4. Об- ратим особое внимание на возможность H2SO4 реагировать с солями — ни- тратами магния и бария. Эти взаимодействия относятся к реакциям ионного обмена, для протекания которых необходимо связывание ионов с выделе- нием осадка или газа. В случае Mg(NO3)2 ни осадка, ни газа не выделяет- ся, значит, реакция с Н2ЗО4не протекает. А вот Ba(NO3)2 реагирует с H2SO4c образованием нерастворимого BaSO4. Теперь и вариант ответа 1 исключён. Но необходимо проверить возможность взаимодействия с серной кислотой других веществ, предложенных в варианте 4. Очевидно, что H2SO4 вступит в реакцию нейтрализации со щёлочью КОН. А вот сульфид натрия — соль, и здесь вновь нужно вспомнить об условиях протекания реакций ионно- го обмена. В данном случае взаимодействие возможно за счёт выделения газа: Na2S + H2SO4= H2ST + Na2SO4 Вариант ответа 4 подойдёт и для раствора сульфата алюминия. При его вза- имодействии с Na2S протекает совместный гидролиз: AI2(SO4)3 + 3Na2S + 6Н2О = 2А1(ОН)3Ф + 3H2ST + 3Na2SO4 Реакция AI2(SO4)3 со щёлочью приводит к образованию осадка А1(ОН)3 и его последующему растворению в избытке КОН: AI2(SO4)3+ 6КОН = 2А1(ОН)3Ф + 3K2SO4 AI(OH)3J, + КОН = К[А1(ОН)4] И снова призываем вас внимательно отнестись к анализу возможности про- текания реакции ионного обмена при взаимодействии растворов двух со- лей — сульфата алюминия и нитрата бария. Обязательно убедитесь, что в этой реакции образуется осадок BaSO4, следовательно, реакция протекает. Ответ: А Б В Г 3 2 4 4 Пример 4. Перманганат калия прокалили. Выделившийся газ смешали с вдвое большим по объёму количеством сероводорода и осторожно нагрели. Образовавшееся светло-жёлтое твёрдое вещество обработали при нагре- вании концентрированной азотной кислотой, наблюдали выделение бурого газа. Образовавшуюся в результате реакции кислоту нейтрализовали раство- ром гидрокарбоната кальция. Напишите уравнения четырёх описанных реак- ций.
108 При прокаливании (т. е. сильном нагревании) перманганата калия он разла- !гается с выделением кислорода (см. способы получения кислорода): 2КМпО4 = К2МпО4 + МпО2 + О2 Анализируя условия второго превращения, обратите внимание на количе- ственное соотношение кислорода и сероводорода. Кислород находится в недостаточном количестве для полного окисления сероводорода до оксида cepbi(IV), поэтому продуктом реакции является сера: t 2H2S + О2( недостаток) = 2S + 2Н2О Взаимодействие серы с концентрированной азотной кислотой приводит к образованию серной кислоты и выделению оксида азота(1\/): S + 6НМО3(конц.) = H2SO4 + 6NO2 + 2Н2О Наконец, нейтрализация серной кислоты раствором гидрокарбоната каль- ция описывается следующим уравнением: H2SO4 + Ca(HCOJ2 = CaS04 + 2Н2О + 2СО ЗАНЯТИЕ 11 Азот и его соединения Основные элементы содержания: общая характеристика азота в связи с его положением в Периодической системе Д. И. Мен- делеева; строение молекулы и химические свойства азота; строение молекулы и химические свойства аммиака; донорно-акцепторный механизм образования ковалентной связи в ионе ам- мония; свойства солей аммония, оксидов азота, азотистой и азотной кислот, нитритов и ни- тратов. Общая характеристика азота в связи с его положением в Периодической системе Д. И. Менделеева Элемент азот расположен во 2-м периоде и VA-группе Периодической системы Д. И. Менделеева. На внешнем электронном уровне атома азота находится пять электронов. В соответствии с положением в Периодической системе и строени- ем внешнего электронного слоя азот, как и другие элементы VA-группы, проявляет высшую степень окисления +5 и низшую степень окисления -3. Одна из особенностей азота состоит в том, что для его атома не реализует- ся валентность, равная номеру группы, т. е. азот не может быть пятивалентным. Атом азота в основном состоянии имеет три неспаренных электрона, поэтому его характерная валентность равна трём. Такую валентность азот имеет, например, в молекулах азота и аммиака (схема 10, а, б). В отличие от атомов многих дру- гих элементов атом азота не может переходить в возбуждённое состояние, по- скольку на его внешнем электронном слое нет свободных орбиталей, на которых мог бы разместиться неспаренный электрон, образовавшийся при разъединении
109 Схема 10 а) электронное строение атома азота 7n б) графические формулы молекул азота и аммиака .-J 11111 л - 2 [н| 2р 2s :n=n: н—n: н в) схема образования катиона аммония н I н—n: н электронной пары с 2в-орбитали. Тем не менее за счёт этой неподелённой элек- тронной пары атом азота может образовывать четвёртую ковалентную связь по донорно-акцепторному механизму, например при образовании катиона аммония (схема 10, в). Таким образом, максимальная валентность азота равна четырём. Такую валентность азот имеет, например, в ионе аммония и в молекуле азотной кислоты (см. материалы занятия 3). Другая особенность азота состоит в том, что он образует устойчивые соедине- ния во всех промежуточных степенях окисления: -2, -1, +1, +2, +3 и +4 (табл. 15). В низшей степени окисления -3 азот образует соединение с водородом — ам- миак, соли аммония и соединения с металлами — нитриды. Во всех положитель- ных степенях окисления азот образует оксиды: N2O, NO, N2O3, NO2 и N2O5. Среди них кислотными являются только три последних, a N2O и NO относятся к несоле- образующим оксидам. Азот также образует кислоты — азотистую HNO2 и азотную HNO3. Соли этих кислот имеют названия «нитриты» и «нитраты» соответственно. Таблица 15 Важнейшие соединения, образованные азотом в различных степенях окисления -3 -2 -1 + 1 +2 +3 +4 4-5 NH3 аммиак Саз^2 нитрид кальция nh4ci хлорид аммония n2h4 гидразин nh2oh гидроксил- амин n2o no оксид оксид азота (I) азота (II) N2°3 оксид азота (III) no2 ОКСИД азота (IV) n2o5 оксид азота (V) несолеобразующие оксиды hno2 азотистая кислота kno2 нитрит калия кислотные оксиды HNO3 азотная кислота KNOo О нитрат калия
110 В природе элемент азот встречается главным образом в виде соответствующе- го простого вещества — газа азота, объёмная доля которого в воздухе составляет 78 %. Кроме того, азот входит в состав органических веществ, в том числе в со- став «молекул жизни»: белков и нуклеиновых кислот. А вот в земной коре азота совсем мало — меньше одной тысячной процента. Азот как простое вещество При обычных условиях азот — газ без цвета и запаха, практически нерастворим в воде. Азот — третий по электроотрицательности элемент после фтора и кислорода, однако как простое вещество он химически довольно инертен. Низкая химическая активность молекулярного азота объясняется чрезвычайно высокой прочностью тройной связи в его молекуле. При обычных условиях азот реагирует только с литием, образуя нитрид лития: N2 + 6Li = 2Li3N При повышенной температуре образуются нитриды и других активных метал- лов — натрия, щелочноземельных металлов, алюминия: 3Mg + N2= Mg3N2 2AI + N2 = 2AIN Нитриды активных металлов IA- и IIA-групп разлагаются водой с выделением аммиака и взаимодействуют с кислотами с образованием солей аммония: Ca3N2 + 6Н2О = ЗСа(ОН)2 + 2NH3 Ca3N2 + 8HCI = 3CaCI2 + 2NH4CI Нитриды многих других элементов (BN, AIN, TiN, TaN и др.), напротив, отлича- ются химической инертностью, твёрдостью и тугоплавкостью. С кислородом азот реагирует при очень высокой температуре (выше 2000 °C), причём образуется оксид азота(Н): N2 + O2 ^2N0-0 Отметим, что эта реакция является примером одной из немногих реакций со- единения, протекающей с поглощением теплоты. В природных условиях её про- текание возможно при разряде молнии во время грозы. Практически важна реакция азота с водородом с образованием аммиака: t, р, кат. N2 + ЗН2 — » 2NH3 + О Эта реакция лежит в основе промышленного способа получения аммиака. В промышленности азот получают ректификацией (разделением) жидкого воз- духа. В лаборатории небольшие количества азота можно получить при термиче- ском разложении нитрита аммония: NH4NO2 = N2T + 2Н2О Нитрит аммония очень неустойчив, поэтому на практике для проведения этой реакции смешивают растворы нитрита натрия и хлорида аммония; при нагревании полученного раствора выделяется азот: NH4CI + NaNO2 = N2t + NaCI + 2H2O
Ill Аммиак Аммиак NH3 — бесцветный газ, очень хорошо растворимый в воде. Аммиак об- ладает резким запахом и в малых дозах оказывает стимулирующее влияние на нервную систему, его водный раствор — нашатырный спирт — дают понюхать че- ловеку, чтобы вывести его из обморочного состояния. В больших дозах аммиак ядовит. Молекула аммиака имеет форму треугольной пирамиды (рис. 11). В этой моле- куле атом азота связан с атомами водорода тремя полярными ковалентными свя- зями. В образовании этих связей задействованы три неспаренных электрона ато- ма азота, а пара электронов, находившаяся на 2з-орбитали, остаётся неподелённой (см. схему 10). Молекула аммиака в целом также полярна: отри- цательный заряд сосредоточен на атоме азота, а по- ложительный — на атомах водорода. Из-за высокой полярности молекул аммиака между ними образуется водородная связь: н н |8- 5+ | Н— N • • • Н—N н н Благодаря водородным связям аммиак легко пере- Рис.11. Модель молекулы ходит в жидкое состояние при повышенном давлении аммиака или охлаждении. Полярностью молекулы NH3 объясняется хорошая растворимость аммиака в воде: при 20 °C в 1л воды растворяется до 700 л аммиака, так что массовая доля аммиака в растворе достигает примерно 34 %. В растворе часть молекул аммиака образует водородные связи с молекулами воды: н н |8- 8+ | Н—N • • • Н—О I н Формулу образовавшегося соединения записывают как NH3 • Н2О и называют его гидратом аммиака. Гидрат аммиака частично диссоциирует с образованием катионов NH4+ и анио- нов ОН' и является, таким образом, слабым основанием6: NH-- Н?О з=ь nh; + oh- Итак, в водном растворе аммиака устанавливаются следующие равновесия: NH3+H2O NH3-H2O NH4+ + OH' Отметим, что при растворении аммиака в воде только небольшая часть его мо- лекул образует гидраты и только небольшая часть гидратов диссоциирует на ионы. Ббльшая часть аммиака находится в водном растворе в виде молекул NH3. Об- разующиеся в растворе аммиака катионы NH4 называются катионами аммония. 6 Водный раствор аммиака иногда называют гидроксидом аммония и обозначают формулой NH4OH. Эта формула хорошо отражает оснбвные свойства аммиака, но не его строение, поскольку в действительности молекула, в которой была бы ионная связь между ионами NH* и ОН", не существует.
112 В катионе NH4+ четыре ковалентные связи, одна из которых образована по донор- но-акцепторному механизму (см. схему 10). Благодаря оснбвным свойствам гидрат аммиака окрашивает лакмус в синий цвет, а фенолфталеин в малиновый. Газообразный аммиак также окрашивает влажную лакмусовую бумажку в синий цвет. Аммиак в водном растворе реагирует с кислотами с образованием солей аммо- ния. Для записи уравнений таких реакций можно использовать как формулу амми- ака, так и формулу его гидрата: 2NH3+ H2SO4= (NH4)2SO4 или 2NH3 • Н2О + H?SO4 = (NH4)?SO4 + 2Н2О сульфат аммония При взаимодействии аммиака с двух- и трёхосновными кислотами в зависимо- сти от соотношения реагентов могут образовываться не только средние, но и кис- лые соли: NH3+ H3P04=NH4H?P04 3 3 4 4 2 4 дигидрофосфат аммония 2NH3 + Н3РО4 = (NH4)2HP04 3 3 4' 4'2 4 гидрофосфат аммония Если кислота летучая, то реакция может протекать не в водном растворе, а не- посредственно между парами кислоты и газообразным аммиаком: NH3„ + HCI(r)=NH.CIw хлорид аммония Как растворимое в воде слабое основание гидрат аммиака может вступать в реакции обмена с растворами солей некоторых металлов, осаждая соответствую- щие гидроксиды: Cr2(SO4)3 + 6NH3 • Н2О = 2Сг(ОН)3Ф + 3(NH4)2SO4 Эти же реакции протекают, если газообразный аммиак пропускать через соот- ветствующие растворы солей металлов. Аммиак проявляет восстановительные свойства за счёт атома азота, находяще- гося в низшей степени окисления. Восстановительные свойства аммиака проявляются прежде всего при его взаи- модействии с кислородом. В зависимости от условий проведения реакции между аммиаком и кислородом могут образовываться различные продукты окисления. Так, при горении аммиака в кислороде образуется азот: 4NH3+ЗО2= 2N2+6Н2О А если реакцию провести в присутствии катализатора — металлической плати- ны, то аммиак окислится до оксида азота(П): 4NH3 + 5О2 = 4NO + 6Н2О Как восстановитель аммиак взаимодействует и со многими другими окислите- лями, причём в подавляющем большинстве случаев продуктом окисления аммиака является азот (схема 11).
113 Схема 11 Восстановительные свойства аммиака + о2 н2О 4- СиО Си 4- Н2О (nh?) 1 ** ( N2 ) + \ +Н2О2 ' н2° + КМпО4 МпО2 + кон + н2о При каталитическом окислении: NH3 + О2 —(no) + Н2О В промышленности аммиак синтезируют из азота и водорода. Синтез проводят при температуре около 300-400 °C, так как повышение температуры приводит к увеличению скорости реакции. Поскольку реакция получения аммиака из азота и водорода является обратимой и протекает с уменьшением объёма, то для сме- щения равновесия в сторону продукта реакции необходимо повышать давление. Обычно синтез ведут при давлении 100-300 атм в зависимости от применяемого оборудования. В качестве катализатора используют губчатое железо, активирован- ное оксидами калия и алюминия. Поскольку даже в таких условиях выход аммиа- ка является невысоким (15-20% от теоретического), для более полного использо- вания азота и водорода используют метод циркуляции. Образовавшийся аммиак охлаждают и отделяют, а непрореагировавшие азот и водород вновь направляют в реактор. Основным аппаратом в производстве аммиака является колонна синтеза. Это реактор в виде цилиндра до 20 м высотой, изготовленный из особых стойких к коррозии сортов сталей. Он способен выдерживать высокое давление и агрессив- ное воздействие химических веществ. В лабораторных условиях аммиак можно получить действием щелочей на соли аммония при нагревании: Са(ОН)2 + 2NH4CI = CaCI2 + 2NH3t + 2Н2О Взаимодействие щелочей с солями аммония используется как качественная ре- акция на присутствие катиона NH4+ в их составе. Выделяющийся аммиак обнару- живают по посинению влажной лакмусовой бумажки или по окрашиванию в мали- новый цвет влажной фенолфталеиновой бумажки. Соли аммония Соли аммония — хорошо растворимые в воде кристаллические вещества. Свой- ства солей аммония во многом напоминают свойства солей калия, что объясняет- ся близостью размеров ионов калия и аммония. Соли аммония вступают в реакции ионного обмена с растворами других солей, щелочей и кислот, например: NH4CI + AgNO3 = NH4NO3 + АдС1Ф (образуется нерастворимое вещество) (NH4)2SO4+ 2NaOH = Na2SO4+ 2NH3 • H2O (образуется слабый электролит — гидрат аммиака)
114 (NH4)2CO3 + 2HCI = 2NH4CI + CO2T + H20 (выделяется газ) Поскольку соли аммония образованы слабым основанием — гидратом аммиака, в растворе протекает их гидролиз по катиону. Поэтому растворы солей аммония, образованных анионами сильных кислот (например, NH4CI, (NH4)2SO4, NH4NO3), имеют кислотную среду. Особенностью солей аммония является их термическая неустойчивость. Все соли аммония разлагаются при нагревании. Состав продуктов термического раз- ложения зависит от того, проявляет ли анион соли окислительные свойства. Если анион не проявляет окислительных свойств, то соль аммония при нагрева- нии разлагается на аммиак и соответствующую кислоту: (NH4)2HPO4 = 2NH3T + H3PO4 NH4CI = NH3t + HClT (NH4)2CO3 = 2NH3T + H2O + CO2T Обратите внимание на то, что продуктами разложения, например, хлорида ам- мония являются только газообразные вещества. В таких случаях говорят, что соль разлагается без образования сухого остатка. Так же, без сухого остатка (конечно, если проводить реакцию при температуре выше 100 °C), разлагаются карбонат и гидрокарбонат аммония. Эти вещества используют в пищевой промышленности как разрыхлители теста. Если в состав соли входит анион, обладающий окислительными свойствами, её термическое разложение протекает с изменением степени окисления азота: NH4NO3= N2O + 2H2O (NH4)2Cr2O7 = N2 + Cr2O3 + 4H2O Оксиды азота Во всех положительных степенях окисления азот образует оксиды (табл. 16). Таблица 16 Свойства оксидов азота Формула и тип оксида Особенности свойств оксида N2O несолеобразующий Разлагается при слабом нагревании: 2N2O = 2N2 + О2 NO несолеобразующий Легко при обычных условиях вступает в реакцию с кислородом с образованием бурого газа — оксида азота(1\/): 2NO + О2 = 2NO2 n2o3 кислотный Термически неустойчив, стабилен только при низких температу- рах: N2O3 = NO + NO2 N2O3 + Н2О(хол.) = 2HNO2 N2O3 + 2NaOH = 2NaNO2 + H2O no2 кислотный Бурый газ. Вступает в реакцию димеризации: 2NO2 N2O4 При взаимодействии NO2 с водой образуются азотная и азоти- стая кислоты: 2NO2 + Н2О = HNO2 + HNO3
115 Продолжение Формула и тип оксида Особенности свойств оксида При взаимодействии NO2 с водой в присутствии кислорода образуется только азотная кислота: 4NO2 + 2Н2О + О2 = 4HNO3 При взаимодействии NO2 с растворами щелочей получаются нитриты и нитраты: 2NO2 + 2NaOH = NaNO2 + NaNO3 + H2O n2o, кислотный Термически неустойчив, стабилен только при низких температурах: 2N2O5 = 4NO2 + О2 N2O5 + H2O = 2HNO3 N2O5 + 2NaOH = 2NaNO3 + H2O Азотистая кислота и нитриты Азотистая кислота HNO2 относится к слабым кислотам. Она очень легко разла- гается и существует только в разбавленных растворах, поэтому не имеет практи- ческого значения. Соли азотистой кислоты — нитриты, наоборот, термически устойчивы. Нитри- ты, как правило, хорошо растворяются в воде. Поскольку атом азота в их составе находится в промежуточной степени окисления, нитриты проявляют свойства как окислителей, так и восстановителей (схема 12). Отметим, что у нитритов восста- новительные свойства преобладают, поэтому в лабораторной практике их исполь- зуют в качестве типичных восстановителей. Схема 12 Окислительно-восстановительные свойства нитритов (на примере нитрита калия) +2 + восстановитель +3 + окислитель +5 NO---------------- KNO2 -------------------------► KNO3 (KI, FeSO4 и др.) z (KMnO4, K2Cr2O7, Н2О2 и др.) a KNO2 является окислителем KNO2 является восстановителем Азотная кислота Азотная кислота HNO3— бесцветная (при хранении желтеющая) жидкость. Кон- центрированной считается кислота с массовой долей HNO3 около 60 %. Можно получить и «дымящую» азотную кислоту с массовой долей HNO3 96—98 %. При нагревании и на свету азотная кислота частично разлагается: 4HNO3 4NO2 + 2Н2О + О2 Азотная кислота является сильной кислотой. Она проявляет все общие свойства кислот: взаимодействует с оснбвными и амфотерными оксидами и гидроксидами, гидратом аммиака, солями слабых кислот. Особенностью азотной кислоты являются её сильные окислительные свойства. Эти свойства проявляются прежде всего при её взаимодействии с металлами. Азотная кислота, как концентрированная, так и разбавленная, реагирует даже с теми металлами, которые расположены правее водорода в ряду напряжений.
116 Схема 13 Устойчивы к действию HNO3 любой концентра- Продукты восстановления азотной кислоты ции только наименее активные металлы, такие как Au, Pt, Rh и некоторые другие. При взаимодействии HNO3 с металлами окислителем является атом азота, находящий- ся в степени окисления +5. Поэтому водород в таких реакциях практически не выделяется, а образуются различные продукты восста- новления нитрат-иона. Состав образующихся продуктов восстановления зависит от многих факторов, прежде всего от концентрации кис- лоты и активности металла (схема 13). Чем более концентрированной является кислота, тем меньше электронов будут склонны прини- мать её молекулы и тем выше будет степень окисления атома азота в продукте реакции. И соответственно чем активнее металл-вос- становитель, тем больше электронов будут принимать молекулы кислоты и тем ниже бу- дет степень окисления атома азота в продукте реакции. Например, при взаимодействии концентрированной HNO3 с малоактивными ме- таллами (Ад, Си, РЬ) будет выделяться NO2: Си + 4НМО3(конц.) = Cu(NO3)2+ 2NO2+ 2Н2О Эти же малоактивные металлы с разбавленной (примерно 30%-ной) HNO3 чаще всего образуют NO: ЗСи + 8НМО3(разб.) = 3Cu(NO3)2 + 2NO + 4Н2О Наконец, при взаимодействии очень сильно разбавленной HNO3 (с массовой до- лей 2—3%) с активными металлами (Zn, Mg) газ не выделяется. Азотная кислота восстанавливается до иона аммония, который с нитрат-анионом образует нитрат аммония: 4Zn + ЮНМО3(очень разб.) = 4Zn(NO3)2+ NH4NO3 + 3H2O Нужно помнить и ещё об одной важной особенности азотной кислоты: кон- центрированная HNO3 при обычной температуре не взаимодействует с Al, Fe, Сг и с некоторыми другими металлами — пассивирует их в результате образования малорастворимой оксидной плёнки. Однако при нагревании взаимодействие ста- новится возможным, и протекает реакция с образованием нитрата металла в его наиболее высокой устойчивой степени окисления: Fe + 6НМО3(конц.) = Fe(NO3)3 + 3NO2+ ЗН2О Азотная кислота окисляет также многие неметаллы и сложные вещества. Обыч- но для осуществления таких реакций используют горячую концентрированную HNO3. Неметалл, как правило, при этом образует высшую кислоту, например: S + 6НМО3(конц.) = H2SO4 + 6NO2 + 2Н2О Р + 5НМО3(конц.) = Н3РО4+ 5NO2+ Н2О Cu2S + 14НМО3(конц.) = 2Cu(NO3)2 + H2SO4 + 10NO2 + 6Н2О
117 В основе промышленного способа получения азотной кислоты лежат следую- щие реакции: t, р, кат. N, + ЗН, <=* 2NK Pt 4NH3 + 5О2 = 4NO + 6Н2О 2NO + O2^2NO2 4NO2+2Н2О + О2 = 4HNO3 В лаборатории небольшие количества азотной кислоты можно получить взаи- модействием твёрдых нитратов с концентрированной серной кислотой при нагре- вании. В этом случае азотная кислота образуется в виде паров, которые затем конденсируются в охлаждаемом приёмнике: KNO3 + H2SO4 = KHSO4 + HNO3T Соли азотной кислоты Соли азотной кислоты — нитраты — проявляют общие свойства солей: вступа- ют в реакции ионного обмена с другими солями, кислотами, щелочами. Практиче- ски все нитраты хорошо растворимы в воде. Особенностью нитратов является их термическая неустойчивость. Все нитраты разлагаются при нагревании. По составу продуктов термического разложения ни- траты можно разделить на три группы. Первую группу составляют нитраты щелочных и щелочноземельных металлов. При разложении большинства из них образуются соответствующие нитриты и кис- лород: 2KNO3 = 2KNO2 + О2Т Вторую группу составляют нитраты почти всех металлов, находящихся в ряду напряжений от магния до меди включительно. Продуктами их разложения являют- ся соответствующие оксиды металлов, оксид азота(1\/) и кислород: 2Cu(NO3)2 = 2CuO + 4NO2T + О2Т Наконец, третью группу составляют нитраты только двух металлов: серебра и ртути. Их нитраты разлагаются до свободного металла, оксида азота(1\/) и кисло- рода: 2AgNO3 = 2Ag + 2NO2T + О2Т В обычных условиях окислительные свойства нитратов выражены слабее, чем у азотной кислоты. Однако в определённых условиях (например, в расплавленном состоянии) они демонстрируют очень сильные окислительные свойства. Например, если уголёк или кусочек серы поместить в расплав селитры, протекает бурная ре- акция: 2КМО3(расплав) + С = 2KNO2 + СО2 2К1МО3(расплав) + S = 2KNO2+ SO2 Выраженные окислительные свойства нитраты проявляют также в растворах, имеющих кислотную среду.
118 Примеры заданий, при выполнении которых нужно продемонстрировать знание свойств азота и его соединений Пример 1. Из предложенного перечня выберите два вещества, реакция разложения которых при нагревании не относится к окислительно-восстано- вительным. 1) гидрокарбонат аммония 4) нитрат калия 2) нитрат аммония 5) нитрат меди(П) 3) хлорид аммония Для выполнения этого задания потребуется проявить знание одной из важ- нейших особенностей химических свойств солей аммония и солей азотной кислоты. И те, и другие соли разлагаются при нагревании. Обратите вни- мание: разложение нитратов металлов всегда сопровождается выделением простого вещества — кислорода, и, значит, является окислительно-восста- новительным процессом. Разложение солей аммония, образованных аниона- ми, не обладающими окислительными свойствами, протекает без изменения степеней окисления. Примерами таких солей являются хлорид, карбонат, ги- дрокарбонат аммония. А вот разложение нитрата аммония — окислительно- восстановительный процесс, причем восстановителем является атом азота в степени окисления -3, входящий в состав катиона аммония, а окислите- лем — атом азота в степени окисления +5, входящий в состав нитрат-иона. Подтвердим рассуждения уравнениями реакций: NH4HCO3 = NH3T + Н2О + СО2Т NH4CI = NH3T + НСГГ NH4NO3= N2OT + 2Н2О 2KNO3= 2KNO2+ O2T 2Cu(NO3)2 = 2CuO + 4NO2? + O2T Ответ: 1 3 Пример 2. Задана следующая схема превращений веществ: Fe -Л- Fe(NO3)2 ЫНз'Нг°> Y Определите, какие из указанных веществ являются веществами X и Y 1) Н1МО3(конц.) 2) FeO 3) Cu(NO3)2 4) Fe(OH)2 5) Fe(OH)3 Рассмотрим каждую из реакций. Можно ли использовать концентрированную азотную кислоту для получения нитрата железа(П) из железа? Нет, нельзя. Дело в том, что под действием концентрированных серной и азотной кислот при обычной температуре железо пассивируется, и реакция протекать не бу- дет. При нагревании реакция становится возможной, но даже в таких усло- виях требуемый продукт получен не будет. Азотная кислота обладает силь- ными окислительными свойствами, поэтому образуется нитрат железа(Ш), а не нитрат железа(П). Fe + 6НМО3(конц.) = Fe(NO3)3+ 3NO2+ ЗН2О Следовательно, в первом превращении в качестве реагента нужно использо- вать нитрат меди(Н):
119 Fe + Cu(NO3)2 = Си + Fe(NO3)2 Для определения продукта второй реакции необходимо вспомнить, что ги- драт аммиака проявляет свойства растворимого слабого основания, поэтому способен осаждать гидроксиды металлов из их солей: ______ Fe(NO3)2+ 2NH3 • Н2О = Fe(OH)2>L + 2NH4NO3 X Y 3 4 Пример 3. Порошок магния нагрели в атмосфере азота. При взаимодей- ствии полученного вещества с водой выделился газ. Газ пропустили над на- гретым оксидом меди(П). На образовавшийся металл подействовали нитра- том калия в присутствии серной кислоты, выделился бурый газ. Напишите уравнения четырёх описанных реакций. Проанализируем описанные превращения. Азот при нагревании взаимодей- ствует с активными металлами с образованием нитридов: 3Mg + N2 = Mg3N2 Как и многие нитриды, Mg3N2 подвергается гидролизу, при этом выделяется аммиак: Mg3N2 + 6Н2О = 3Mg(OH)2 + 2NH3 Аммиак способен восстанавливать малоактивные металлы из их оксидов. Как правило, продуктом окисления NH3 является N2 (см. схему 11): ЗСиО + 2NH3 = ЗСи + N2 + ЗН2О Нитраты в кислотной среде проявляют сильные окислительные свойства: Си + 2KNO3 + 2H2SO4 = CuSO4 + 2NO, + K2SO4 + 2Н2О Пример 4. При взаимодействии растворов сульфата цинка и нитрата ба- рия образовался белый осадок. Осадок отделили фильтрованием, фильтрат осторожно выпарили, сухой остаток прокалили. Смесь выделившихся при прокаливании газов растворили в воде. В разбавленном растворе полу- ченной кислоты растворили магний, при этом в растворе образовалось две соли, а газ не выделялся. Напишите уравнения четырёх описанных реакций. При взаимодействии растворов сульфата цинка и нитрата бария протекает реакция ионного обмена: ZnSO4 + Ba(NO3)2= BaSOj + Zn(NO3)2 После отделения осадка в растворе остаётся нитрат цинка, его прокалива- ние приводит к получению оксида цинка, оксида азота(1\/) и кислорода: 2Zn(NO3)2 = 2ZnO + 4NO2 + О2 При взаимодействии NO2c водой в присутствии кислорода образуется азот- ная кислота: 4NO2+2H2O + O2 = 4HNO3
120 Наконец, при взаимодействии сильно разбавленной HNO3 с таким активным металлом, как Мд, азотная кислота восстанавливается до иона аммония, ко- торый с избытком кислоты образует нитрат: 4Мд + 10НМО3(очень разб.) = 4Mg(NO3)2 + NH4NO3 + ЗН2О ЗАНЯТИЕ 12 Фосфор и его соединения Основные элементы содержания: общая характеристика фосфора в связи с его положением в Периодической системе Д. И. Мен- делеева; аллотропные модификации фосфора, химические свойства фосфора; химические свойства фосфина, оксидов фосфора, фосфорной кислоты и её солей. Общая характеристика фосфора в связи с его положением в Периодической системе Д. И. Менделеева Элемент фосфор расположен в 3-м периоде и VA-группе Периодической систе- мы Д. И. Менделеева. Фосфор является электронным аналогом азота: на внешнем электронном уровне его атома, как и у атома азота, находится пять электронов. Однако химические свойства азота и фосфора существенно различаются. Одна из причин такого различия состоит в том, что на внешнем электронном уровне атом фосфора имеет пять свободных d-орбиталей, на которые при переходе атома в возбуждённое состояние могут перемещаться электроны с Зе-орбитали (схема 14). Поэтому, в отличие от азота, фосфор может быть пятивалентным. Именно эта ва- лентность для фосфора наиболее характерна. В соответствии с положением в Периодической системе и строением внешнего электронного слоя фосфор, как и другие элементы VA-группы, проявляет высшую степень окисления +5 и низшую степень окисления -3. Также фосфор образует соединения в степенях окисления +1 и +3 (см. табл. 17). Наиболее характерна для фосфора степень окисления +5. Именно в виде соединений с этой степенью окис- ления фосфор находится в природе. Основными природными соединениями фос- фора являются фторапатит ЗСа3(РО4)2 • CaF2 и фосфорит Са3(РО4)2. Фосфор также входит в состав органических соединений, например нуклеиновых кислот. Схема 14 Электронная конфигурация атома фосфора в основном и возбуждённом состояниях 15Р ls22s22p63s23p3 15Р ls22s22p63s13p33d1 возбуждённое состояние основное состояние
121 Фосфор необходим для жизнедеятельности растений и животных. Содержание фосфора в организме взрослого человека составляет около 1% от массы тела. Эта масса почти полностью приходится на костную ткань, которая состоит в основном из гидроксилапатита ЗСа3(РО4)2 • Са(ОН)2. Эмаль зубов содержит гидроксиапатит с примесью фторапатита. Среди других органов наибольшее содержание фосфора отмечается в головном мозге. Таблица 17 Важнейшие соединения, образованные фосфором в различных степенях окисления -3 +1 +3 +5 РН3 — фосфин; фосфиды: Са3Р2 н3ро2 фосфорноватистая кислота и её соли — гипофосфиты: КН2РО2 Р2О3 оксид фосфора(1Н); н3ро3 фосфористая кислота и её соли — фосфиты: К^РОз P2OS оксид фосфора(\/); Н3РО4 ортофосфорная кис- лота и её соли — ортофосфаты: КзРО4 Фосфор как простое вещество Известно более десяти аллотропных модификаций фосфора. Наиболее важны- ми из них являются белый, красный и чёрный фосфор. Белый фосфор состоит из тетраэдрических молекул Р4. Он имеет молекулярную кристаллическую решётку, в узлах которой и находятся эти молекулы. Свойства бе- лого фосфора типичны для вещества молекулярного строения: это воскообразное мягкое прозрачное вещество, имеющее обычно желтоватый оттенок. Белый фосфор имеет низкую температуру плавления — всего 44 °C. Он растворяется в сероугле- роде и некоторых других органических растворителях. Белый фосфор очень ядовит и чрезвычайно химически активен. Так, он окисляется и самовоспламеняется на воздухе, поэтому его хранят под водой. При окислении белый фосфор светится в темноте — часть выделяющейся в ходе реакции энергии выделяется в виде света. При нагревании белого фосфора без доступа воздуха происходит разрыв одной из связей Р—Р в молекуле Р4 и начинается полимеризация, приводящая к образо- ванию красного фосфора: белый фосфор -ф-ф-ф- красный фосфор Красный фосфор имеет полимерное строение: в его структуре присутствуют цепи разной длины, состоящие из разного количества тетраэдров Р4, связанных между собой. Красный фосфор обычно аморфен, однако его можно получить и в кристаллической форме. В этом случае его кристаллическая решётка будет атом-
122 ной. Красный фосфор представляет собой порошок бурого цвета. Он не раство- ряется в большинстве растворителей, не ядовит. Химическая активность красного фосфора заметно ниже, чем у белого. Чёрный фосфор также имеет полимерное строение. По внешнему виду он по- хож на графит, обладает полупроводниковыми свойствами. На практике чёрный фосфор используется редко. Химическая активность фосфора зависит от того, в какой из аллотропных форм он находится. Как уже было отмечено, белый фосфор значительно более активен, чем красный. Тем не менее, большинство реакций, характерных для белого фос- фора, возможны и для красного, только протекают они в более жёстких условиях. Химические взаимодействия с участием белого и красного фосфора чаще всего описываются одинаковыми уравнениями реакций, поэтому обычно в уравнениях используют символ Р, а не Р4. Фосфор реагирует с активными металлами, кислородом (белый фосфор мед- ленно окисляется уже при контакте с воздухом, красный фосфор загорается при поджигании), галогенами, серой (схема 15). При взаимодействии с металлами фосфор является окислителем и восстанав- ливается до низшей степени окисления -3: t 2Р + ЗСа = Са3Р2 Взаимодействуя с кислородом, галогенами, серой, фосфор выступает в роли восстановителя. При нагревании с достаточным или избыточным количеством ре- агента фосфор окисляется до его наиболее устойчивой степени окисления +5: 4Р + 5О2 = 2Р2О5 (горение на воздухе) 2Р + 5CI2 = 2РС15 При более низких температурах или недостатке реагента могут образоваться соединения фосфора(Ш): 4Р + ЗО2 = 2Р2О3 (медленное окисление) 2Р + 3CI2= 2РС13 Схема 15 Взаимодействие фосфора с простыми веществами СазРг P2S3> P2S5
123 Важно запомнить, что с водородом фосфор практически не взаимодействует. Многие бинарные соединения фосфора (фосфиды металлов, галогениды фос- фора) подвергаются полному гидролизу, т. е. разлагаются под действием воды, а также растворов кислот или щелочей. Например, как и нитриды, фосфиды разлагаются водой и разбавленными кис- лотами с выделением фосфина: Са3Р2 + 6Н2О = ЗСа(ОН)2 + 2РН3Т Са3Р2 + 6HCI = ЗСаС12 + 2РН3Т Напомним, что при обработке нитридов кислотами аммиак не выделяется, а связывается избытком кислоты в соли аммония. Фосфин же, в отличие от амми- ака, обладает очень слабыми оснбвными свойствами и не образует солей с боль- шинством кислот, поэтому при взаимодействии фосфидов с кислотами выделяется газообразный РН3. Галогениды фосфора(\/) разлагаются водой с образованием ортофосфорной и галогеноводородной кислоты; при щелочном гидролизе получаются соли этих кис- лот: РС15+ 4Н2О = Н3РО4+ 5HCI РС15 + 8КОН = К3РО4 + 5KCI + 4Н2О Для фосфора характерны восстановительные свойства. Так, он окисляется до высшей кислоты при действии концентрированной серной или азотной кислоты: 2Р + 5Н25О4(конц.) = 2Н3РО4+ 5SO,T + 2Н2О Р + 5Н1\Ю3(конц.) = Н3РО4 + 5NO2T + Н2О Фосфор окисляется и другими окислителями: например, перманганатом калия, пероксидом водорода, хлоратом калия. Во всех случаях образуются соединения фосфора в высшей степени окисления, например: 6Р + 5КСЮ3 = ЗР2О5 +5KCI (эта реакция протекает при зажигании спички). При нагревании с растворами щелочей фосфор диспропорционирует (реакция характерна для белого фосфора): 4Р + 3NaOH + ЗН2О = РН3Т + 3NaH2PO2 В этой реакции образуются фосфин и соли фосфорноватистой кислоты — гипо- фосфиты, в составе которых фосфор имеет степень окисления +1. Фосфор получают прокаливанием фосфата кальция в электрических печах с углём (коксом) и песком: Са3(РО4)2 + 5С + 3SiO2 -U 2Р + 3CaSiO3 + 5СОТ Углерод служит в этой реакции восстановителем, a SiO2 добавляют для связыва- ния ионов кальция в силикат кальция. Фосфин Фосфин РН3 — бесцветный газ с чесночным запахом, очень ядовит. Несмотря на то что фосфин является аналогом аммиака, свойства этих веществ сильно раз- личаются. Основные свойства фосфина выражены очень слабо. Так, в отличие от аммиака, фосфин не растворяется в воде и не реагирует с ней. Взаимодействие
124 с растворами кислот для него не характерно. Соли фосфония образуются только при взаимодействии фосфина с наиболее сильными кислотами, например: РН3+ Н1 = РН41 иодид фосфония Соли фосфония очень неустойчивы. Поскольку фосфор в составе фосфина находится в своей низшей степени окис- ления -3, фосфин проявляет сильнейшие восстановительные свойства. Восстано- вительные свойства у фосфина выражены гораздо сильнее, чем у аммиака. Так, фосфин самовоспламеняется на воздухе: 2РН3 +4О2= Р2О5+ЗН2О Продукты этой реакции далее взаимодействуют друг с другом, образуя орто- фосфорную кислоту, поэтому уравнение реакции фосфина с кислородом можно записать и по-другому: 2РН3 + 4О2= 2НэРО4 Фосфин взаимодействует и с другими типичными окислителями: концентриро- ванными серной и азотной кислотами, галогенами, пероксидом водорода, пер- манганатом калия и т. д. Продуктом окисления фосфина является, как правило, фосфорная кислота или её соль (в зависимости от характера среды, в которой протекает реакция), например: РН3 + 8НМО3(конц.) = Н3РО4 + 8NO2? + 4Н2О Напомним, что фосфор с водородом практически не реагирует, поэтому полу- чить РН3 напрямую из этих веществ нельзя. Получают фосфин гидролизом фосфи- дов или при взаимодействии белого фосфора с растворами щелочей. Оксиды и кислоты фосфора Фосфор образует несколько оксидов, важнейшими из которых являются Р2О3 и Р2О5. Оба эти оксида являются кислотными. Они представляют собой белые кри- сталлические вещества. Оксид фосфора(Ш) Р2О3 образуется при медленном окислении белого фосфора кислородом воздуха. При взаимодействии этого оксида с водой образуется фос- фористая кислота: Р2О3 + ЗН2О = 2Н3РО3 Оксид фосфора(Ш) проявляет восстановительные свойства. Так, при взаимодей- ствии с кислородом он окисляется до высшего оксида: Р2О3 + О2 = Р2О5 Оксид фосфора(\/) Р2О5 представляет собой твёрдое, белое, очень гигроскопич- ное вещество; проявляет все свойства типичного кислотного оксида. Окислитель- ные свойства для него не характерны. При взаимодействии Р2О5 с водой возмож- но образование различных фосфорных кислот: Р2О5 + Н2О = 2НРО3 (метафосфорная кислота)1; Р2О5+ 2Н2О = Н4Р2О7 (пирофосфорная кислота); Р2О5+ ЗН2О = 2Н3РО4 (ортофосфорная кислота). 1 Дано упрощённое уравнение реакции. Формулу метафосфорной кислоты принято записывать как (НРО3)П.
125 Как кислотный оксид Р2О5 реагирует с основаниями и основными оксидами, на- пример: Р2О5 + ЗСаО = Са3(РО4)2 Р2О5 + 6КОН = 2К3РО4 + ЗН2О При взаимодействии Р2О5 со щелочами в зависимости от соотношения реаген- тов могут образовываться не только средние, но и кислые соли: Р2О5 + 4КОН = 2К2НРО4 + Н2О Р2О5 + 2КОН + Н2О = 2КН2РО4 с. Э £ е. 4 Несмотря на то что фосфор в составе Р2О5 находится в высшей степени окис- ления, окислительные свойства для него не характерны, так как для фосфора сте- пень окисления +5 является очень устойчивой. Оксид фосфора(\/) является одним из самых сильных водоотнимающих и водопоглащающих реагентов. Его исполь- зуют как осушитель жидкостей и газов, для проведения реакций дегидратации в органической химии и т. п. При попадании на кожу Р2О5 вызывает тяжёлые ожоги. Ортофосфорная кислота Н3РО4 представляет собой прозрачные кристаллы, рас- плывающиеся на воздухе, неограниченно растворимые в воде. Это наиболее важ- ная в практическом отношении из кислот фосфора, поэтому часто её называют просто фосфорной кислотой. В водном растворе Н3РО4 диссоциирует по трём сту- пеням: Н3РО4*±Н++Н2РС)з- н2ро4-^н++нро2- нро2-^н+ + роз- Фосфорная кислота относится к слабым кислотам. Тем не менее степень дис- социации Н3РО4 по первой ступени довольно значительна и составляет в разбав- ленном растворе лишь немногим меньше 30 %. Степень диссоциации по второй ступени существенно ниже (меньше 0,1 %), а по третьей ступени пренебрежимо мала. Фосфорная кислота проявляет все общие свойства кислот. Она взаимодейству- ет с металлами, находящимися в ряду напряжений левее водорода, с выделением водорода; с основными и амфотерными оксидами и гидроксидами, с аммиаком; вступает в реакции ионного обмена. Окислительные свойства для фосфорной кислоты и фосфатов не характерны. Фосфорная кислота образует три типа солей (табл. 18). Соли фосфорной кислоты Таблица 18 дигидрофосфаты: гидрофосфаты: фосфаты: NaH2PO4, Са(Н2РО4)2 Na2HPO4, СаНРО4 Na3PO4, Са3(РО4)2 кислые соли средние соли Например, при нейтрализации фосфорной кислоты гидроксидом калия в зави- симости от количественного соотношения реагентов могут протекать следующие реакции:
12В Н3РО4+ КОН = КН2РО4+ Н2О Н3РО4+ 2КОН = К2НРО4+ 2Н2О Н3РО4 + ЭКОН = К3РО4 + ЗН2О Фосфорную кислоту в промышленности получают действием более сильной серной кислоты на фосфат кальция: Са3(РО4)2 + 3H2SO4 = 2Н3РО4 + 3CaSO4 Как и Р2О5, фосфорная кислота и фосфаты не проявляют выраженных окисли- тельных свойств. Качественной реакцией на фосфат-ион в составе растворимых фосфатов яв- ляется реакция с нитратом серебра, приводящая к образованию жёлтого осадка Ад3РО4. Фосфат серебра растворим в кислотах, поэтому при действии нитрата се- ребра на фосфорную кислоту осадок не образуется. Фосфорная кислота и её соли широко используются при производстве мине- ральных удобрений. Наиболее распространены следующие фосфорные удобрения: смесь Са(Н2РО4)2 и CaSO4— простой суперфосфат; СаНРО4 • 2Н2О — преципитат; смесь NH4H2PO4 и (NH4)2HPO4— аммофос. Примеры заданий, при выполнении которых нужно продемонстрировать знание свойств фосфора и его соединений Пример 1. Задана следующая схема превращений веществ: Н3РО4 -Л- Na3PO4 -Л- Na2HPO4 Определите, какие из указанных веществ являются веществами X и Y 1) NaH2PO4 2) Na2O 3) HCI(изб.) 4) NaCI 5) РН3 Для осуществления первого из превращений нужно выбрать вещество, со- держащее натрий и взаимодействующее с фосфорной кислотой. Таким ве- ществом является основный оксид Na2O: 2Н3РО4 + 3Na2O = 2Na3PO4 + ЗН2О В перечне веществ предложен также хлорид натрия, однако взаимодействие соли с кислотами подчиняется закономерностям реакций ионного обмена, в ходе которых обязательно должны получиться нерастворимое вещество, газ или слабый электролит. Ни одно из этих условий для взаимодействия NaCI с Н3РО4 выполнено не будет, следовательно, реакция между ними не про- текает. По этой же причине невозможно взаимодействие Н3РО4 и NaH2PO4. Второе превращение представляет собой переход средней соли в кислую соль. Логично предположить, что подходящим реагентом будет кислота — HCI. Однако в условии указано, что кислота взята в избытке; в этом случае она полностью вытеснит более слабую фосфорную кислоту из её соли: Na3PO4 + 3HCI = Н3РО4 + 3NaCI Нужным реагентом в данном случае будет дигидрофосфат натрия. Протека- ющий процесс отразим сокращённым ионным уравнением: Н2РО4 + РОз- = 2НРО2-
127 Видно, что в данном случае происходит образование гидрофосфат-иона НРО|-, являющегося более слабым электролитом, чем дигидрофосфат-ион Н2РО4. Ответ: X Y 2 1 Пример 2. Установите соответствие между исходными веществами продуктами, которые преимущественно образуются в ходе реакции. и ИСХОДНЫЕ ВЕЩЕСТВА A) Na3PO4+Н3РО4(изб.)-> Б) Na3PO4 + NaH2PO4 -> В) KNO34 Г) (NH4)2co34 ПРОДУКТЫ РЕАКЦИИ 1) 2) 3) 4) 5) 6) KNO2 + О2 NH3+CO2 + H2O N2+CO2 + H2O + H2 NaH2PO4 К2О + NO2 + 02 Na2HPO4 Определить продукты реакций А и Б средних солей. При взаимодействии фосфата натрия с избытком фосфорной кислоты образуется дигидрофосфат натрия: Na3PO4 + 2Н3РО4 = 3NaH2PO4 Взаимодействие фосфата натрия с ди гидрофосфатом натрия приводит к получению гидрофосфата натрия: Na3PO4 + NaH2PO4 = 2Na2HPO4 Характерной особенностью как нитратов, так и солей аммония является их способность к разложению при нагревании. Известно, что нитраты щелочных металлов разлагаются с образованием соответствующего нитрита и кислорода: 2KNO3 = 2KNO2 + О2 Состав продуктов разложения солей аммония зависит от того, проявляет ли кислотный остаток окислительные свойства. Если это так, то протекает внутримолекулярная окислительно-восстановительная реакция, причём про- дуктом окисления атома азота, входящего в состав катиона аммония, чаще всего бывает N2. Если в состав соли входит кислотный остаток, не прояв- ляющий окислительных свойств, то разложение происходит без изменения степеней окисления элементов и сопровождается выделением газообразно- го аммиака. Именно к таким солям и относится карбонат аммония. Его раз- ложение протекает в соответствии со следующим уравнением: (NH4)2CO3 = 2NH3 + СО2 + Н2О помогут знания о свойствах кислых и I' Ответ: А Б в г 4 6 1 2
128 Пример 3. К раствору дигидрофосфата кальция добавили избыток рас- твора гидроксида кальция. Выпавший осадок отделили, высушили и прока- лили с оксидом кремния и углеродом. Образовавшееся простое вещество прореагировало при нагревании с избытком хлора. Продукт реакции обра- ботали избытком водного раствора гидроксида натрия. Напишите уравнения четырёх описанных реакций. При взаимодействии кислой соли — ди гидрофосфата кальция — с избытком раствора щёлочи образуется средняя соль. Са(Н2РО4)2 + 2Са(ОН)2 = Са3(РО4)2Ф + 4Н20 Обратите внимание на то, что дигидрофосфат кальция растворим в воде, а фосфат кальция — нет, поэтому образуется осадок Са3(РО4)2. Прокаливание Са3(РО4)2с оксидом кремния и углеродом является способом получения фосфора: Са3(РО4)2 + 5С + 3SiO, 4 2Р + 3CaSiO3 + 5СОТ При взаимодействии фосфора с избытком хлора при повышенной темпера- туре преимущественно образуется хлорид фосфора(У): 2Р + 5С12 = 2РС15 Хлорид фосфора(У) подвергается щелочному гидролизу: PCL + 8NaOH = Na3PO4 + 5NaCI + 4Н2О 5 3 4 2 ЗАНЯТИЕ 13 Неметаллы IVA-группы: углерод, кремний Основные элементы содержания: общая характеристика углерода и кремния в связи с их положением в Периодической системе Д. И. Менделеева; аллотропные модификации углерода, химические свойства углерода и его соединений: метана, оксида углерода(П) и оксида углерода(1\/), угольной кислоты и её солей; химические свойства кремния и его соединений: силана, оксида кремния(1\/), кремниевой кис- лоты и её солей. УГЛЕРОД И ЕГО СОЕДИНЕНИЯ Общая характеристика углерода в связи с его положением в Периодической системе Д. И. Менделеева Элемент углерод расположен во 2-м периоде и IVA-группе Периодической си- стемы Д. И. Менделеева. На внешнем электронном уровне атома углерода на- ходятся четыре электрона. В основном состоянии атома два из них не спарены
129 (схема 16, слева). За счёт этих неспаренных электронов атом углерода может образовы- вать две ковалентные связи и проявлять ва- лентность II. Ещё одна ковалентная связь мо- жет образоваться по донорно-акцепторному механизму за счёт неподелённой электрон- ной пары, в этом случае валентность углеро- да будет равна трём. Эта валентность атома углерода реализуется, например, в молекуле оксида углерода(П). Однако наиболее харак- терна для углерода валентность IV, соответ- ствующая возбуждённому состоянию его ато- ма (схема 16, справа). Схема 16 Электронная конфигурация атома углерода в основном и возбуждённом состоянии 12с 1822822р2 12С ls22s12p3 п = 2 2з основное состояние 2р 2s возбуждённое состояние п = 2 t t 1 ♦ Таблица 19 Важнейшие соединения, образованные углеродом в различных степенях окисления -4 —1 +2 +4 СН4 — метан; А14С3 — карбид алюминия С2Н2 — ацетилен; СаС2 — карбид кальция СО — оксид углерода(П), угарный газ СО2 — оксид углерода(1\/), углекис- лый газ; Н2СО3 — угольная кислота; соли угольной кислоты — карбонаты и гидрокарбонаты: К.СО,, кнсо3 В соответствии с положением в Периодической системе и строением внешне- го электронного слоя углерод проявляет высшую степень окисления +4 и низшую степень окисления -4. Углерод образует соединения и в других степенях окисле- ния (табл. 19). Углерод находится в природе как в свободном состоянии, так и в виде мно- гочисленных соединений — как неорганических, так и органических. Среди не- органических соединений углерода в земной коре наиболее распространены карбонаты: СаСО3 (мел, мрамор, известняк), МдСО3 (магнезит). Минерал ма- лахит образован оснбвной солью — карбонатом гидроксомеди(Н) (СиОН)2СО3. В атмосферном воздухе находится углекислый газ СО2. Хотя его содержание и не- велико — всего около 0,04% по объёму — он играет важную роль в поддержании мягкого климата на нашей планете, поскольку является одним из парниковых га- зов. Ископаемые угли — антрацит, каменный уголь, бурый уголь — содержат от 70 до 95% углерода, а также множество других неорганических и органических ве- ществ. Органические соединения — углеводороды и их производные — основные компоненты нефти и природного газа. И конечно, органические соединения входят в состав всех растительных и животных организмов. В организме человека массой 70 кг содержится примерно 16 кг углерода. Простые вещества, образованные углеродом Углерод образует несколько простых веществ. Встречающиеся в природе кри- сталлические формы углерода — это алмаз и графит.
130 Алмаз — бесцветное кристаллическое вещество с атомной кристаллической ре- шёткой (рис. 12, а). Электронные орбитали атомов углерода в кристалле алмаза находятся в состоянии зр3-гибридизации, каждый атом углерода связан прочны- ми ковалентными неполярными связями с четырьмя другими атомами углерода. Такое строение обусловливает высочайшую твёрдость алмаза: по этому показателю алмаз превосходит все другие известные вещества. Технические алмазы широко применяют для обработки других твёрдых материалов, резки стекла, бурения гор- ных пород, шлифовки драгоценных камней. Кристаллы алмаза сильно преломляют лучи света, поэтому обладают очень сильным красивым блеском. Специальным образом отшлифованные алмазы называются бриллиантами. Технические алма- зы получают из графита при очень высоких температурах и сверхвысоких давле- ниях. Графит представляет собой тёмно-серое вещество со слабым металлическим блеском. Его кристаллическая решётка, так же как и у алмаза, является атомной, однако имеет слоистое строение (рис. 12, б). Электронные орбитали атомов угле- рода в кристалле графита находятся в состоянии sp2-гибридизации. Внутри слоёв атомы углерода связаны друг с другом прочными ковалентными связями, а связи между атомами, находящимися в разных слоях, значительно слабее. Поэтому слои атомов в кристалле графита достаточно легко отделяются друг от друга. Когда мы проводим кусочком графита по бумаге, тонкие чешуйки графита отщепляются от кристалла и остаются на её поверхности. На этом явлении основано применение графита в качестве карандашных грифелей. Графит хуже, чем металлы, но всё же довольно хорошо проводит электрический ток. Из него изготавливают электроды для промышленных электролизеров, используют в химических источниках тока. Другие формы углерода — древесный уголь, кокс и сажа имеют неупорядочен- ную структуру. Они состоят из мельчайших частичек, строение которых подобно графиту; обычно эти вещества содержат значительное количество примесей, су- щественно влияющих на их свойства. Древесный уголь имеет пористую структуру. При его нагревании до высоких температур в присутствии водяного пара получают так называемый активированный уголь, обладающий ещё большей пористостью. Активированный уголь обладает очень высокой адсорбционной (поглощающей) способностью. Он применяется в противогазах, фильтрах для очистки воды и дру- гих жидкостей от нежелательных примесей. В медицине таблетки активированного Рис. 12. Модели кристаллических решёток алмаза (а) и графита (б)
131 Рис. 13. Строение фуллерена (а), графена (б), одностенной углеродной нанотрубки (в) угля применяются при отравлениях для поглощения токсичных веществ из желу- дочно-кишечного тракта. Известны и другие модификации углерода: карбин, фуллерены, графен, углерод- ные нанотрубки (рис. 13). Все эти модификации углерода получены искусственным путём. Их свойства в настоящее время интенсивно изучаются. Далее речь пойдёт о химических свойствах углерода в виде графита и кокса. Химические свойства углерода При обычных условиях углерод химически малоактивен. При нагревании он всту- пает в реакции с кислородом, водородом, серой, другими неметаллами: t С + О2 = СО2 (горение на воздухе) С + 2Нг = СН4 С + 2S = CS2 (сероуглерод) Активные металлы при взаимодействии с углеродом образуют карбиды, напри- мер: t 4AI + ЗС = А14С3 Са + 2С = СаС2 Напомним, что при обработке водой или растворами кислот карбид алюминия и карбид кальция подвергаются гидролизу с выделением метана и ацетилена со- ответственно: AI4C3 + 12Н2О = 4А1(ОН)3 + ЗСН4Т СаС2 + 2Н2О = Са(ОН)2 + С2Н2Т В промышленности карбид кальция получают прокаливанием оксида кальция с коксом в электрических печах: СаО + ЗС = СаС2 + СО В целом для углерода характерны восстановительные свойства. Так, кокс вос- станавливает железо, медь, цинк, свинец и многие другие металлы из их оксидов, что используется в металлургии для получения этих металлов, например: 2СиО + С = 2Си + СО2 2ZnO + С = 2Zn + СО2
132 С растворами кислот, солей, щелочей уголь не взаимодействует, однако он окисляется концентрированными серной и азотной кислотами при нагревании: С + 2Н28О4(конц.) = CO2t + 2SO2? + 2Н2О С + 4НМО3(конц.) = CO2t + 4NO2T + 2Н2О Метан Метан СН4 — бесцветный нерастворимый в воде газ без запаха. Он не прояв- ляет ни кислотных, ни оснбвных свойств. В большинстве реакций он выступает в качестве восстановителя. Метан горит синеватым пламенем: СН4+2О2-> СО2+2Н2О При горении метана выделяется большое количество теплоты, поэтому он явля- ется важнейшим энергоносителем, широко используется в качестве топлива. Не- обходимо помнить, что смесь метана с воздухом взрывоопасна. При нагревании без доступа воздуха метан разлагается с образованием сажи: СН4 Д С + 2Н2 Путём взаимодействия метана с водяным паром (конверсии) получают смесь оксида углерода(П) и водорода — синтез-газ: t, р, кат. СН4 + Н2О ц > со + зн2 Синтез-газ является важнейшим сырьём для промышленного получения органи- ческих веществ. Более подробно свойства метана рассматриваются в курсе органической химии. Оксид углерода(Н) Оксид углерода(П), или угарный газ, СО образуется при неполном сгорании угле- рода или углеводородов. В молекуле угарного газа атомы углерода и кислорода связаны между собой тройной связью. Одна из этих связей образована по донор- но-акцепторному механизму за счёт неподелённой электронной пары атома кисло- рода и свободной орбитали атома углерода (схема 17). Схема 17 Образование химической связи в молекуле оксида углерода(Н) свободная орбиталь с щЕП© □ -»--------------------------- -----► : С......О: -------* С ^== О z^jН| 11 11 °Г неподелённая электронная пара
133 Оксид углерода(П) — это бесцветный нерастворимый в воде газ. Являясь не- солеобразующим оксидом, он не взаимодействует ни с водой, ни с кислотами, ни со щелочами. Реакционная способность СО связана с его восстановительной активностью. Оксид углерода(П) горит на воздухе синеватым пламенем: 2СО + О2=2СО2 Его используют в промышленности в качестве восстановителя при получении металлов из их оксидов, например: t CuO + СО = Си + СО2 t Fe2O3 + ЗСО = 2Fe + ЗСО2 Другое важнейшее направление промышленного использования СО — синтез разнообразных органических соединений, например метанола, из его смеси с во- дородом (синтез-газа): t, р, кат. СО + 2Н2 < » СН3ОН В лаборатории небольшие количества СО можно получить при взаимодействии муравьиной НСООН и концентрированной серной кислот (серная кислота выступа- ет в этой реакции в качестве водоотнимающего агента): Н,8О.(конц.) НСООН -----------► СО + Н2О Эта реакция показывает, что СО можно условно считать ангидридом муравьи- ной кислоты. Хотя муравьиную кислоту нельзя получить взаимодействием угарно- го газа и воды, но при взаимодействии твёрдых щелочей с СО при температуре • 150—200 °C образуются соли этой кислоты — формиаты: NaOH + СО = HCOONa формиат натрия Угарный газ образуется также при взаимодействии углекислого газа с углём при высоких температурах: f С + СО2 = 2СО Угарный газ очень ядовит. Оксид углерода(И) легко соединяется с гемоглоби- ном крови и блокирует его способность переносить кислород. СО не имеет запа- ха и, поскольку отравление может произойти незаметно, особенно опасен. Важно помнить, что угарный газ может образоваться при неполном сгорании в условиях недостатка воздуха любого углеродсодержащего топлива, в том числе бензина в двигателях внутреннего сгорания, и соблюдать меры предосторожности. При по- дозрении на отравление угарным газом пострадавшего нужно немедленно выне- сти на свежий воздух и обратиться за медицинской помощью. Оксид углерода(1У), угольная кислота и её соли Оксид углерода(1\/) при обычных условиях представляет собой бесцветный газ. При температурах ниже -78,5 °C он существует в виде похожего на снег твёрдого вещества — «сухого льда». В этом состоянии СО2 состоит из мелких кристалликов, имеющих молекулярное строение. При повышении температуры «сухой лёд», не переходя в жидкое состояние, испаряется с образованием газообразного СО2.
134 Оксид углерода(1У) — кислотный оксид. Он немного растворим в воде; его реак- ция с водой, приводящая к образованию угольной кислоты, является обратимой: СО2 + Н,О Н2СО3 Как кислотный оксид СО2 реагирует с оснбвными оксидами и с основаниями. Образуются соли угольной кислоты — карбонаты: СаО + СО2 = СаСО3 При взаимодействии СО2 со щелочами в зависимости от соотношения реагентов могут образовываться не только средние, но и кислые соли. Если в избытке на- ходится щёлочь, то образуются карбонаты, например: 2NaOH + СО2 = Na2CO3 + Н2О Ва(ОН)2 + СО2 = ВаСО3 + Н2О А если через раствор щёлочи пропустить избыток СО2, то получатся кислые соли — гидрокарбонаты: NaOH + СО2 = NaHCO3 Ва(ОН)2 + 2СО2= Ва(НСО3)2 Обратите внимание на то, что карбонаты щелочноземельных металлов в воде не растворяются, а соответствующие гидрокарбонаты растворимы. Если пропускать угле- кислый газ через раствор Са(ОН)2 (так называемую известковую воду), сначала будет наблюдаться помутнение раствора из-за образования нерастворимого СаСО3, а за- тем раствор вновь станет прозрачным вследствие образования раствора Са(НСО3)2: Са(ОН)2 + СО2 = СаСО31 + Н2О СаСО3 + Н2О + СО2 = Са(НСО3)2 (р-р) Это качественная реакция на СО2. Угольная кислота Н2СО3 существует только в растворе. Это очень неустойчивое вещество, поэтому при действии растворов кислот на карбонаты выделяется про- дукт её разложения — углекислый газ: Na2CO3 + 2HCI = 2NaCI + Н2О + СО2? Са(НСО3)2 + 2HNO3 = Ca(NO3)2 + 2Н2О + 2СО2Т Угольная кислота относится к слабым кислотам. Особенностью солей угольной кислоты является их термическая неустойчивость. Все карбонаты, кроме карбонатов щелочных металлов, разлагаются на оксиды ме- таллов и углекислый газ, например: t СаСО, = СаО + СО2Т (CuOH)2CO3 = 2СиО + СО2Т + Н2О Карбонаты щелочных металлов можно расплавить без их разложения. А вот гидрокарбонаты щелочных металлов при нагревании переходят в карбонаты: 2NaHCO3 = Na2CO3 + СО2Т+ Н2О При несильном нагревании, например при кипячении раствора, в карбонаты переходят и гидрокарбонаты щелочноземельных металлов: Са(НСО3)2 = СаСО3 + СО2Т+ Н2О ж О' £ О £ £
135 Жёсткая вода всегда содержит некоторое количество растворённых гидрокар- бонатов кальция и магния. При нагревании такой воды они переходят в нераство- римые карбонаты, которые оседают на стенках сосудов в виде накипи. Поскольку карбонаты и гидрокарбонаты образованы слабой угольной кисло- той, для них характерен гидролиз по аниону. При смешивании растворов солей железа(Ш), хрома(Ш) или алюминия с растворами карбонатов протекает совмест- ный гидролиз. При этом в осадок выпадает гидроксид соответствующего металла, а СО2 выделяется в виде газа, например: 2AICL + ЗК,СО, + ЗН2О = 2А1(ОН)3 + 6KCI + ЗСО2Т КРЕМНИЙ И ЕГО СОЕДИНЕНИЯ Кремний занимает второе место по распространённости в земной коре после кислорода. В отличие от углерода в природе кремний в виде простого вещества не встречается. Он входит в состав соединений, главным образом оксида кремния(1\/) и солей кремниевой кислоты — силикатов. В соответствии с положением в Пери- одической системе и строением внешнего электронного слоя кремний, как и угле- род, проявляет высшую степень окисления +4 и низшую степень окисления -4. Наиболее устойчивы и распространены соединения кремния в степени окисления +4. Кремний как простое вещество Наиболее известны две аллотропные модификации кремния: кристаллический кремний и аморфный. Кристаллический кремний — вещество серого цвета с металлическим блеском, химически малоактивен. Аморфный кремний — порошок белого цвета, который из-за присутствия при- месей может иметь коричневый цвет, обладает более высокой химической ак- тивностью, чем кристаллический. Обе модификации кремния имеют атомную кристаллическую решётку, поэтому обладают хрупкостью и высокой твёрдостью. Кристаллический кремний — полупроводник. При обычных условиях кремний химически малоактивен. Даже с кислородом он взаимодействует только при сильном нагревании, с водородом кремний в реак- цию не вступает. Без нагревания он реагирует только с фтором: Si + 2F2=SiF4 При нагревании кремний реагирует с активными металлами с образованием си- лицидов: t 2Mg + Si = Mg2Si силицид магния Силициды разлагаются растворами кислот с выделением газообразного соеди- нения кремния с водородом — силана: Mg2Si + 4HCI = 2MgCI2 + SiH4t силан Также при очень высоких температурах кремний соединяется с углеродом: t Si + С = SiC карбид кремния
136 Карбид кремния SiC называют также карборундом. Карборунд имеет алмазопо- добную атомную кристаллическую решётку. Это очень твёрдое вещество. На его ос- нове производят, например, точильные бруски, шлифовальную (наждачную) бумагу. Растворы кислот на кремний не действуют. При нагревании он реагирует только с HF: t Si + 4HF = SiF4 + 2Н2 А вот с растворами щелочей кремний реагирует довольно энергично: Si + 2NaOH + Н2О = Na2SiO3+ 2Н2Т Кремний можно получить при нагревании SiO2c восстановителями, например с углеродом или с активными металлами: SiO2 + 2С = Si + 2СО t SiO2 + 2Mg = Si + 2MgO Кремний обладает полупроводниковыми свойствами и широко используется в электронике и микроэлектронике, для изготовления солнечных батарей. Соединения кремния Соединение кремния с водородом SiH4 носит название силан. Это бесцветный неустойчивый нерастворимый в воде газ. Силан — сильный восстановитель, он са- мовоспламеняется на воздухе и сгорает с образованием оксида кремния и воды: SiH4 +2О2 = SiO2+2Н2О Наиболее распространённым соединением кремния является оксид кремния(1\/) SiO2 — кремнезём. Кристаллы кварца, горного хрусталя, агата, яшмы, речной и морской песок образованы именно этим веществом. Кремнезём имеет атомную кристаллическую решётку, это твёрдое тугоплавкое вещество. Оксид кремния(1\/) является кислотным оксидом, но не взаимодействует с во- дой. При его взаимодействии с растворами или расплавами щелочей образуются соли кремниевой кислоты — силикаты: SiO2 + 2NaOH = Na2SiO3 + Н2О Силикаты образуются также при нагревании SiO2 с оксидами и карбонатами ак- тивных металлов: . SiO2 + СаО = CaSiO3 SiO2 + Na2CO3 = Na2SiO3 + CO2 Особенностью оксида кремния является его взаимодействие с HF: SiO2 +4HF = SiF4+2Н2О На протекании этой реакции основано травление стекла с помощью плавиковой кислоты. Поскольку SiO2 не взаимодействует с водой, соответствующую ему кремниевую кислоту можно получить только косвенным путём, действуя кислотой на раствор силиката калия или натрия: Na2SiO3 + 2HCI = H2SiO34< + 2NaCI Кремниевая кислота нерастворима в воде, она является слабой неустойчивой кислотой. При нагревании H2SiO3 постепенно разлагается на оксид кремния и воду: H2SiO3 = Н2О + SiO2
137 Среди солей кремниевой кислоты растворимы в воде только силикаты натрия и калия. Их называют растворимым стеклом, а водные растворы этих солей — жид- ким стеклом. Силикаты широко распространены в природе. К природным силика- там относятся полевой шпат, слюда, асбест, тальк и др. Силикаты входят в состав целого ряда горных пород: гранита, базальта и др. В технике на основе силикатов изготовляют стекло, цемент, другие материалы. Примеры заданий, при выполнении которых нужно продемонстрировать знание свойств углерода, кремния и их соединений Пример 1. Установите соответствие между реагирующими веществами и продуктами их взаимодействия. РЕАГИРУЮЩИЕ ВЕЩЕСТВА А) Са(НСО3)2+Са(ОН)2-> Б) Ca(HCO3)2+H2SO4-> В) Са(НСО3)2 4 Г) СаСО3+СО2+Н2О-> ПРОДУКТЫ ВЗАИМОДЕЙСТВИЯ 1) СаСО3 + Н2О 2) СаСО3 + СО2 + Н2О 3) CaSO4 + Н2О 4) CaSO4 + CO2+H2O 5) СаСО3 + СО2+Н2 6) Са(НСО3)2 При выполнении этого задания потребуется применить знание свойств кис- лых и средних солей угольной кислоты — карбонатов и гидрокарбонатов. Кислые соли могут взаимодействовать с основаниями с образованием сред- них солей, поэтому в примере А продуктами реакции будут карбонат каль- ция и вода: Са(НСО3)2+ Са(ОН)2= 2СаСО3+ 2Н2О И гидрокарбонаты, и карбонаты взаимодействуют с кислотами с выделением СО2. В примере Б продуктами реакции являются сульфат кальция, углекис- лый газ и вода: Са(НСО3)2 + H2SO4 = CaSO4 + 2СО2 + 2Н2О Характерным свойством гидрокарбонатов является их способность разла- гаться с образованием карбонатов при слабом нагревании: Са(НСО3)2 -U- СаСО3 + СО2 + Н2О Такая реакция протекает, например, при кипячении жёсткой воды. Наконец, возможен и обратный переход карбоната кальция в гидрокарбонат. Это происходит в том случае, если пропускать углекислый газ через взвесь СаСО3 в воде. Поскольку гидрокарбонат кальция в отличие от карбоната рас- творим, наблюдается исчезновение мути, образованной частичками СаСО3: СаСО, + СО2 + Н2О = Са(НСО3)2 О £ с- * £ Ответ:
138 СО СО2 SiO2 Na2SiO3(p-p) Пример 2. Установите соответствие между формулой вещества и реаген- тами, с каждым из которых это вещество может взаимодействовать. ФОРМУЛА ВЕЩЕСТВА А) Б) В) Г) 2) 3) 4) 5) РЕАГЕНТЫ HF, Mg, BaO H2O, C, Na2CO3(p-p) H2, Fe3O4, O2 H2, N2O5, Ca(OH)2 H2SO4, CO2, CaCI2 При выполнении этого задания следует учитывать не только общие химиче- ские свойства несолеобразующих и кислотных оксидов, но и характерные особенности каждого из предложенных веществ. Оксид углерода(П) — несолеобразующий оксид. Он не взаимодействует ни с водой, ни с кислотами, ни с растворами щелочей. Однако СО горит, ис- пользуется как восстановитель при получении металлов из их оксидов, вза- имодействует с водородом с образованием метанола. Следовательно, веще- ства, с которыми будет вступать во взаимодействие оксид углерода(Н), — это Н2, Fe3O4, О2 (ответ 3). Оксид углерода(1\/) — кислотный, поэтому его взаимодействие с кисло- тами и кислотными оксидами невозможно; СО2 не проявляет восстанови- тельных свойств, следовательно, и с кислородом он тоже не реагирует. По этой причине можно исключить из рассмотрения все варианты отве- тов, кроме варианта 2. Проверим, протекают ли реакции СО2 с предло- женными реагентами. Как кислотный оксид СО2 может взаимодейство- вать с водой с образованием угольной кислоты. Взаимодействие с углём и раствором карбоната натрия описывается следующими уравнениями реакций: С + СО2 = 2СО Na2CO3 + Н2О + СО2 = 2NaHCO3 Оксид кремния(1\/) — кислотный, поэтому взаимодействует с основными ок- сидами и щелочами. Особенность SiO2 в том, что он не реагирует с водой, но вступает во взаимодействие с НЕ Кроме того, SiO2 можно восстановить до кремния под действием активных металлов, например магния. Таким об- разом, подходит вариант ответа 1. Наконец, силикат натрия — соль, поэтому для неё характерно протекание реакций ионного обмена. В этом случае важно обратить внимание на то, что в каждой из реакций должен образоваться или осадок, или газ, или слабый электролит. Эти условия выполняются для H2SO4, СО2 и СаС12: Na2SiO3 + H2SO4 = H2SiO3>l + Na2SO4 Na2SiO3+ H2O + CO2= H2SiO3>L + Na2CO3 Na2SiO3 + CaCI2 = CaSiO34- + 2NaCI Ответ: А Б В г 3 2 1 5
139 Пример 3. Установите соответствие между двумя веществами и призна- ком реакции, протекающей между ними. ВЕЩЕСТВА A) Na2CO3(p-p) и А1С13(р-р) Б) Са(НСО3)2(р-р) и КОН В) Na2SiO3 и HNO3 Г) СаСО3, Н20 и С02 ПРИЗНАК РЕАКЦИИ 1) выделение бесцветного газа 2) и образование осадка, и выделение газа 3) образование осадка 4) растворение осадка 5) видимые признаки реакции отсутствуют При смешивании раствора хлорида алюминия с раствором карбоната натрия протекает совместный гидролиз, при этом наблюдается как образование осадка, так и выделение газа: 2AICI3 + 3Na2CO3 + ЗН2О = 2А1(ОН)3+ + 6NaCI + ЗСО2Т Кислая соль Са(НСО3)2 под действием щёлочи переходит в среднюю соль. Образующийся карбонат кальция нерастворим, поэтому наблюдается обра- зование осадка: Са(НСО3)2 + 2КОН = СаСО34< + К2СО3 + 2Н2О При действии кислоты на раствор силиката натрия образуется осадок крем- ниевой кислоты: Na,SiO3+ 2HNO3= H2SiOj + 2NaNO3 Если пропускать углекислый газ через взвесь СаСО3 в воде, то образуется гидрокарбонат кальция, который в отличие от карбоната растворим. Наблю- дается растворение осадка: СаСО3 + СО2 + Н2О = Са(НСО3)2 Ответ: А Б в г 2 3 3 4 Пример 4. Кремний сожгли в атмосфере хлора. Полученный хлорид об- работали водой. Выделившийся при этом осадок прокалили. Сухой остаток прокалили с фосфатом кальция и углём. Составьте уравнения четырёх опи- санных реакций. При взаимодействии кремния с хлором образуется хлорид кремния(1\/): Si + 2CI2=SiCI4 Полученное вещество, как и многие бинарные соединения, подвергается ги- дролизу: SiCI4 + ЗН2О = H2SiO3>l + 4HCI Кремниевая кислота разлагается при нагревании: H2SiO3 = SiO2 + Н2О Прокаливанием фосфата кальция в электрических печах с углём (коксом) и песком получают фосфор: Са3(РО4)2 + 5С + 3SiO2 = 2Р + 3CaSiO3 + 5СОТ ihj®
140 ЗАНЯТИЕ 14 Общие свойства металлов. Металлы А-групп Основные элементы содержания: общая характеристика металлов 1А-ША-групп в связи с их положением в Периодической систе- ме химических элементов Д. И. Менделеева и особенностями строения их атомов. Характерные химические свойства простых веществ — металлов: щелочных, щелочноземельных, алюминия; общие способы получения металлов. Положение металлов в Периодической системе элементов Д. И. Менделеева К элементам-металлам относятся все s-элементы, кроме водорода и гелия, все элементы побочных подгрупп (d- и f-элементы), а также значительная часть р-элементов. Свойства металлов обусловлены следующими особенностями строения их атомов: • относительно большие радиусы атомов; сравним, например, радиусы атомов щелочного металла цезия и наиболее активного неметалла фтора: R^Cs) = 0,26 нм, RaT(F) = 0,06 нм; • на внешнем слое атомов активных металлов находится 1—3 электрона; из-за большого радиуса атома эти электроны довольно слабо связаны с ядром, по- этому атомы металлов легко отдают эти электроны. Таким образом, для металлов характерна способность отдавать электроны, т. е. проявление восстановительных свойств. Чем активней атом металла отдаёт элек- троны, тем сильнее выражены его металлические свойства. Химическая активность s- и p-металлов в подгруппах растёт сверху вниз. В периодах с возрастанием за- ряда ядра атомов (слева направо) металлические свойства ослабевают. Наиболее сильными металлическими свойствами обладают находящиеся в IA-группе Периодической системы щелочные металлы Li, Na, К, Rb, Cs, Fr. Они очень активны химически: горят на воздухе, реагируют при обычных условиях с водой. Их приходится хранить в запаянных ампулах или под слоем керосина. Четыре металла IIA-группы Периодической системы — Са, Sr, Ba, Ra — называ- ют щелочноземельными. Как и щелочные металлы, они весьма химически активны и способны взаимодействовать с водой в обычных условиях. Находящиеся в IIIA- и IVA-rpynnax p-металлы, например алюминий, олово, сви- нец, имеют меньшую химическую активность, чем щелочные и щелочноземельные металлы. Также менее активны находящиеся в побочных подгруппах d-металлы, такие, как железо, хром, никель, медь, цинк, платина и многие другие. Металлы обладают целым рядом характерных физических свойств, таких, как: • высокие электропроводность и теплопроводность; • пластичность, ковкость; • металлический блеск. Наличие этих свойств объясняется особенностями металлической связи и метал- лической кристаллической решётки. Напомним, что в узлах металлической кристал- лической решётки находятся атомы и ионы металлов. Валентные электроны внутри кристаллической решётки обобществляются, образуя электронное облако, общее для всего кристалла металла. Общее электронное облако продолжает связывать атомы и ионы металлов, даже если они в результате деформации окажутся сме-
141 щёнными со своих прежних положений. Благодаря пластичности металлы можно ковать, штамповать, прокатывать из них листы, изготавливать тончайшую проволоку. Наличие большого числа свободных электронов в металлах обусловливает так- же их электропроводность и теплопроводность. По остальным свойствам (темпе- ратуре плавления, твёрдости, плотности) металлы могут значительно различаться. Общая характеристика химических свойств металлов Напомним, что главное и наиболее общее свойство металлов состоит в их спо- собности быть восстановителями. Это свойство проявляется при взаимодействии с неметаллами, кислотами, растворами солей, другими окислителями. Схема 18 Электрохимический ряд напряжений металлов взаимодействуют с водой при обычных условиях взаимодействуют с водяным паром при* = 150-500 °C пассивируются H2SO4 (конц.) и HNO3 (конц.) на холоду Li К Ba Sr Са Na Mg(AljMn Zn(Cr )Fe)Cd Co Ni Sn Pb (H2)Cu Hg Ag Pt Au вытесняют водород из растворов кислот Для характеристики химической активности металлов можно использовать элек- трохимический ряд напряжений (схема 18). Напомним наиболее характерные хи- мические свойства металлов. 1. Металлы взаимодействуют с неметаллами кислородом, галогенами, серой, азотом, фосфором, углеродом, например: Cu + CI2 = CuCI2 ЗМд + 2Р = Мд3Р2 Fe + S = FeS 2. Наиболее активные металлы, находящиеся в ряду напряжений от Li по Na включительно, взаимодействуют с водой при обычных условиях: 2Na + 2Н2О = 2NaOH + Н2Т Ва + 2Н2О = Ва(ОН)2 + Н2Т Менее активные металлы — с магния по железо включительно — могут взаимо- действовать с водяным паром при повышенной температуре: Мд + 2Н2О = Мд(ОН)2+ + Н2Т 3Fe + 4H2O = Fe3O4 + 4H2T 3. Металлы, расположенные в ряду напряжений левее водорода, вытесняют Н2 из кислот: Zn + 2HCI = ZnCI2+ Н2Т Fe + Н2ЗО4(разб.) = FeSO4+ H2T 4. Практически все металлы, кроме золота и платины, взаимодействуют с кис- лотами-окислителями: азотной и концентрированной серной кислотами. Эти кис- лоты содержат в составе аниона элементы (S+6 и N+5), которые являются более сильными окислителями, чем ион Н+. При взаимодействии металлов с этими кис-
142 лотами Н2 не выделяется, а образуется другой продукт восстановления, например NO, NO2, SO2, H2S: Си + 4HNO3(kohu.) = Cu(NO3)2 + 2NO2T + 2H2O 2H2SO4(kohu.) + 2Ag = Ag2SO4 + SO2T + 2H2O Эти реакции могут и не сопровождаться выделением газообразных продуктов: 4Мд+ ЮНМО3(очень разб.) = 4Mg(NO3)2+ NH4NO3 + ЗН2О 4Н2ЭО4(конц.) + 3Zn = 3ZnSO4 + + 4Н2О 5. Более активные металлы вытесняют менее активные из растворов их солей. Например, железо может вытеснить медь из раствора её соли, но не вытесняет цинк или магний: CuSO4 + Fe = FeSO4 + Си Это правило неприменимо к активным металлам, расположенным в ряду напря- жений левее магния. Если внести какой-либо из этих металлов в раствор соли, будет прежде всего протекать его реакция с водой с образованием соответствую- щего гидроксида и выделением водорода. 6. Для большинства металлов взаимодействие со щелочами не характерно. Од- нако активные металлы, образующие амфотерные оксиды и гидроксиды — алю- миний и цинк, взаимодействуют с растворами щелочей. В процессе реакции образуются комплексные соли и выделяется водород: 2AI + 2NaOH + 6Н2О = 2Na[AI(OH)4] + ЗН2Т тетрагидроксоалюминат Zn + 2NaOH + 2Н2О = Na2[Zn(OH)4] + Н2Т тетрагидроксоцинкат натрия Рядом напряжений металлов следует пользоваться с учётом особенностей рас- сматриваемых процессов. Так, хотя щелочные металлы и способны взаимодей- ствовать с кислотами, в действительности такие реакции проводить чрезвычайно опасно из-за слишком бурного их протекания. Общие способы получения металлов Главная задача металлургии — получение металлов из руд. В зависимости от способов получения металлов различают пирометаллургию, гидрометаллургию и электрометаллургию. Пирометаллургия (от греческого «руг» — огонь) объединяет методы восстановле- ния металлов из их соединений при высоких температурах. В большинстве случаев в качестве исходного сырья используют оксиды металлов: CuO, NiO, Fe2O3, Fe3O4 и др. Если руда изначально содержит не оксид, а сульфид металла, то её подвергают обжигу, в ходе которого образуется оксид металла и сернистый газ, например: 2ZnS + ЗО2 = 2ZnO + 2SO2T Когда оксиды металлов восстанавливают до свободных металлов, в качестве восстановителей используют уголь (кокс), оксид углерода(П), водород: 2ZnO + С = 2Zn + СО2 t FeO + СО = Fe + СО2 WO3 + ЗН2 = W + ЗН2О
143 В качестве восстановителей иногда используют другой, более активный металл, чаще всего магний и алюминий. Если применяют алюминий, то процесс называ- ется алюминотермией, а если магний — магнийтермией: Cr2O3 + 2AI = 2Сг + А12О3 TiCI4 + 2Mg = Ti + 2MgCI2 Гидрометаллургией называют методы переработки руд металлов в водных раст- ворах. Соединение металла при этом переводят в раствор, затем металл восста- навливают, при этом часто используют электролиз. В основе электрометаллургии лежит получение металлов с помощью электро- лиза. Наиболее активные металлы — щелочные, щелочноземельные, алюминий — получают электролизом расплавов их соединений, например: 2NaCI э"ектр°лиз> 2Na ♦ С12 расплав Характерные химические свойства металлов IA- и 11А-групп Щелочные (Li, Na, К, Rb, Cs, Fr) и щелочноземельные (Са, Sr, Ba, Ra) метал- лы — это самые активные металлы и сильные восстановители. Они способны взаимодействовать с водой (см. выше) и атмосферным кислородом уже при обыч- ных условиях, поэтому их нужно хранить в запаянных ампулах или в банках под слоем керосина. Щелочные металлы довольно мягкие (режутся ножом), щелочно- земельные металлы более твёрдые. Щелочные и щелочноземельные металлы энергично взаимодействуют с про- стыми веществами-неметаллами — галогенами, серой, фосфором, азотом, водо- родом. Важной особенностью щелочных металлов (кроме лития) является то, что при горении они образуют не оксиды, а пероксиды7: 2Na + O2=Na2O2 Оксиды щелочных металлов можно получить взаимодействием их пероксидов с металлами, например: Na2O2 + 2Na = 2Na2O Оксиды щелочных металлов являются типичными основными оксидами. Они взаимодействуют с водой с образованием щелочей, а также проявляют другие свойства, характерные для оснбвных оксидов. Магний не относится к щелочноземельным металлам. Он не реагирует с водой в обычных условиях, его оксид МдО не реагирует с водой, гидроксид Мд(ОН)2 практически нерастворим. Тем не менее магний химически очень активен, для него характерны сильные восстановительные свойства. Горение магния на возду- хе сопровождается ярким белым слепящим пламенем. При высоких температурах магний способен реагировать с водой и с углекислым газом: Мд + 2Н2О = Мд(ОН)2+ Н2 2Мд + СО2 = 2МдО + С Поэтому горящий магний нельзя гасить ни водой, ни углекислотным огнетуши- телем. 7 Кроме пероксидов, при горении щелочных металлов образуются их соединения с кислородом и другого состава, например надпероксид КО2.
144 Магний используют как восстановитель при получении кремния и ряда метал- лов, например титана (магнийтермия): f 2Mg + TiCI4 = 2MgO + Ti Особенности свойств алюминия Очень важная особенность алюминия состоит в том, что на его поверхности на- ходится тонкая (толщиной всего несколько десятков нанометров!), но очень проч- ная и гибкая плёнка из А12О3, которая защищает его от взаимодействия со многими веществами. Алюминий настолько активен, что должен при комнатной температуре реагировать с водой. Но мы хорошо знаем, что в алюминиевой посуде воду можно даже кипятить без всяких признаков протекания химической реакции. От взаимо- действия с водой алюминий и предохраняет оксидная плёнка. Эту плёнку с по- верхности алюминия практически невозможно удалить механически, при контакте алюминия с кислородом воздуха она сразу же образуется вновь. Если опустить алюминиевую деталь в раствор нитрата ртути, то алюминий через некоторое время покроется слоем амальгамы — его раствора в ртути. Такой амаль- гамированный алюминий свободен от оксидной плёнки и взаимодействует с водой: 2AI + 6Н2О = 2А1(ОН)3 + ЗН2Т без оксидной плёнки Как и другие активные металлы, алюминий реагирует с неметаллами: 2AI + 312= 2АИ3 (реакция начинается при добавлении капли воды к смеси порошков алюминия и иода); t 2AI + N2 = 2AIN 4AI + ЗС = А14С3 Сульфид, нитрид и карбид алюминия подвергаются необратимому гидролизу, например: AIN + ЗН2О = А1(ОН )31 + NH3? Напомним, что алюминий пассивируется на холоду концентрированными серной и азотной кислотами. Однако с другими кислотами, в том числе с разбавленной серной и азотной, алюминий взаимодействует. Важной особенностью алюминия является то, что в отличие от большинства ме- таллов он взаимодействует с растворами щелочей: 2AI + 2NaOH + 6Н2О = 2Na[AI(OH)4] + ЗН2Т Оксид и гидроксид алюминия амфотерны. Алюминий получают электролизом раствора А12О3 в расплавленном криолите — Na^AIF • 2А12О3 —Na3AIFs » 4AI + ЗО2 электролиз Алюминий, как и магний, используют как восстановитель при получении других металлов (алюминотермия): Cr2O3 + 2AI = А12О3 + 2Сг 3Fe3O4 + 8AI = 4А12О3 + 9Fe
145 При протекании последней реакции выделяется такое большое количество те- плоты, что его достаточно для расплавления железа. Поэтому смесь порошков же- лезной окалины и алюминия (термитную смесь) можно использовать для сварки стальных изделий. Алюминий легко обрабатывается, образует разнообразные сплавы с другими металлами. Благодаря такому сочетанию свойств алюминий незаменим в совре- менной технике. Его сплавы особенно ценятся в самолётостроении, поэтому алю- миний называют «крылатым металлом». Из алюминия изготавливают кабели для линий электропередач, корпуса вагонов, посуду и др. ЗАНЯТИЕ 15 Металлы В-групп Основные элементы содержания: общая характеристика переходных металлов (меди, цинка, хрома, железа) в связи с их положе- нием в Периодической системе химических элементов Д. И. Менделеева и особенностями стро- ения их атомов. Характерные химические свойства переходных металлов (меди, цинка, хрома, железа) и их соединений. В В-группах (побочных подгруппах) Периодической системы элементов Д. И. Мен- делеева расположены так называемые переходные металлы. Их называют также d-металлами, поскольку в атомах этих элементов сначала происходит заполнение электронами не внешних орбиталей, а d-орбиталей предвнешнего электронного уровня. К переходным металлам относят также лантаноиды и актиноиды, в атомах которых происходит заполнение электронами 4f- и 5^-орбиталей. Как правило, на внешнем уровне атомов d-металлов содержится два s-электрона, которые атомы могут отдавать с образованием двухзарядных катионов; поэтому для большинства металлов побочных подгрупп характерна степень окисления +2. В отличие от элементов A-групп (главных подгрупп) у металлов побочных подгрупп валентными являются не только электроны внешнего слоя, но и d-электроны пред- внешнего слоя, благодаря чему обеспечивается большое разнообразие степеней окисления (табл. 20). В атомах меди, хрома и некоторых других элементов наблюдается «проскок» («провал») электрона. Рассмотрим это явление подробнее на примере атома меди. Известно, что полностью или наполовину заполненные подуровни обладают повышенной устойчивостью. Если бы у атома меди сохранилось два s-электрона на внешнем уровне, то для завершения Sd-подуровня не хватало бы всего одного электрона (схема 19, слева). Энергетически более выгодным оказывается переход одного из электронов с 4s- на Sd-орбиталь для полного заполнения d-подуровня; при этом полностью заполненным оказывается и весь третий уровень (схема 19, справа). Таким образом, у атома меди реализуется следующая электронная кон- фигурация: 29Cu: 1s* 2 * * *2s22p6 * В *3s23p63d104s1. В целом d-металлы характеризуются следующими особенностями (см. табл. 20): • радиусы их атомов меньше, чем у s-металлов того же периода; • электроотрицательность выше, чем у металлов главных подгрупп;
Некоторые свойства s- и d-металлов 4-го периода8 Таблица 20 s-металлы d- металлы Металл К Са Sc Ti V Cr Mn Fe Co Ni Cu Zn нм атома1 0,227 0,197 0,160 0,144 0,132 0,125 0,124 0,124 0,125 0,125 0,128 0,133 Электроотрица- тельность 0,82 1,00 1,36 1,54 1,63 1,66 1,55 1,83 1,88 1,91 1,90 1,65 Валентные электроны 4s1 4s2 3cP4s2 3cP4s2 3cP4s2 3cf>4s’ 3cP4s2 3d®4s2 3cP4s2 3d84s2 3<d104s1 (3d10)4s2 Характерные степени окисле- ния +1 +2 +3 +3, +4 +2, +3, +4, +5 +2, +3, +6 +2, +4, +6, +7 +2, +3, +6 +2, +3 +2, +3 +1, +2 . +2 Плотность, г/см3 0,9 1,6 3,0 4,5 6,1 7,2 7,4 7,9 8,9 8,9 9,0 7,1 Т , °C плавления' v 64 839 1541 1660 1887 1857 1244 1535 1495 1453 1084 419 Содержание в земной коре, % 2,1 4,1 1,6 103 0,56 1,610-2 0,01 9,5-10-’ 4,1 2-105 8-IO3 5-Ю-3 7,5-10-3 8 Данные взяты из книги Дж. Эмсли «Элементы», (М.: Мир, 1993)
147 Схема 19 Электронная конфигурация атома меди 29Cu ls22s22p63823p33d10481 • металлические свойства и восстановительная способность выражены слабее, чем у щелочных и щелочноземельных металлов, поэтому они расположены в середине и в правой части ряда напряжений; • как правило, образуют соединения в нескольких степенях окисления; однако для большинства d-металлов известна степень окисления +2. Металлы побочных подгрупп широко применяются в современной технике как конструкционные материалы, компоненты сталей и сплавов, а также в качестве ка- тализаторов. ЦИНК Цинк — один из наиболее активных металлов побочных подгрупп. В соединени- ях цинк проявляет единственную степень окисления +2. При нагревании он соединяется с кислородом, галогенами, серой. В соответ- ствии со своим положением в ряду напряжений цинк вытесняет водород из рас- творов кислот: Zn + Н28О4(разб.) = ZnSO4 + Н2Т Высокая восстановительная способность цинка проявляется в том, что при вза- имодействии с концентрированной серной кислотой сера может принять даже во- семь электронов и восстановиться до низшей степени окисления, в результате вы- деляется сероводород9: 4Zn + 5Н28О4(конц.) = 4ZnSO4 + H2ST + 4Н2О При взаимодействии цинка с азотной кислотой в зависимости от её концентра- ции могут образоваться различные продукты восстановления: NO2, NO, N2O, N2 и даже NH4NO3. Если взята горячая концентрированная азотная кислота, будет вы- деляться бурый газ — оксид азота(1\/): Zn + 4НМО3(конц.) = Zn(NO3)2 + 2NO2T + 2Н2О В случае взаимодействия цинка с очень разбавленной азотной кислотой полу- чается нитрат аммония: 4Zn + 10HNO- = 4Zn(NO3)? + NH4NO3 + ЗН2О Цинк имеет важную особенность: он взаимодействует с водными растворами щелочей. В результате реакции образуется гидроксокомплекс цинка и выделяется водород: Zn + 2NaOH + 2Н2О = Na2[Zn(OH)4] + Н2Т тетрагидроксоцинкат натрия 9 В этой реакции в зависимости от условий возможно образование серы, а также примеси SO2.
148 Цинк применяется для нанесения покрытий на железные и стальные изделия с целью защиты их от коррозии, он входит в состав латуни — сплава с медью — и других сплавов, используется в электротехнике для изготовления патронов элек- троламп, корпусов химических источников тока и т. д. Оксид и гидроксид цинка амфотерны. Свойства амфотерных оксидов и гидроксидов на примере соединений алюминия и цинка Амфотерными называются такие оксиды и гидроксиды, которые проявляют как оснбвные, так и кислотные свойства. Такие оксиды и гидроксиды взаимодействуют как с веществами, имеющими кислотные свойства, так и с веществами, прояв- ляющими оснбвные свойства. В реакциях с кислотными оксидами и с кислотами образуются соли соответствующих металлов, например: ZnO + 2HNO3 = Zn(NO3)2 + Н2О 2А1(ОН)3 + 3SO3 = AI2(SO4)3 + ЗН2О При взаимодействии амфотерных оксидов и гидроксидов с растворами щело- чей образуются комплексные соли, например: ZnO + 2NaOH + Н2О = Na2[Zn(OH)4] Zn(OH)2+ 2NaOH = Na2[Zn(OH)4] Образующаяся в данной реакции комплексная соль растворима; в растворе она практически нацело диссоциирует на катионы натрия и комплексные анионы: Na2[Zn(OH)4] = 2Na++ [Zn(OH)4]2- Рассмотрим строение комплексной частицы на примере аниона [Zn(OH)4]2*. В центре её находится катион цинка, он называется центральным атомом или ком- плексообразователем. Он окружён четырьмя гидроксогруппами: Между гидроксогруппами и катионом цинка за счёт неподелённых электронных пар атома кислорода и свободных орбиталей Zn2+ возникают ковалентные связи по донорно-акцепторному механизму (показаны стрелками). Итак, в комплексной частице [Zn(OH)4]2- центральный атом прочно связан с че- тырьмя группами ОН-. Разумеется, суммарный заряд комплексной частицы скла- дывается из зарядов иона Zn2+ и четырёх ионов ОН" и равен 2-. Такой комплексной частице для компенсации заряда требуется два положительно заряженных иона. Таким образом и образуется комплексная соль состава Na2[Zn(OH)4]. Аналогично построен и комплексный анион алюминия. Но поскольку катион алю- миния имеет заряд 3+, то суммарный заряд комплексной частицы будет равен 1-, и для компенсации заряда ей потребуется только один положительно заряженный ион: Na[AI(OH)4].
149 Рассмотренные комплексные соли имеют названия тетрагидроксоцинкат натрия и тетрагидроксоалюминат натрия соответственно (приставка тетра- означает «че- тыре»). Данные комплексные соединения относят к гидроксокомплексам, посколь- ку они образованы гидроксогруппами10. Образованием гидроксокомплексов объясняется способность алюминия и цин- ка, их оксидов и гидроксидов взаимодействовать с растворами щелочей: 2AI + 2NaOH + 6Н2О = 2Na[AI(OH)J + ЗН2Т AI2O3+ 2NaOH + ЗН2О = 2Na[AI(OH)4] AI(OH)3+ NaOH = Na[AI(OH)4] Взаимодействие расторов солей цинка и алюминия с растворами щелочей име- ет следующую особенность. Если к раствору соли алюминия или цинка постепенно (по каплям) добавлять раствор щёлочи, то будет наблюдаться сначала образова- ние осадка гидроксида металла, а затем его растворение (схема 20): AICI3 + 3NaOH = А1(ОН)3Ф +3NaCI А1(ОН)34, + NaOH = Na[AI(OH)4] Если к раствору соли алюминия добавить избыток щёлочи, то соль алюминия можно перевести в гидроксокомплекс в одну стадию: AICI3 + 4NaOH = Na[AI(OH)J + 3NaCI Если к раствору гидроксокомплекса алюминия или цинка добавлять постепенно кислоту, то наблюдается аналогичная картина: сначала выпадает осадок гидрокси- да, а затем он растворяется: Na[AI(OH)4] + HCI = А1(ОН)3Ф + NaCI + Н2О А1(ОН)34, + 3HCI = А1С13 + ЗН2О И соответственно при добавлении к гидроксокомплексу избытка кислоты сразу образуется соль алюминия: Na[AI(0H)4] + 4HCI = AICI3 + NaCI + 4Н2О Аналогичные реакции характерны и для соединений цинка. Для того чтобы получить гидроксид алюминия или цинка в виде осадка и пре- дотвратить его дальнейшее растворение, можно использовать слабые кислоты (H2S, СО2 вместо угольной кислоты) или слабое основание (NH3 • Н2О). Схема 20 Взаимные превращения соли, амфотерного гидроксида и гидроксокомплекса на примере соединений алюминия NaOH избыток 3NaOH I NaOH А1С1я 4 - А1(0Н)3| Na[Al(OH)4] раствор ЗНС1 НС1 раствор НС1 избыток 10 Образуются также гидроксокомплексы алюминия состава Na[AI(OH)4(H2O)2] и Na3[AI(OH)6].
150 Na[AI(OH)4] + CO2 = А1(ОН)3Ф + NaHCO3 AICI3 + 3NH3 • H2O = AI(OH)34, + 3NH4CI Оксиды цинка и алюминия взаимодействуют со щелочами не только в раство- рах, но и в твёрдом виде при сплавлении: ZnO + 2КОН = K2ZnO2 + Н2О AI2O3+ 2NaOH = 2NaAIO2+ Н2О Образующиеся соли называются цинкат калия и алюминат натрия соответствен- но. При растворении в воде они превращаются в соответствующие гидроксоком- плексы: K2ZnO2+ 2Н2О = K2[Zn(OH)4] NaAIO2 + 2Н2О = Na[AI(OH )4] Цинкаты и алюминаты можно получить при сплавлении соответствующих окси- дов не только со щелочами, но и с карбонатами щелочных металлов, например: t ZnO + Na2CO3 = Na2ZnO2 + CO2 AI2O3 + K2CO3 = 2KAIO2 + CO2 МЕДЬ Медь — элемент IB-группы. Это мягкий ковкий металл красного цвета, имею- щий высокую тепло- и электропроводность. В атоме меди за счёт проскока электрона 3d-подуровень полностью заполнен, а на внешнем электронном слое находится только один s-электрон: гдСи: 1 s22s22pe3s23p63d'°4s' Тем не менее в химических реакциях медь может отдавать не только 4з-электрон, но и один из электронов с ЗсУ-подуровня. Таким образом, в соединениях медь проявляет две степени окисления: +1 и +2, причём степень окисления +2 для неё более характерна. Медь расположена в ряду напряжений после водорода, т. е. относится к числу малоактивных металлов. Она не взаимодействует ни с водяным паром, ни с раз- бавленными кислотами, ни с растворами щелочей. При обычной температуре медь взаимодействует с хлором и бромом, а при на- гревании с серой, образуя соли меди(Н): CuCI2, CuBr2 и CuS соответственно. При нагревании на воздухе медь покрывается слоем чёрного оксида меди(П) СиО. Медь реагирует с кислотами-окислителями. При её взаимодействии с концен- трированной и разбавленной азотной кислотой наблюдается выделение бурого ок- сида азота(1\/) и бесцветного оксида азота(П) соответственно: Си + 4НМО3(конц.) = Cu(NO3)2 + 2NO2T + 2Н2О ЗСи + 8НМО3(разб.) = 3Cu(NO3)2 + 2NOT + 4Н2О Реакция с концентрированной серной кислотой протекает при нагревании и со- провождается выделением сернистого газа: Си + 2H2SO4 = CuSO4 + SO2? +2Н2О Наиболее устойчивы и распространены соединения меди(Н): различные соли, оксид СиО, гидроксид Си(ОН)2. Для оксида и гидроксида меди(П) характерны ос-
151 нбвные свойства11: они легко взаимодействуют с кислотами с образованием соот- ветствующих солей: CuO + H2SO4 = CuSO4+ Н2О Си(ОН)2 + 2HCI = СиС12 + 2Н2О Соединения меди(И) проявляют слабые окислительные свойства, причём в за- висимости от условий проведения реакции и силы восстановителя медь может пе- реходить как в степень окисления +1, так и восстанавливаться до металлической меди. Обычно соединения меди(1) образуются при действии иодид-иона на соли меди(Н): 2CuSO4 + 4KI = 2Cul^ + l2 + 2K2SO4 а также при нагревании Си(ОН)2с альдегидами: СН,СНО + 2Си(ОН)2 СН3СООН + Си2О>1 + 2Н2О Металлическая медь образуется при восстановлении оксида меди(Н) водоро- дом, оксидом углерода(П), коксом: t CuO + Н2 = Си + Н2О t CuO + СО = Си + СО2 t 2СиО + С = 2Си + СО2 Эти реакции лежат в основе промышленных способов получения меди. Кроме того, медь получают и электрохимически; обычно при этом проводят электролиз водного раствора сульфата меди(П): электролиз 2CuSO4 + 2Н2О = 2Cu + 2H2SO4 + О2 Медь широко применяется в технике. Особенно много её идёт на изготовление электрических проводов и для производства различных сплавов: бронзы (сплав меди с оловом), латуни (сплав меди с цинком), мельхиора (сплав меди с никелем) и др. Сплавы меди применяют для изготовления различной аппаратуры, деталей машин и инструментов, а также художественно-промышленных изделий. ХРОМ И ЕГО СОЕДИНЕНИЯ Хром — элемент VIB-группы. Строение атома хро- ма имеет интересную особенность: за счет проскока электрона в нем содержится шесть неспаренных элек- тронов (схема 21). Для соединений хрома характерна яркая разно- образная окраска. Отсюда название элемента, проис- ходящее от греческого слова chroma — цвет, краска. Известны соединения хрома во всех степенях окис- ления от +2 до +6, однако наиболее характерны сте- пени окисления +2, +3 и +6 (табл. 21). 8 Схема 21 Строение атома хрома 24Cr l«22s22p63s23p63<Z54s1 s ♦ I н и d п = 3 Н р 11 Строго говоря, оксид и гидроксид меди(П) относят к амфотерным. Однако основные свойства преобладают, а кислотные свойства выражены очень слабо. Так, Си(ОН)2 растворяется только в очень концентрированных растворах щелочей при нагревании.
152 Таблица 21 Важнейшие соединения, образованные хромом в различных степенях окисления Степень окисления хрома +2 +3 +6 Оксид СгО основный Сг2О3 амфотерный СгО3 кислотный кислотный характер оксидов и гидроксидов усиливается Гидроксид Сг(ОН)2 основный Сг(ОН)3 амфотерный Н2СгО4 хромовая кислота, Н2Сг2О7 дихромовая кислота Примеры солей СгС12, CrSO4 СгС13, Cr2(SO4)3 Кз[Сг(ОН)в] гексагидроксохромат( III) калия КСгО2 хромит калия К,СгО4 хромат калия, К2Сг2О7 дихромат калия Окислительно-восста- новительные свой- ства сильнейшие восста- новители: ♦2 +3 Сг - ё -> Сг наиболее устойчивая степень окисления; возможно проявление как окислительных, так и восстановительных свойств: +3 +€ Сг - Зё -> Сг +3 +2 Сг + ё-> Сг сильные окислители: ♦6 *3 Сг+ Зё->Сг Обратите внимание: хром образует три оксида, причём с повышением степени окисления хрома в оксиде их кислотный характер усиливается (см. табл. 21). Та- кое явление характерно и для других d-элементов, например для оксидов железа, марганца, ванадия и др. Металлический хром — блестящий, светло-серый, очень твёрдый и тугоплав- кий металл. Хотя хром находится в ряду напряжений левее водорода, в обычных условиях он малоактивен благодаря наличию на поверхности оксидной плёнки. При комнатной температуре хром медленно взаимодействует только с разбавлен- ными соляной и серной кислотами с образованием солей хрома(Н): Сг + 2HCI = CrCI2 + Н2Т Cr+H2SO4=CrSO4+H2T Под действием и концентрированной, и разбавленной азотной кислоты хром пассивируется; с растворами щелочей он не взаимодействует. Даже с галогенами реакции начинаются только при очень сильном нагревании, преимущественно об- разуются соединения хрома(Ш): 2Cr + 3CI2 = 2CrCI3 t 2Сг + ЗВг2 = 2СгВг3
153 Соединения хрома(П) Оксид хрома(П) СгО имеет основный характер, ему соответствует слабое ос- нование Сг(ОН)2. Этот гидроксид получают в виде жёлтого осадка при действии раствора щёлочи на раствор соли хрома(П): CrCI2+ 2NaOH = Сг(ОН)2Ф + 2NaCI Все соединения хрома(П) — сильнейшие восстановители; в присутствии окис- лителей они неустойчивы и легко окисляются кислородом воздуха в соединения хрома(Ш), например: 4Сг(ОН)2 + О2 + 2Н2О = 4Сг(ОН)3 Поэтому на практике соединения хрома(П) очень трудно получить, и делать это необходимо без доступа кислорода. Соединения хрома (III) В степени окисления +3 хром образует амфотерный оксид Сг2О3 и соответству- ющий ему амфотерный гидроксид Сг(ОН)3. Гидроксид хрома(Ш) образуется в виде осадка серо-зелёного цвета при дей- ствии растворов щелочей на растворы солей хрома(Ш): CrCI3 + ЗКОН = Cr(OH)3l + 3KCI Полученный осадок легко растворим как в кислотах, так и в щелочах с образо- ванием солей хрома(Ш) и гексагидроксохроматов соответственно: 2Сг(ОН)3 + 3H2SO4 = Cr2(SO4)3 + 6Н2О Cr(OH)3+ 3NaOH = Na3[Cr(OH)6] Оксид хрома(1П) — тугоплавкий порошок зелёного цвета. Тип его кристалличе- ской решётки такой же, как у корунда, поэтому он очень твёрдый и применяется для полировки металлов. В лаборатории оксид хрома(Ш) можно получить при термическом разложении дихромата аммония: (NH4)2Cr2O7 = Cr2O3 + N2T + 4Н2О Хотя Сг2О3, как и Сг(ОН)3, амфотерен, но с растворами кислот и щелочей он реагирует чрезвычайно медленно из-за особой устойчивости кристаллической структуры. Наиболее типичная реакция с его участием — сплавление с твёрдыми щелочами или с карбонатами щелочных металлов с образованием хромитов: Cr2O3 + 2NaOH = 2NaCrO2 + Н2О хромит натрия Cr2O3 + Na2CO3 = 2NaCrO2 + СО2 При действии кислот хромиты переходят в соли хрома(Ш): 2NaCrO2 + 4H2SO4 = Cr2(SO4)3 + Na2SO4 +4H2O Соли хрома(Ш) проявляют все общие свойства солей, в частности вступают в реакции ионного обмена. В растворах солей хрома(Ш), образованных анионом сильной кислоты, среда кислотная из-за гидролиза по катиону. При сливании рас- творов солей хрома(Ш) с растворами карбонатов, сульфидов, сульфитов протекает совместный гидролиз: 2СгС13 + ЗК2СО3 + ЗН2О = 2Сг(ОН)Д + 6KCI + ЗСО2Т Cr2(SO4)3 + 3Na2S + 6Н2О = 2Cr(OH)3l + 3H2ST + 3Na2SO4
154 Соединения xpoMa(VI) Оксид хрома(\/1) — тёмно-красное кристаллическое вещество, типичный кислот- ный оксид. Ему соответствуют две кислоты: хромовая Н2СгО4 и дихромовая Н2Сг2О7, существующие только в водном растворе. Это сильные кислоты. Они образуются при взаимодействии СгО3с водой, причём дихромовая кислота получается при до- бавлении небольшого количества воды, а хромовая — при дальнейшем разбавле- нии раствора. Соли хромовой кислоты называются хроматами, а соли дихромовой кислоты называются дихроматами. Растворимые хроматы жёлтые, дихроматы — оранже- вые. Хроматы существуют только в щелочной среде. При добавлении кислот они переходят в дихроматы и жёлтый раствор становится оранжевым: 2Na2CrO4 + H2SO4 = Na2Cr2O7 + Na2SO4 + H2O Если к раствору дихромата прибавить щёлочь, дихромат вновь превращается в хромат и оранжевая окраска раствора меняется на жёлтую: Na2Cr2O7 + 2NaOH = 2Na2CrO4 + Н2О Хроматы Ва2+, Pb2+, Ад+ и некоторых других металлов нерастворимы и образуют окрашенные осадки: г Na2CrO4 + Ba(NO3)3 = BaCr04i + 2NaNO3 жёлтый Окислительно-восстановительные свойства соединений хрома По мере возрастания степени окисления хрома последовательно усиливаются окислительные свойства его соединений. Так, соединения хрома(П) — сильные восстановители, под действием даже слабых окислителей превращаются в соеди- нения хрома(Ш) (см. выше). Соединения xpoMa(VI), содержащие хром в высшей степени окисления, — силь- ные окислители, легко восстанавливаются в соединения хрома(Ш): 4CrO3 + 3S = 3SO2 + 2Сг2О3 Соединения с промежуточной степенью окисления, т. е. соединения хрома(Ш), могут при взаимодействии с сильными восстановителями проявлять окислитель- ные свойства, переходя в соединения хрома(П): 2CrCI3 + Zn = 2CrCI2 + ZnCI2 При взаимодействии с сильными окислителями соединения хрома(Ш) могут про- являть восстановительные свойства, превращаясь в соединения xpoMa(VI): Сг2О3 + КСЮ3 + 4КОН = 2К2СгО4+ KCI + 2Н2О Прогнозируя продукты окислительно-восстановительных реакций с участием со- единений хрома, необходимо учитывать характер среды раствора и форму соеди- нения, которое образует хром в данной степени окисления с учётом среды (схе- ма 22). Так, хроматы и дихроматы — сильные окислители. В кислотной среде они вос- станавливаются до катиона Сг3+, образующего соль хрома(Ш) с анионом кислоты, использующейся для создания кислотной среды. Например, бромиды и иодиды окисляются дихроматом калия до свободных галогенов, сульфид-ион — до серы: К2Сг2О7 + 6KI + 7H2SO4 = Cr2(SO4)3 + 3I2 + 4K2SO4 + 7Н2О 2Na2CrO4 + 3Na2S + 8H2SO4 = Cr2(SO4)3 + 3S + 5Na2SO4 + 8H2O
155 В нейтральной и щелочной среде окислительные свойства соединений хрома(\/1) выражены слабее; продуктом восстановления будет гидроксокомплекс хрома(Ш): 2K2CrO4 + 3Zn + 8КОН + 8Н2О = 2K3[Cr(OH)6] + 3K2[Zn(OH)4] Чтобы из соединений хрома(Ш) получить соединение xpoMa(VI), нужно использо- вать сильный окислитель (пероксид водорода, бром и др.), причём такие реакции легче протекают в щелочной среде. В этом случае продуктом окисления соедине- ния хрома будет не дихромат, а хромат, поскольку именно он устойчив в щелочной среде (см. схему 22), например: 2Cr(OH)3 + ЗН2О2 + 4КОН = 2К2СгО4 + 8Н2О Хроматы могут быть получены и сплавлением оксида хрома(Ш) с каким-либо окислителем и щёлочью или карбонатом щелочного металла. Например: Cr2O3 + 3KNO, + 2K-CCL = 2К?СгО4 + 3KNCL + 2СО, Чистый хром чаще всего получают методом алюминотермии: Cr2O3 + 2AI = А12О3 + 2Сг Наибольшее количество хрома применяется в металлургии для получения хро- мистых сталей, которые отличаются большой твёрдостью и прочностью, поэтому используются для изготовления инструментов и деталей машин, несущих большую нагрузку. Добавка хрома к другим сплавам значительно повышает их твёрдость; хромирование стальных деталей применяется для защиты их от коррозии. Схема 22 Окислительно-восстановительные переходы соединений хрома(Ш) и хрома(У1) кислая среда щелочная среда +3 кислотная среда щелочная среда
156 ЖЕЛЕЗО И ЕГО СОЕДИНЕНИЯ Железо находится в VIIIB-rpynne Периодической системы элементов Д. И. Мен- делеева. Атом железа имеет следующую электронную конфигурацию: 26Fe: ls22s22p63s23p63cF4s2 Как и хром, железо проявляет в соединениях степени окисления +2, +3 и +6 (табл. 22). Однако по сравнению с соответствующими соединениями хрома соеди- нения железа(Н) более устойчивы, а соединения железа(\/1) встречаются редко и являются сильнейшими окислителями. Таблица 22 Важнейшие соединения, образованные железом в различных степенях окисления Степень окисления железа +2 +3 +6 Оксид FeO основный амфотерный не существует кислотный характер оксидов и гидроксидов усиливается Гидроксид Fe(OH)2 основный Fe(OH)3 амфотерный не существует Примеры солей FeCI2, FeSO4 FeCls, Fe2(SO4)3 KFeO2 феррит калия K2FeO4 феррат калия Окислительно-вос- становительные свойства сильные восстано- вители: ♦2 +3 Fe - ё -> Fe наиболее устойчивая степень окисления; воз- можно проявление как окислительных, так и вос- становительных свойств, окислительные свойства преобладают: +3 *6 Fe - Зё -> Fe +3 +2 Fe + ё -> Fe сильнейшие окис- лители: +6 +3 Fe + Зё -> Fe С химической точки зрения железо — металл средней активности. Во влажном воздухе железо быстро корродирует — ржавеет, покрываясь бурым налётом ги- дратированного оксида железа: 2Fe + 3/2О2 + nH2O = Fe2O3 • пН2О При сильном нагревании железо реагирует с кислородом воздуха с образова- нием красно-бурого оксида Fe2O3. Раскалённая стальная проволока горит в чистом кислороде, при этом образуется чёрный порошок железной окалины — двойного оксида состава Fe3O4: f 3Fe + 2О2 = Fe3O4 При высокой температуре железо реагирует с парами воды: 3Fe + 4Н2О = Fe3O4 + 4Н2
157 Железо взаимодействует с соляной и разбавленной серной кислотами с выде- лением водорода: Fe + 2HCI = FeCI2 + Н2Т Fe + H2SO4 = FeSO4 + H2T Обратите внимание: железо в этих реакциях приобретает степень окисления +2, поскольку участвующий в них окислитель — ион Н+ — слабый. Под действием более сильных окислителей, например хлора или брома, железо приобретает степень окисления +3, например: 2Fe + 3CI2= 2FeCI3 Концентрированные азотная и серная кислоты при нагревании также окисляют металлическое железо до Fe(lll): Fe + 6НМО3(конц.) = Fe(NO3)3+ 3NO2T + ЗН2О 2Fe + 6Н2ЗО4(конц.) = Fe2(SO4)3 + 3SO2t + 6H2O Очень концентрированная азотная кислота (плотностью 1,4 г/см3) и серная кис- лота, концентрация которой близка к 100 %, пассивируют железо на холоду (т. е. реакция не протекает), поэтому серную кислоту такой концентрации можно пере- возить в стальных цистернах. Соединения железа(П) Оксид железа(Н) FeO имеет основный характер, ему соответствует слабое осно- вание Fe(OH)2. Этот гидроксид получают в виде светло-зелёного осадка при дей- ствии растворов щелочей на растворы солей железа(П): FeS04 + 2NaOH = Fe(OH)2>L + Na2SO4 Гидроксид железа(П), как и другие соединения железа в этой степени окисле- ния, обладают выраженными восстановительными свойствами. Он легко окисляет- ся кислородом воздуха или другими окислителями: 4Fe(OH)2 + О2 + 2Н2О = 4Fe(OH)3 2Fe(OH)2 + Н2О2 = 2Fe(OH)3 Соли железа(Н) легко окисляются до солей железа(Ш) под действием различных окислителей: 2FeCI2 +Cl2= 2FeCI3 6FeSO4 + K2Cr2O7 + 7H2SO4 = 3Fe2(SO4)3 + Cr2(SO4)3 + K2SO4 + 7H2O Соединения железа(Ш) Гидроксид и оксид железа(Ш) обладают слабо выраженной амфотерностью, при- чём оснбвные свойства преобладают. Гидроксид легко взаимодействует с раство- рами кислот: 2Fe(OH)3 + 3H2SO4 = Fe2(SO4)3 + 6Н2О Но со щелочами гидроксид и оксид железа(Ш) реагируют при нагревании в сухом виде (при сплавлении), образуя ферриты: Fe2O3 + 2КОН = 2KFeO2 + Н2О
158 Соли железа(Ш) могут проявлять как окислительные, так и восстановительные свойства вследствие того, что железо в этих солях находится в промежуточной степени окисления. Так, хлорид железа(Ш) при взаимодействии с бромом окисля- ется до феррата: 2FeCI3 + 3Br2 + 16NaOH = 2Na2FeO4 + 6NaBr + 6NaCI + 8H2O Окислительные свойства для соединений железа(Ш) более характерны; в каче- стве восстановителей можно использовать иодиды, сульфиды, металлические же- лезо и медь, оксид cepbi(IV) и др.: 2FeCI3 + 2KI = 2FeCL + l2 + 2KCI 2FeCI3 + 3K2S = 2FeS>f + Si + 6KCI Fe2(SO4)3 + Fe = 3FeSO4 Fe2(SO4)3 + Cu = 2FeSO4 + CuSO4 Fe2(SO4)3 + SO2 + 2H2O = 2FeSO4 + 2H2SO4 Напомним, что при сливании растворов солей железа(Ш) с растворами карбона- тов протекает совместный гидролиз: 2FeCI3 + ЗК2СО3 + ЗН2О = 2Fe(OH)J + 6KCI + ЗСО2Т В природе железо встречается в составе оксидных (Fe2O3, Fe3O4) и сульфидных (FeS, FeS2) руд. Получение железа из оксидов основано на их восстановлении оксидом углерода(П) и коксом. Восстановление оксида железа(Ш) до железа протекает по- стадийно через ряд оксидов с более низкими степенями окисления, что можно представить в виде схемы: Fe2O3 -> Fe3O4 -> FeO -> Fe и отразить следующими уравнениями реакций: 3Fe2O3 + СО = 2Fe3O4 + СО2 (при температуре около 450—500 °C) Fe3O4+ СО = 3FeO + СО2 (при температуре около 600 °C) FeO + СО = Fe + СО2 (при температуре около 700 °C) В результате образуется не чистое железо, а его сплав с углеродом — чугун. Выплавку чугуна проводят в доменных печах, часть чугуна затем перерабатывают в сталь. Первой стадией переработки сульфидов железа (например, в сернокислотном производстве) является их обжиг: 4FeS + 7О2 = 2Fe2O3 + 4SO2 4FeS2 + 1102 = 2Fe2O3 + 8SO2 Железо используют в основном в виде сплавов чугуна, стали и др.
159 Модуль III. Органическая химия ЗАНЯТИЕ 16 Классификация и номенклатура органических веществ. Теория строения органических соединений Основные элементы содержания: Классификация органических соединений. Номенклатура органических веществ (тривиальная и международная). Теория строения органических соединений: гомология и изомерия (структур- ная и пространственная). Для классификации органических соединений в их молекулах принято выделять углеродный скелет и функциональные группы. Углеродный скелет представляет собой последовательность химически связанных между собой атомов углерода. Соединения, в молекулах которых к атомам углерода присоединены только ато- мы водорода, называют углеводородами. Если молекула органического вещества содержит только простые (одинарные) связи углерод—углерод, то вещество явля- ется предельным, или насыщенным. Соединения с двойными или тройными свя- зями углерод—углерод являются непредельными, или ненасыщенными. В молеку- лах органических веществ, кроме углерода и водорода, могут содержатся атомы других элементов — кислорода, азота, серы. Эти атомы входят в состав особых группировок, называемых функциональными группами. Присутствие той или иной функциональной группы определяет принадлежность органических соединений к определённым классам (табл. 23). Таблица 23 Функциональные группы и классы органических соединений Функциональная группа Класс соединений Примеры соединений Формула Название Галогены -F, -Cl, -Br, -I Галогенпроизводные углеводородов СН3—СН2—Вг Бромэтан Гидроксильная -ОН Спирты СН3—СН2—ОН Этанол Фенолы О Фенол Карбонильная ^С=О Альдегиды сн3—с хн Этаналь Кетоны сн3—с—СНз О Пропанон Карбоксильная — с х хон Карбоновые кислоты ^0 сн3—сн2—с хон Пропановая кислота Нитрогруппа -NO2 Нитросоединения ch3-no2 Нитрометан Аминогруппа -NH2 Амины сн3—nh2 Метиламин
160 При классификации органических соединений широко используют понятие го- мологии. Ряд соединений, обладающих сходным химическим строением и отли- чающихся друг от друга по составу на одну или несколько групп —СН2—, назы- вают гомологическим рядом. Члены этого ряда носят название гомологов. Имея сходное строение, гомологи обладают и сходными химическими свойствами. Это позволяет вместо изучения химических свойств отдельных многочисленных соеди- нений изучать свойства гомологических рядов в целом. В гомологических рядах углеводородов, спиртов, карбоновых кислот и т. п. температуры плавления и кипе- ния по мере роста числа атомов углерода в молекулах закономерно повышаются. Приведённые в таблице 24 данные иллюстрируют эту закономерность на примере гомологического ряда алканов. Изомеры — это вещества, имеющие одинаковый качественный и количествен- ный состав, но различное строение и, следовательно, разные свойства. Виды изо- мерии разделяют на две большие группы: структурную изомерию и пространствен- ную изомерию. Структурная изомерия может быть обусловлена принадлежностью соединений, имеющих одну и ту же молекулярную формулу, к разным классам ор- ганических веществ (межклассовая изомерия), различным строением углеродно- го скелета (изомерия углеродного скелета), различным положением заместителей или кратных связей в молекуле (изомерия положения). Пространственную изоме- рию разделяют на геометрическую (цис-транс-) и оптическую изомерию. Таблица 24 Формулы, названия и свойства некоторых представителей гомологического ряда алканов Название Формула Тпл, °C Т , ’С кип’ Состояние при обычных условиях Метан сн„ -184 -162 4 газы без запаха Этан С2Н6 -172 -88 Пропан С3Н8 -190 -42 Бутан с4н,0 -135 -0,5 Пентан ^5^12 -132 36 жидкости с запахом > нефти или бензина Гексан с6нм -95 68 Гептан с7н,в -90 98 Октан С8Н18 -57 126 Нонан -54 151 Декан С10Н22 -30 174 Проиллюстрируем существование изомеров различных видов на примере од- ного из представителей гомологического ряда алкенов — вещества состава С4Н8 (схема 23). Для алкенов характерна как структурная, так и геометрическая изомерия. По- скольку общая формула С„Н2л соответствует не только алкенам, но и циклоалка- нам, то алкены и циклоалканы являются межклассовыми изомерами. Структурная изомерия алкенов обусловливается также различным строением углеродного ске- лета молекулы и различным положением двойной связи в углеродной цепи.
161 Виды изомерии алкенов Схема 23 н2с=сн—сн2—сн3 н3с —сн=сн— сн3 бутен-1 бутен-2 Пространственная изомерия алкенов обусловлена различным положением за- местителей относительно линии двойной связи. Это геометрическая, или цис- транс-изомерия. Этот вид изомерии проявляется тогда, когда каждый из атомов углерода при связи С=С связан с двумя разными заместителями (атомами или группами атомов). Тот изомер, у которого одинаковые заместители при разных атомах углерода (например, группы СН3— или атомы водорода) оказываются рас- положенными по одну сторону линии двойной связи, называется цис-изомером (от латинского cis — по эту сторону): и .и /с=сх н3с сн2 сн3 цис-пентен-2 Тот изомер, у которого одинаковые заместители оказываются расположенными по разные стороны линии двойной связи, называется транс-изомером (от латин- ского trans — через): н3с н /с=с\ н сн2—сн3 тпранс-пентен-2 Отметим, что, поскольку вокруг двойной связи С=С свободное вращение невоз- можно, цис- и транс-изомеры не могут в обычных условиях переходить друг в друга. Такое превращение требует разрыва л-связи и может происходить только при со- общении молекуле достаточного количества энергии, например при нагревании или
162 освещении. Поэтому цис- и транс-изомеры представляют собой разные индивиду- альные вещества, которые имеют отличия по физическим, а иногда и по химическим свойствам. Алкены, у которых хотя бы один из атомов углерода при двойной связи соединён с двумя одинаковыми заместителями, не имеют цис- и транс-изомеров. Например, геометрическая изомерия невозможна для бутена-1 и 2-метилпропена. НАЗВАНИЕ ВЕЩЕСТВА 2) 3) 4) А) ацетилен Б) этиленгликоль В) глицил ал ан ин 2) 3) 4) Чтобы успешно выполнить это задание, необходимо знать не только систе- матические, но и тривиальные (исторически сложившиеся) названия важней- Примеры заданий Пример 1. Установите соответствие между молекулярной формулой ве- щества и его принадлежностью к определённому(ой) классу (группе) орга- нических соединений. Запишите в таблицу выбранные цифры под соответ- ствующими буквами. МОЛЕКУЛЯРНАЯ ФОРМУЛА ВЕЩЕСТВА А) С4Н8 Б) С8Н10 В) С2Н6О2 КЛАСС (ГРУППА) ОРГАНИЧЕСКИХ СОЕДИНЕНИЙ 1) арены циклоалканы многоатомные спирты сложные эфиры Для выполнения этого задания необходимо установить, каким общим форму- лам соответствует состав каждого из веществ, и на основании этого отнести эти вещества к определённому классу (группе). Так, состав вещества С4Н8 соответствует общей формуле СпН2п, которую имеют как циклоалканы, так и алкены. Поскольку в представленном перечне вариант ответа «алкены» от- сутствует, выбираем вариант «циклоалканы». Состав вещества С8Н10 отвечает общей формуле СпН2п_6, что позволяет отнести его к ароматическим углево- дородам аренам. Состав вещества С2Н6О2 описывается формулой СпН2п + 2О2 или СпН2л(ОН)2. Следовательно, оно содержит две гидроксильные группы и относится к многоатомным спиртам. Ответ: А Б в 2 1 3 Пример 2. Установите соответствие между названием вещества и клас- сом (группой) органических соединений, к которому^ ой) оно принадлежит. Запишите в таблицу выбранные цифры под соответствующими буквами. КЛАСС (ГРУППА) ОРГАНИЧЕСКИХ СОЕДИНЕНИЙ альдегиды спирты алкины пептиды
163 ших органических соединений. Ацетилен (этин) — это первый представитель гомологического ряда алкинов. Этиленгликоль (этандилол-1,2) является мно- гоатомным спиртом. Глицилаланин — это дипептид, молекула которого со- стоит из остатков аминокислот глицина и аланина. Ответ: А Б В 3 2 4 Ответ: Пример 3. Из предложенного перечня выберите два вещества, молекулы которых содержат как гидроксильные группы, так и карбонильную группу. 1) глицин 2) глицерин 3) глюкоза 4) фруктоза 5) этилацетат Это задание проверяет знание характерных функциональных групп органиче- ских соединений. Глицин является простейшей аминокислотой, его молекула содержит карбоксильную группу —СООН и аминогруппу —NH2. Глицерин от- носится к многоатомным спиртам, его молекула содержит три гидроксиль- ные группы —ОН. И фруктоза, и глюкоза являются углеводами. Молекулы углеводов содержат, как правило, одну карбонильную (альдегидную или ке- тонную) группу и несколько гидроксильных групп. Фруктоза относится к ке- тоноспиртам (в молекуле содержится пять гидроксильных и одна кетонная группа), а глюкоза — к альдегидоспиртам (в молекуле пять гидроксильных и одна альдегидная группа). Этилацетат относится к сложным эфирам. 3 4 Пример 4. Из предложенного перечня выберите два вещества, которые являются гомологами. 1) пентан 2) пентен-1 3) пропин 4) циклопентан 5) этан Напомним, что гомологами называют вещества, принадлежащие к одному гомологическому ряду, близкие по строению и различающиеся по составу на одну или несколько групп —СН2—. Исходя из этого определения, при- ходим к выводу, что среди представленных веществ гомологами являются пентан и этан. Ответ: Пример 5. Из предложенного перечня выберите два вещества, которые являются структурными изомерами бутанола-2. 1) бутаналь 4) диэтиловый эфир 2) 2-метилпропанол-2 5) пентанол-2 3) бутандиол-1,2 Напомним, что изомерами называются вещества, имеющие один и тот же состав, но разное строение. Вещества-изомеры имеют одну и ту же моле- SSBSSSSSSSi
164 :-лЪ ' транс-бутен-2 гексен-1 пропилен сн3—сн2 н /с=сС н сн2— сн3 кулярную формулу, но различаются строением углеродного скелета, положе- нием функциональной группы или кратной связи и даже могут принадлежать к разным классам органических веществ (межклассовая изомерия). Для од- ноатомных спиртов, к которым и относится бутанол-2, характерны изомерия углеродного скелета, изомерия положения гидроксильной группы, а также межклассовая изомерия с простыми эфирами. Таким образом, изомерами бутанола-2 будут 2-метилпропанол-2 и диэтиловый эфир. Все эти вещества имеют молекулярную формулу С4Н,0О. Ответ: 2 Пример 6. Из предложенного перечня выберите два вещества, для кото- рых возможна цис-транс-изомерия. 1) 2,3-диметилбутен-2 4) гексен-3 2) бутен-2 5) гексен-1 3) пропилен Геометрические (цис-транс-) изомеры возможны для алкенов, в молекулах которых каждый из атомов углерода, соединённых двойной связью, в свою очередь, связан с двумя разными атомами или группами атомов. Среди предложенных веществ такими алкенами являются бутен-2 и гексен-3: н3с х /Н /с=с\ н сн3 н\ /Н хс=с\ Н3С сн3 цис-бутен-2 нх ,н хс=сх сн3— сн2 сн2— сн3 цис-гексен-3 транс-гексен- 3 Остальные алкены, предложенные в задании, геометрических изомеров имеют. Убедимся в этом, обратившись к их структурным формулам: н н /с=с\ н сн2—сн2—сн2—сн3 Н3С СН3 хс=сС Н3с СН3 2,3-диметилбутен-2 Ответ: Н JCH3 хс=сС н н
165 Пример 7. В молекуле пропина 1) одна л- и три ст-связи 3) две л- и три ст-связи 2) одна л- и шесть ст-связей 4) две л- и шесть ст-связей В молекуле пропина атомы углерода при тройной связи соединены одной ст- и двумя л-связями, остальные связи являются ст-связями (схема 24). ст- и л-связи в молекуле пропина Схема 24 л-связи о-связи о-связи н— Всего в молекуле пропина две л- и шесть ст-связей. Ответ: 4 Пример 8. Укажите число атомов углерода в молекуле малеиновой кислоты НООС—СН=СН—СООН, электронные орбитали которых находятся в состоянии эр2-гибридизации. 1)1 2) 2 3) 3 4) 4 Каждый атом углерода при двойной связи С=С, а также каждый атом углеро- да в составе карбоксильной группы находится в состоянии ер2-гибридизации. Таким образом, в молекуле малеиновой кислоты все четыре атома углерода находятся в состоянии зр2-гибридизации. Ответ: 4 ЗАНЯТИЕ 17 Алканы и циклоалканы Основные элементы содержания: характерные химические свойства алканов и циклоалканов; основные способы получения алка- нов и циклоалканов. Алканы — это предельные углеводороды нециклического строения. Их называют также парафинами. Общая формула алканов — СпН2п+2- Каждый атом углерода в молекулах алканов образует четыре простые a-связи с атомами водорода или другими атомами углерода. Электронные орбитали атомов
166 углерода находятся в состоянии зр3-гибридизации. Гибридные орбитали, а значит, и связи каждого атома углерода направлены к вершинам тетраэдра (рис. 14). Рис. 14. Модели молекул метана (а) и гексана (б) В обычных условиях алканы химически малоактивны: не взаимодействуют с кис- лотами, щелочами; не обесцвечивают раствор КМпО4 и бромную воду. Ковалентная связь С—С в молекулах алканов неполярна, а связь С—Н мало- полярна. Они проявляют склонность к гомолитическому разрыву с образованием свободных радикалов. Следовательно, для алканов характерны реакции, протека- ющие по радикальному механизму (табл. 25). В промышленных количествах алканы выделяют из нефти и природного газа. Методы, использующиеся для получения алканов в лаборатории, приведены в таб- лице 26. Таблица 25 Химические свойства алканов Типы реакций Уравнения реакций Замещение атомов водорода. Протекает по радикальному механизму на свету или при на- гревании Хлорирование, бромиро- вание СН4 + С12 СН3С1 + НС1 (образуются также СНгС1г, СНС13, СС14) Вг сн3—СН—СН2—СН3 + Вг2 —*-► СН3—с—СН2—СН3+ НВг сн3 сн3 Нитрование no2 сн3—сн2—СН3 + НЫО3(разб.) -J->- СН3—СН—СН3 + Н2О При бромировании и нитровании гомологов метана соблюдается следующий порядок замещения атомов водорода: у третичного атома С -> у вторичного атома С -*> у первичного атома С Реакции с разрывом связи С—С. Протекают при нагревании и действии катализаторов Крекинг н3с—сн2— сн2—сн2—сн2— сн2—сн2—сн2—СН3 450-600 °с > н3с—сн2—сн2— СН=сн2 + н3с—сн2— сн2—сн3 Изомеризация СНз н3с—сн2—сн2—сн3 А1С18’ - - н3с—сн—сн3
167 Продолжение Типы реакций Уравнения реакций Дегидрирование и дегидроциклизация. Протекают под действием катализаторов Ni, Pt, Cr?O3 Дегидрирование н3с—СН2—сн3 кат-’ - ► Н2С=СН—сн3 + н2 Дегидроциклизация (ароматизация) сн3 н3с—сн2—сн2—сн2—сн2—сн2—СН3 каТ|* ► +4Нг Горение и каталитическое окисление Гзрение СН4 + 2О2 -> СО2 + 2Н2О + 882 кДж/моль Каталитическое окисле- ние 2Н3С—СН2—СН2—СН3 + 502 кат~' - ► 4СН3 —COOH + 2Н20 Таблица 26 Основные способы получения алканов Способы получения Уравнения реакций Гидролиз карбида алюминия (можно получить только метан). А14С3+ 12НгО -» 4AI(OH)3 + ЗСН4 Сплавление натриевых и калие- вых солей карбоновых кислот со щелочами. CH3CH,COONa + NaOH 4 CH3CH3 + Na,CO3 Реакция Вюрца — взаимодей- ствие галогеналканов с натрием. 2СН3СН2Вг + 2Na -> СН3СН2СН2СН3 + 2NaBr Синтез Кольбе — электролиз вод- ных растворов натриевых или ка- лиевых солей карбоновых кислот. 2CH3COONa + 2Н,0 электР°ли^ СН3СН3 + 2СО, + 2NaOH + Н3 Циклоалканы — это предельные углеводороды циклического строения (рис. 15). Как и в молекулах алканов, электронные орбитали атомов углерода в молекулах циклоалканов находятся в состоянии зр3-гибридизации. Общая формула циклоал- канов — С„Н,„. Л 2л /СН2^ СН, А Н2С СН, /—\ а Н2С -СН2 Z_\ б н2С СН2 О СН/ Рис. 15. Модели молекул и структурные формулы циклопропана (а) и циклогексана (б)
168 Химические свойства циклоалканов зависят от размеров цикла. Для углеводо- родов с малыми циклами (С3 — С4) характерны реакции присоединения водорода, галогенов и галогеноводородов, приводящие к размыканию цикла. Циклоалканы с пятью и более атомами углерода в цикле по химическим свойствам практически не отличаются от алканов; для них наиболее характерны реакции замещения ато- мов водорода (табл. 27). Основные способы получения циклоалканов приведены в таблице 28. Таблица 27 Химические свойства циклоалканов Типы реакций Уравнения реакций Реакции присоединения с размыканием цикла. Характерны для малых циклов — циклопро- пана и циклобутана Гидрирование /С\2 + Н2 Nb80°c> н3с—сн2—сн3 н2с—сн2 циклопропан пропан Галогенирование сн2 / \ + Вг2 — > н2с—сн2 циклопропан ► сн2—сн2—сн2 Вт Вт 1,3-дибромпропан Гидрогалогенирование н2с—СН2 + НВг * > СН3 Н2С — СН2 циклобутан —сн2— сн2— СН2—Вг 1-бромбутан Замещение атомов водорода. Характерно для циклоалканов с рода в цикле пятью и более атомами угле- Галогенирование \ Av \ + С12 ► + НС1 циклогексан хлорциклогексан Нитрование 1 + HNO3 Il + Н2О циклогексан нитроциклогексан Ароматизация — дегидрирование с образованием ароматических углеводородов. Харак- терна для циклоалканов, содержащих шесть атомов углерода в цикле Ароматизация pt’ -г 1 циклогексан С бензол 1 +зн2 I
169 Таблица 28 Способы получения циклоалканов Способы получения Действие активных металлов (на- трия, лития, магния, цинка) на ди- галогенпроизводные алканов Уравнения реакций ХСН2—Вг а Н2С + 2Na ----►/ \ + 2NaBr Хсн2—Вг „ Л циклопропан 1,3-дибромпропан ZCH2—СН2—Вг Н2С 4- Mg ХСН2—СН2—Вг 1,5-дибромпентан Гидрирование бензола и его го- мологов. Можно получить толь- ко циклоалканы с шестью атомами углерода в цикле + ЗН2 бензол кат., t,p^ циклогексан ЗАНЯТИЕ 18 Алкены и алкадиены Основные элементы содержания: характерные химические свойства алкенов и алкадиенов; основные способы получения алкенов и алкадиенов. Алкены (олефины) — это непредельные углеводороды, молекула которых содер- жит одну двойную связь, которая представляет собой сочетание одной о- и одной л-связи. Электронные орбитали атомов углерода, связанных двойной связью, на- ходятся в состоянии зр2-гибридизации. Общая формула алкенов — СпН2п. Для алкенов характерна как структурная изомерия (углеродного скелета, поло- жения кратной связи, межклассовая), так и геометрическая изомерия (цис-транс-). Алкены гораздо более химически активны, чем алканы, причём двойная связь является реакционным центром их молекул. Для алкенов наиболее характерны ре- акции присоединения, окисления и полимеризации (табл. 29). Основные способы получения алкенов приведены в таблице 30.
170 Таблица 29 Химические свойства алкенов Типы реакций Уравнения реакций Реакции присоединения Гидрирование. Протека- ет в присутствии метал- лических катализаторов (никель, платина, палла- дий) н3с—сн= сн2 + н2 Ni,t ► н3с—сн2—сн3 пропилен пропан Галогенирование. Про- текает в обычных услови- ях. Механизм реакции — ионный Н3С—СН = СН2 + Вг2 ► н3с—сн—сн2 пропилен Вг Вг 1,2-дибромпропан Гидрогалогенирование. Протекает в обычных ус- ловиях. Механизм реак- ции — ионный Н2С=СН2 + НС1 * н3с—СН2—С1 этилен хлорэтан При присоединении галогеноводородов к несимметричным алке- нам выполняется правило Марковникова: атом водорода преиму- щественно присоединяется к более гидрированному атому углеро- да при двойной связи, например: СН3 СН3 н3с—с =сн—СН3 + НС1 ► н3с—с—сн2—сн3 2-метилбутен-2 С1 2-метил-2-хлорбутан Гидратация. Протека- ет при нагревании в присутствии кислотных катализаторов. Механизм реакции — ионный Н2С=СН2 + Н2О —Н3Р04,300°С > НзС—Сн2—он этилен этиловый спирт При присоединении воды к несимметричным алкенам также вы- полняется правило Марковникова: ОН н2с=сн—сн2—сн3 + н2о H2SO<,t > н3с—сн—сн2—сн3 бутен-1 бутанол-2 Окисление Мягкое окисление — взаимодействие с КМпО4 в нейтральной или сла- бощелочной среде без нагревания ЗН3С—СН = СН2 + 2КМпО4 + 4Н2О *• пропилен ЗН3С —СН—СН2 + 2МпО2 + 2КОН ОН ОН пропандиол-1,2 Жёсткое окисление — взаимодействие с КМпО4 в кислотной среде при нагревании Происходит расщепление молекулы по месту двойной связи, об- разуются карбоновые кислоты: 5Н3С—СН2—СН = СН—СН3 + 8КМпО4 + 12H2SO4 -*-* °Ч 5Н3С—СН2 — С +5 С — СН3 + 8MnSO4 + 4K2SO4 + 12Н2О ХОН НО
171 Продолжение Типы реакций Уравнения реакций Если двойная связь концевая, то одним из продуктов окисления алкена будет углекислый газ: Н3с—СН = СН2+ 2КМпО4 + 3H2SO4 ► Л) Н3С—С + СО2 + 2MnSO4 + K2SO4 + 4Н2О ХОН Если углеродная цепь алкена имеет разветвление при двойной связи, образуется кетон: 5Н3С—С=СН2+ 8КМпО4 + 12H2SO4 —- сн3 5Н3С—С = О + 5СО2+ 8MnSO4+ 4K2SO4 + 17Н2О сн3 Полимеризация Полимеризация про- текает при воздействии различных факторов (повышении давления, УФ-облучении, действии органических пероксидов) или в присутствии ката- лизаторов пн2с = сн2 —кат‘ » — СН2 — СН2 — этилен полиэтилен П Г — СН — сн2 — пН3С — СН = СН2 -—► | СН3 полипропилен L J п полипропилен Таблица 30 Способы получения алкенов Способы получения Уравнения реакций Дегидрирование алканов н3с — сн2 — сн3 кат”* » Н2С = СН — сн3 + н2 Дегалогенирование дигалоген- замещённых алканов при дей- ствии на них цинка или магния Н3С — СН — СН — СН3 + Zn ► 1 1 Вг Вг Н3С — СН = СН — СН3 + ZnBr2 Дегидратация спиртов H2SO4 (конц.), t Н3С — СН2 — ОН ► Н2С = сн2 + Н2О Выполняется правило Зайцева: атом водорода отщепля- ется преимущественно от наименее гидрированного атома углерода: ОН H2SO4 (конц.), t Н3С — СН — СН2 — СН3 ► Н3С — СН = СН — сн3 + Н2О
172 Продолжение Способы получения Уравнения реакций Дегидрогалогенирование при нагревании моногалогензаме- щённых алканов со спиртовыми растворами щелочей Как и дегидратация, протекает в соответствии с правилом Зайцева: СН3 I t Н3С — СН — СН — СН3 + КОН (спирт, р-р) —► Вг СН3 I Н3с — С = СН — СН3 + КВг + Н2О Алкадиены (диеновые углеводороды) — это непредельные углеводороды, моле- кулы которых содержат две двойные связи. Общая формула диеновых углеводо- родов С„Н2п _ 2. Взаимное расположение двойных связей в молекулах алкадиенов может быть различным. Если две двойные связи разделены одной простой связью, то их назы- вают сопряжёнными двойными связями, а соответствующие алкадиены — сопря- жёнными диенами. Наибольшее практическое значение имеют два простейших представителя этих углеводородов: бутадиен-1,3 (дивинил) и 2-метилбутади- ен-1,3 (изопрен): сн3 н2с = сн — сн = сн2 Н2С = с — сн = сн2 бутадиен-1,3 2 - мети л бутадиен -1,3 дивинил изопрен Все четыре атома углерода в молекуле простейшего сопряжённого диена — бу- тадиена-1,3 находятся в состоянии зр2-гибридизации и располагаются в одной плоскости. Негибридные р-орбитали расположены перпендикулярно плоскости. Боковое перекрывание этих орбиталей с образованием л-связей происходит не только между первым и вторым, третьим и четвёртым атомами углерода, между которыми обозначена двойная связь, но и частично между вторым и третьим ато- мами. Такое взаимодействие двух соседних л-связей называют сопряжением. Че- тыре р-электрона образуют общее n-электронное облако (рис. 16). Рис. 16. Схема образования л-связей в молекуле бутадиена-1,3 Имея двойные связи в молекулах, диеновые углеводороды вступают в обычные реакции присоединения: обесцвечива- ют бромную воду, присоединяют водород и галогеноводоро- ды (табл. 31). Однако эффект сопряжения накладывает свои особенности на протекание этих реакций. Присоединение может протекать в двух направлениях: к одной из двойных связей (1,2-присоединение) или к крайним атомам сопря- жённой системы с образованием новой двойной связи в центре молекулы (1,4-присоединение). Практически важной является полимеризация сопряжён- ных диенов, которая также протекает в основном по типу 1,4-присоединения. Полимеризацией бутадиена-1,3 получа- ют бутадиеновый (дивиниловый) каучук, а полимеризацией
173 2-метилбутадиена-1,3 — изопреновый каучук, который по химическому строению является аналогом натурального каучука. Основные способы получения алкадиенов приведены в таблице 32. Таблица 31 Химические свойства алкадиенов Типы реакций Уравнения реакций Гидрирование При взаимодействии с избытком водо СН3 I к Н2С = С — СН = СН2 + 2Н2 Если реагенты взяты в молярном cooti продуктов как 1,2-присоединения, так Н2С = СН — СН = СН2 + Н2 рода образуется алкан: сн3 ат. ► СН3 — СН — СН2 — СН3 ношении 1:1, возможно образование и 1,4-присоединения: —► н3с — сн2 — сн = сн2 ► н3с — сн = сн — сн3 Галогенирование При взаимодействии с избытком брома мирования: Н2С = СН — СН = СН2 + 2Вг2 Если реагенты взяты в молярном соотнс продуктов как 1,2-присоединения, так и Н2С = сн — СН = СН2 + Вг2 В обычных условиях основным является i образуется продукт полного бро- Вг Вг Вг Вт ► Н2С — СН — СН — СН2 >шении 1:1, возможно образование 1,4-присоединения: Вт Вг ► Н2С — СН — СН = СН2 Вг Вг ► Н2С — СН = СН — сн2 (основной продукт) i продукт 1,4-присоединения Полимеризация Полимеризация протекает в основном пн2с = сн — сн = сн2 —*ат’- > бутадиен-1,3 (дивинил) сн3 кят пН2С = С — СН = сн2 —* изопрен г б ю типу 1,4-присоединения: — СН2 — СН = СН — СН2 — Jn утадиеновый (дивиниловый) каучук сн3 — сн2 — с = сн — сн2 — - п изопреновый каучук Окисление Диеновые углеводороды окисляются КМпО4 и другими окислителями
174 Таблица 32 Способы получения алкадиенов Способы получения Уравнения реакций Дегидрирование алканов (катализаторы Ni, Pt, CrfiJ кат., t Н3С — сн2 —сн2—СН3 Н2С = СН — СН = СН2 + 2Н2 бутан бутадиен-1,3 (дивинил) СН3 СН3 кат., t Н3С — СН — СН2 — СН3 * Н2С = С — СН = СН2 + 2Н2 2-метилбутан 2-метилбутадиен-1,3 (изопрен) Реакция Лебедева — получение бутадиена из этанола ZnO, А12О3, 450 °C 2Н3С — СН2 — ОН ► Н2С = СН — СН = СН2 + 2Н2О + Н2 этанол бутадиен-1,3 (дивинил) ЗАНЯТИЕ 19 Алкины Основные элементы содержания: характерные химические свойства алкинов: реакции присоединения, окисления, тримеризации, замещения атомов водорода при тройной связи; основные способы получения алкинов. Ненасыщенные углеводороды, содержащие тройную связь С=С и образующие гомологические ряды общей формулы СпН2п_2, называют алкинами. Тройная связь в молекулах алкинов представляет собой сочетание одной о- и двух л-связей. Атомы углерода, связанные тройной связью, находятся в состоянии sp - гибридизации. Наличие кратной связи в молекулах ацетиленовых углеводородов обуславливает их склонность к реакциям присоединения (табл. 33). Во многих реакциях они ведут себя аналогично алкенам, но есть и некоторые особенности. Так, реакции присо- единения могут протекать по стадиям, первая из которых приводит к образованию продукта с двойной связью, а вторая — к получению вещества предельного стро- ения. Гидратация алкинов приводит к образованию карбонильных соединений — альдегидов и кетонов. Реакции полимеризации для алкинов менее характерны, чем для алкенов и алкадиенов, однако практически важной является тримериза- ция ацетилена, приводящая к получению бензола. Связь С—Н при тройной связи более полярна, чем связи С—Н в молекулах алканов или алкенов, поэтому атом водорода при тройной связи способен замещаться на металл. Эти реакции иногда
175 рассматривают как проявление алкинами с концевой тройной связью очень сла- бых кислотных свойств. Основные способы получения ацетилена и его гомологов приведены в таблице 34. Таблица 33 Химические свойства алкинов Типы реакций Уравнения реакций Реакции присоединения Гидрирование Протекает под действи- ем катализатора Н3С — С = СН + 2Н2 —кат‘ ► н3с — сн2 — сн3 Галогенирование Протекает в обычных условиях. Механизм реакции — ионный Вг Вг Н3С - С = СН + 2Вг2 ► Н3С — С — СН 1 1 Вг Вг 1,1,2,2 -тетрабромпропан Гидрогалогенирование Протекает в обычных условиях. Механизм реакции — ионный. Присоединение осу- ществляется по правилу Марковникова С1 Н3С — С = СН + НС1 ► Н3С — с = сн2 С1 С1 1 1 Н3С — С = СН2 + НС1 ► н3с — с — сн3 С1 суммарно: С1 Н3С — С = СН + 2НС1 ► Н3С — С — СН3 1 С1 2,2-дихлорпропан Гидратация (реакция Кучерова) Присоединение осу- ществляется по правилу Марковникова H2so4 нс = сн + н2о HgSO4 HgSO4 н3с — с = сн + н2о H2so4 пропин Н2С = сн он Н3С —с = но .0 ► Н3С— С С хн этаналь, уксусный альдегид = сн2 —► н3с —с —сн3 II J 0 ацетон
176 Продолжение Типы реакций Уравнения реакций Полимеризация Тримеризация ацетилена Протекает при исполь- зовании активирован- ного угля в качестве катализатора при темпе- ратуре 400—600 °C С акт., 450 °C ЗНС = СН ► бензол Реакции замещения в реакции замещения вступают только алкины с концевой тройной связью Взаимодействие со щелочными метал- лами HC=CH + Na *- НС = CNa + l/2H2f ацетиленид натрия НС = СН + 2Na * NaC = CNa + Н2 f диацетиленид натрия Взаимодействие с гидридами щелоч- ных металлов H3C-C = CH + NaH ► Н3С — C = CNa + H2 Взаимодействие с аммиачным раствором оксида серебра Н3С — С = СН + [Ag(NH3)2]OH ► Н3С — С = CAg| + 2NH3 + Н2О метилацетиленид серебра Окисление Гзрение 2С2Н2 + 5О2 = 4СО2 + 2Н2О Жёсткое окисление — в зависимости от стро- ения исходного алкина образуются различные продукты 5Н3С —С = СН + 8КМпО4 + 12H2SO4-> -> 5СО2 + 5Н3С — СООН + 8MnSO4 + 4^50, + 12Н2О 5Н3С — С = С — СН3 + 6KMnO4 + 9H2SO4 -> -> 10СН3 — СООН + 6MnSO4 + ЗК^О, + 4Н2О Мягкое окисление ацетилена ЗС2Н2 + 8КМпО4 -> 3KjC2O4 + 8МпО2 + 2КОН + 2Н2О Таблица 34 Способы получения алкинов Способы получения Уравнения реакций Промышленные способы получения Карбидный метод СаО + ЗС _1800 °9» СаС2 + СО СаС2 + 2Н2О ► НС 2 СН + Са(ОН)2
in Продолжение Способы получения Уравнения реакций Крекинг метана и этилена 2СН4 1500 °С» НС s СН + ЗН2 н2с = сн2 1200 °с» нс=сн + н2 Лабораторные способы получения Взаимодействие кар- бида кальция с кис- лотами (или водой) СаС2 + 2HCI ► НС в СН + СаС12 Дегидрогалогениро - ванне дигалогенопроизвод • ных алканов протекает при нагрева- нии ди галоген- замещённых алканов со спиртовыми раство- рами щелочей t Н3С — СН — СН2 + 2КОН(спирт. р -р) —► Н3С — С = СН + 2КС1 + 2Н2О 1 1 С1 С1 Вг 1 t Н3С — С — СН3 + 2КОН (спирт. р-р) —► Н3С — С = СН + 2КВг + 2Н2О Вг Дегалогенирование тетрагалогенопроиз - водных алканов Вг Вт । । t Н3С — С —С —CH3 + 2Zn ► Н3С—С = С — СН3 + 2ZnBr2 Вг Вг Взаимодействие ме- таллических производ- ных алкинов с галоге- нопроизводными алка- нов. Реакция позволяет удлинять углеродную цепь молекулы Н3С — Вт + NaC = СН —► Н3С — С = СН + NaBr 2Н3С — Вг + NaC = CNa ——* Н3С — С = С — СН3 + 2NaBr ЗАНЯТИЕ 20 Ароматические углеводороды Основные элементы содержания: ароматичность, единое л-электронное облако, круговое сопряжение; характерные химические свойства ароматических углеводородов; основные способы получения ароматических углеводо- родов. Ароматические углеводороды (арены) — это углеводороды, молекулы которых содержат одно или несколько бензольных колец. Простейшим представителем аро- матических углеводородов является бензол, молекулярная формула которого С6Н6. Общая формула гомологического ряда ароматических углеводородов СпН2п 6 Ближайшим гомологом бензола является метилбензол. Для изображения молекулы бензола пользуются следующими структурными формулами:
178 н н /Ч. с с II I с Н н н а Формулы а и б предложил в 1865 г. немецкий химик Фридрих Август Кекуле. Несмотря на то что они недостаточно точно передают строение молекулы бензола, их используют до сих пор и называют формулами Кекуле. Исторически название «ароматические углеводороды» сложилось потому, что многие производные бензола, которые первыми были выделены из природных источников, обладали приятным запахом. В настоящее время под понятием «аро- матичность» подразумевают прежде всего особый характер реакционной способ- ности веществ, обусловленный, в свою очередь, особенностями строения молекул этих соединений. В чём же состоят эти особенности? В соответствии с молекулярной формулой С6Н6 бензол является ненасыщенным соединением, и можно ожидать, что для него были бы характерны типичные для ал- кенов реакции присоединения. Однако в условиях, в которых алкены быстро вступа- ют в реакции присоединения, бензол не реагирует или реагирует медленно. Бензол не даёт и характерных качественных реакций, свойственных непредельным углеводо- родам: он не обесцвечивает бромную воду и водный раствор перманганата калия. Такой характер реакционной способности объясняется наличием в ароматиче- ском кольце сопряжённой системы — единого n-электронного облака. В молекуле бензола каждый атом углерода находится в состоянии sp2-гибриди- зации и связан тремя о-связями с двумя атомами углерода и одним атомом водо- рода. Четвёртый валентный электрон атома углерода находится на р-орбитали, перпендикулярной плоскости молекулы. В молекуле бензола происходит боковое перекрывание р-орбиталей каждого атома углерода с р-орбиталями обоих сосед- них атомов углерода (рис. 17). В результате такого сопряжения образуется единое 0,14нм Рис. 17. Электронное строение молекулы бензола: схема перекрывания р-орбиталей (слева) и единое п-электронное облако (справа)
179 п-электронное облако, расположенное над и под плоскостью бензольного коль- ца, — осуществляется круговое сопряжение. Такая циклическая система с общим облаком из шести электронов очень устой- чива, энергетически выгодна; поэтому бензол преимущественно вступает в те ре- акции, в которых ароматическое кольцо сохраняется (табл. 35). Подчеркнём еще раз, что в молекуле бензола нельзя выделить трёх двойных и трёх простых связей. Электронная плотность распределяется в молекуле равно- мерно, и все связи между атомами углерода оказываются совершенно одинаковы- ми. Поэтому нужно помнить, что формула Кекуле, которая часто используется для изображения бензола, условна и не отражает реального строения его молекулы. Итак, ароматическими являются соединения, в молекулах которых имеется устойчивая циклическая группировка с особым характером связи. Имея молеку- лярную формулу, указывающую на высокую степень ненасыщенности, эти веще- ства, однако, не реагируют как ненасыщенные, а вступают преимущественно в ре- акции замещения с сохранением ароматической системы. Важнейшие способы получения ароматических углеводородов приведены в та- блице 36, Таблица 35 Химические свойства ароматических углеводородов
180 Продолжение Типы реакций Нитрование При нитровании гомоло- гов бензола образуется смесь орто- и пара- изомеров Алкилирование Алкилирование аренов можно осуществить раз- личными способами: а) действием на бензол галогензамещёнными алканами в присутствии галогенидов алюминия; б) взаимодействием бензола с алкенами в присутствии кислотных катализаторов Реакции присоединения Гидрирование Протекает в присут- ствии катализатора. Реакция гидрирования протекает аналогично для бензола и его гомо- логов
181 Продолжение Типы реакций Уравнения реакций Присоединение хлора к бензолу Гомологи бензола в данных условиях всту- пают в реакцию за- мещения (см. раздел «Особенности химиче- ских свойств гомологов бензола») гексахлорциклогексан Реакции окисления Горение 2CfiH6 + 150, —> 12СО2 + 6Н2О С6Н5—СН3 + 9О2 -> 7СО2 + 4Н2О Особенности химических свойств гомологов бензола Галогенирование этилбензол (1-хлорэтил) бензол Окисление гомологов бензола (бензол устойчив к окислению, с раствором КМпО4 не реагирует) Окислению подвергаются боковые углеводородные цепи. Продуктом окисления является бензойная кислота (в кислотной среде) или её соли — бензоаты (в нейтральной или щелочной среде): бензойная кислота + 12MnSO4 + 6K2SO4 + 28Н2О + 2МпО2 + КОН + Н20 бензоат калия
182 Способы получения бензола и его гомологов Таблица 36 Способы получения Уравнения реакций Дегидроциклизация алканов кат., t ' сн3—сн2 —сн2—сн2—сн2—сн3 ► ( ) +4Н2 гексан бензол ^СНз ^СНд н2с сн2 Х2\/Снз 1 1 Кат-Ч О +4Н2 н2с сн2 LVVJ гептан толуол • • е >•, Дегидрирование циклоалканов сн3 сн3 Ц)| +зн2 метилциклогексан толуол Тримеризация ацетилена (реакция Зелинского) С акт., t зсн=сн —► / ) Сплавление солей бензойной кислоты со щёлочью COONa + NaOH ——*• + Na2c°3 Взаимодействие аромати- ческих галогенпроизвод- ных с галогеналканами (реакция Вюрца—Фиттига) (^2^) + Вг — сн2 — CH3 + 2Na- ► бромэтан бромбензол СН2— сн3 (^2^) + 2NaBr этилбензол
183 Продолжение Способы получения Алкилирование бензола Другие примеры реакций алкилирования см. раздел «Химические свойства аро- матических углеводородов. Реакции замещения» (табл. 35) Уравнения реакций Примеры заданий, при выполнении которых нужно продемонстрировать знание химических свойств и способов получения углеводородов Пример 1. Из предложенного перечня выберите два вещества, при ги- дратации каждого из которых образуется бутанол-2. 1) бутен-1 2) бутин-1 3) бутен-2 4) бутин-2 5) бутадиен-1,3 Определим, какие продукты образуются при гидратации, т. е. взаимодей- ствии с водой каждого из предложенных веществ. При гидратации алкинов (бутина-1 и бутина-2) образуются карбонильные соединения, следовательно, ответы 2 и 4 неверны. Бутен-1 является углеводородом несимметричного строения. Присоединение воды к нему протекает в соответствии с правилом Марковникова: атом водорода преимущественно присоединяется к наиболее гидрогенизированному атому углерода при двойной связи. Таким образом, преимущественным продуктом гидратации бутена-1 является бутанол-2: h,so4, t сн3— сн2—сн = сн2 + н2о ——-—► сн3— сн2—сн — сн3 он Бутанол-2 образуется и при гидратации бутена-2: h,so., t сн3— сн = сн—сн3 + н2о ——!—► сн3— сн2—сн — сн3 он Гидратация бутадиена-1,3 приводит к образованию двухатомного или непре- дельного спирта. Ответ: 3
184 Пример 2. И пропан, 1) с бромной водой 2) с раствором КМпО4 3) с водородом и пропен реагируют 4) с хлором 5) с кислородом пропана и пропена к реактивам, перечисленным в пропан относится к алканам, т. е. предельным угле- к алкенам — непредельным углеводородам. Рассмотрим отношение условии. Напомним, что водородам, а пропен — 1) Бромная вода представляет собой очень разбавленный водный раствор брома. В органической химии бромная вода является качественным реак- тивом на непредельные соединения (алкены, диены, алкины). В частности, пропен легко обесцвечивает бромную воду с образованием 1,2-дибром- пропана. Пропан же бромную воду не обесцвечивает. Подчеркнём, что не с бромной водой, а с бромом пропан реагировать может, но реакция проис- ходит в довольно жёстких условиях (не в растворе). 2) Раствор перманганата калия также легко обесцвечивается под действием непредельных соединений, поэтому пропен будет реагировать с ним, а про- пан нет. 3) Присоединение водорода — гидрирование — характерно для веществ, содержащих кратные связи и малые циклы, следовательно, пропен будет реагировать с водородом, а пропан нет. 4) Хлор может реагировать со всеми углеводородами, при этом возможны реакции замещения (с предельными углеводородами) или присоединения (с непредельными углеводородами). Таким образом, и пропан, и пропен могут реагировать с хлором: сн3— сн2— СН3 + С12 сн3— СН=СН2 + С12 Av —► сн3— сн — СН3 + НС1 С1 сн3—сн —сн 2 С1 С1 5) Как родом и все углеводороды, пропан и — горят. пропен Ответ: 4 5 вступают в реакцию с кисло- Пример 3. Хлорирование метана протекает по ионному механизму начинается с разрыва связи в молекуле хлора сопровождается выделением Н2 относится к эндотермическим процессам приводит к образованию нескольких хлорпроизводных 2) 3) 4) 5)
185 МАЯЛ ‘Да RW Хлорирование метана относится к реакциям радикального замещения, про- текает на свету и сопровождается образованием хлороводорода (но не Н2, исключаем ответ 3): СН4 + Cl2 СН3 - Cl + HCI Обычно реакция не останавливается на стадии образования хлорметана, об- разуются также и другие хлорпроизводные — дихлорметан, трихлорметан и тетрахлорметан: CH3CI + Cl2 СН2С12 + HCI СН2С12 + Cl2 СНС13 + HCI СНС13 + С12 СС14 + HCI Первой стадией рассматриваемой реакции является разрыв ковалентной связи в молекулах хлора, на что и затрачивается энергия кванта света: Cl—Cl CI- + Cl- ИЖ Образовавшиеся атомы хлора имеют в наружном слое по одному неспа- ренному электрону, т. е. являются свободными радикалами. Далее реакция протекает по цепному радикальному механизму (не ионному, исключаем ответ 1) и сопровождается выделением значительного количества теплоты, т. е. является экзотермической (исключаем ответ 4). Ответ: 2 5 4) бутадиен-1,3 5) циклопентан Пример 4. С какими из перечисленных веществ хлор вступает в реакцию замещения? 1) метан 2) этен 3) циклопропан При выполнении подобных заданий рекомендуем не только выбирать верные ответы, но и исключать неверные. Такой двойной контроль своих действий поможет вам не совершить случайной ошибки. При взаимодействии галоге- нов с непредельными соединениями протекает реакция присоединения: сн2=сн2 + с12-----*• сн2—сн2 Cl Cl сн2 = сн—сн=сн2 + С12 сн9— сн = сн—сн2 ,2 I Cl Cl ж® ай следовательно, исключаем ответы 2 и 4. Взаимодействие циклопропана с хлором тоже является реакцией присоеди- нения, сопровождающейся раскрытием цикла (исключаем ответ 3): СН2 /\ +С12 СН2—сн2 С1СН2СН2СН2С1
186 Ответ: Для алканов и циклоалканов, начиная с циклопентана, характерны реакции замещения: + HC1 Cl 2) 3) В Пример 5. В заданной схеме превращений С12, А1С13 1 -бромпропан----► X----► бензол--------Y веществами X и Y соответственно являются пропанол-1 4) хлорбензол пропилен 5) гексахлорциклогексан гексан качестве вещества X из предложенного списка нужно выбрать такое, ко- торое можно получить из 1-бромпропана в одну стадию и далее в одну стадию превратить в бензол. Таким веществом является гексан. Действи- тельно, его можно получить с помощью реакции Вюрца из 1-бромпропана (реагент — натрий); при дегидрировании и циклизации гексана образуется бензол (катализатор — металл платиновой группы). При хлорировании бен- зола в присутствии катализатора образуется хлорбензол. Напомним, что гексахлорциклогексан можно получить при хлорировании бензола в условиях УФ-облучения. X Ответ: 3 Пример 6. Установите соответствие между исходными веществами и ор- ганическим веществом, преимущественно образующимся при их взаимо- действии. ИСХОДНЫЕ ВЕЩЕСТВА ,сн3 А) + С12 А1С13 ПРОДУКТ ВЗАИМОДЕЙСТВИЯ сн„ Б) + С12 £Н3 Av С1
187 Продолжение ИСХОДНЫЕ ВЕЩЕСТВА ПРОДУКТ ВЗАИМОДЕЙСТВИЯ Рассмотрим каждую из приведённых схем реакций. При взаимодействии то- луола с хлором в присутствии катализатора А1С13 протекает реакция замеще- ния атомов водорода в ароматическом ядре, причём заместитель под вли- янием радикала — СН3 направляется преимущественно в положения 2, 4, 6: Таким образом, продуктом данной реакции является 4-хлортолуол (ответ 4). Отметим, что в данной реакции образуется также 2-хлортолуол. При взаимодействии толуола с хлором на свету происходит замещение ато- ма водорода в метильном радикале (ответ 5): Взаимодействие бензола с хлором в присутствии катализатора А1С13 приво- дит к образованию хлорбензола (ответ 3): Наконец, бензол может и присоединить хлор, если реакционную смесь под- вергнуть УФ-облучению. Продуктом реакции будет гексахлорциклогексан (ответ 2):
188 ЗАНЯТИЕ 21 Спирты и фенолы Основные элементы содержания: характерные химические свойства одноатомных, многоатомных спиртов и фенола; основные способы получения одноатомных, многоатомных спиртов и фенола. Спирты — это производные углеводородов, содержащие в молекуле одну или несколько гидроксильных групп —ОН у насыщенных атомов углерода. В зависимо- сти от числа гидроксильных групп в молекуле различают одноатомные и много- атомные спирты. Изомерия предельных одноатомных спиртов связана со строе- нием углеродного скелета и положением группы —ОН. Кроме того, одноатомные спирты изомерны простым эфирам (межклассовая изомерия). Атомы углерода в предельных спиртах находятся в состоянии зр3-гибридизации. Молекулы спиртов содержат полярные связи С—Н, С—О, О—Н. Полярность связи О—Н больше полярности связи С—О вследствие большей разности электроотри- цательностей кислорода и водорода. Однако полярность этой связи недостаточна для диссоциации её с образованием ионов Н+. Поэтому спирты являются неэлек- тролитами. Присутствие в молекулах спиртов полярных групп —ОН определяет их физиче- ские свойства. Температуры кипения спиртов существенно выше температур кипе- ния соответствующих алканов с тем же числом атомов углерода. Это объясняется ассоциацией молекул спиртов вследствие образования межмолекулярных водо- родных связей. Спирты — активные в химическом отношении вещества. Важнейшие химиче- ские реакции, характерные для спиртов, обусловлены присутствием в составе их молекул гидроксильной группы и могут протекать как с разрывом связи С—О, так и с разрывом связи О—Н (табл. 37). Основные способы получения одноатомных спиртов приведены в таблице 38.
189 Таблица 37 Химические свойства одноатомных спиртов Типы реакций Уравнения реакций Кислотные свойства спиртов Взаимодействие с активными метал- лами Кислотные свойства у спиртов выражены очень слабо, слабее, чем у воды Образующиеся соединения называются алкоголятами: 2Н3С — СН2— ОН + 2Na ► 2Н3С — СН2—ONa + Н2 | этилат натрия При действии воды или кислот алкоголяты разлагаются: сн3—ок + н2о —► сн3—он + кон метилат калия Замещение группы —ОН на атом гало- гена Протекают при действии на спирты галогеноводородов: СН3— ОН + НС1СН3 — С1 + н2о Для более полного превращения спиртов в галогеналканы можно ис- пользовать галогениды фосфора (РС13, РС15): СН3ОН + РС15 —► СН3С1 + РОС13 + HCI Внутримолекулярная и межмолекулярная дегидратация Протекает при дей- ствии на спирты водоотнимающих ре- агентов: концентриро- ванной серной кислоты или оксида алюминия А190о 2С,Н5ОН —С2н5 —о — с2н5 + н2о 2 5 250°С диэтиловыи эфир А1?Оо с2н5он—сн2 = сн2 + н2о 2 5 350 °C этилен Внутримолекулярная дегидратация протекает при более высокой температуре, чем межмолекулярная: H9SO4 2С2Н5ОН *- С2н5 о — с2н5 + н2о 2 5 130 °C диэтиловый эфир H9SO4 С2Н5ОН ► СН2 = сн2 + н2о 2 5 170 °C этилен Внутримолекулярная дегидратация спиртов протекает в соответ- ствии с правилом Зайцева: H2SO4 (конц.) н3с—сн—сн— сн3 —-— ► н3с—сн=сн— сн3 + н2о г 1 1-п 170 °C 1 Н ОН 1 Образование слож- ных эфиров При взаимодействии спиртов с кислотами образуются сложные эфиры — протекает реакция этерифика- ции Реакция этерификации обратима. В качестве катализатора использу- ют обычно небольшие количества сильных кислот, например серной кислоты. е,Н+ н3с—с о — сн3 „ ► Н3с—С\ +Н2О :он Hf о—сн3 уксусная метиловый эфир кислота уксусной кислоты
190 Продолжение Типы реакций Уравнения реакций Сложные эфиры могут образовываться с участием неорганических кислот (азотной, серной): О II t сн3—сн2— ОН + НО—S — он сн3—сн2—О—SO3H + н2о о Реакции окисления Гзрение С2Н50Н + 302 —► 2СО2 + ЗН2О Окисление перманга- натом калия и дихро- матом калия При окислении первичных спиртов сначала образуются альдегиды, которые, в свою очередь, могут окислиться до карбоновых кислот: о ш [0] о [О1 „ R СН2 ОН R С ► R С н ОН первичный альдегид карбоновая спирт кислота ЗС2Н5ОН + К2Сг2О7 4- 4H2SO4 —► ЗСН3СНО+ Cr,(SO4k + K2SO4 + 7Н9О; СО С С 1 2 4 о 2х 4Z о 24 2 7 ЗС2Н.ОН + 2К,Сг,О7 + 8H2SO —► ЗСН.СООН + 2Cr,(SOJ, + 2 0 & С 1 2 4 о 2х 4zo 4-2K2SO4 4- 11Н2О Вторичные спирты окисляются до кетонов: [0] R, CH R« ► R-j С R9 1 II он ° вторичный кетон спирт Третичные спирты могут окисляться только с разрывом связей угле- род—углерод Окисление оксидом меди(П) Из первичных спиртов образуются альдегиды: С2Н6ОН + СиО -U- СН3СНО + Си+ Н2О этаналь Из вторичных спиртов образуются кетоны: О t II Н3С —СН —СН34-СиО ► Н3с — С— СН3 4-Си 4-Н2О ОН пропанон Третичные спирты в реакцию с оксидом меди(П) не вступают Дегидрирование Протекает при пропу- скании паров спиртов над нагретым металли- ческим катализатором, например медью или серебром Си, t С2Н5ОН —+ СН3СНО 4- н2 этаналь сн3он -Ag’ *-» НСНО + Н2 метаналь
191 Таблица 38 Способы получения предельных одноатомных спиртов Способы получения Уравнения реакций Гидратация алкенов Происходит в присутствии разбавленной серной или фосфорной кислоты. Эта реакция протекает в со- ответствии с правилом Марковникова Н3РО4, t н9с=сн9 + н9о ——-—► н,с—сн,—он этилен этиловый спирт сн3 сн3 1 H,SO4 , t н2с=с—сн3 + н2о —- ► н3с—с—сн3 2-метил пропен ОН (изобутилен) 2-метилпропанол-2 Замещение атома гало- гена на группу —ОН Реакция протекает при действии водных раство- ров щелочей на галоген- замещённые углеводороды Нзс — сн2— Вг + ЫаОН(водн. р-р) ► Н3С — СН2— ОН + NaBr бромэтан этиловый спирт сн3 Н3с — СН— I + КОН(водн. р-р) *- Н8с —СН—СН3 + К1 2-иодпропан ОН пропанол-2 Гидрирование альдеги- дов и кетонов при дей- ствии водорода в присут- ствии катализаторов (Ni, Pt, Pd) Альдегиды восстанавливаются до первичных спиртов: Ni _ сн3—сн2—сн2—с +н2 * 1_Г бутаналь ► сн3—сн2—сн2— сн2—он бутанол-1 Кетоны — до вторичных спиртов: О II Ni _ н3с—с — сн2— сн3 + н2 *• бутанон-2 ► н3с—сн—сн2 — сн3 он бутанол-2 Специфические способы получения метанола и этанола Получение метанола из синтез-газа р. t, кат. СО + 2Н2 > СН3ОН синтез-газ метанол Спиртовое брожение глюкозы СвН12О„ ФвР1*611™» 2С2Н5ОН + 2СО2 глюкоза этанол
192 Важнейшими представителями многоатомных спиртов являются двухатомный спирт этандиол-1,2 (этиленгликоль) и трёхатомный спирт пропантриол-1,2,3 (гли- церин): сн9—сн2 сн9 — сн—сн9 I I III он он он он он. этиленгликоль глицерин Этиленгликоль и глицерин — вязкие бесцветные жидкости, смешивающиеся с водой в любых соотношениях. Между молекулами многоатомных спиртов, так же как и между молекулами одноатомных спиртов, действуют водородные связи. Химические свойства многоатомных спиртов во многом схожи со свойствами одноатомных спиртов. Для них характерны кислотные свойства (более сильные, чем у одноатомных спиртов, но всё ещё более слабые, чем у воды); реакции за- мещения групп —ОН; образование сложных эфиров; различные виды дегидрата- ции и окисления (табл. 39). Основные способы получения многатомных спиртов приведены в таблице 40. Таблица 39 Химические свойства многоатомных спиртов Типы реакций Уравнения реакций Кислотные свойства спиртов Взаимодействие с актив- ными металлами При действии щелочных металлов на этиленгликоль сначала за- мещается атом водорода у одной гидроксильной группы, а за- тем и у второй: 2СНр—CH,+ 2Na ► 2СН9—СН9 + Н9 I I I I ОН ОН ОН ONa 2СН2— СН2 + 2Na *- 2СН2— СН2 + Н2 ОН ONa ONa ONa Реакция с гидроксидом меди(П) Эта реакция является качественной реакцией, позволяющей обнаружить в составе органического ве- щества наличие нескольких гидроксильных групп Голубой осадок Cu(OH)2 растворяется, и наблюдается образова- ние ярко-синего гликолята меди: Н н2с —он н2с—/О—сн2 2 | +Си(ОН)2 | I Н2с —ОН - н2° н2с—^-о —сн2 н гликолят меди Замещение ОН-группы на атом галогена Протекают при действии на спирты галогеноводородов: СН9 — СН2 + 2НС1 ► СН9 — СН2 + 2Н9О I I I I ОН ОН С1 С1
193 Продолжение Типы реакций Уравнения реакций Образование сложных эфиров При взаимодействии спир- тов с кислотами образуются сложные эфиры — про- текает реакция этерифи- кации Сложные эфиры глицерина и высших жирных кислот называются жирами Сложные эфиры могут образовываться также с участием неорганических кислот (азотной, серной) 0 н2с—°—н | +2Н0—С — СН3 н2с—О—Н О н2с—о—с—сн3 *• I + 2Н,0 Н2С—О—С—СНо 2 II 3 О диацетат этиленгликоля н2с—о—н н2с—о—no2 2| 1 нс—0—Н 4- 3HONO2 —► нс—о—no2 + зн2о н2с—о—н н2с—о—no2 тринитрат глицерина Реакции окисления Гзрение Взаимодействуют с кислородом с образованием углекислого газа и воды Таблица 40 Способы получения предельных многоатомных спиртов Способы получения Уравнения реакций Мягкое окисление алкенов «Мягкое» окисление алкенов водным или слабощелочным раство- ром КМпО4 приводит к получению двухатомных спиртов: ЗН3С—СН=СН2 + 2КМпО4 + 4Н2О ► пропилен ► ЗН3С—CH—СН2 + 2МпО2 + 2К0Н ОН ОН пропандиол-1,2 Замещение атома галогена на группу -ОН Реакция протекает при действии водных растворов щелочей на га- логензамещённые углеводороды: сн2—сн—СН2—снз+ 2МаОН(водн.) —► С1 С1 1,2 - дихл орбутан СН2—СН—СН2—СН3 4- 2NaCl ОН ОН бутандиол-1,2
194 Продолжение Способы получения Уравнения реакций Глицерин можно полу- чить гидролизом жиров 0 II н2с-о-с-с17н36 о н,с—он II Н+ 2| НС—0—с—С17н35 + ЗН2О нс—он + зс17н35соон 0 * 1 н2с—он н2с-0-с-с17н35 Фенолами называют производные ароматических углеводородов, в молекуле которых гидроксильная группа (—ОН) непосредственно связана с атомом углеро- да бензольного кольца. Простейшим и наиболее известным представителем этого класса соединений является фенол С6Н5ОН. Ближайшие гомологи фенола — кре- золы: орто-, мета-, лара-метилфенолы. Фенол — твёрдое бесцветное кристаллическое вещество, очень гигроскопичное, с характерным резким запахом. Кристаллическое состояние фенола обусловлено наличием водородных связей между его молекулами. Он малорастворим в воде при комнатной температуре, но его растворимость заметно увеличивается при повышении температуры. Важнейшие химические свойства фенола приведены в таблице 41, а основные способы его получения — в таблице 42. Таблица 41 Химические свойства фенолов Типы реакций Уравнения реакций Кислотные свойства По своим кислотным свойствам фенолы значительно превосходят алифатические спирты и воду, но уступают органическим и неорганическим кислотам, даже таким слабым, как угольная кислота Взаимодействие с актив- ными металлами Взаимодействие со щело- чами В отличие от спиртов фено- лы взаимодействуют с рас- творами щелочей фенолят натрия фенолят натрия При действии кислот, а также углекислого газа на водные рас- творы фенолятов происходит разрушение последних и выделе- ние фенола:
195 Продолжение Типы реакций Уравнения реакций Реакции замещения атомов водорода бензольного кольца Бромирование фенола Протекает без катализаторов под действием даже бром- ной воды Эта реакция используется как качественная реакция на фенол и его гомологи Образовавшийся 2,4,6-трибромфенол представляет собой бе- лое соединение, выделяющееся в виде осадка Нитрование фенола При действии разбавленной азотной кислоты при комнат- ной температуре образуется смесь орто- и пара-фенолов Если нитрование фенола проводить концентрированной азот- ной кислотой в присутствии концентрированной серной кисло- ты, то продуктом реакции будет 2,4,6-тринитрофенол: 2,4,6- тринитрофенол Реакции поликонденсации с альдегидами Получение феноло- формальдегидных смол ОН фенолоформальдегидная смола
196 Продолжение Типы реакций Уравнения реакций Окисление и восстановление фенола Гидрирование фенола (восстановление) Бензольное кольцо фенолов гидрируется водородом при нагре- вании в присутствии катализатора: /0Н /0Н + зн2 —’-** | Окисление фенола За счёт гидроксильной группы фенолы легко окисляются, при- чём образуется смесь темноокрашенных продуктов. Окисле- ние может протекать даже под действием кислорода воздуха, вследствие чего при длительном хранении кристаллы фенола розовеют Реакция с хлоридом железа(Ш) Взаимодействие с FeCI3 яв- ляется качественной реакци- ей на фенолы В водных растворах фенолы взаимодействуют с FeCI3 с образо- ванием смеси яркоокрашенных комплексных соединений. Фенол даёт фиолетовое окрашивание, его ближайшие гомо- логи — синее Таблица 42 Способы получения фенолов Способы получения Уравнения реакций Кумольный метод Изопропилбензол окисляют кислородом воздуха в присут- ствии катализатора. Образую- щееся соединение разлагают под действием серной кислоты СН А. 1 °2 > 1 1 + Н3С—с H2SO4 \н3 кумол фенол ацетон Действие на хлорбензол раствора гидроксида натрия При проведении этой реакции необходимо нагревание до 300 °C и давление 200 атм C1 ,ONa (i t, р г; + 2NaOH *—*- +NaCl + H2O Образующийся на первой стадии фенолят натрия обрабаты- вают затем соляной кислотой для выделения фенола: ^^.ONa || 1 + НС1 * || 1 + NaC1
197 ЗАНЯТИЕ 22 Карбонильные соединения: альдегиды и кетоны Основные элементы содержания: характерные химические свойства альдегидов и кетонов; основные способы получения альде- гидов и кетонов. Карбонильными соединениями называются органические вещества, в моле- кулах которых содержится карбонильная группа ^>С = О. В зависимости от того, с какими заместителями связана эта группа, карбонильные соединения разделяют на альдегиды и кетоны: Карбонильные соединения альдегиды кетоны О R—С Н 1Ц—с—r2 Примеры: Примеры: н—С' Н метаналь СН3—Q н этаналь О сн3—с — сн3 пропанон О СН3—С — СН2— СН3 бутанон Состав предельных альдегидов и кетонов выражается одной и той же общей формулой — СпН2пО. Изомерия альдегидов связана со строением углеродного скелета, изомерия кетонов может быть обусловлена как строением углеродного скелета, так и по- ложением карбонильной группы. Альдегиды изомерны кетонам с тем же числом углеродных атомов (межклассовая изомерия). Атом углерода, входящий в состав карбонильной группы, находится в состо- янии зр2-гибридизации. Он образует две a-связи, направленные на соседние атомы углерода или водорода, с атомом кислорода он связан одной о- и од- ной л-связью. Поскольку атом кислорода более электроотрицателен, чем атом углерода, связь С = 0 в альдегидах и кетонах полярна. Атом кислорода имеет частичный отрицательный заряд, атом углерода — частичный положительный заряд: н3с —с н
198 Из-за наличия полярной карбонильной группы альдегиды и кетоны с небольши- ми углеводородными радикалами хорошо растворимы в воде. Между молекулами карбонильных соединений не образуется водородных свя- зей, поскольку в их молекулах нет положительно заряженного атома водорода. Альдегиды — химически активные вещества. Для них характерны реакции при- соединения по двойной связи карбонильной группы, реакции окисления, конден- сации, замещения карбонильного атома кислорода (табл. 43). Эти же типы ре- акционной способности, за исключением реакций окисления, характерны и для кетонов, однако они менее активны, чем альдегиды. Основные способы получения альдегидов и кетонов приведены в таблице 44. Таблица 43 Химические свойства альдегидов и кетонов Типы реакций Уравнения реакций Реакции присоединения Характерны для альдегидов и кетонов Каталитическое гидри- рование в присутствии металлических катализа- торов — никеля, плати- ны, палладия Из альдегидов образуются первичные спирты: сн3—+н2 N1,*> сн3—сн2—он хн уксусный этиловый альдегид спирт Из кетонов — вторичные: О ОН И N1 < । сн3—с —сн3 + н2 ’ ► сн3—сн—сн3 ацетон пропанол-2 Присоединение воды /он Н—С + о * н—с —он \ н \ формальдегид гидрат формальдегида Образующиеся соединения являются, по сути, двухатомными спир- тами, обе гидроксильные группы которых присоединены к одному и тому же атому углерода. Такие спирты неустойчивы, в большин- стве случаев их невозможно выделить из растворов в свободном состоянии Присоединение спир- тов Первоначально образуются полуацетали, которые, взаимодействуя ещё с одной молекулой спирта, дают ацетали: —-х с' S+£—^87Н /ОН сн3—С + о *- н3с—с—о—сн3 н сн3 н ацетальдегид полуацеталь
199 Продолжение Типы реакций Уравнения реакций хон хо—сн3 НоС — С — 0— сн, + но— СНо ► нчс —с + Н„О О v о о О |S. Z н 1 о—сн« н 3 полуацеталь ацеталь Реакции окисления Альдегиды и кетоны по-разному относятся к действию окислителей. Альдегидная группа легко окисляется до карбоксильной, кетоны практически не окисляются Окисление перманга- натами и дихроматами В кислотной среде продуктами окисления альдегидов являются карбоновые кислоты, например: /° t ЗН3С — + К2Сг2О7 4- 4H2SO4 ► ' н /° *- ЗН3С — (\ 4- Cr2(SO4)3 4- K2SO4 + 4Н2О он В нейтральной или щелочной среде образуются не сами карбоно- вые кислоты, а их соли: t снз— СН2— С 4- 2КМпО4 4- ЗКОН ► хн ► снз— сн2— 4- 2К2МпО4 4- 2Н2О хок При окислении формальдегида в качестве продукта окисления об- разуется углекислый газ: 0 5Н — 4- 4КМпО4 4- 6H2SO4 —► хн ► 5СО2 + 4MnSO4 + 2K2SO44- 11Н2О Реакция «серебряного зеркала» Является качественной реакцией на альдегид- ную группу Н3С — С + 2[Ag(NH3)2 ]ОН ► И 0 ►НдС—+ 2Ag|+3NH3 + Н2 О onh4 Окисление формальдегида приводит к образованию карбоната ам- мония: t 1 Н— С + 4[Ag(NH3)2]OH —► (NH4)2CO3 + 4Ag|+ 6NH3 + 2H2 О ХН
200 Продолжение Типы реакций Уравнения реакций Реакция с гидрокси- дом меди(П) хо СН3— СН2— с' + 2Cu(OH)2 1 н Является качественной реакцией на альдегид- ную группу пропаналь голубой ► сн3— сн2— + Си2О + 2Н2О ОН пропановая кирпично- кислота красный Окисление формальдегида: 1 ‘ 1 Н— С + 4Си(ОН)2^ ► СО2+ 2Cu2o| + 5Н2О хн Кетоны в реакцию «серебряного зеркала» и реакцию окисления гидроксидом меди(П) не вступают. Кетоны окисляются только под действием сильных окислителей, например перманга- ната калия, причём эти реакции всегда сопровождаются разрывом связи С—С в их молекулах Гзрение Альдегиды и кетоны взаимодействуют с кислородом с образовани- ем углекислого газа и воды Реакции конденсации Для альдегидов характерны реакции конденсации, примером которых может служить конденса- ция фенола с формальдегидом Взаимодействие фор- мальдегида с фенолом фенолоформальдегидная смола Способы получения альдегидов и кетонов Таблица 44 Способы получения Уравнения реакций Дегидрирование и окис- ление спиртов Чтобы предотвратить дальнейшее окисление альдегида, исполь- зуют мягкий окислитель — СиО: Из первичных спиртов об- разуются альдегиды, из вторичных — кетоны С2Н5ОН + СиО -U- СН3СНО + Си + Н2О Дегидрирование спиртов протекает при пропускании их паров над нагретым металлическим катализатором: сн3он Ag' нсно + н2
201 Продолжение Способы получения Уравнения реакций Гидратация алкинов (ре- Альдегид можно получить только из ацетилена: акция Кучерова) H9S04 Н9С = СН нс=сн+н9о—-—1 ► н,с —сх H8SO< L ohJ ' хн Остальные алкины при гидратации дают кетоны: HgSO4 Г Н3С — С = СН2 н3с — с — сн + н2о *- I *- пропин H2SO4 [_ НО ► н,с — с—сн. II 0 ацетон Пиролиз кальциевых Х° \ и бариевых солей карбо- СНо—/(; новых кислот ч>ххз \ У Са / ► НоС — С — СН,+ СаСО, V0 •/ II сн3-с. О х О ацетон Гидролиз дигалогенал- Вг канов Н3С — СН— Br 4- 2МаОН(водн. р-р) —► Реакция протекает при действии водных раство- 1,1-дибромэтан ров щелочей на дигало- гензамещённые углево- —► СНо — С 4- 2NaBr 4- Н9О о \ ТТ дороды, содержащие два н атома галогена у одного и этаналь того же атома углерода С1 1 Н3С — С— СН3 4- 2КОН(водн. р-р) —► С1 2,2 -дихлорпропан 0 II —*- Н3С — С— СН3 + 2КС1 + Н2О ацетон Каталитическое окисле- CuCl2, PdCl2 ние алкенов 2СН2 — СН2+ О2 ► 2Н3С — С Н Кумольный способ полу- снзххснз чения ацетона приводит к СН образованию двух ценных /°Н продуктов — ацетона и фенола 1)О2; 2)H2SO4 +ИС-/ сн3 кумол фенол ацетон
202 ЗАНЯТИЕ 23 Карбоновые кислоты и сложные эфиры Основные элементы содержания: характерные химические свойства карбоновых кислот; основные способы получения карбоно- вых кислот; реакция этерификации, гидролиз сложных эфиров в кислотной и щелочной средах. Карбоновыми кислотами называют соединения, содержащие в составе моле- кулы карбоксильную группу (в сокращённой записи — СООН). По характеру радикала, связанного с карбоксильной группой, карбоновые кис- лоты классифицируют на предельные, непредельные и ароматические. Молекулы карбоновых кислот могут содержать различное количество карбо- ксильных групп. Кислоты, в составе молекул которых содержится одна карбоксиль- ная группа, относят к монокарбоновым, или одноосновным. Кислоты, содержащие в молекуле две карбоксильные группы, называются дикарбоновыми или двух- основными. Общая формула предельных одноосновных кислот — СпН2пО2. Для предельных одноосновных кислот характерна изомерия углеродного ске- лета. Кроме того, эти вещества изомерны сложным эфирам (межклассовая изо- мерия). Так, уксусная кислота является изомером метилового эфира муравьиной кислоты: сщ—С он уксусная кислота н—сС О — СЩ о метиловый эфир муравьиной кислоты (метилформиат) Первые девять представителей гомологического ряда предельных одноосновных кислот — жидкости, гомологи с ббльшим числом атомов углерода в молекуле — твёрдые, жирные на ощупь вещества. Из-за наличия водородных связей между молекулами карбоновые кислоты обладают аномально высокими температурами кипения даже по сравнению с соответствующими спиртами. Поскольку группа —СООН является гидрофильной, карбоновые кислоты с небольшим числом атомов углерода хорошо растворяются в воде. С увеличением числа углеродных атомов растёт влияние гидрофобного углеводородного радикала, и растворимость карбо- новых кислот в воде уменьшается. Для предельных одноосновных карбоновых кислот можно выделить следующие группы реакционной способности: кислотные свойства, проявляющиеся в способ- ности диссоциировать в водных растворах с образованием иона Н+ и образовывать соли; реакции замещения с участием углеводородного радикала; образование сложных эфиров и других функциональных производных кислот (табл. 45). Основ- ные способы получения карбоновых кислот приведены в таблице 46.
203 Таблица 45 Химические свойства карбоновых кислот Типы реакций Уравнения реакций Кислотные свойства Карбоновые кислоты в водном растворе диссоциируют с образованием катиона Н*: + R— Сх « » R—Сх + H ОН О" Предельные одноосновные кислоты относятся к слабым электролитам. Кислотные свойства их выражены слабее, чем, например, у соляной, фтороводородной и ортофосфорной кислот, но сильнее, чем у угольной и сероводородной кислот Образование солей при взаимодействии с металлами, оснбв- ными и амфотерными оксидами, основания- ми, аммиаком Взаимодействие с металлами, расположенными в ряду напряжений до водорода: 2НСООН + Мд -> (НСОО)2Мд + Н2? Взаимодействие с оснбвными и амфотерными оксидами: 2СН3СООН + СиО -» (СН3СОО)2Си + Н2О Взаимодействие с основаниями и аммиаком: С2Н5СООН + NaOH -» C2H5COONa + Н2О СН3СООН + NH3 -> CH3COONH4 Соли муравьиной кислоты называются формиатами, а соли уксусной кислоты — ацетатами Реакции замещения с участием углеводородного радикала Независимо от размеров углеводородного радикала замещение атомов водорода на галоген происходит у ближайшего к карбоксильной группе атома углерода Взаимодействие с га- логенами происходит в присутствии ката- лизатора — красного фосфора хо сн3— сн2— сн2 — С^ + С12 он бутановая (масляная) кислота ---► сн»— сн,— сн — С + НС1 I он С1 2-хлорбутановая кислота Образование функциональных производных Функциональные производные карбоновых кислот — это продукты замещения группы —ОН дру- гими атомами или группами атомов. Их общую формулу условно можно записать как R— С^ X Характерным для этих соединений является то, что при гидролизе из них вновь образуются кис- лоты: ^О R —С^ +н9О R— + НХ X ОН
204 Продолжение Типы реакций Уравнения реакций Получение сложных эфиров (реакция эте- рификации) <.н‘ Н3С с\. *. О—СНо -« -Н3С~С\ +н2о он к о-сн3 уксусная метиловый эфир кислота уксусной кислоты Сложные эфиры называют, прибавляя в качестве приставки название спиртового радикала к названию кислотного остатка: О сщ II 1 с сн н о снч изопропил формиат сн3— сн2— СН2 — с' СН3— Сх о —сн2—сн3 о —сн2—сн3 этилбутаноат этилацетат Получение хлоранги- дридов карбоновых кислот СНЧ—СХ + РС15 *• СНо—С^ + РОС1Ч + НС1 О X. ° о ОН С1 уксусная хлорангидрид кислота уксусной кислоты Получение анги- дридов карбоновых кислот сн,—с^ 1 он ; РоОс снз~ сх он ; — снз- < СНз С% о ангидрид уксусная уксусной кислоты кислота z - ч (уксусный ангидрид) Хлорангидриды и ангидриды — более реакционноспособные вещества, чем сами карбоновые кислоты. Так, например, в отличие от спиртов фенолы не образуют сложных эфиров при непо- средственном взаимодействии с карбоновыми кислотами, т. е. не вступают в реакции этерифи- кации. Однако сложные эфиры фенолов можно получить, действуя на них хлорангидридами или ангидридами кислот: НС1 хлорангидрид фенол уксусной кислоты фениловый эфир уксусной кислоты
205 Типы реакций Уравнения реакций сщ-с О X. о СН3-С^ о ангидрид уксусной кислоты Особенности свойств муравьиной кислоты Муравьиная кислота обладает как свойствами карбоновой кислоты, так и свойствами альдегида: она вступает в реакцию «серебряного зеркала» и окисляется гидроксидом меди(П) при нагре- вании Реакции окисления: «серебряного зер- кала», взаимодей- ствие с гидроксидом меди(П), хлором Дегидратация под действием водоотнима- ющих средств, напри- мер концентрирован- ной серной кислоты НСООН + 2[Ag(NH3)2]OH -> 2AgJ- + (NH„)2CO3 + 2NH3 + H2O HCOOH + 2Cu(OH)2 -> СО2Т + Cu2OJ- + ЗН2О НСООН + С12 -> СО2Т + 2HCI НСООН H2SO4(kohu.) + СО + Н2О Особенности свойств непредельных карбоновых кислот Эти вещества проявляют как свойства карбоновых кислот, так и свойства алкенов. Они присо- единяют галогены, водород, галогеноводороды, легко полимеризуются Гидрирование кат t сн, = сн — с + н2------------► СН3— сн2 — сх ОН ОН акриловая (пропеновая) пропановая кислота кислота Полимеризация п СН9 = СН кат., t О ОН Присоединение галоге- новодородов Из-за влияния карбоксильной группы присоединение галогеноводо- родов к акриловой и метакриловой кислотам протекает против пра- вила Марковникова /О СН2 = СН — + НВг ----* сн2—СН2 С^ ОН I ОН Вг
206 Способы получения карбоновых кислот Таблица 46 Способы получения Уравнения реакций Способы, основанные на окислении Жёсткое окисление алкенов t 5Н3С — СН2— СН = сн — СН3 + 8КМпО4 + 12H2SO4 ► /О °Ч ► 5Н3С — СН2— +5 ^С—СН3 + ОН но пропановая кислота этановая кислота + 8MnSO4 4- 4K2SO4 + 12Н2О Окисление толуола и других гомологов бензола 5 ( ) + 6KMnO4 + 9H,SO, *- \ / 4 Z 4 СООН * 5 + 6MnSO4 + 3K2SO4 + 14Н2О бензойная кислота Окисление первичных спиртов ЗС2Н5ОН + 2К2Сг2О7 + 8H2SO4 -> -> ЗСН3СООН + 2Cr2(SO4)3 + 2K2SO4+ 11Н2О Окисление альдегидов ^,0 t СН3— СН2— Сх + 2Cu(OH)2J—► Н пропаналь ^0 —► сн3— СН2 — Сх + Си2с4 + 2Н2О ОН пропановая кислота Окисление алканов Промышленный способ получения некоторых карбоновых кислот кат., t 2СН3 — СН2— СН2 — СН3 + 5О2 ► 4СН3 + 2Н2О ОН
Продолжение Способы получения Уравнения реакций Способы, основанные на гидролизе Гидролиз сложных эфиров в кислотной и щелочной среде В кислотной среде образуются исходная карбоновая кислота и спирт: СН3— Сх о — сн2— сн2— сн3 пропиловый эфир уксусной кислоты H2SO4 + н2о ^.о « > сн3—+ сн3—сн2—сн2—он он уксусная кислота пропиловый спирт Под действием щелочей гидролиз сложных эфиров протекает не- обратимо, причём продуктами являются спирт и соль соответству- ющей карбоновой кислоты: СН3— Сх О— СН2— сн2— СН3+ NaOH ^,0 СН3— + СН3— СН2— СН2— ОН ONa ацетат натрия пропиловый спирт Из тригалогенопроиз- водных алканов В качестве исходных веществ используют галогензамещённые угле- водороды, содержащие три атома галогена у одного углеродного атома: С1 I Н2О СН»— СН,— С — Cl + 4NaOH-------► I Cl 1, 1, 1-трихлорпропан СН3— СН2 — С + 3NaCl + 2Н2О ONa пропионат натрия
208 Продолжение Способы получения Уравнения реакций Гидролиз проводят водным раствором щёлочи, поэтому сначала по- лучаются соответствующие соли, которые после обработки сильны- ми кислотами дают карбоновые кислоты: СН,— СН,— С + НС1 —► О & Х^ ONa ► СН,— СН9— С + NaCl о £ х^ ОН пропионовая кислота Получение муравьиной кислоты При нагревании оксида углерода(И) с порошкообразным гидрокси- дом натрия при повышенном давлении образуется формиат натрия: t, р СО + NaOH > HCOONa, из которого действием более сильной кислоты получают муравьи- ную кислоту: HCOONa + HCI -> НСООН + NaCl Сложные эфиры можно рассматривать как производные карбоновых кислот, в которых атом водорода гидроксильной группы замещён на углеводородный радикал: ,о хо --------- R-< О —Н----О—R1 В состав сложных эфиров могут входить фрагменты самых различных органиче- ских и неорганических кислот и различные спирты (одноатомные и многоатомные). К сложным эфирам относятся также жиры, представляющие собой эфиры трёх- атомного спирта глицерина и высших карбоновых кислот. Сложные эфиры называют, прибавляя в качестве приставки название спиртово- го радикала к названию кислотного остатка. При этом название кислотного остатка получается из названия соответствующей кислоты путём замены окончания -овая на -оат. Для остатков муравьиной и уксусной кислот используются названия фор- миат и ацетат соответственно. Также часто используются названия, образуемые из названий спиртов и кислот, дающих эфир, например: Н— С\ /СН3 о—сн хсн, о изопропилформиат, или изопропиловый эфир муравьиной кислоты метилбензоат, или метиловый эфир бензойной кислоты Наиболее важной реакцией сложных эфиров является их гидролиз, который мо- жет протекать как в кислотной, так и в щелочной среде (см. табл. 46). В кислот- ной среде сложные эфиры гидролизуются с образованием исходных карбоновых кислот и спиртов:
203 О ^2 SO,i сн3— + н2о « * хо — сн9— сн9— сщ пропиловый эфир уксусной кислоты 2 СН3— Сх + СН3— сн2— сн2— он он уксусная кислота пропиловый спирт Кислотный гидролиз является обратимым, по сути, это реакция, обратная реак- ции этерификации. Под действием щелочей гидролиз сложных эфиров протекает необратимо, при- чём продуктами являются спирт и соль соответствующей карбоновой кислоты. Ги- дролиз сложных эфиров в щелочной среде также называют омылением: сн3— с. ХО — сн2— СН2— СН3 + NaOH ---* СН3— Сх + сн3— сн2— сн2— он ONa ацетат натрия пропиловый спирт Сложные эфиры образуются в результате реакции этерификации (см. табл. 45). Примеры заданий, при выполнении которых нужно продемонстрировать знание химических свойств и способов получения кислородсодержащих органических соединений 4) дипропиловый эфир 5) пропановая кислота может протекать под действием различных реагентов: меди(П), дихромата калия, перманганата калия и т. д. зависит от строения молекулы спирта. Так, первичные сначала до альдегидов, а при избытке окислителя далее Пример 1. /7ри окислении пропанола-1 может образоваться 1) пропилен 2) пропанон 3) пропаналь Окисление спиртов кислорода, оксида Продукт окисления спирты окисляются до карбоновых кислот; вторичные спирты окисляются до кетонов; третичные спирты могут окисляться только в жёстких условиях разрывом связи С—С. Пропанол-1 относится к первичным спиртам, поэтому его окисление приво- дит к образованию соответствующего альдегида — пропаналя; при избытке окислителя возможно образование и пропановой кислоты. Ответ
210 Пример 2. Реакция с аммиачным раствором оксида серебра характерна 1) для метанола 4) для диметилового эфира 2) для пропаналя 5) для муравьиной кислоты 3) для пропанона Аммиачный раствор оксида серебра проявляет слабые окислительные свой- ства за счёт иона Ад*. Взаимодействие с этим реактивом является каче- ственной реакцией на альдегидную группу, поэтому с [Ag(NH3)2]OH взаи- модействуют альдегиды, муравьиная кислота, углеводы (альдегидоспирты). Признаком реакции является образование слоя металлического серебра. Ответ: 2 5 натрием гидроксидом меди(И) 4) реакция гидрирования 5) реакция полимеризации Глицерин является трёхатомным Для этого вещества характер- из вариантов ответа. СН(ОН) — СН2(ОН). Пример 3. Для глицерина характерна(о) 1) плохая растворимость в воде 2) взаимодействие с 3) взаимодействие с Рассмотрим каждый спиртом: СН2(ОН) — на хорошая растворимость в воде, поэтому вариант ответа 1 неверен. Как представитель класса спиртов глицерин взаимодействует с натрием (ответ 2 верный). Взаимодействие с гидроксидом меди(П) является качественной реакцией на многоатомные спирты, в том числе и на глицерин (вариант 3 верный). Реакции гидрирования и полимеризации невозможны для глицери- на, так как в его молекуле нет кратных связей (варианты 4 и 5 неверные). Ответ: 2 3 Пример 4. Установите соответствие между исходными веществами и ор- ганическим веществом, которое преимущественно образуется при их взаи- модействии. ИСХОДНЫЕ ВЕЩЕСТВА h2so4, t сн3— сн — сн2— сн3---- ПРОДУКТ ВЗАИМОДЕЙСТВИЯ 1) метилэтиловый эфир 2) бутанон 3) бутен-2 4) бутен-1 5) эти л ацетат 6) этилформиат А) он Б) ОН В) Г) СН„— СН,— ONa + СН„С1 3 “ о h,so4, t сн3—сн2—он + нсоон-------- t СН3—сн — СН2—СН3 + СиО — Рассмотрим каждую из приведённых схем реакций. Под действием концен- трированной серной кислоты при нагревании спирты подвергаются дегидра-
211 тации. Как правило, при несильном нагревании протекает межмолекулярная дегидратация, приводящая к образованию простых эфиров; при более высо- ких температурах протекает внутримолекулярная дегидратация и образуются алкены. Для первой из реакций конкретная температура не указана, следо- вательно, необходимо рассмотреть оба направления дегидратации. При межмолекулярной дегидратации бутанола-2 образуется ди(втор-бутило- вый) эфир: V.-' ’.г : « • • . И’ H2so4, t 2CH3— сн — сн2— сн3------► СН3— сн2— сн — о— сн — сн2— сн3 + н2о он сн3 сн3 V л. При внутримолекулярной дегидратации бутанола-2 образуется алкен: H2SO4, t 4 сн3— сн — сн2— сн3-------► СН3— сн = сн — сн3 + н2о он В данном случае для определения продукта реакции следует использовать правило Зайцева, в соответствии с которым преимущественно образуется бутен-2, а не бутен-1. Итак, продуктом первой из предложенных реакций может быть ди(втор-бутиловый) эфир или бутен-2. Но первый из вариантов ответа не предусмотрен условием задания, следовательно, выбираем вари- ант ответа 3 — бутен-2. При нагревании в присутствии СиО спирты подвергаются окислению. Бута- нол-2 относится ко вторичным спиртам, при окислении которых образуются кетоны: СН3—СН —сн2—CH3 + CuO —► сн3—с—СН2—CH3 + Cu + H2O I II он о Таким образом, продуктом второй реакции является бутанон. Этиловый спирт взаимодействует с муравьиной кислотой с образованием сложного эфира — этилформиата: у) h2so4, t н — с + но—сн2— СН3 ч * Н — С\ + н2о он о—сн2—сн3 Наконец, при взаимодействии этилата натрия и хлорметана образуется про- стой эфир — метилэтиловый СН„—СН,— ONa + СН,С1 ----► СН,—СН,— О —СН,+ NaCl О <6 о о о Ответ: А Б В г 3 2 6 1
212 Пример 5. В заданной схеме превращений х сн3— сн —сн2 СН3— С—СН3 сн3—сн = сн2 3 3 ОН О tf веществами X и Y соответственно являются 1) Н2О 2) СиО 3) Си(ОН)2 4) Превращение алкена в спирт происходит присоединении воды, вилом Марковникова, Этот вторичный спирт СН3ОН 5) НСНО гидратации, т.е. при соответствии с пра- в процессе протекает в продуктом является пропанол-2, кетона с помощью СиО. Поскольку реакция преимущественным можно окислить до Ответ: X Y 1 2 Пример 6- В заданной схеме превращений нс = сн уО сн3— сх ХО—сн3 х i веществами X и Y соответственно являются 1) этилен 4) метанол 2) ацетальдегид 5) уксусная кислота 3) этанол Рассмотрим первое из предложенных превращений. В одну стадию из аце- тилена можно получить как этилен (путём гидрирования), так и ацетальдегид (путём гидратации). Из этих двух веществ нужно выбрать такое, из которого в одну стадию можно получить уксусную кислоту, необходимую для получе- ния метилового эфира уксусной кислоты. Уксусная кислота образуется при окислении ацетальдегида. Таким образом, вещество X — ацетальдегид, ве- щество Y — уксусная кислота. х Ответ: 2 5 Пример 7. Напишите уравнения реакций, с помощью которых можно осу- ществить следующие превращения: КОН (спирт, р-р), t 1, 2-дибромэтан ----------------i н2о, Hgz+ X,---------- [Ag(NH3)2] он, t Х2 Ва(ОН)2, t t ----------~ Х3 — сн3 — с —сн3 сн3— с/ onh4 О При написании уравнений реакций используйте структурные формулы орга- нических веществ.
213 Х3 При обработке галогензамещённых алканов спиртовым раствором щёлочи протекает реакция дегидрогалогенирования. При этом из моногалогензаме- щённых алканов образуются алкены, а из дигалогензамещённых алканов — алкины: 1) СН2 СН2 (спиртовой р-р), t | + 2КОН --------------► СН = СН + 2КВг + 2Н2О Вг Вг Итак, вещество X, — это ацетилен. При взаимодействии ацетилена с водой в присутствии солей ртути протекает реакция Кучерова, приводящая к об- разованию уксусного альдегида: Hg2+ 2) СН = СН + Н2О-----► СН3— С^ хн Уксусный альдегид вступает в реакцию «серебряного зеркала». Так как в ка- честве окислителя в этой реакции используется аммиачный раствор оксида серебра (т. е. раствор содержит избыток аммиака), образуется не уксусная кислота, а её аммиачная соль — ацетат аммония: .о ( о 3) СН3 — + 2[Ag(NH3)2]OH-*- сн3 — С + 2Ag + 3NH3 + Н2О ХН XONH, Как и из других солей аммония, из ацетата аммония можно вытеснить ам- миак при нагревании со щёлочью — в данном случае с гидроксидом бария: 4) 2СН3—+Ва(ОН)2 (СН3СОО)2Ва + 2NH3 + 2Н2О xonh4 Разложение при нагревании кальциевых или бариевых солей карбоновых кислот (пиролиз) приводит к получению кетонов: о 5) (СН3СОО)2Ва СН3—с —СН3+ВаСО3 Пример 8. Напишите уравнения реакций, с помощью которых можно осу- ществить следующие превращения: „ Сакт.^_ „„ ci2, FeCl3 Х| ► CzL’H Х2 no2 KMnO4, H2so4, t *" Х4 При написании уравнений реакций используйте структурные формулы орга- нических веществ. В результате первого превращения требуется из неизвестного вещества X, получить бензол. Существуют различные способы получения бензола из алканов, циклоалканов, других углеводородов, но в данном случае выбор исходного вещества X, задаётся указанными в схеме условиями: реакция
214 должна протекать при нагревании в присутствии активированного угля. Это тримеризация ацетилена (реакция Зелинского): Сакт ’ 1) зсн = сн — Следующая реакция — хлорирование бензола: ci 2) FeCl. + ci2 --------- 4-HCI Далее из хлорбензола нужно перейти к веществу Хд, из которого можно полу- чить 4-нитротолуол. Для этого можно использовать реакцию Вюрца—Фиттига: сн, О 3) + 2Na + CHoCl О + 2NaCl В этой реакции образуются побочные продукты — этан и дифенил С6Н5С6Н5. Желательно указать их при записи уравнения реакции, но если этого не сде- лать, оценка за выполнение не снижается при условии правильной записи уравнения 3. Следующая стадия — нитрование толуола. Здесь также возможно обра- зование второго и даже третьего продуктов реакции — 2-нитротолуола и 2,4,6-тринитротолуола. Как же быть? Нужно учесть, что по условию задания требуется написать уравнение (а не схему!) реакции и расставить коэффи- циенты. Поэтому записываем уравнение реакции нитрования толуола с об- разованием требуемого продукта 4-нитротолуола. Побочные продукты при желании можно указать в скобках. Но как и в случае с предыдущей реакци- ей, если уравнение реакции 4 составлено правильно, а побочные продукты не указаны, оценка снижена не будет. сн, сн сн» \ no2| Й' Cl NO + H2O + HNO, <3 А если сначала цию Вюрца—Фиттига? Такой порядок действий тоже допустим: no2 провести нитрование хлорбензола, а затем провести реак- H2so4, 4) || 3) II H2SO + hno3 —-— Cl
215 сн„ + 2NaCl 4) С1 4- СЩС14- 2Na о no2 no2 Такое решение тоже будет оценено полным количеством баллов, недаром в критериях оценивания рассматриваемых заданий присутствует фраза «допу- скаются иные формулировки ответа, не искажающие его смысла». И наконец, последняя реакция — окисление 4-нитротолуола перманганатом ка- лия в кислой среде. Как и в случае толуола, в этих условиях метильная группа 4-нитробензола окисляется до карбоксильной, нитрогруппа не затрагивается: сн„ соон 5) 4- 6KMnO4 4- 9H2SO4 4- 6MnSO4 4- 3K2SO4 + 14H2O . .. no2 no2 Обращаем ваше внимание на то, что по условию задания и для процесса окисления нужно составить не схему, а именно уравнение реакции. Это зна- чит, что нужно указать в том числе и все неорганические продукты реакции и правильно расставить стехиометрические коэффициенты. Пример 9. Напишите уравнения реакций, с помощью которых можно осу- ществить следующие превращения: Zn НВг СН3СНВгСН2СН2Вг кат., t КМпО4, Н2 *"хз 3,4-диметилгексан 1) CH3CHBrCH2CH2Br 4- Zn При написании уравнений реакций используйте структурные формулы орга- нических веществ. При обработке 1,3-дибромбутана металлическим цинком происходит замы- кание цикла и образуется метилциклопропан: Н2С ZnBr2 | снз н2с Малые циклы неустойчивы и склонны к реакциям присоединения с раскры- тием цикла. При взаимодействии метилциклопропана с бромоводородом преимущественно образуется 2-бромбутан: Н2С\ 2) Q.CH — СНд+НВг НоС t —► СН3— СНВг— сн2— сн3
216 Другим продуктом этой реакции может быть 1-бромбутан, но запись урав- нения с его участием будет ошибкой, поскольку из этого вещества нельзя получить следующее вещество цепочки — 3,4-диметилгексан. При обработке 2-бромбутана натрием протекает реакция Вюрца: сн3 сн3 сн3 3) 2СН3— СН2— СН — Вг 4- 2Na - ► СН3— СН2~ СН — СН — CH2~ СН3+ 2NaBr Затем протекает дегидроциклизация (ароматизация) 3,4-диметилгексана: Последняя реакция — окисление полученного 1,2-диметилбензола, обе ме- тильные группы которого при этом окисляются до карбоксильных: сн3 соон J^.CHg А.СООН 5) 5 Н | + 12КМпО4 + 18H2SO4--► 5 || | + 12MnSO4 + 6K2SO4 + 28Н2О ЗАНЯТИЕ 24 Амины и аминокислоты Основные элементы содержания: характерные химические свойства алифатических аминов и анилина; основные способы получе- ния аминов; характерные химические свойства аминокислот, основные способы их получения. Амины — это органические производные аммиака, в молекуле которого один или несколько атомов водорода замещены углеводородными радикалами. В зависимости от числа углеводородных радикалов, связанных непосредственно с атомом азота, различают первичные, вторичные и третичные амины, например: ,н сн3— сн2— сн2— хн пропиламин первичный амин ,Н СН3— СН2—N сн3 метилэтамин вторичный амин /СН3 СН3— СН2— N сн3 диметилэтамин третичный амин Общая формула предельных алифатических аминов — CnH2n+1NH2, или CnH2n+3N. Запах простейших аминов напоминает запах аммиака, другие амины также име- ют характерный, часто неприятный запах. Между молекулами аминов образуются водородные связи, однако они слабее, чем те, которые существуют между молекулами воды или спиртов. Поэтому тем- пературы кипения аминов ниже, чем у спиртов с близкой молекулярной массой.
217 Низшие амины (метиламин, диметиламин, триметиламин и этиламин) при обыч- ных условиях газообразные вещества. Амины с большей молекулярной массой — жидкости или твёрдые вещества. Низшие амины, как и аммиак, хорошо растворимы в воде, по мере роста угле- водородного радикала растворимость в воде уменьшается. В водных растворах водородные связи образуются не только между молекулами аминов, но также между молекулами аминов и воды. Важнейшим представителем ароматических аминов является анилин. Это бесцветная жидкость, темнеющая на воздухе из-за окисления. В воде анилин рас- творяется очень плохо, его оснбвные свойства выражены слабее, чем у аммиака и алифатических аминов. Характерные химические свойства аминов приведены в таблицах 47 и 48, а ос- новные способы их получения — в таблице 49. Таблица 47 Химические свойства аминов Типы реакций Уравнения реакций Оснбвные свойства. Алифатические амины проявляют свойства слабых оснований. Однако их оснбвные свойства выражены немного сильнее, чем у аммиака. Оснбвные свойства аромати- ческих аминов слабее, чем у аммиака. В порядке усиления оснбвных свойств вещества можно расположить в следующий ряд: основные свойства усиливаются ароматические первичные вторичные амины 3 амины амины Взаимодействие с водой сн3— nh2 + Н2о сн3— NH3 ОН Образование гидроксид-ионов в растворах аминов подтверждает- ся изменением окраски индикаторов: лакмус приобретает синюю, а фенолфталеин малиновую окраску Взаимодействие с кислотами При взаимодействии аминов с кислотами образуются соли алкилам- мония: сн3— nh2 + НС1 —* сн3— ын3сГ метиламин хлорид метиламмония СНЧ СНЧ 1 1 . СН3—N + HC1 *- СН3—NH С1 сн3 сн3 триметиламин хлорид триметиламмония 2 сн3— сн2— nh2 + H2SO4 —*- [сн3—сн2—nh3+]2so4’ этиламин сульфат этиламмония Соли аминов — твёрдые, хорошо растворимые в воде вещества. Щёлочи, как более сильные основания, вытесняют из них амины: . _ t СН3— NHgCl + КОН ► СН3— NH2 4- КС1 4- Н2О
218 Продолжение Типы реакций Уравнения реакций Взаимодействие с азотистой кислотой. Реакции аминов с азотистой кислотой протекают по- особому и позволяют различать первичные, вторичные, третичные амины между собой Реакция с HNO2 Первичные амины взаимодействуют с HNO2 с выделением азота и образованием спирта: сн3— сн2— nh2 + о = n—он —► СН3— сн2— он + n2 + н2о этиламин этанол Вторичные амины превращаются в нитрозоамины — желтоватые жидкости, малорастворимые в воде. Третичные амины реагируют с HNO2 так же, как и с другими кисло- тами, образуя растворимые в воде соли азотистой кислоты Алкилирование — введение в молекулу исходного амина новых алкильных групп. Позволяет из первичного амина получить вторичный, а из вторичного — третичный Взаимодействие с галогеналканами Атомы водорода аминогруппы замещаются на углеводородные ради- калы. При этом образуется соль: н сн3—сн2 СН3 — СН2— N +Вг — СН, — СН, ► ^NH9*Br ХН СН3 —CHj,/ этиламин бромэтан бромид диэтиламмония Действуя на эту соль щёлочью, можно получить свободный амин: сн3—сн2 сн3—сн2 NH2Br + NaOH ► NH + NaBr + Н,О сн3—сн2х сн3—сн2х 2 бромид диэтиламмония диэтиламин Горение аминов. При горении аминов образуются азот, углекислый газ и вода, например: 4C2H5NH2 + 15О2 -► 8СО2 + 2N2 + 14Н2О Таблица 48 Особенности химических свойств анилина Типы реакций Уравнения реакций Оснбвные свойства. Выра Так, анилин не взаимодейс жены слабее, чем у аммиака и алифатических аминов: основные свойства усиливаются сн3 ► NH, ► .NH I* J сн3х гвует с водой (т. е. не изменяет окраску индикатора)
210 Продолжение Типы реакций Уравнения реакций Взаимодействие с кис- лотами При взаимодействии анилина с кислотами образуются соли фенил- аммония: хлорид фениламмония Если на раствор образовавшейся соли подействовать щёлочью, то снова образуется анилин: Реакции по бензольному ядру. Из-за электронодонорного влияния аминогруппы реакции элек- трофильного замещения для анилина протекают легче, чем для бензола. Заместители направля- ются в положения 2, 4 и 6 по отношению к аминогруппе Взаимодействие с бромной водой Для бромирования анилина можно использовать бромную воду: 2, 4, 6-триброманилин, белый осадок Данную реакцию можно использовать как качественную для иден- тификации анилина и его производных
220 Таблица 49 Способы получения аминов Способы получения Уравнения реакций Взаимодействие галоген- алканов с аммиаком В этой реакции образуется не свободный амин, а его соль. Для получения амина её обрабатывают щёлочью: C.H.Br + NH3 [CJ-LNHJ’Br [C2H5NH/]Br- + NaOH —> CJH.NH, + NaBr + H20 Взаимодействие спиртов с аммиаком C2H5OH + NH, - кат'» c5h,nh5+ н.о Восстановление нитросо- единений Этим путем получают первичные алифатические амины и ани- лин. В промышленности используют водород в присутствии катали- заторов: кат. сн3— no2+ зн2 ► сн3— NH2+ 2Н2О no2 nh2 h кат. С + ЗН2 ► | +2Н2О В лаборатории можно использовать цинк, железо и другие вос- становители. Если реакция проходит в кислотной среде, обра- зуются соли аминов: СНо— NOo + 3Zn + 7НС1 ► СН,— NHoCl- + 3ZnCl, + 2Н,О О с. о о А Л NO2 NHgCl” +3Fe 7нс1 —* + згес1г+2Н2° Аминокислоты — это соединения, которые содержат две функциональные группы: аминогруппу —NH2 и карбоксильную группу —СООН. Простейшей амино- кислотой является аминоуксусная кислота (глицин): H2N—СН2—СООН В молекулах аминокислот может содержаться более одной амино- и карбо- ксильной групп. Также в составе этих соединений могут содержаться ароматиче- ские радикалы, гидроксильные группы —ОН, тиольные группы —SH и другие за- местители (табл. 50). Аминокислоты проявляют свойства, характерные как для кислот, так и для ами- нов. Они взаимодействуют и с кислотами, и со щелочами. Таким образом, амино- кислоты проявляют амфотерные свойства (табл. 51). Основные способы получения аминокислот приведены в таблице 52.
221 Таблица 50 Некоторые а-аминокислоты, входящие в состав белков Название аминокислоты Формула аминокислоты Глицин ^0 h2n—сн2 — хон Аланин /° НоС — сн — с^ | хон NH2 Фенилаланин \/ ХО —сн — с' | хон nh2 Валин нзс\ сн—СН— Н3СХ I хон nh2 Тирозин s to 1 Z— о X я “ 1 о / О о я Лизин /° h2n—сн2—сн2— сн2— сн2— сн—с хон nh2 Глутаминовая кислота Х с — сн2— сн2— сн — с нох I хон nh2 Серин но—сн?—сн—с 2 1 хон nh2 Цистеин HS—сн?—сн—с 2 1 хон nh2
222 Таблица 51 Химические свойства аминокислот Типы реакций Уравнения реакций Реакции с участием аминогруппы Взаимодействие с кисло- тами сн9—С + НС1 —► сн9—с 1 2 хон 1 \ \)Н NH„ NH, С1 м о глицин гидрохлорид глицина (хлорид глициния) Взаимодействие с азоти- стой кислотой Реагируя с азотистой кислотой, аминокислоты образуют гидрок- сикислоты: / / сн2—С +НО—№0 —► сн2—С +н2о + n2 1 он 1 он nh2 он гликолевая кислота Реакции с участием карбоксильной группы Взаимодействие со ще- лочами СН2—С +NaOH ► СН2—С + Н20 1 ОН 1 ONa nh2 nh2 глицин натриевая соль глицина (аминоацетат натрия) Образование сложных эфиров / /> сн2 с + С2Н5ОН СН2—с' +н2о NH 0Н L О-СН2-СН3 1тГ12 ^**^2 этиловый эфир аминоуксусной кислоты Образование пептидов Конденсация аминокислот При конденсации аминокислот образуется пептидная связь -CO-NH-. Соединение, которое образуется при взаимодей- ствии двух аминокислот, называется дипептидом: 0 0 II II H,N—СН—С— OH + Н—NH—СН—С—ОН ► —► h2n—сн—с—NH—сн—с—он + н2о R1 R2 дипептид
223 Типы реакций Уравнения реакций Название дипептида складывается из названий аминокислот, ко- торые его образуют, например глицилаланин: О О II II h2n—сн2—с—ОН 4- н—NH—сн—с—он —► сн3 О О II II ► H2N—сн2—с—NH—СН—с—ОН 4- Н2О сн3 глицилаланин Возможно дальнейшее присоединение к дипептиду новых мо- лекул аминокислот с образованием полипептидов. Дипептиды и полипептиды подвергаются гидролизу с образованием смеси аминокислот Таблица 52 Способы получения аминокислот Аминокислоты относятся к веществам природного происхождения. Чаще всего их получают при гидролизе белков. При этом образуется сложная смесь а-аминокислот К синтетическим способам получения аминокислот можно отнести реакцию замещения галогена (хлора или брома) на аминогруппу в галогенкарбоновой кислоте: Н3С—СН—С + 2NH3 ХОН С1 НоС—сн—с 3 I хон nh2 + NH4C1 Необходимое для этого синтеза галогенопроизводное можно получить из соответствующей кар- боновой кислоты (реакция Гелля—Фольгарда—Зелинского): Z' Р красн. zz СНп—сн9—С 4- С19 ------------► СНо—сн—С + НС1 о Z \ Z о I X он I он С1
224 ЗАНЯТИЕ 25 Жиры. Углеводы Основные элементы содержания: строение и характерные химические свойства жиров; строение и характерные химические свой- ства углеводов (моносахаридов, дисахаридов, полисахаридов). Природные животные и растительные жиры представляют собой сложные эфи- ры трёхатомного спирта глицерина и высших жирных карбоновых кислот. Общая формула жиров ° сн,—о—с—R I о I II 1 сн—о—С —R1 I ? , сн2—о—С—R где R, R' и R2 — углеводородные остатки карбоновых кислот. Поскольку все жиры образованы остатком одного и того же спирта — глицери- на, их называют также триглицеридами. Высшие жирные кислоты, как правило, содержат одну карбоксильную группу, чётное достаточно большое число атомов углерода (обычно от 10 до 22) и нераз- ветвлённую углеродную цепь. Они могут быть предельными (не содержат кратных связей) или непредельными (обычно содержат от одной до четырёх двойных свя- зей) (табл. 53). Таблица 53 Наиболее распространённые высшие жирные кислоты Название кислоты Молеку- лярная формула Структурная формула Предельные кислоты Пальмитино- вая С,5Н3,СООН .. _,СООН о Стеариновая С,7Н35СООН ~ .. .соон и3с Непредельные кислоты Олеиновая с17нмсоон СН,—(СН,)7—СН=СН—(СН2)7—соон 10 9 Линолевая С17Н3,СООН СН,—(СН,)4—сн=сн—сн,—СН=СН—(СН,)7—соон 13 12 10 9 Линолено- вая с^н^соон СНЧ—сн,—СН=СН—СН,—СН=СН—сн,—СН=СН—(СН,)7—соон 3 2 16 15 2 13 12 2 10 9 2 7
225 Жиры проявляют химические свойства, присущие сложным эфирам. Прежде всего это гидролиз, который может протекать как в кислотной, так и в щелочной среде. Кроме того, жиры, содержащие остатки непредельных кислот, вступают в реакции, характерные для алкенов: они присоединяют водород и галогены, под- вергаются окислению и полимеризации (табл. 54). Таблица 54 Химические свойства жиров Типы реакций Уравнения реакций Гидролиз жиров Гидролиз в кислотной среде 0 II сн2 о—ч? с17н35 1 II н+ сн—о—с—с17н35 + зн2о =*=₽ 0 сн2—о—с—с15н31 сн9—он 1 =*=₽ сн—OH + C15H31COOH+2C17H35COOH сн2—он Реакция обратима, её продуктами являются глицерин и карбоновые кислоты Гидролиз в щелочной среде (омыление) 0 II сн2 0 с с17н35 1 II t сн—о—С—С15н31 + 3NaOH =а=₽ О сн2—о с с15н31 сн?—он 1 =*=₽ сн—OH + 2C15H31COONa + C17H35COONa сн2—ОН Щелочной гидролиз протекает необратимо и приводит к образова- нию глицерина и натриевых или калиевых солей карбоновых кис- лот, называемых мылами
Продолжение Типы реакций Уравнения реакций Особенности свойств жиров, содержащих остатки непредельных кислот. Для непредельных жиров характерны свойства непредельных соединений: реакции присоединения, окисления, по- лимеризации. Непредельные жиры обесцвечивают бромную воду и раствор перманганата калия Взаимодействие с бромной водой О II сн2 О—< с17н33 СН 0—С С17Н33 + ЗВг2(водн. р-р) ► о СН2 О С С17Н33 О сн2 О Сj 7Н33Вг2 ► СН О С С17Н33Вг2 0 1 II СН2 О С С17Н33Вг2 Гидрирование О сн2 о—с17н33 1 II Ni сн о—с с17н33 + зн2 -ыи. о 1 II сн2 о с с17н33 триолеат глицерина (жидкий жир) О си2-oU-с17н35 1 II —- сн-о-с с17н35 О сн2—о—с—с17н35 тристеарат глицерина (твёрдый жир) Полимеризация Непредельные жиры, полимеризуясь на воздухе или под действием инициирующих агентов, образуют эластичные, блестящие и прочные плёнки Окисление Может протекать на воздухе или под действием типичных лабора- торных окислителей. Непредельные жиры, в отличие от насыщенных жиров, обесцвечивают раствор перманганата калия
227 Углеводы — это органические вещества природного происхождения, содержа- щие несколько гидроксильных и одну карбонильную группу. К ним относятся глю- коза, сахароза, фруктоза, целлюлоза, крахмал и многие другие вещества. Часто углеводы называют также сахарами. Углеводы подразделяют на моносахариды, дисахариды (их молекулы состоят из двух остатков моносахаридов) и полисахариды (молекулы состоят из многих остатков моносахаридов). Важнейшими из моносахаридов являются глюкоза, фруктоза, рибоза и дезок- с и рибоза: ,0 Z СН2—ОН Z Z 1 н н—с—он 1 с=о | Н I Н 1 н—с—он н—с—н но—с—н но—с—н н—с—он н—с—он н—с—он н—с—он н—с—он н—с—он н—с—он н—с—он н2с—он н2с—он н2с—он н2с—он Рибоза Дезоксирибоза Глюкоза Фруктоза С5Н10°5 СбН10°4 ^6^12^6 ^6^12^6 Альдегидоспирты Кетоноспирт Глюкоза, рибоза и дезоксирибоза содержат одну альдегидную группу и несколь- ко спиртовых; по этому признаку их относят к альдегидоспиртам. Фруктоза, со- держащая одну кетонную группу и пять спиртовых, является кетоноспиртом. Поскольку молекулы моносахаридов содержат одну карбонильную и несколько спиртовых групп, эти вещества вступают как в реакции, характерные для альдеги- дов или кетонов, так и в реакции, характерные для многоатомных спиртов. Кроме того, глюкоза проявляет свои особые, специфические свойства, например подвер- гается различным видам брожения (табл. 55). Таблица 55 Химические свойства моносахаридов на примере глюкозы Типы реакций Уравнения реакций Реакции с участием альдегидной группы Восстановление р /он < н2с рн 1 н-с-он н-с-он но-А- Н кат. НО—С—Н + н, ► н-с-он н-с-он н—с—он н—с—он н,А н2с хон он глюкоза сорбит
228 Продолжение Типы реакций Уравнения реакций Окисление Как и альдегиды, глюкоза вступает при нагревании во взаимодействие с аммиачным раствором оксида серебра(1) (реакция «серебряного зерка- ла») и со свежеосаждённым гидроксидом меди(П). Это качественные ре- акции на глюкозу: Z Z 1 хн I onh4 н— с—он н—с-он но—i—н t но—с—н + 2[Ag(NH3)2]OH ► | + 2Agj + 3NH3 + Н2О н—с—он н—с—он н—с—он н—с—он Н2С Н2С \)Н он глюкоза соль аммония глюконовой кислоты I н 1 ОН н—С—ОН Н—с—он но—с— н t но—с— н + 2Cu(OH), ► | +Си9О| + 2Н,О н—с-он н—с—он 2 2 Н—С—ОН H—с— он 1 хД Н,С. Н2С иН ОН глюкоза глюконовая кислота Реакции с участием гидроксильных групп Взаимодействие с Си(ОН)2 Водный раствор глюкозы даёт качественную реакцию на многоатомные спирты — взаимодействует со свежеосаждённым гидроксидом меди(Н) с образованием комплексного соединения тёмно-синего цвета Образование сложных эфиров При взаимодействии глюкозы с ангидридами кислот образуются сложные эфиры. Например, в реакции с уксусным ангидридом образуется пентаа- цетат глюкозы: СНз о=с о—сн2 Н X . О-s. н \/ 1 /СН3 \ | о с; н с ;с о Ч । ох X н У1 II /С—° X । ° \У О—С—СН3 Н,С х 'СГ 1 1 н о—с—сн, II О пентаацетат глюкозы
229 Продолжение Типы реакций Уравнения реакций Образование простых эфиров Простые эфиры глюкозы образуются при действии на неё спиртов или галогеналканов. Например, при взаимодействии с метанолом образуется монометиловый эфир: но—сн2 н ,с О, н н УС Ан\?Н f о—сн3 т -сг н он монометиловый эфир глюкозы Брожение — реакции разложения углеводов с образованием более простых органических ве- ществ, протекающие под действием ферментов Спиртовое брожение ферменты дрожжей» + О 14 D £ О Другие виды ферментов могут вызывать молочнокислое, маслянокислое, лимоннокислое и другие виды брожения, приводящие к образованию соответствующих органических кислот: молочной, масляной, лимонной и т. д. Дисахариды состоят из двух остатков моносахаридов. К дисахаридам относят- ся, например, сахароза (состоит из остатков а-глюкозы и 0-фруктозы), лактоза (молочный сахар), мальтоза (образована двумя остатками а-глюкозы), целлобиоза (образована двумя остатками 0-глюкозы) и многие другие вещества. Полисахариды — это природные полимеры, которые можно рассматривать как продукт поликонденсации моносахаридов. К полисахаридам относятся крахмал (образован остатками а-глюкозы), целлюлоза (образована остатками р-глюкозы), гликоген, хитин и другие вещества. Важнейшим свойством полисахаридов являет- ся их гидролиз, а также реакции с участием гидроксильных групп (табл. 56). Таблица 56 Химические свойства дисахаридов и полисахаридов Типы реакций Уравнения реакций Свойства дисахаридов Гидролиз Гидролиз дисахаридов приводит к образованию соответствую- щих моносахаридов, например: С^НггОп + Н2О ► С6Н12О6 + С6Н12О6 сахароза глюкоза фруктоза Ci2H220n + Н2О ► 2С6Н12О6 мальтоза глюкоза
230 Продолжение Типы реакций Уравнения реакций Реакции с участием гидроксильных групп Водные растворы дисахаридов взаимодействуют со свежеосаж- дённым гидроксидом меди(11) с образованием комплексного со- единения тёмно-синего цвета Окисление Мальтоза, целлобиоза и лактоза, как и моносахариды, вступают в реакцию «серебряного зеркала» и реакцию с Си(ОН)2 при на- гревании. Эти дисахариды называют восстанавливающими. Сахароза в эти реакции не вступает. Её называют невосстанав- ливающим дисахаридом Свойства полисахаридов Гидролиз Конечным продуктом гидролиза крахмала и целлюлозы является глюкоза: (СвН,0О5)„ + (л- 1)Н2О лСвН12О6 крахмал глюкоза (целлюлоза) Образование сложных эфиров Практическое значение имеют сложные эфиры целлюлозы — нитраты и ацетаты, которые образуются при действии на цел- люлозу азотной кислоты и уксусного ангидрида соответственно: О2№. сн2 Н /С (к l/l н \ xl X 0N02 Н УX \i IZ н с с н ono2 L J n тринитрат целлюлозы г 1”* и О-СН2 и ,с ск с; н сх 3 > х н о—с—сн3 II L о _|„ триацетат целлюлозы Иодкрахмальная реакция В результате взаимодействия крахмала с иодом образуется ком- плекс синего цвета. Эту реакцию используют для качественного обнаружения как иода, так и крахмала
231 4 5) NaCl 3 4 Ответ: Ответ: 2) 3) 4) Примеры заданий, при выполнении которых нужно продемонстрировать знание химических свойств аминов, аминокислот, жиров, углеводов Пример 1. В каком ряду вещества расположены в порядке усиления ос- нбвных свойств? NH3 -> С2Н5—NH—С2Н5 -> С3Н7—NH2 CJ-L—NH2-> С3Н7—NH2-> NH, О О с. о f с о СН3—NH2-> NH3 -> с6н5—nh2 NH3 -> C2H4—NH2 -> C2HS—NH—C2H5 Амины являются органическими производными аммиака и, как и аммиак, проявляют оснбвные свойства. Из-за влияния радикала С6Н5— оснбвные свойства ароматических аминов выражены слабее, чем у аммиака. В случае алифатических аминов, наоборот, появление углеводородных радикалов в молекуле способствует усилению оснбвных свойств, причём первичные ами- ны являются более сильными основаниями, чем аммиак, а вторичные амины ещё более сильными основаниями, чем первичные амины. Пример 2. Из предложенного перечня выберите два вещества, с которы- ми взаимодействует метилэтиламин. 1) С6Н6 2) NaOH 3) HCI 4) C2H5CI Метилэтиламин относится к вторичным аминам: СН3—NH—С2Н5. Амины явля- ются органическими производными аммиака и проявляют оснбвные свойства, поэтому реагируют с кислотами (как органическими, так и неорганическими) с образованием солей (ответ 3 верен). Взаимодействие с углеводородами, щелочами и солями для аминов нехарактерно (ответы 1, 2 и 5 неверны). Ещё одним свойством аминов является способность вступать в реакции ал- килирования с галогеналканами: СН3—NH—С2Н5 + C2H5CI -> [CH3N(C2H5)2]CI Пример 3. Из предложенного перечня выберите два вещества, которые взаимодействуют с 2-аминопропановой кислотой. 1) этан 4) толуол 2) сульфат натрия 5) бромоводород 3) пропанол-1 СН3—CH(NH2)—СООН
232 Обратим внимание на наличие двух функциональных групп в молекуле. Вспомним, какие реакции возможны с участием каждой из этих групп. По аминогруппе возможны реакции с кислотами (ответ 5). По карбоксильной группе возможны реакции с металлами, основаниями, спиртами (образова- ние сложных эфиров), другими аминокислотами (образование пептидов), по- этому верным является также вариант ответа 3. Ответ: 3 5 Пример 4. Установите соответствие между реагирующими веществами и признаком протекающей между ними реакции. РЕАГИРУЮЩИЕ ВЕЩЕСТВА А) фенол и бромная вода Б) этен и бромная вода В) этиленгликоль и гидроксид меди(Н) Г) белок и азотная кислота ПРИЗНАК РЕАКЦИИ 1) появление фиолетового окрашива- ния 2) появление жёлтого окрашивания 3) образование белого осадка 4) обесцвечивание раствора 5) образование ярко-синего раствора При выполнении данного задания следует опираться на знание химических свойств и качественных реакций органических веществ (приложение 6). А) Фенол реагирует с бромной водой с образованием белого осадка 2,4,6-трибромфенола (ответ 3). Б) Этен также реагирует с бромной водой, при этом жёлтая окраска брома исчезает — наблюдается обесцвечивание раствора (ответ 4). В) Этиленгликоль является многоатомным спиртом. Многоатомные спирты реагируют с гидроксидом меди(Н), при этом образуется ярко-синий раствор (ответ 5). Г) При действии на белок концентрированной азотной кислоты наблюдается жёлтое окрашивание — ксантепротеиновая реакция (ответ 2). Ответ: А Б В Г 3 4 5 2 Пример 5. Установите соответствие между двумя веществами и реаген- том, с помощью которого можно различить эти два вещества. ВЕЩЕСТВА А) пропанол и пропанон Б) метиламин (водный р-р) и метанол (водный р-р) В) бутин-1 и бутин-2 Г) пропанол и глицерин РЕАГЕНТ 1) натрий 2) гидроксид натрия 3) гидроксид меди(Н) 4) [Ag(NH3)2]OH 5) фенолфталеин
233 Ответ: А) Отличить кетон от спирта можно при помощи металлического натрия. Спирты реагируют с натрием с выделением водорода: 2СН3СН2СН2ОН + 2Na -> 2CH3CH2CH2ONa + H2t О С. С. & £ £ £ (ответ 1). Б) Среда водного раствора метанола — нейтральная. Метиламин хорошо растворим в воде, среда его раствора — щелочная, поскольку амины про- являют оснбвные свойства. Фенолфталеин в водном растворе метиламина окрашивается в малиновый цвет (ответ 5). В) Бутин-1, в отличие от бутина-2, содержит концевую тройную связь и реа- гирует с аммиачным раствором оксида серебра с образованием осадка: СН3СН2С н СН + [Ag(NH3)2]OH -> СН3СН2С = CAg^ + 2NH3 + Н2О Бутин-2 в эту реакцию не вступает (ответ 4). Г) И пропанол, и глицерин — спирты. Однако глицерин, в отличие от пропа- нола, является многоатомным спиртом. Многоатомные спирты реагируют с ги- дроксидом меди(Н), при этом образуется раствор ярко-синего цвета (ответ 3). Ответ: А Б В Г 1 5 4 3 Пример 6. Из предложенного перечня выберите два вещества, которые не подвергаются гидролизу. 1) фруктоза 4) сахароза 2) крахмал 5) целлюлоза 3) дезоксирибоза Известно, что гидролизу подвергаются дисахариды и полисахариды, моно- сахариды не гидролизуются. Следовательно, для ответа на поставленный во- прос достаточно выбрать из приведённого списка углеводов моносахариды. Это фруктоза, дезоксирибоза.
234 Модуль IV. Количественные отношения в химии: расчёты по химическим формулам и уравнениям химических реакций Основные элементы содержания: количество вещества, молярная масса, молярный объём, объёмные отношения газов, массовая доля вещества в растворе и в смеси веществ. Основой успеха решения любой расчётной задачи является тщательный анализ того, о каких физических величинах идёт речь в условии задания и какие форму- лы, связывающие эти величины, необходимо использовать при расчётах. При решении расчётных задач по химии понадобятся следующие физические величины и формулы для их вычисления (табл, 57). Таблица 57 Некоторые физические величины и формулы для их вычисления Физическая величина Размерность физической величины Формулы для вычисления Название Обозна- чение Масса т г, 1 мг= ю-3 г, 1 кг= 103 г о. ! ? I * SEVf 11 11 11 I ? I II ее а о. Е Е~ Е Е" Объём V 1 л = 1 дм3, 1 МЛ = 1 СМ3 = 10"3 л, 1 м3= 103 л = 106 мл 1/ = ^Р-Ра) <₽•₽•> р = п Vu Количество вещества п моль, 1 ММОЛЬ = 10'3 моль, 1 кмоль = 103 моль т п= — М _ у_ П(газа) ~ 1/ Массовая доля а) элемента в со- единении б) вещества в рас- творе (смеси) W величина безразмер- ная или % a) w(9)~ (J , где Аг(Э) — мг атомная масса элемента, п — число атомов элемента в соеди- нении, Mr — относительная молекулярная масса. б) IV = т(вещ) W (вещ.) га(р-ра) Плотность Р г/мл, 1 кг/м3 = 1 г/мл п _ га (р.ра) ” (р-ра) у v (Р-ра) = М- Р (газа) 1/ VM Относительная плот- ность газа А по газу В D DB(A) величина безразмерная DBW=^ в М(В) М (воздуха) = 29 г/моль
235 Продолжнение Физическая величина Размерность физической величины Формулы для вычисления Название Обозна- чение Молярная масса М г/моль Численно равна относительной молекулярной массе вещества или относительной атомной массе элемента Молярный объём газа VM л/моль 22,4 л/моль при нормальных усло- виях (н. у.) Доля выхода продук- та реакции п величина безразмерная или % _ ^(практ.) _ У(практ.) _ п(практ.) ш(теор.) ^(теор.) п(теор.) где т(теор), V(Teop), л(теор) - теорети- чески вычисленные соответствен- но масса, объём или количество вещества продукта реакции, ™(практ.)> Цпракт). "(практ.)- ПРЭКТИЧе- ски полученные соответственно масса, объём или количество ве- щества продукта реакции ЗАНЯТИЕ 26 Расчёты массы вещества или объёма газа по известному коли- честву вещества, массе или объёму одного из участвующих в реакции веществ Алгоритм решения большинства задач этого типа предполагает следующую после- довательность действий: — По формуле п = или л(газа) = находим количество вещества того реагента, масса или объём которого известны. — По уравнению реакции определяем количество вещества того реагента, массу или объём которого надо найти. т V — Отвечаем на вопрос задачи, используя формулу л = — или л(газа) = у. Приведём примеры выполнения подобных задач. Задача 1. Аммиак объёмом 1,12 л (н. у.) поглотили раствором, содержащим 4,9 г серной кислоты. Определите массу полученной соли. Дано: Найти: У(МН3)=1,12 л т(соли) — ? Л7(Н25О4) = 4,9 г Решение 1) Записывая в краткой форме условие задачи, мы не можем сразу указать, массу какой соли надо найти. Серная кислота двухосновная, в зависимости от соотноше- ния реагентов она образует два типа солей — кислые и средние.
236 Приведём уравнения двух возможных реакций: NH3+H2SO4=NH4HSO4 (1) 2NH3+ H2SO4= (NH4)2SO4 (2) При соотношении реагентов n(NH3): n(H2SO4) = 1 : 1 образуется кислая соль, а при соотношении n(NH3): n(H2SO4) = 2:1 — средняя соль. 2) В данной задаче даны количества двух реагирующих веществ, поэтому нахо- дим количество вещества для каждого из них, т. е. количества веществ аммиа- ка и серной кислоты. Определяем количество вещества аммиака по формуле п = —• n(NH3) = 1,12 л : 22,4 л/моль = 0,05 моль и количество вещества серной кислоты по формуле п = : м n(H2SO4) = 4,9 г : 98 г/моль = 0,05 моль. Получаем соотношение реагентов 1:1, следовательно, в результате реакции об- разуется кислая соль NH4HSO4. 3) Определяем количество вещества гидросульфата аммония по уравнению реак- ции (1): n(NH4HSO4) = n(H2SO4) = n(NH3) = 0,05 моль 4) Находим массу гидросульфата аммония, исходя из формулы т = п- М: m(NH4HSO4) = 0,05моль • 115 г/моль = 5,75 г Ответ: 5,75 г. Расчёты по химическим уравнениям, если одно из реагирующих веществ дано в избытке Задача 2. Сернистый газ объёмом 5,6 л (н. у.) поглотили раствором, содержа- щим 16 г гидроксида натрия. Определите массу каждой соли в образовавшемся растворе. Дано: Найти: l/(SO2) = 5,6 л т(солей) — ? m(NaOH) = 16 г Решение 1) При пропускании сернистого газа через раствор гидроксида натрия в начале реакции гидроксид натрия находится в избытке. Поэтому образуется средняя соль: SO2 + 2NaOH = Na2SO3 + Н2О (1) 2) Находим количества веществ сернистого газа и гидроксида натрия по форму- I/ m ЛЭМ П(газа) = 77~ • И П = 77 СООТВеТСТВвННО: n(SO2) = 5,6 л : 22,4 л/моль = 0,25 моль n(NaOH) = 16 г : 40 г/моль = 0,4 моль n(SO2): n(NaOH) = 0,25 : 0,4 3) Найденное соотношение реагентов указывает на то, что в конце реакции, кото- рая описывается уравнением 1, в избытке находится сернистый газ. Поэтому рас-
237 чёт ведём по гидроксиду натрия. Находим количество вещества образовавшейся в ходе реакции средней соли: n(Na2SO3) = 0,5n(NaOH) = 0,20 моль. Находим количества веществ прореагировавшего сернистого газа: n(SO2) = 0,5n(NaOH) = 0,20 моль Следовательно, сернистого газа осталось n(SO,)„„nnr = 0,25 моль - 0,20 моль = 0,05 моль 4 z UV I cUiOVD При дальнейшем пропускании через полученный раствор сернистый газ реагирует с находящейся в растворе средней солью. При этом образуется кислая соль: Na2SO3 + SO2 +Н2О = 2NaHSO3 (2) n(NaHS03)p6pa30eajl0Cb — 2 )осталось ’ n(Na2SO3)npopearapoeaJlo = = 0,05 моль Следовательно, сульфита натрия осталось n( Na,SO, )„„„„„ = 0,20 моль - 0,05 моль = 0,15 моль 4) Находим массу каждой соли в растворе по формуле т = п • М: m(Na2SO3) = 0,15 моль • 126 г/моль=18,9 г m(NaHSO3) = 0,1 моль • 104 г/моль=10,4 г Ответ: m(Na2SO3) = 18,9 г, m(NaHSO3) = 10,4 г. Расчёты объёмных отношений газов при химических реакциях Задача 3. Какой объём азота (н. у.) выделится при полном сгорании 100 л ам- миака? Дано: Найти: V(NH3) = 100 л V(N2) — ? Решение 1) Записываем уравнение реакции, расставляем коэффициенты, подчёркиваем нужные нам вещества: 4NH3 + ЗО2= 6Н2О + 2N2 И аммиак, и азот при нормальных условиях являются газами. А для газов соот- ношение количества вещества (коэффициентов в уравнении реакции) равно соотношению объёмов. 2) По уравнению реакции соотношение количества вещества аммиака и азота рав- но 2:1. Это соотношение равно объёмному отношению этих веществ: 2:1= U(NH3): l/(N2). Из полученной пропорции находим l/(N2): V(N2) = 0,5 V(NH3) l/(N2) = 0,5 • 100 л = 50 л Ответ: 50 л.
238 Расчёты по термохимическим уравнениям Задача 4. По термохимическому уравнению реакции 2НгО(ж) - 2Н2(г) + О2(г) - 571 кДж рассчитайте количество теплоты, поглощённой при образовании 2,24 л водорода (н. у.). Дано: Найти: 1/(Н2) = 2,24 л Q —? Решение 1) Уравнение реакции уже записано. Подчеркнём в нем интересующие нас веще- ства и величины: 2Н2О = 2Нг + 02 - 571 кДж 2) Определим количество вещества водорода по формуле л(гаэа) = : л(Н2) = 2,24 л : 22,4 л/моль = 0,1 моль 3) По уравнению на получение 2 моль водорода затрачивается 571 кДж теплоты, а на получение 0,1 моль водорода затрачивается О кДж теплоты. Составим про- порцию: 2 моль — 571 кДж 0,1 моль — Q кДж Найдём количество теплоты Q, поглощённой в ходе реакции: Q = 571 • 0,1 : 2 = 28,55 кДж Ответ: 28,55 кДж. ЗАНЯТИЕ 27 Расчёты с использованием понятия «массовая доля вещества в растворе» Раствор — это однородная (гомогенная) система, состоящая из частиц растворён- ного вещества и растворителя, чаще всего воды. Масса раствора складывается из массы растворённого вещества и массы растворителя: ™ (р-ра) — ^(вещ.) + ^(р-теля) В условии некоторых задач может быть приведена не масса раствора или раство- рителя, а объём. В этом случае для вычисления массы необходимо воспользовать- ся формулой т(р-ра) — ^(р-ра) Р Необходимо запомнить, что плотность воды р=1 г/мл, или 1 кг/л. Массовая доля вещества в растворе и/ вычисляется по формуле iv = т<тецч т(р-ра) Чтобы выразить массовую долю вещества в процентах, нужно полученное значе- ние умножить на 100. Указанную выше формулу можно использовать для расчёта массы веществ в рас- творе:
239 П7/__,|1 к — W • ГП. , (вещ.) (Р-ра) Рассмотрим решение некоторых задач данного типа. Задача 5. Какая масса сульфата лития содержится в 400 г 12%-ного раствора этой соли? Дано: Найти: т(р-ра) = 400 Г П7(вещ) - ? iv=0,12 Решение Для нахождения массы вещества воспользуемся формулой Л7(вещ ,= iv • т(вещ) = 400 • 0,12 = 48 г Ответ: 48 г. Задача 6. Определите массовую долю соли в растворе, полученном при до- бавлении 50 мл воды к 350 г 20%-ного раствора бромида натрия. Дано: Найти: ^(р-ра) — 350 Г ^2(соли) 2 IV, = 0,2 т(р.твля)=50 Г Решение Важно знать, что при добавлении воды к имеющемуся раствору не происходит изменения массы вещества в растворе, увеличивается только масса раствора. 1) Определяем массу полученного раствора: m(DDa)2=350 г+ 50 г = 400 г ре/ 2) Определяем массу вещества в исходном растворе по формуле т(вещ.) — W * m(p-pa)’ т(вещ.) = 0-2 • 350 г = 70 г 3) Находим массовую долю вещества в конечном растворе по формуле и/2(С0ЛИ)= 70/400 = 0,175, или 17,5% Ответ: 17,5%. Задача 7. Определите массовую долю соли в растворе, полученном при добав- лении 5 г хлорида калия к 150 г 10%-ного раствора этой соли. Дано: Найти: т(р-ра)=150г ^(соли)-? IV, = 0,1 ^(вещ.) — 3 Г Решение Важно знать, что при добавлении вещества к уже имеющемуся раствору данного вещества происходит как увеличение массы самого вещества в растворе, так и увеличение массы раствора.
240 1) Определяем массу полученного раствора: <”(р-ра2) =150 г + 5 г= 155 г 2) Определяем массу вещества в исходном растворе: т(Вещ.> = 0'1- 15° г =15 г 3) Определяем массу вещества в полученном растворе: т(вещ.2)= 15 Г + 5 г = 20 г 4) Находим массовую долю вещества в конечном растворе: и<2(вещ)= 20/155 = 0,129, или 12,9 % Ответ: 12,9%. Задача 8. К 200 мл 10%-ного раствора серной кислоты (р = 1,07 г/мл) добавили 20 мл концентрированной серной кислоты (массовая доля кислоты 96 %, плот- ность 1,84 г/мл). Определите массовую долю кислоты в полученном растворе. Дано: Найти: Ир-ра 1)= 200 мл и/3(КИСЛ0ТЫ) - ? W| = 0,10 р, = 1,07 г/мл ia па= 20 мл (р-ра 2) и/2 = 0,96 р2 =1,84 г/мл Решение Важно знать, что масса раствора, полученного при сливании двух растворов, рав- на сумме масс исходных растворов. Также и масса вещества в полученном рас- творе равна сумме масс вещества в исходных растворах. 1) Определяем массы исходных растворов: Л7(Р.ра1)= 1,07 г/мл • 200 мл = 214 г т(р-ра2)= 1>84 г/мл ’ 20 мл = 36,8 г 2) Определяем массу полученного раствора: Л7(ооа3)= 214 г+ 36,8 г =250,8 г 3) Определяем массу вещества в исходных растворах: л1(ВеЩ.1) = 0,1 • 214 г = 21,4 г П7(вещ.2) = 0-96 • 36,8 г = 35,33 г 4) Определяем массу вещества в полученном растворе: л^(вещ. з)— 21 >4 г + 35,33 г = 56,73 г 5) Находим массовую долю вещества в конечном растворе: и/3 = 56,73/250,8 = 0,226, или 22,6 % Ответ: 22,6%. Задача 9. 56 л аммиака (н. у.) растворили в 500 мл воды. Определите массо- вую долю аммиака в полученном растворе. Дано: Найти: V(NH3) = 56 л w(NH3) — ? V(H2O) = 500 мл
241 Решение 1) Для нахождения массовой доли аммиака в растворе необходимо знать массу растворённого вещества. Находим количество вещества аммиака: n(NH3) = 56 л / 22,4 л/моль = 2,5 моль m(NH3) = 2,5 моль • 17 г/ моль = 42,5 г 2) Определяем массу раствора аммиака: ^(р-теля) = V • р = 500 мл • 1 г/мл = 500 г = т(р-теля) + ^(вещ.) = 500 г + 42,5 г = 542,5 г 3) Определяем массовую долю аммиака в полученном растворе: w(NH3) = 42,5 г : 542,5 г = 0,078 Ответ: 0,078, или 7,8%. При нахождении массовой доли вещества в растворе необходимо учитывать тот факт, что растворяемое вещество может уже содержать воду или может взаимодействовать с водой. Рассмотрим несколько примеров таких задач. Задача 10. Определите массовую долю вещества в растворе, полученном при растворении в 90 г воды 10 г глауберовой соли (Na?SO4 • ЮН2О). В данном случае для приготовления раствора используется кристаллогидрат, в со- став которого входит не только соль, но и вода. Поэтому масса растворённого ве- щества — сульфата натрия — будет меньше 10 г. Дано: Найти: m (Na2SO4 • ЮН2О) = 10 г iv(Na2SO4) - ? ^<р-твля) = 90 г Решение 1) Находим массу раствора: т(р-ра) = 90 + 10 = 100 г 2) Определяем молярную массу глауберовой соли: M(Na2SO4 • ЮН2О) = M(Na2SO4) + ЮМ(Н20) = 142 + 180 = 322 г/моль 3) Находим количество вещества глауберовой соли: n(Na2SO4 • ЮН2О) = 10/322 = 0,03 моль 4) Количество вещества сульфата натрия равно количеству вещества глауберовой соли: n(Na2SO4) = n(Na2SO4- ЮН20) =0,03 моль 5) Находим массу сульфата натрия в растворе: m(Na2S04) = 0,03 • 142 = 4,26 г 6) Находим массовую долю соли в растворе: w = 4,26/100 = 0,0426, или 4,26 % Ответ: 4,26%. Задача 11. Определите массовую долю вещества в растворе, полученном при добавлении 10 г оксида натрия к 90 г воды.
242 При решении этой задачи необходимо учитывать, что оксид натрия взаимодейству- ет с водой. При этом образуется раствор гидроксида натрия. Поэтому определить надо массовую долю гидроксида натрия в растворе. Дано: Найти: m(Na2O) = 10 г w(NaOH) — ? ^(р-теля) — 90 Г Решение 1) Записываем уравнение реакции, расставляем коэффициенты: Na2O + Н2О = 2NaOH 2) Определяем количество вещества оксида натрия по формуле п = : n(Na2O) = 10 г : 62 г/моль = 0,16 моль 3) Определяем количество вещества гидроксида натрия: по уравнению реакции из одного моля оксида натрия образуется два моля гидроксида: n(NaOH) = 2n(Na2O) = 2 • 0,16 моль = 0,32 моль 4) Определяем массу гидроксида натрия в растворе: m(NaOH) = 0,32 моль • 40 г/моль = 12,8 г 5) Находим массу раствора: = 90 + 10 = 100 г 6) Находим массовую долю гидроксида натрия в растворе: w = 12,8 : 100= 0,128, или 12,8% Ответ: 12,8%. Задача 12. Определите массовую долю вещества в растворе, полученном при добавлении 10 г натрия к 90 г воды. Как и в предыдущем случае, при решении этой задачи необходимо учитывать вза- имодействие натрия с водой. Но при взаимодействии натрия с водой, кроме ги- дроксида натрия, образуется и водород, который уходит из реакционной смеси. Масса конечного раствора при этом будет меньше суммы масс реагирую- щих веществ. Дано: Найти: m(Na) = 10 г w(NaOH) — ? ^(р-теля) ~ 90 Г Решение 1) Составляем уравнение реакции: 2Na + 2Н2О = 2NaOH + Н,Т 2) Определяем количество вещества натрия по формуле п = : м n(Na) =10 г: 23 г/моль = 0,43 моль 3) Определяем количество вещества гидроксида натрия: n(NaOH) = n(Na) = 0,43 моль 4) Определяем количество вещества выделившегося водорода:
243 л(Н2) = 0,5n(Na) = 0,215 моль 5) Определяем массу гидроксида натрия в растворе: m(NaOH) = 0,43 моль • 40 г/моль= 17,2 г 6) Определяем массу выделившегося водорода: т(Н2) = 0,215 моль • 2 г/моль = 0,43 г 7) Находим массу раствора: т(р-ра>= 90 + 10 - °-43 = ">57 г 8) Находим массовую долю гидроксида натрия в растворе: w(NaOH) = 17,2:99,57 = 0,173, или 17,3%. Ответ: 17,3%. Задача 13. Определите массовую долю вещества в растворе, полученном при добавлении 10 г тетрахлорсилана к 90 г воды. Тетрахлорсилан взаимодействует с водой. При этом образуются хлороводо- род, который растворяется в воде, и нерастворимая кремниевая кислота. Масса конечного раствора и в этом случае будет меньше суммы масс реагирую- щих веществ. Дано: Найти: m(SiCI4) = 10 г w(HCI) — ? т(р-гелЯ) = 90 г Решение 1) Составляем уравнение реакции: SiCI4 + ЗН2О = H2SiO3+ + 4HCI 2) Определяем количество вещества тетрахлорсилана по формуле п = : n(SiCI4) = 10 г : 170 г/моль = 0,06 моль 3) Определяем количество вещества кремниевой кислоты: n(H2SiO3) = n(SiCI4) = 0,06 моль 4) Определяем количество вещества хлороводорода: n(HCI) = 4n(SiCI4) =0,24 моль 5) Определяем массу кремниевой кислоты: m(H2SiO3) = 0,06 моль • 78 г/моль = 4,68 г 6) Определяем массу хлороводорода в растворе: m(HCI) = 0,24 моль • 36,5 г/моль = 8,76 г 7) Находим массу раствора: пт = 90 + 10 — 4,68 = 95,32 г ip-pa; 8) Находим массовую долю хлороводорода в растворе: w(HCI) = 8,76 : 95,32 = 0,092, или 9,2% Ответ: 9,2%.
244 Таким образом, мы рассмотрели основные алгоритмы решения задач на нахожде- ние массовой доли вещества в растворе. Теперь разберём способы решения бо- лее сложных задач, которые требуют выполнения большего числа арифметических действий, однако эти действия выполняются по алгоритмам, описанным выше. Задача 14. К раствору, полученному при добавлении 28,4 г оксида фосфора(\/) к 200 г горячей воды, добавили 200 г 16%-ного раствора гидроксида натрия. Вы- числите массовую долю соли в полученном растворе. При растворении в воде оксида фосфора(\/) образуется трёхосновная фосфорная кислота. Эта кислота в реакции с гидроксидом натрия в зависимости от соотно- шения реагентов может давать три типа солей: NaOH + Н3РО4 = NaH2PO4 + Н2О 2NaOH + Н3РО4 = Na2HPO4 + 2Н2О 3NaOH + Н3РО4 = Na3PO4 + ЗН2О Поэтому, чтобы установить, какая соль получится в результате реакции, сначала надо найти количества веществ реагентов, а затем, зная их соотношение, опреде- лить образующуюся соль. Решение 1) Составляем уравнение реакции оксида фосфора(У) с водой: Р2О5 + ЗН2О = 2Н3РО4 2) Определяем количество вещества оксида фосфора(\/): л(Р2О5) = 28,4 г : 142 г/моль = 0,2 моль 3) Определяем количество вещества фосфорной кислоты: л(Н3РО4) = 2л(Р2О5) = 0,4 моль 4) Определяем массу гидроксида натрия в растворе: m(NaOH) = 200 г • 0,16 = 32 г 5) Определяем количество вещества гидроксида натрия в растворе: n(NaOH) = 32 г : 40 г/моль = 0,8 моль 6) Составляем уравнение реакции гидроксида натрия с фосфорной кислотой: так как соотношение n(NaOH): л(Н3РО4) = 0,8 моль : 0,4 моль = 2 : 1, то в результате ре- акции образуется гидрофосфат натрия: 2NaOH + Н3РО = Na2HPO4+ 2Н,0 о 4 2 4 2 7) Определяем количество вещества гидрофосфата натрия в растворе: л(Ыа2НРО4) = л(Н3РО4) = 0,4 моль 8) Определяем массу гидрофосфата натрия в растворе: m(Na2HPO4) = 0,4 моль • 142 г/моль = 56,8 г 9) Находим массу раствора: масса раствора будет складываться из массы оксида фосфора(\/), массы воды и массы раствора гидроксида натрия: m.D м) = 28,4 г + 200 г + 200 г = 428,4 г
245 10) Находим массовую долю гидроксида натрия в растворе: w=56,8 / 428,4 = 0,133, или 13,3% Ответ: 13,3%. Задача 15. Смесь меди и оксида меди(П) может прореагировать с 219 г 10%- ного раствора соляной кислоты или 61,25 г 80%-ного раствора серной кислоты. Определите массовую долю меди в смеси. Дано: Найти: т(р.ра)(НС1) = 219 г w(Cu) - ? w(HCI) = 0,1 Л1(мж) (H2SO4) = 61,25 г w(H2SO4) = 0,8 Решение 1) Учитывая, что оксид меди(П) реагирует как с соляной, так и с серной кислотой, а медь — только с концентрированной серной кислотой, записываем уравнения реакций: CuO + 2HCI = CuCI2+Н2О СиО + H2SO4 = CuS04 + Н2О Си + 2H2SO4 = CuSO4 + SO2 + 2Н2О 2) Рассчитываем количество вещества хлороводорода: m(HCI) = 219 г • 0,1 =21,9 г л(НС1) = 21,9 г : 36,5 г/моль = 0,6 моль 3) Соляная кислота реагирует только с оксидом меди(И), поэтому, зная количество вещества хлороводорода, можно определить количество вещества оксида меди(П): л(СиО) = 0,5л(НС1) = 0,3 моль 4) Рассчитываем количество вещества серной кислоты общее и то, которое затра- чено на каждую реакцию: m(H2SO4) = 61,25 г • 0,8 = 49 г л(Н2ЗО4)(общее) = 49 г: 98 г/моль = 0,5 моль n(H2SO4)(B реакции с СиО) = л(СиО) = 0,3 моль л(Н2ЗО4)(в реакции с Си) = 0,5 моль - 0,3 моль = 0,2 моль 5) Рассчитываем количество вещества меди: л(Си) = 0,5л(Н2ЗО4) = 0,1 моль 6) Вычисляем массы веществ и массу их смеси: m(CuO) = 0,3 моль • 80 г/моль = 24,0 г m(Cu) = 0,1 моль • 64 г/моль = 6,4 г т(смеси) = 24,0 + 6,4 = 30,4 г 7) Вычисляем массовую долю меди в смеси: iv(Cu) = 6,4/30,4 = 0,21, или 21% Ответ: 21 %. Задача 16. Карбид кальция массой 6,4 г растворили в 87 мл бромоводородной кислоты (р = 1,12 г/мл) с массовой долей 20%. Какова массовая доля бромово- дорода в образовавшемся растворе?
246 Решение 1) Составляем уравнение реакции: СаС2 + 2НВг = С2Н2Т + СаВг2 2) Рассчитываем количества веществ реагентов: л(НВг)исх = 87 • 1,12 • 0,2 : 81 = 0,24 моль л(СаС2) = 6,4 : 64 = 0,1 моль Делаем вывод, что бромоводород взят в избытке. Поэтому расчёты будем вести по карбиду кальция. 3) Рассчитываем количество бромоводорода — прореагировавшего и избытка: n(HBr)npopear = 2л(СаС2) = 0,2 моль л(НВг)иэб = 0,24 - 0,2 = 0,04 моль 4) Находим массу избытка бромоводорода в растворе: m(HBr) = 0,04 • 81 =3,24 г 5) В результате реакции выделилось газообразное вещество — ацетилен. Его мас- са не входит в массу полученного раствора. Рассчитываем количество вещества и массу ацетилена: л(С2Н2) = л(СаС2) = 0,1 моль т(С2Н2) = 0,1 • 26 = 2,6 г 6) Рассчитываем массу образовавшегося раствора: лг, =87 ’ 1,12 + 6,4 - 2,6= 101,24 г 7) Рассчитываем массовую долю бромоводорода в конечном растворе: w(HBr) = 3,24 : 101,24 = 0,032, или 3,2% Ответ: 3,2%. ЗАНЯТИЕ 28 Задачи на установление молекулярной и структурной формулы вещества Установление молекулярной формулы вещества по уравнению протекающей реакции Для решения задач этого типа необходимо знать общие формулы классов органи- ческих веществ и общие формулы для вычисления молярной массы веществ этих классов (см. приложение 3). Большинство задач этого типа решается в соответствии со следующим ал- горитмом: 1) Вначале составляем уравнение реакции в общем виде. 2) По формуле п= £ или гтя.= определяем количество вещества известного /V/ вещества, для которого даны масса или объём или массу или объём которого можно вычислить по условию задачи. 3) По уравнению реакции определяем количество (моль) того вещества, формулу которого нужно установить. 4) Вычисляем молярную массу неизвестного вещества по формуле М = т/п
247 5) Подставляем полученное значение молярной массы в формулу для вычисления молярной массы веществ данного класса. Получаем математическое уравнение с одним неизвестным. 6) Решая уравнение, находим число атомов углерода в молекуле. 7) Зная число атомов углерода в молекуле, составляем молекулярную формулу ве- щества. Задача 17. При взаимодействии 22 г предельного одноатомного спирта с из- бытком металлического натрия выделилось 2,8 л газа (н. у.). Определите молеку- лярную формулу спирта. Решение Спирты реагируют с активными металлами с выделением водорода. Для решения этой задачи общую формулу предельного одноатомного спирта удоб- нее записать в таком виде: С„Н2л + 1ОН. 1) Составляем уравнение реакции в общем виде, подчёркиваем формулы нужных нам веществ: 2СПН^,ОН + 2Na = 2С„Н2л + ,ONa + у 2) Находим количество вещества водорода по формуле л(газа) = —: л(Н2) = 2,8 л : 22,4 л/моль = 0,125 моль Записываем математическое выражение для вычисления молярной массы спирта по общей формуле: М(СПН^,ОН) = 12л + 2л + 1 + 16+1 = (14л + 18) г/моль и выражение для расчёта количества вещества спирта в соответствии с формулой _ _ m. Г7 “• • л^Н^ОН) = 22 г : (14л + 18) г/моль 3) В соответствии с уравнением реакции л(СпН2п+1ОН) = 2л(Н2) = 0,25 моль Следовательно, 22 : (14л + 18) = 0,25 14л + 18 = 88 14л = 70 л = 5 Ответ: Молекулярная формула спирта — СдН^ОН. Задача 18. Установите молекулярную формулу ацетиленового углеводорода, если молярная масса продукта его реакции с избытком бромоводорода в 4 раза больше, чем молярная масса исходного углеводорода. Дано: Найти: М(продукта): М(исходного углеводорода) = 4 С^Н^ _ 2 —? Решение 1) Составляем уравнение реакции в общем виде: СпН2п.2 + 2НВг-+СпН2лВг2
248 2) Записываем математическое выражение для вычисления молярной массы алки- на и дибромалкана: М(С/)Н2п_2)=14л - 2 М(С„Н2лВг2) = 14л + 160 Исходя из того что молярная масса дибромалкана в 4 раза больше молярной мас- сы алкина, составляем уравнение: (14л+160): (14л-2) = 4 14л + 160 = 4(14л-2) л = 4 Записываем молекулярную формулу алкина: С4Нв. Нахождение молекулярной формулы вещества по массе (объёму) продуктов сгорания Задача 19. При сгорании органического вещества массой 3,9 г образовались оксид углерода(1\/) массой 13,2 г и вода массой 2,7 г. Выведите формулу веще- ства, зная, что плотность паров этого вещества по водороду равна 39. Дано: Найти: Л7(СО2) = 13,2 г СхНуОг л?(Н2О) = 2,7 г лт(СхНуОг) = 3,9 г DH =39 Решение Так как продуктами горения органического вещества являются только углекислый газ и вода, то искомое вещество содержит только углерод, водород и, возможно, кислород. Обозначим число атомов углерода х, число атомов водорода у и число атомов кислорода z. Тогда формула искомого вещества будет иметь следующий молярную массу искомого вещества исходя из формулы 1) Вычисляем D (А) = ИА). ) М(В) • М(СхНуОг) = ОНг • М(Н2) = 39 • 2 г/моль = 78 г/моль 2) Находим количества всех веществ, приведённых в условии: л(СО2) = 13,2 г: 44 г/моль = 0,3 моль л(Н2О) = 2,7 г: 18 г/моль = 0,15 моль л(СхНуОг) = 3,9 : 78 г/моль = 0,05 моль 3) Определяем количество вещества атомов углерода и водорода. В молекуле СО2 один атом углерода, поэтому л(С) = л(СО2) = 0,3 моль В молекуле Н2О два атома водорода, следовательно, л(Н) = 2л(Н2О) = 0,3 моль. 4) Находим х и у: х = л(С): л(СхНуО,) = 0,3 : 0,05 = 6 у = л(Н): л(СхНуОг) = 0,3 : 0,05 = 6
249 5) Вычисляем z. Подставляем в формулу искомого вещества х и у: С6Н6Ог. Запи- сываем выражение для вычисления молярной массы: Л4(С6Н6Ог) = 12 • 6 + 6 + 16 • z = 78 г/моль z = 0 Записываем молекулярную формулу вещества: С6Н6. Задача 20. /7ри сгорании органического вещества массой 15 г образовались оксид углерода(IV) объёмом 16,8 л (н. у.) и вода массой 18 г. Выведите формулу вещества, зная, что плотность паров этого вещества по фтороводороду равна 3. Дано: Найти: У(СО2) = 16,8 л СхНуОг т(Н20)= 18 г m(CxHyOz) = 15 г dhf ~ 3 Решение Так как продуктами горения органического вещества являются только углекислый газ и вода, то искомое вещество содержит только углерод, водород и, возможно, кислород. Обозначим число атомов углерода х, число атомов водорода у и число атомов кислорода z. Тогда формула вещества будет иметь следующий вид: СхНу02. 1) Вычисляем молярную массу искомого вещества исходя из формулы D (А)=^ в ' ' М(В) M(CxHyOz) = Dhf • M(HF) = 3-20 г/моль = 60 г/моль 2) Находим количества всех веществ, приведённых в условии: л(СО2) = 16,8 л : 22,4 л/моль = 0,75 моль л(Н2О) = 18 г : 18 г/моль = 1 моль л(СхНуОг) =15 г : 60 г/моль = 0,25 моль 3) Определяем количество вещества атомов углерода и водорода. В молекуле СО2 один атом углерода, поэтому л(С) = л(СО2) = 0,75 моль В молекуле Н2О два атома водорода, следовательно, л(Н) = 2л(Н2О)= 2 моль 4) Находим х и у: х = л(С): л(СхНуОг) = 0,75 : 0,25 = 3 у = л(Н): л(СхНуОг) = 2 : 0,25 = 8 5) Вычисляем z. Подставляем в формулу искомого вещества х и у: С3Н8Ог. Запи- сываем выражение для вычисления молярной массы: М(С3Н8Ог) = 12 • 3 + 8 + 16 • z = 60 г/моль z = 1 Записываем молекулярную формулу вещества: С3Н0О. Задача 21. При сгорании 0,45 г газообразного органического вещества выде- лилось 0,448 л углекислого газа, 0,63 г воды и 0,112 л азота (н. у.). Плотность исходного газообразного вещества по азоту 1,607. Установите молекулярную фор- мулу этого вещества.
250 Дано: 1/(СО2) = 0,448 л m(H2O) = 0,63 г m(CxHyOz) = 0,45 г l/(N2) = 0,112 л DM = 1,607 n2 Найти: Решение Так как продуктами горения органического вещества являются углекислый газ, азот и вода, то искомое вещество содержит углерод, водород, азот и, возможно, кислород. Обозначим число атомов углерода х, число атомов водорода у, число атомов азота р и число атомов кислорода z. Тогда формула вещества будет иметь следующий вид: CxHyNp02. 1) Вычисляем молярную массу искомого вещества исходя из формулы D (А) = в 1 ' М(В) M(CxHyNpOJ = DN2 • М(N2) = 1,607 • 28 г/моль = 45 г/моль 2) Находим количества всех веществ, приведённых в условии: л(СО2) = 0,448 л : 22,4 л/моль = 0,02 моль л(Н2О) = 0,63 г: 18 г/моль = 0,035 моль л(М2) = 0,448 л : 22,4 л/моль = 0,02 моль n(CxHyNpOJ = 0,45 г : 45 г/моль = 0,01 моль 3) Определяем количество вещества атомов углерода и водорода. В молекуле СО2 один атом углерода, поэтому л(С) = л(СО2) = 0,02 моль В молекуле Н2О два атома водорода, следовательно, л(Н) = 2л(Н2О) = 0,07 моль В молекуле Ь12два атома азота, следовательно, л(Ы) = 2л(М2) = 0,01 моль 4) Находим х, у и р: х = л(С): л(СхНуОг) = 0,02 : 0,01 = 2 у = л(Н): n(CxHyOJ = 0,07 : 0,01 = 7 р = л(N): л(СхНуОг) = 0,01 : 0,01 = 1 5) Вычисляем z. Подставляем в формулу искомого вещества х, у и р: C2H7NOZ. За- писываем выражение для вычисления молярной массы: M(C2H7NOJ = 12 • 2 + 7 + 14 + 16 • z = 45 г/моль z= 0 Записываем молекулярную формулу вещества: C2H7N. ♦ 4 •• Задача 22. При сгорании бескислородного органического вещества образова- лось 4,48 л углекислого газа (н. у.), 3,6 г воды и 3,65 г хлороводорода. Опреде- лите молекулярную формулу сгоревшего соединения.
251 Дано: Найти: 1/(СО2) = 4,48 л CxHyClz m(H2O) = 3,6 г m(HCI) = 3,65 г Решение Так как продуктом горения органического вещества, кроме углекислого газа и воды, является и хлороводород, то искомое вещество содержит, кроме углерода и водорода, ещё и хлор. Обозначим число атомов углерода х, число атомов водо- рода у и число атомов хлора z. Тогда формула вещества будет иметь следующий вид: СхНуС1г 1) Находим количества всех веществ, приведённых в условии: л(СО2) = 4,48 л : 22,4 л/моль = 0,2 моль л(Н2О) = 3,6 г: 18 г/моль = 0,2 моль л(НС1) = 3,65 г : 36,5 г/моль = 0,1 моль 2) Определяем количества атомов углерода, водорода и хлора. В молекуле СО2 один атом углерода, поэтому л(С) = л(СО2) = 0,2 моль Атомы водорода содержатся и в молекуле Н2О (два атома водорода), и в молекуле HCI, следовательно, л(Н) = 2л(Н2О) + л(НС1) = 0,4 + 0,1 = 0,5 моль В молекуле хлороводорода один атом хлора, поэтому л(С1) = л(НС1) = 0,1 моль 3) Находим соотношение х: у: z. x:y:z = 0,2:0,5:0,1 =2:5: 1 Записываем формулу искомого вещества: С2Н5С1. Задача 23. При сгорании органического вещества массой 9,2 г образовались оксид углерода(1\/) объёмом 6,72 л (н. у.) и вода массой 7,2 г. Выведите формулу вещества. Дано: Найти: 1/(СО2) = 6,72 л СхНуОг т(Н2О) = 7,2 г лэ(СхНуОг) = 9,2 г Решение 1) Находим количества всех веществ, приведённых в условии: л(СО2) = 6,72 л : 22,4 л/моль = 0,3 моль л(Н2О) = 7,2 г: 18 г/моль = 0,4 моль 2) Определяем количества атомов углерода и водорода. В молекуле СО2один атом углерода, поэтому л(С) = л(СО2) = 0,3 моль
252 В молекуле Н2О два атома водорода, следовательно, л(Н) = 2л(Н2О) = 0,8 моль 3) Находим массу углерода и водорода, входящих в данную порцию вещества: лэ(С) = л(С) • М(С) = 0,3 моль • 12 г/моль = 3,6 г л7(Н) = л(Н) • М(Н) = 0,8 моль • 1 г/моль = 0,8 г 4) Находим массу атомов кислорода, входящих в данную порцию вещества: т(О) = m(CxHxOJ - л?(С) - m(H) = 9,2 - 3,6 - 0,8 = 4,8 г 5) Определяем количество вещества атомов кислорода: л(О) = 4,8 г: 16 г/моль = 0,3 моль 6) Находим соотношение х: у: z: х: у: z = 0,3 : 0,8 : 0,3 = 3 : 8 : 3 Записываем формулу искомого вещества: С3Н8О3. Задача 24. При сгорании органического вещества массой 3 г образовались ок- сид углерода(1\/) объёмом 2,24 л (н. у.) и вода массой 1,8 г. Известно, что это вещество реагирует с цинком. На основании данных условия задания: 1) произведите вычисления, необходимые для установления молекулярной форму- лы органического вещества; 2) запишите молекулярную формулу исходного органического вещества; 3) составьте структурную формулу этого вещества, которая однозначно отражает порядок связи атомов в его молекуле; 4) напишите уравнение реакции этого вещества с цинком. Дано: Найти: У(СО2) = 2,24 л СХНД л?(Н2О) = 1,8 г Л7(СхНуОг) = 3 г Решение 1) Находим количества всех веществ, приведённых в условии: л(СО2) = 2,24 л : 22,4 л/моль = 0,1 моль л(Н2О) = 1,8 г: 18 г/моль = 0,1 моль 2) Определяем количества веществ атомов углерода и водорода. В молекуле С02один атом углерода, поэтому л(С) = л(СО2) = 0,1 моль В молекуле Н2О два атома водорода, следовательно, л(Н) = 2л(Н2О) = 0,2 моль 3) Находим массу углерода и водорода, входящих в данную порцию вещества: л?(С) = л(С) • М(С) = 0,1 моль • 12 г/моль =1,2 г т(Н) = л(Н) • А4(Н) = 0,2 моль • 1 г/моль = 0,2 г 4) Находим массу атомов кислорода, входящих в данную порцию вещества: Л1(О) = Л7(СХН О ) - Л7(С) - лэ(Н) = 3 - 1,2 - 0,2 =1,6 г А у
253 5) Определяем количество вещества атомов кислорода: л(0) = 1,6 г: 16 г/моль = 0,1 моль 6) Находим соотношение х: y:z: x:y:z = 0,1 : 0,2:0,1 = 1:2:1 Записываем простейшую (эмпирическую) формулу искомого вещества: СН2О. Эта формула соответствует формальдегиду. Но формальдегид не реагирует с цинком. Если удвоить число атомов всех элементов в молекуле, то получаем формулу С2Н4О2. Такую молекулярную формулу имеет уксусная кислота СН3СООН. Уксус- ная кислота реагирует с цинком: 2СН3СООН + Zn -> (CH3COO)2Zn + Н2 Задача 25. При сгорании 40,95 г органического вещества получили 39,2 л угле- кислого газа (н. у.), 3,92 л азота и 34,65 г воды. При нагревании с соляной кис- лотой данное вещество подвергается гидролизу, продуктами которого являются соединение состава C/igNO^CI и вторичный спирт. На основании данных условия задания: 1) произведите вычисления, необходимые для установления молекулярной форму- лы органического вещества; 2) запишите молекулярную формулу исходного органического вещества; 3) составьте структурную формулу этого вещества, которая однозначно отражает порядок связи атомов в его молекуле; 4) напишите уравнение реакции гидролиза исходного вещества в присутствии со- ляной кислоты. Решение 1) Находим количество вещества продуктов сгорания: л(СО2) = 39,2 : 22,4 = 1,75 моль; л(С) = 1,75 моль л(Н2О) = 34,65/18= 1,925 моль; л(Н) = 1,925 • 2 = 3,85 моль n(N2) = 3,92/22,4 = 0,175 моль; л(М) = 0,175 • 2 = 0,35 моль 2) Установим массу и количество вещества атомов кислорода, определим молеку- лярную формулу вещества: л?(С + Н + N) = 1,75 • 12 + 3,85 • 1 +0,35 • 14 = 29,75 г л?(О) = 40,95 - 29,75=11,2 г л(О) = 11,2/16 = 0,7 моль л(С): л(Н): л(М): л(О) = 1,75 : 3,85 : 0,35 : 0,7 = 5 : 11 : 1 : 2 Молекулярная формула — CgH^NO^ 3) Составим структурную формулу вещества: nh2—сн2—с' о—сн—сн3 снч 4) Напишем уравнение реакции гидролиза вещества: о он / I nh2—сн2—С +Н2О + НС1 ►C1[H3N—сн2—соон] + сн3—сн—сн3 о—сн—сн3 сн3
254 Нахождение молекулярной формулы органического вещества по массовой доле элементов в соединении Задача 26. Массовая доля кислорода в двухатомном предельном спирте со- ставляет 42,05 %. Известно, что это вещество не может быть получено окислением соответствующе- го алкена. На основании данных условия задания: 1) произведите вычисления, необходимые для установления молекулярной форму- лы органического вещества; 2) запишите молекулярную формулу исходного органического вещества; 3) составьте структурную формулу этого вещества, которая однозначно отражает порядок связи атомов в его молекуле. Решение 1) Общая формула вещества — С„Н2п + 2О2. Воспользовавшись формулой и/(Э) = л , вычислим молярную массу двухатом- м ного спирта: М(СлН2п + 2О2) = 16 • 2 : 0,4205 = 76 г/моль 2) Молярная масса двухатомного спирта может быть вычислена по формуле М = 12л + 2л + 2 + 16 • 2 Составляем уравнение и находим п — число атомов углерода в спирте: 14л + 34 = 76 14л = 42 л = 3 Молекулярная формула вещества — С3Н8О2. 3) Приводим структурную формулу вещества. Поскольку по условию задачи двухатомный спирт не может быть получен окисле- нием соответствующего алкена, то делаем вывод, что гидроксильные группы на- ходятся не у соседних атомов углерода: НО-СН2-СН2-СН2-ОН Задача 27. Массовая доля углерода в некотором органическом соединении со- ставляет 64,82 %, а массовая доля кислорода — 21,58 %. В ходе исследования химических свойств этого вещества установлено, что оно взаимодействует с уксусной кислотой. Известно также, что вещество не может быть получено восстановлением альдегида и что молекула данного вещества не содержит третичных атомов углерода. На основании данных условия задания: 1) произведите вычисления, необходимые для установления молекулярной форму- лы органического вещества; 2) запишите молекулярную формулу исходного органического вещества; 3) составьте структурную формулу этого вещества, которая однозначно отражает порядок связи атомов в его молекуле; 4) напишите уравнение реакции этого вещества с уксусной кислотой.
255 Решение 1) Находим соотношение числа атомов элементов в соединении. Общая формула вещества от Пусть взято 100 г вещества, тогда т(С) = 64,82 г л?(О) = 21,58 г /т?(Н) = 100 - 21,58 - 64,82 = 13,6 г л(С) = 64,82 : 12 = 5,40 моль л(О) = 21,58 : 16= 1,35 моль л(Н)= 13,6 : 1 = 13,6 моль 2) Определяем молекулярную формула вещества: х:у:г = л(С): л(Н): л(О) = 5,40 : 13,6: 1,35 = 4: 10 : 1 Простейшая молекулярная формула вещества — С4Н10О. Это может быть спирт или простой эфир. 3) Приводим структурную формулу вещества: так так вещество реагирует с уксус- ной кислотой, то это спирт; поскольку по условию данный спирт не может быть получен восстановлением соответствующего альдегида, а третичные атомы угле- рода в его молекуле отсутствуют, спирт является вторичным: сн,—сн— сн,—снч о । А 9 он 4) Составляем уравнение реакции вещества с уксусной кислотой: н+ сн3—сн—сн2—сн3 + сн3соон —*- сн3—сн—сн2—сн3 + н2о он о—с—сн3 о Ещё раз повторим, что многие задачи могут быть решены несколькими способа- ми. Учащиеся могут выбирать наиболее удобный для них способ решения. При- ведём два способа решения следующей задачи. Задача 28. При сгорании 17,5 г органического вещества получили 28 л угле- кислого газа (н. у.) и 22,5 мл воды. Плотность паров этого вещества составляет 3,125 г/л. Известно также, что это вещество было получено в результате дегид- ратации третичного спирта. На основании данных условия задачи: 1) произведите вычисления, необходимые для установления молекулярной форму- лы органического вещества; 2) запишите молекулярную формулу органического вещества; 3) составьте структурную формулу исходного вещества, которая однозначно отра- жает порядок связи атомов в его молекуле; 4) напишите уравнение реакции получения данного вещества дегидратацией соот- ветствующего третичного спирта.
256 Решение 1 Общая формула вещества — СхНуО2. 1) Найдём количества веществ углекислого газа, воды и органического вещества: л(СО2) = 28 : 22,4 = 1,25 моль л(Н2О) = 22,5 : 18 = 1,25 моль M(CxHyOJ = 3,125 • 22,4 = 70 г/моль л(С Н О ) = 17,5 : 70 = 0,25 моль 2) Определим молекулярную формулу вещества: л(С) = л(СО2) = 1,25 моль л(Н) = 2л(Н2О) = 2,5 моль х = л(С): л(СхНу02) = 1,25 : 0,25 = 5 у = л(Н): л(СхНуОг) = 2,5 : 0,25 = 10 5 • 12+ 10 + z • 16 = 70 z- 0 Молекулярная формула вещества — С5Н10. 3) Составим структурную формулу вещества: сн3 сн3—с=сн—сн3 4) Составим уравнение реакции получения вещества из третичного спирта: сн3 сн3 Н28О.(конц.), t сн3—с—сн2—сн3-------------*- сн3—с=сн—сн3 + н2о он Решение 2 Общая формула вещества — СхНу07. 1) Найдём количества веществ углекислого газа и воды, определим молярную массу органического вещества: л(СО2) = 28 : 22,4 = 1,25 моль л(Н2О) = 22,5 : 18 = 1,25 моль M(CxHyOJ = 3,125 • 22,4 = 70 г/моль 2) Определим молекулярную формулу вещества: л(С) = л(СО2) = 1,25 моль л(Н) = 2л(Н2О) = 2,5 моль При дегидратации спирта может образоваться алкен или простой эфир. Получен- ное соотношение атомов углерода и водорода в молекуле соответствует формуле алкена СхН2х. Молярная масса алкена вычисляется по формуле MQJIJ = 14х 14х = 70 х = 5 Молекулярная формула вещества — С5Н10. 3) Составим структурную формулу вещества: сн3— С=СН—сн3
Содержание Предисловие...................................................... 3 Тематическое планирование курса «Я СДАМ ЕГЭ»..................... 4 МОДУЛЬ I. ТЕОРЕТИЧЕСКИЕ ОСНОВЫ ХИМИИ............................. 7 Занятие 1. Строение электронных оболочек и электронная конфигу- рация атомов. Периодический закон и Периодическая система химиче- ских элементов Д. И. Менделеева. Закономерности изменения химических свойств элементов и их соединений по периодам и группам................. Занятие 2. Электроотрицательность, степень окисления и валентность хи- мических элементов. Химическая связь. Вещества молекулярного и не- молекулярного строения................................................ 17 Занятие 3. Классификация и номенклатура неорганических веществ. Классификация химических реакций...................................... 30 Занятие 4. Скорость реакции, её зависимость от различных факторов. Обратимые и необратимые химические реакции. Химическое равновесие 41 Занятие 5. Электролитическая диссоциация. Сильные и слабые электро- литы. Реакции ионного обмена ......................................... 46 Занятие 6. Гидролиз .................................................. 51 Занятие 7. Окислительно-восстановительные реакции .................... 55 МОДУЛЬ II. НЕОРГАНИЧЕСКАЯ ХИМИЯ ........................................ 71 Занятие 8. Характерные химические свойства представителей основных классов неорганических веществ ........................................ — Занятие 9. Свойства галогенов и их соединений ........................ 83 Занятие 10. Неметаллы VIA-группы: кислород, сера ..................... 94 Занятие 11. Азот и его соединения ................................... 108 Занятие 12. Фосфор и его соединения ................................. 120 Занятие 13. Неметаллы IVA-группы: углерод, кремний................... 128 Занятие 14. Общие свойства металлов. Металлы А-групп ................ 140 Занятие 15. Металлы В-групп .......................................... 145 МОДУЛЬ III. ОРГАНИЧЕСКАЯ ХИМИЯ......................................... 159 Занятие 16. Классификация и номенклатура органических веществ. Теория строения органических соединений ...................................... — Занятие 17. Алканы и циклоалканы .................................... 165 Занятие 18. Алкены и алкадиены ...................................... 169 Занятие 19. Алкины................................................... 174 Занятие 20. Ароматические углеводороды................................ 177 Занятие 21. Спирты и фенолы ......................................... 188 Занятие 22. Карбонильные соединения: альдегиды и кетоны ............. 197 Занятие 23. Карбоновые кислоты и сложные эфиры ...................... 202 Занятие 24. Амины и аминокислоты .................................... 216 Занятие 25. Жиры. Углеводы........................................... 224 МОДУЛЬ IV. КОЛИЧЕСТВЕННЫЕ ОТНОШЕНИЯ В ХИМИИ: РАСЧЁТЫ ПО ХИМИЧЕСКИМ ФОРМУЛАМ И УРАВНЕНИЯМ ХИМИЧЕСКИХ РЕАКЦИЙ................... 234 Занятие 26. Расчёты массы вещества или объёма газа по известному ко- личеству вещества, массе или объёму одного из участвующих в реакции веществ.............................................................. 235 Занятие 27. Расчёты с использованием понятия «массовая доля вещества в растворе».......................................................... 238 Занятие 28. Задачи на установление молекулярной и структурной формулы вещества ............................................................ 246